273
Pg 1 NIESV STUDY PACK - PPE STUDY PACK FOR P P R R O O F F E E S S S S I I O O N N A A L L P P R R A A C C T T I I C C E E ( ( P P P P E E ) ) THE NIGERIAN INSTITUTION OF ESTATE SURVEYORS AND VALUERS (NIESV)

unitedemsa.files.wordpress.com...Pg 2 NIESV STUDY PACK - PPE KNOWLEDGE AND STRUCTURE OF NIGERIA INSTITUTION OF ESTATE SURVEYORS AND VALUERS Learning Objectives At the end of this lecture,

  • Upload
    others

  • View
    1

  • Download
    0

Embed Size (px)

Citation preview

Page 1: unitedemsa.files.wordpress.com...Pg 2 NIESV STUDY PACK - PPE KNOWLEDGE AND STRUCTURE OF NIGERIA INSTITUTION OF ESTATE SURVEYORS AND VALUERS Learning Objectives At the end of this lecture,

Pg 1

NIESV STUDY PACK - PPE

STUDY PACK

FOR

PPRROOFFEESSSSIIOONNAALL PPRRAACCTTIICCEE ((PPPPEE))

THE NIGERIAN INSTITUTION OF ESTATE SURVEYORS AND VALUERS (NIESV)

Page 2: unitedemsa.files.wordpress.com...Pg 2 NIESV STUDY PACK - PPE KNOWLEDGE AND STRUCTURE OF NIGERIA INSTITUTION OF ESTATE SURVEYORS AND VALUERS Learning Objectives At the end of this lecture,

Pg 2

NIESV STUDY PACK - PPE

KNOWLEDGE AND STRUCTURE OF NIGERIA INSTITUTION OF ESTATE

SURVEYORS AND VALUERS

Learning Objectives

At the end of this lecture, students are expected to have acquired basic understanding of the

structure as well as operations of Nigeria Institution of Estate Surveyors and Valuers (NIESV)

and should be able to:

a. Demonstrate in clear terms good knowledge of the institution.

b. Understand the broad idea of the structure of the institution and their inter relationship.

c. Establish an understanding of the basic requirements for membership of the institution.

d. Understand the qualifications required at every stage of the institution.

e. Acquire a basic understanding of firm‟s registration with the institution.

f. Demonstrate high understanding of the various cadres of the institution.

Introduction

The Nigerian Institution of Estate Surveyors and Valuers (NIESV) was founded in 1969 by the

few qualified General Practice Chartered Surveyors who were trained mainly in the United

Kingdom. The Institution was granted government recognition by the enactment of the Estate

Surveyors and Valuers (Registration Act)" Decree No. 24 of 1975.

The first Annual Conference was held at Ibadan in 1969. The Estate Surveyors and Valuers

Registration Board of Nigeria (ESVARBON) is empowered to regulate and control the practice

of the profession of Estate Surveying and Valuation in the country. The Institution is affiliated to

the International Real Estate Federation (FIABCI), Commonwealth Association of Surveying

and Land Economy (CASLE), International Federation of Surveyors (FIG), Royal Institution of

Chartered Surveyors (RICS), Association of Professional Bodies of Nigeria (APBN) and The

International Valuation Standards Council (IVSC).

Page 3: unitedemsa.files.wordpress.com...Pg 2 NIESV STUDY PACK - PPE KNOWLEDGE AND STRUCTURE OF NIGERIA INSTITUTION OF ESTATE SURVEYORS AND VALUERS Learning Objectives At the end of this lecture,

Pg 3

NIESV STUDY PACK - PPE

BASIC REQUIREMENTS FOR MEMBERSHIP OF THE INSTITUTION

GRADUATES: To become a graduate member, candidates are expected to possess the

following qualifications:

1. B.Sc. Degree Certificate in Estate Management

2. FIVE (5) O'level results with credits in Mathematics, English, Economics and any two

(2) of the following subjects; Biology, Chemistry, Physics, Geography, Commerce,

Technical Drawing, Accounts and Fine Arts.

3. NYSC discharge certificate or exemption letter; and

4. Recommendation letter from an approved establishment under the supervision of an

Associate Member of the Institution with at least 5 years experience as an Associate.

PROBATIONERS: The qualifications that are required are:

1. HND Certificate and OND Certificates in Estate management from an accredited

Polytechnic;

2. FIVE (5) O' level results with credits in Mathematics, English, Economics and any two

(2) of the following subjects; Biology, Chemistry, Physics, Geography, Commerce,

Technical Drawing, Accounts and Fine Arts.

3. NYSC discharge certificate or exemption letter; and

4. Recommendation letter from an approved establishment under the supervision of an

Associate Member of the Institution with at least 5 years experience as an Associate.

STUDENTS: candidates who aspire to join the institution as student members should be holders

of the following qualifications:

1. OND Certificate

Page 4: unitedemsa.files.wordpress.com...Pg 2 NIESV STUDY PACK - PPE KNOWLEDGE AND STRUCTURE OF NIGERIA INSTITUTION OF ESTATE SURVEYORS AND VALUERS Learning Objectives At the end of this lecture,

Pg 4

NIESV STUDY PACK - PPE

2. O' Level Certificate - FIVE (5) O' level results with credits in Mathematics, English,

Economics and any two (2) of the following subjects; Biology, Chemistry, Physics,

Geography, Commerce, Technical Drawing, Accounts and Fine Arts; and a

3. Letter of recommendation from an approved industry bases supervisor/Employer.

SENIOR PROFESSIONAL: These categories of members are matured candidates that fall into

the following categories:

1) A degree or Diploma in Estate Management and Valuation.

2) A member of the Royal Institution of Chartered Surveyors (RICS), at Associate level for

not less than five (5) years.

3) Must have held or is holding a Management position in the Real Estate Section of a

company (if in employment) or has been working in an Estate Surveying Firm recognized

by the Institution.

4) Candidate shall submit a detailed resume detailing qualification and experience (resume

template is obtainable at the NIESV's Secretariat).

5) Candidate shall submit a case study on any aspect of Real Estate Market in Nigeria or

the Profession, of not more than 5000 words.

6) Candidate shall submit his application for election through the Branch Chairman of his

place of residence or work.

7) Candidate shall be invited to appear before the Management Committee

FIRM: Corporate organisations that desire to register with the institution for professional firms

shall:

1.) Purchase of registration form for ₦10,000

2.) make payment for processing of registration form for ₦25,000

3.) produce a letter of resignation & letter of acceptance from previous employer

4.) Certificate of compliance from ESVARBON

Page 5: unitedemsa.files.wordpress.com...Pg 2 NIESV STUDY PACK - PPE KNOWLEDGE AND STRUCTURE OF NIGERIA INSTITUTION OF ESTATE SURVEYORS AND VALUERS Learning Objectives At the end of this lecture,

Pg 5

NIESV STUDY PACK - PPE

5.) Certificate of business registration from CAC to read "real estate surveying & valuation only

6.) Up to date payment of N.I.E.S.V. Dues and pledges

7.) Two recent passport photographs

8.) Certificate of professional indemnity minimum of ₦1 m cover

9.) Evidence of client dedicated account

10.) Evidence of account opening with the business name

11.) Recent phone numbers

A member aspiring to be upgraded from Student to Probationer or Graduate membership after

he/she must have past the NIESV professional examinations or has obtained additional

qualifications should write formally to the Head Education Committee for processing.

Recommendations from the Membership Committee:

1.0 STUDENT MEMBERSHIP

Student members are those who decide to join The Nigerian Institution of Estate Surveyors and

Valuers (NIESV) by writing the Institution's Examinations starting from Foundation

Examination (FE)to Professional Practice Examination (PPE).

This category includes those:

i. Studying Estate Management and Valuation in accredited Institutions,

ii. From Allied Professional Bodies; and

iii. Working in Organizations/Firms approved by the Institution.

In all cases, the candidate must have the requisite entry qualification for admission to read Estate

Management and Valuation.

2.0 GRADUATES/PROBATIONER MEMBERSHIP

These are graduates from Universities and Polytechnics offering Estate Management and

Valuation. These categories of members are required to take the following Institution's

Examinations.

i. Graduates from accredited Universities are required to sit for Professional Practice

Examination (PPE).

Page 6: unitedemsa.files.wordpress.com...Pg 2 NIESV STUDY PACK - PPE KNOWLEDGE AND STRUCTURE OF NIGERIA INSTITUTION OF ESTATE SURVEYORS AND VALUERS Learning Objectives At the end of this lecture,

Pg 6

NIESV STUDY PACK - PPE

If however the university was not accredited at the time the candidate graduated, such a

candidate will be required to write Professional Qualifying Examination 3 (PQE3) and

Professional Practice Examination (PPE).

ii. Graduates from Polytechnics are required to sit for Professional Qualifying Examination 3

(PQE 3) and Professional Practice Examination (PPE).

If however, the Polytechnic was not accredited at the time the candidate graduated, such a

candidate will sit for Professional Qualifying Examination 2 (PQE 2), Professional Qualifying

Examination 3 (PQE 3) and Professional Practice Examination (PPE).

3.0 ASSOCIATE MEMBERSHIP

Student or Graduate/Probationer member, who is successful in the Institution's Examinations as

in paragraph 1.0 and 2.0 above, respectively shall be eligible for election as an Associate

Member of the Institution.

3.1 ELECTION INTERVIEW:

To qualify for election into the corporate membership of the institution requires the following:

i) The candidate must have gained practical experience in a recognized office for a minimum

period of two years, post graduation/final qualifying examination.

ii) The Candidates shall submit a Topic and synopsis for a Critical Analysis in one of his/her area

of practical experience during the two year tutelage for approval.

iii) The approval synopsis expires after two year from the date to approval and can be revalidated

on the payment of N10,000.

iv) The Critical Analysis must be supervised by a financially update Estate Surveyor and Valuer

of not less than five (5) years post Associate Election.

v) Candidate can apply for approval of a supervisor where there is no qualified estate Surveyor

and Valuer in his/her place of employment.

vi) Candidate shall apply for approval for change of supervisor on change of employment

vii) The candidate shall keep a Log Book and Diary on all his/her practical work within the two

(2) years tutelage.

viii) Within the period of probation, the candidate must have attended a minimum of two (2)

national Conferences and Continuing Professional Development Seminar, thus, obtaining a

Page 7: unitedemsa.files.wordpress.com...Pg 2 NIESV STUDY PACK - PPE KNOWLEDGE AND STRUCTURE OF NIGERIA INSTITUTION OF ESTATE SURVEYORS AND VALUERS Learning Objectives At the end of this lecture,

Pg 7

NIESV STUDY PACK - PPE

minimum total of 20 credit points.

The credit points are assigned as follows:

National Conference - 7 points per attendance

National MCPD - 5 points

One Day State Branch organized MCPD - 3 points

Two Day State Branch organized MCPD - 5 points

3.2 APPLICATION FOR INTERVIEW

i. Eligible candidate shall complete the Institution's prescribed Application form

ii. The Form must be signed by:

- a. Candidate's employer

- b. Two (2) financially update Fellows of the Institution.

- c. Two (2) financially update Associates of not less than five years post Associate election.

- d. The Branch Chairman of the Candidate.

iii. The application Form shall be submitted along with:

a. Stipulated copies of Critical Analysis duly certified and sealed by the Supervisor.

b. Log books, Diary and Analysis Sheets duly completed.

c. Payment of the required Application Fee including any outstanding due to the Institution.

3.3 ELECTION INTERVIEW

i. Candidates who have satisfied the above stipulated requirements shall be invited to appear

before the Membership Committee for interview.

ii. At the interview, Candidate is required to:

- a. Present all the Original Certificates for sighting by the Interview Panel.

- b. Satisfy the Interview Panel on his/her competence on the Critical Analysis, Professional

knowledge and Current Affairs.

- c. Score a minimum of 50% average for a pass mark

- d. Donate a minimum of two standard textbooks in any area of Estate Management and

Valuation.

- e. Make a handsome donation to the Institution's Benevolent Fund.

Page 8: unitedemsa.files.wordpress.com...Pg 2 NIESV STUDY PACK - PPE KNOWLEDGE AND STRUCTURE OF NIGERIA INSTITUTION OF ESTATE SURVEYORS AND VALUERS Learning Objectives At the end of this lecture,

Pg 8

NIESV STUDY PACK - PPE

iii. Candidates who did not honour invitation to interview can be reinvited on payment of

N5,000.00

iv. Candidates who were not successful at the interview can be reinvited on payment of

N5,000.00

3.4 ELECTION THROUGH THE SENIOR PROFESSIONAL ROUTE

A candidate to be considered under this route must possess:

i) A degree or Diploma in Estate Management and Valuation.

ii) Be a member of the Royal Institution of Chartered Surveyors (RICS), at Associate level for

not less than five (5) years.

iii) Must have held or is holding a Management position in the Real Estate Section of a company

(if in employment) or has been working in an Estate Surveying Firm recognized by the

Institution.

iv) Candidate shall submit a detailed resume detailing qualification and experience (resume

template is obtainable at the NIESV's Secretariat).

v) Candidate shall submit a case study on any aspect of Real Estate Market in Nigeria or the

Profession, of not more than 5000 words.

vi) Candidate shall submit his application for election through the Branch Chairman of his place

of residence or work.

vii) Candidate shall be invited to appear before the Management Committee and shall be

required to fulfill the conditions in paragraph 3.3 (ii) above.

viii) Candidates must have put in 25 years in service

4.0 FELLOWS

Associates above ten years could apply to be admitted into the fellowship cadre of the Institution

through the Privileges Committee.

Grades of Membership

Membership of The Nigerian Institution of Estates Surveyors and Valuers is open to all

Nigerians, and in certain cases to foreigners, who have obtained the requisite training in estate

Page 9: unitedemsa.files.wordpress.com...Pg 2 NIESV STUDY PACK - PPE KNOWLEDGE AND STRUCTURE OF NIGERIA INSTITUTION OF ESTATE SURVEYORS AND VALUERS Learning Objectives At the end of this lecture,

Pg 9

NIESV STUDY PACK - PPE

surveying and valuation and in the broad field of estate management at approved Institutions

both within and outside the country. Foreigners desiring to be members must show proof that

they are conversant with laws guiding the ownership of land and property in Nigeria.

Membership can also be attained through sitting for the Institutions qualifying professional

examinations.

There are four categories of Membership, namely :

1. Professional

2. Non-professional

3. Corporate or firm

4. Honorary

1. Professional membership is made up of Associates and Fellows, and they are those who have

undergone the prescribed training, passed the qualifying examinations and interviews and

obtained the necessary Practical experience required to practice Estate Surveying.

2. Non-Professional membership includes Graduates, Probationers and Students who are in the

process of taking the qualifying professional examination or interview but are attached to firms

of the Institution for the furtherance of their professional practice.

3. Corporate or Firm Membership includes registered organization which are licensed to practice

Estate Surveying as a legal entity.

4. Honorary Members are persons who are not engaged in the Practice of the profession, but by

reason of position, experience or eminence assist in promoting the objects of the Institution. i.e.:

The Grand Patron of The Institution is the President and Commander-in-Chief of the Armed

Forces of the Federal republic of Nigeria. The State Governors are the Patrons of The Institution

in their respective states.

Abbreviation

A Past President of The Institution (NIESV) may have after his or her name the initials

"PPNIVS", a Fellow may use the abbreviation "FNIVS", and an associate may use the letters

"ANIVS" while an Honorary Member may attach after his or her name the initials "HNIVS".

Code of Practice

Page 10: unitedemsa.files.wordpress.com...Pg 2 NIESV STUDY PACK - PPE KNOWLEDGE AND STRUCTURE OF NIGERIA INSTITUTION OF ESTATE SURVEYORS AND VALUERS Learning Objectives At the end of this lecture,

Pg 10

NIESV STUDY PACK - PPE

All members of The Institution shall conduct themselves in accordance with the Code of

Professional Ethics and Practice that has been passed and approved by the general meeting of the

Institution.

Code of Conducts for ESV

Decree 24 of 1975; the law regulating estate surveying and valuation practice in Nigeria set out

code of conduct for professional Estate Surveyor and Valuers (ESV), some of the rules are as

follows:

No Estate Surveyor & Valuer shall be connected in any way with any occupation or

business which in the opinion of the Board is incompatible with his professional status as

Estate Surveyor & Valuer.

It shall be the duty of Estate Surveyor & Valuer when acting for a client whose interest

conflicts or may conflict with his own to disclose the relevant facts to his client. He

should also not act for two or more parties with conflicting interest without disclosing the

relevant facts to those parties.

Every Estate Surveyor & Valuer who is a sole principal of a practice or a partner of a

firm carrying on the practice of estate surveyors and valuers shall be responsible for any

contravention of regulation committed by any other partner or any member of the staff of

such practice firm or company.

No Estate Surveyor & Valuer shall in his professional capacity accept otherwise than for

the benefit of the client any trade or other commercial discount or commission from any

person whose business consist in provision of goods and service used in the building

construction or building maintenance industries.

No Estate Surveyor & Valuer shall offer to provide whether to a third party any gift or

favour whether in money or kind designed to secure instruction for work or introduction

of a client.

No Estate Surveyor & Valuer shall solicit instructions for work in any manner whatever.

Every Estate Surveyor & Valuer shall keep in one or more Bank Accounts, separate from

his own, his firm‟s or bank account any money held by or entrusted to him or his firm in

Page 11: unitedemsa.files.wordpress.com...Pg 2 NIESV STUDY PACK - PPE KNOWLEDGE AND STRUCTURE OF NIGERIA INSTITUTION OF ESTATE SURVEYORS AND VALUERS Learning Objectives At the end of this lecture,

Pg 11

NIESV STUDY PACK - PPE

any capacity other than of beneficial owner. He should also account at the due time for

the moneys held to the parties entitled to it

No person shall practice estate surveying and valuation under more than one business

name.

No Estate Surveyor & Valuer shall buy property of client except he discloses to the client

all the material fact about the property and secure the prior approval of his client.

An Estate Surveyor & Valuer shall charge his fees in accordance with the current

approved scale of professional charges.

Hanging of multiple “FOR SALE” or “TO LET” board by different estate surveying and

valuation firm is not allowed.

An Estate Surveyor & Valuer employed by a limited liability company or government

agencies in active service cannot undertake service of estate surveying and valuation to

member of public for fee.

DISCIPLINE OF MEMBERS

The Nigerian Institution of Estate Surveyors and Valuers has an effective self-regulating and

disciplinary system which has been tested and up-dated over time.

Upon the receipt of a report or petition of an act of indiscipline or professional misconduct

against a member, the National Council of the Institution refers the matter to its disciplinary

organ the Professional Practice Committee (PPC) to inquire into the allegation in all its

ramifications; and where such a member if found liable, recommend appropriate sanctions in line

with Constitution of The Institution.

ENTRY GUIDELINES FOR NIESV PROFESSIONAL EXAMINATION

Basic Entry Requirements Candidate seeking entry into The Nigerian Institution of Estate

Surveyors and Valuers (NIESV) Professional Qualifying Examinations (PQE) must possess

Senior Secondary School Certificate of Education (SSCE), or its equivalent with credit passes in

at least five subjects which must include Mathematics, English Language, Economics, one basic

Page 12: unitedemsa.files.wordpress.com...Pg 2 NIESV STUDY PACK - PPE KNOWLEDGE AND STRUCTURE OF NIGERIA INSTITUTION OF ESTATE SURVEYORS AND VALUERS Learning Objectives At the end of this lecture,

Pg 12

NIESV STUDY PACK - PPE

science from (Biology, Physics, Chemistry, Agricultural Science) and any one from Geography,

Government, Commerce, Business Studies, Technical Drawing, Fine Arts, Accounting, Building

Construction, Wood Work and Metal Work at not more than two sittings.

Page 13: unitedemsa.files.wordpress.com...Pg 2 NIESV STUDY PACK - PPE KNOWLEDGE AND STRUCTURE OF NIGERIA INSTITUTION OF ESTATE SURVEYORS AND VALUERS Learning Objectives At the end of this lecture,

Pg 13

NIESV STUDY PACK - PPE

ESTATE SURVEYORS AND VALUERS REGISTRATION BOARD OF NIGERIA

INTRODUCTION

In 1954, the first Nigerian Estate Surveyor and Valuer, John Wood Ekpeyong, graduated from

the University of London with B.Sc degree in Estate Management. Since then, the profession has

witnessed phenomenal and tremendous growth.

The profession gained limelight in the country when, in 1969, a group of qualified Chartered

(General Practice) Surveyors formed what is known as the Nigeria Institution of Estate

Surveyors and Valuers (NIESV), as a non-profit voluntary professional organization to cater for

the interests of the Landed profession in Nigeria, Six years later, it was accorded Government

recognition by the promulgation of the Estate Surveyors and Valuers (Registration, ETC) Decree

No. 24 of 1975, now CAP III (Laws of the federal Republic of Nigeria) 1990

COMMITTEES OF THE BOARD

“The Board operates by committees. Those currently in place are:

INVESTIGATING PANEL

This Committee is charged with responsibility and investigating petitions levied against

members alleged to have committed professional misconduct. Member found culpable are

charged to the tribunal for further trial. The committee has been involved in instilling values

pertinent to the committee.

PROFESSIONAL PRACTICE COMMITTEE

The Committee conducts registration interviews for candidates who apply to be registered Estate

Surveyors and valuers. It also handles issues relating to professional practice.

RULES AND REGULATION COMMITTEE

The committee comprises professionals vast with knowledge to formulate and prepare rules and

regulations for the practice of Real Estate profession in Nigeria. The committee also prepares

adequate procedures for registration of Estate Surveyors and Valuers and formulates the rules

and regulation for the training of Estate Surveyors and Valuers.

Page 14: unitedemsa.files.wordpress.com...Pg 2 NIESV STUDY PACK - PPE KNOWLEDGE AND STRUCTURE OF NIGERIA INSTITUTION OF ESTATE SURVEYORS AND VALUERS Learning Objectives At the end of this lecture,

Pg 14

NIESV STUDY PACK - PPE

The aforementioned functions and other function are still subject to the approval of the Board.

EDUCATION COMMITTEE

This is the Committee that is saddled with matters pertaining to education. The committee has

experience scholars as members and regularly carries out accreditation visitations to tertiary

institutions offering programmes in Estate Management and Valuation. As part of its activities,

the committee also conducts a programme tagged "Train the Trainer" for lecturers in Tertiary

institutions in the area of Information Communication Technology (ICT).

THE TRIBUNAL

The Tribunal is a court of appellate jurisdiction. This is because its awards are only appealable in

the Supreme Court of Nigeria. It is headed by the Chairman of the Board and its membership

comprises experienced elder of the noble profession. As provided by the enabling law, an

Assessor guides the Tribunal on its proceedings relating to matters of the law. This Assessor

must be a senior Advocate of Nigeria, appointed by the committees‟ recommendation are subject

to ratification and approval of the board except those of the Tribunal.

EXECUTIVE COMMITTEE/FINANCE AND GENERAL PURPOSE COMMITTEE

The Committee is charged with the responsibility of handing staff matters, staff welfare and

financial issues of the Board

HOW TO BECOME A MEMBER

(1) Subject to section 15 and to rules made under section 6 (4) of ESVARBON Act, a person

shall be entitled to be registered as an estate surveyor and valuer if:

(a) he passes the qualifying examination for membership recognized or conducted by The

Board under this Act and immediately before the commencement of this Act, he was a

member of The Institution; or

(b) he holds a qualification granted outside Nigeria which for the time being is accepted

by The Board and he is by law entitled to practice for all purpose as estate surveyor and

valuer in the country in which the qualification was granted and, if The Board so,

Page 15: unitedemsa.files.wordpress.com...Pg 2 NIESV STUDY PACK - PPE KNOWLEDGE AND STRUCTURE OF NIGERIA INSTITUTION OF ESTATE SURVEYORS AND VALUERS Learning Objectives At the end of this lecture,

Pg 15

NIESV STUDY PACK - PPE

requires, he satisfies The Board that he has had sufficient practical experience as estate

surveyor and valuer.

(2) Subject as aforesaid, a person shall be entitled to be registered under this Act if

(a) he holds a certificate recognized by the Board and has had not less than two years post

qualification practical experience in the profession; or

(b) he has passed an examination approved by the Board and has had not less than two

years post qualification practical experience in the profession.

(3) An applicant for registration under this Act shall, in addition to evidence of qualification,

satisfy The Board

(a) that he is of good character

(b) that he has attained the age of 21 years; and

(c) that he has not been convicted in Nigeria or elsewhere of an offence involving fraud

or dishonesty.

(4) The Board may in its absolute discretion provisionally accept a qualification produced in

respect of an application for registration under this section, or direct that the application be

renewed within such period as may be specified in the direction.

(5) Any entry directed to be made in the register under section (4) of this section shall show that

registration is provisional, and no entry the Board specified in writing in that behalf.

(6) The Board shall from time to time publish in the Gazette particulars of the qualifications for

the time being accepted for registration under this Act.” (www.esvabon.gov.ng).

Page 16: unitedemsa.files.wordpress.com...Pg 2 NIESV STUDY PACK - PPE KNOWLEDGE AND STRUCTURE OF NIGERIA INSTITUTION OF ESTATE SURVEYORS AND VALUERS Learning Objectives At the end of this lecture,

Pg 16

NIESV STUDY PACK - PPE

“ESTATE SURVEYORS AND VALUERS (REGISTRATION, ETC.) ACT

ARRANGEMENT OF SECTIONS

Estate Surveyors and Valuers Registration Board

SECTION

1. Establishment of the Estate Surveyors and Valuers Registration Board of

Nigeria.

2. Functions of the Board.

3. Membership of the Board.

4. Financial provisions.

5. Directions by the Minister.

The Registrar

6. Appointment of Registrar, preparation and maintenance of the register.

7. Publication of the register and list of corrections.

Registration

8. Registration as estate surveyors and valuers.

9. Approval of institutions, courses of training and qualifications by Board.

10. Supervision of instructions and of examinations leading to approved qualifications.

11. Certificate by the Registrar to be evidence of registration, etc.

12. Registrar to notify Institution of entries in register.

Professional discipline

13. Disciplinary powers of the Board.

14. Penalties for unprofessional conduct, etc.

Miscellaneous and general

15. Temporary registration of persons not citizens of Nigeria.

16. Offences.

17. Miscellaneous supplementary provisions.

18. Regulations, rules and orders.

19. Interpretation.

20. Short title.

Page 17: unitedemsa.files.wordpress.com...Pg 2 NIESV STUDY PACK - PPE KNOWLEDGE AND STRUCTURE OF NIGERIA INSTITUTION OF ESTATE SURVEYORS AND VALUERS Learning Objectives At the end of this lecture,

Pg 17

NIESV STUDY PACK - PPE

SCHEDULE

FIRST SCHEDULE

Supplementary provisions relating to the Board

SECONDSCHEDULE

Supplementary provisions relating to the Disciplinary Tribunal

and Investigating Panel

__________________

ESTATE SURVEYORS AND VALUERS (REGISTRATION, ETC.) ACT

An Act to establish the Estate Surveyors and Valuers Registration Board of Nigeria as a

body corporate and to empower the Board inter alia to determine persons who are to

become estate surveyors and valuers and what standards of knowledge and skill are to be

attained by such persons and to make provisions for other matters connected therewith.

[1975 No.4.]

[29th August, 1975]

[Commencement.]

Estate Surveyors and Valuers Registration Board

1. Establishment of the Estate Surveyors and Valuers Registration Board of Nigeria

(1) There is hereby established a body to be known as the Estate Surveyors and

Valuers Registration Board of Nigeria (in this Act referred to as "the Board").

(2) The Board shall be a body corporate with perpetual succession and a common seal.

2. Functions of the Board

The Board is hereby charged with the general duty of-

(a) determining who are estate surveyors and valuers for the purpose of this Act;

(b) determining what standards of knowledge and skill are to be attained by persons seeking to

become registered as estate surveyors and valuers and reviewing such standards, from time to

time, for the purpose of raising them;

(c) securing, in accordance with the provisions of this Act, the establishment and maintenance

of a register of persons entitled to practice as estate surveyors and valuers and the publication,

from time to time, of the lists of such persons;

Page 18: unitedemsa.files.wordpress.com...Pg 2 NIESV STUDY PACK - PPE KNOWLEDGE AND STRUCTURE OF NIGERIA INSTITUTION OF ESTATE SURVEYORS AND VALUERS Learning Objectives At the end of this lecture,

Pg 18

NIESV STUDY PACK - PPE

(d) regulating and controlling the practice of estate surveying and valuation (in this Act referred

to as "the profession") in all its aspects and ramifications; and

(e) performing the other functions conferred on the Board by this Act.

3. Membership of the Board

1. Subject to the provisions of this Act, the Board shall consist of-

(a) a chairman who shall be a person of distinction in the profession to be appointed by the

Minister on the recommendation of the Nigerian Institute of Estate Surveyors and Valuers (in

this Act referred to as "the Institute");

(b) five persons to be appointed by the Minister of whom at least one but not more than two

shall be employed by the Ministry under his control and the others not so employed shall be

appointed by him from amongst other interests in the field of estate surveying and valuation

covered by this Act, which in the opinion of the Minister ought to be adequately represented;

(c) six persons to be appointed by the Minister to represent the States in rotation for two years at

a time, no two of whom shall come from one State at anyone time;

(d) six persons elected by the Institute in the manner for the time being provided by its

constitution;

(e) four persons nominated by the universities and appointed by the Minister one each from any

four of the universities offering courses leading to an approved qualification.

2. The provisions of the First Schedule to this Act shall have effect with respect to the

qualifications and tenure of office of members of the Board, powers and procedure of the Board

and the other matters therein mentioned.

[First Schedule.]

(3) The Minister may make regulations to provide for increasing or reducing the membership of

the Board, and may make such consequential amendments of paragraph 1 of the First Schedule

to this Act as he deems fit.

4. Financial provisions

(1) The Board shall prepare and submit to the Minister not later than the 31st day of

December in the year in which this Act comes into force (so however that for that year the

Page 19: unitedemsa.files.wordpress.com...Pg 2 NIESV STUDY PACK - PPE KNOWLEDGE AND STRUCTURE OF NIGERIA INSTITUTION OF ESTATE SURVEYORS AND VALUERS Learning Objectives At the end of this lecture,

Pg 19

NIESV STUDY PACK - PPE

Minister may, if he considers it necessary, extend the period) and of each subsequent year, an

estimate of its expenditure and income during the next succeeding financial year.

(2) The Board shall keep proper accounts in respect of each financial year, and proper

records in relation to those accounts to be audited as soon as may be after the end of the financial

year to which the accounts relate by a firm of auditors approved as respects that year by the

Minister.

5. Directions by the Minister

(1) The Minister may give to the Board directions of a general character or relating to particular

matters (but not to any individual person or case) with regard to the exercise by the Board of its

functions, and it shall be the duty of the Board to comply with the directions.

(2) Before giving a direction under subsection (1) of this section, the Minister shall serve a copy

of the proposed direction on the Board and shall afford the Board an opportunity of making

representations to him with respect to the direction; and after considering any representations

made to him in pursuance of this subsection, the Minister may give the direction either without

modification, or with such modifications as appear to him to be appropriate having regard to the

representations.

The Registrar

6. Appointment of Registrar, preparation and maintenance of the register

(1) The Board shall appoint a fit and proper person to be the Registrar for the purposes of this

Act. (2) The Registrar shall prepare and maintain, in accordance with the rules made by the

Board under this section, a register of estate surveyors and valuers (in this Act referred to as "the

register") and the register shall contain the names, addresses, approved qualifications and such

other particulars as may be specified by the Board, of all persons who are entitled and who apply

in accordance with the provisions of this Act to be registered as estate surveyors and valuers.

(3) The register shall consist of two parts of which one shall be in respect of fully registered

persons and the other in respect of provisionally registered persons.

(4) Subject to the following provisions of this section, the Board shall make rules with respect to

the form and keeping of the register and the making of entries therein, and in particular-

Page 20: unitedemsa.files.wordpress.com...Pg 2 NIESV STUDY PACK - PPE KNOWLEDGE AND STRUCTURE OF NIGERIA INSTITUTION OF ESTATE SURVEYORS AND VALUERS Learning Objectives At the end of this lecture,

Pg 20

NIESV STUDY PACK - PPE

(a) Regulating the making of applications for registration and providing for the evidence to be

produced in support of applications;

(b) Providing for the notification to the Registrar, by the person to whom any registered

particulars relate, of any change in those particulars;

(c) Authorising a registered person to have any qualification which is registered in relation to

his name in addition to or, as he may elect, in substitution for, any

other qualifications so registered;

(d) Specifying the fees to be paid to the Board in respect of the entry of names on the register

and authorising the Registrar to refuse to enter a name on the register until any fee specified for

the entry has been paid;

(e) Specifying anything falling to be specified under the foregoing provisions of this section.

(5) Any rules made for the purpose of paragraph (d) of subsection (4) of this section, shall not

come into force until they are confirmed by the Minister.

(6) It shall be the duty of the Registrar to-

(a) Correct, in accordance with the Board's directions, any entry in the register which the

Board directs him to correct as being in the Board's opinion an entry which was incorrectly

made;

(b) Make, from time to time, any necessary alterations to the registered particulars of

registered persons;

(c) Remove from the relevant part of the register the name of any registered person who has

died, `or as the case may be, has ceased to be entitled to be registered.

(7) If the Registrar-

(a) sends by post to any registered person a registered letter addressed to him at his address on

the register enquiring whether the registered particulars relating to him are correct and

receives no reply to the letter within the period of six months from the date of posting it; and

(b) upon the expiration of that period sends in like manner to the person in question a second

similar letter and receives no reply to that letter within three months from the date of posting it,

the Registrar may remove the particulars relating to the person in question from the relevant part

of the register provided that the Board may for any reason which seems to it to be sufficient,

Page 21: unitedemsa.files.wordpress.com...Pg 2 NIESV STUDY PACK - PPE KNOWLEDGE AND STRUCTURE OF NIGERIA INSTITUTION OF ESTATE SURVEYORS AND VALUERS Learning Objectives At the end of this lecture,

Pg 21

NIESV STUDY PACK - PPE

direct the Registrar to restore to the appropriate part of the register any particulars removed

therefrom under this subsection.

7. Publication of the register and list of corrections

(1) It shall be the duty of the Registrar-

(a) to cause a list of persons whose names and qualifications are indicated in the

register to be printed, published and put on sale to members of the public not later than two

years from the beginning of the year in which this Act comes to force;

(b) in each year after that in which a register is first published under paragraph (a) of

this subsection, to cause to be printed, published and put on sale as aforesaid either a corrected

edition of the register since it was last printed or a list of alterations made to the register since it

was last printed; and

(c) to cause a print of each edition of the register and of each list of corrections to be

deposited at the principal office of the Board, and it shall be the duty of the Board to keep the

register and lists so deposited open at all reasonable times for inspection by members of the

public.

(2) A document purporting to be a print of an edition of a register published under this section by

authority of the Registrar in the current year, or documents purporting to be prints of an edition

of a register so published in a previous year and of a list of corrections to that edition so

published in the current year, shall (without prejudice to any other mode of proof) be admissible

in any proceedings as evidence that any person specified in the document, or the documents read

together, as being fully or provisionally registered, is so registered, and that any person not so

specified, is not so registered.

Registration

8. Registration as estate surveyors and valuers

(1) Subject to section 15 and to rules made under section 6 (4) of this Act, a person shall be

entitled to be registered as an estate surveyor and valuer if-

(a) he passes the qualifying examination for membership recognized or

conducted by the Board under this Act and immediately before the commencement of this Act,

he was a member of the Institution; or

Page 22: unitedemsa.files.wordpress.com...Pg 2 NIESV STUDY PACK - PPE KNOWLEDGE AND STRUCTURE OF NIGERIA INSTITUTION OF ESTATE SURVEYORS AND VALUERS Learning Objectives At the end of this lecture,

Pg 22

NIESV STUDY PACK - PPE

(b) he holds a qualification granted outside Nigeria which for the time being

is accepted by the Board and he is by law entitled to practice for all purposes as an estate

surveyor and valuer in the country in which the qualification was granted and, if the Board so

requires, he satisfies the Board that he has had sufficient practical experience as an estate

surveyor and valuer.

(2) Subject as aforesaid, a person shall be entitled to be registered under this Act if-

(a) he holds a certificate recognized by the Board and has had not less than two

years' post qualification practical experience in the profession; or

(b) he has passed an examination approved by the Board and has had not less

than two years' post-qualification practical experience in the profession.

(3) An applicant for registration under this Act shall, in addition to evidence of

qualification, satisfy the Board-

(a) that he is of good character;

(b) that he has attained the age of 21 years; and

(c) that he has not been convicted in Nigeria or elsewhere of an offence involving

fraud or dishonesty.

(4) The Board may in its absolute discretion provisionally accept a qualification

produced in respect of an application for registration under this section, or direct that the

application for registration be renewed within such period as may be specified in the direction.

(5) Any entry directed to be made in the register under subsection (4) of this section

shall show that registration is provisional, and no entry so made shall be converted into full

registration without the consent of the Board specified in writing in that behalf.

(6) The Board shall, from time to time, publish in the Federal Gazette particulars of the

qualifications for the time being accepted for registration under this Act.

9. Approval of institutions, courses of training and qualifications by Board

(1) For the purpose of section 8 of this Act, the Board may approve-

(a) any institution, whether in Nigeria or elsewhere, which the Board considers is

properly organised and equipped for conducting a course of training approved by the Board

under this section;

Page 23: unitedemsa.files.wordpress.com...Pg 2 NIESV STUDY PACK - PPE KNOWLEDGE AND STRUCTURE OF NIGERIA INSTITUTION OF ESTATE SURVEYORS AND VALUERS Learning Objectives At the end of this lecture,

Pg 23

NIESV STUDY PACK - PPE

(b) any course of training which is intended for persons who are seeking to become,

or are already members of the profession and which the Board considers is designed to confer on

persons completing it sufficient knowledge and skill for the practice of the profession; and

(c) any qualification which, as a result of examination taken in conjunction with the

course of training approved by the Board under this section, is granted to

candidates reaching a standard at the examination indicating, in the opinion of the Board,

that they have sufficient knowledge and skill to practise the profession.

(2) The Board shall, from time to time, publish in the Federal Gazette a list of

qualifications approved by it, and subject thereto shall not approve any qualification granted by

an institution in Nigeria unless such qualification has been included in the list published by the

Board.

(3) The Board may, if it thinks fit, withdraw any approval given under this section in

respect of any course, qualification or institution but before withdrawing such an approval it

shall-

(a) give notice that it proposes to do so to persons in Nigeria appearing to the Board

to be persons by whom the course is conducted or the qualification is

granted or the institution is controlled, as the case may be;

(b) afford each such person an opportunity of making to the Board representations

with regard to the proposal; and

(c) take into consideration any representations made as respects the proposals in

pursuance of paragraph (b) of this subsection.

(4) As respects any period during which the approval of the Board under this section for a

course, qualification or institution is withdrawn, the course, qualification or institution shall not

be treated as approved under this section; but the withdrawal of such an approval shall not

prejudice the registration or eligibility for registration of any person who by virtue of the

approval was registered or eligible for registration immediately before the approval was

withdrawn.

(5) The giving or withdrawal of an approval under this section shall have the effect

from such date, either before or after the execution of the instrument signifying the giving or

withdrawal of the approval, as the Board may specify in the instrument; and the Board shall-

Page 24: unitedemsa.files.wordpress.com...Pg 2 NIESV STUDY PACK - PPE KNOWLEDGE AND STRUCTURE OF NIGERIA INSTITUTION OF ESTATE SURVEYORS AND VALUERS Learning Objectives At the end of this lecture,

Pg 24

NIESV STUDY PACK - PPE

(a) as soon as may be, publish a copy of every such instrument in the Federal

Gazette; and

(b) not later than seven days before its publication as aforesaid, send a copy of the

instrument to the Minister.

10. Supervision of instructions and of examinations leading to approved qualifications

(1) It shall be the duty of the Board to keep itself informed of the nature of-

(a) the instructions given at approved institutions to persons attending approved

courses of training; and

(b) the examinations as a result of which approved qualifications are granted,

and for the purpose of performing that duty the Board may appoint, either from among its own

members or otherwise, persons to visit approved institutions or to attend such examinations.

(2) It shall be the duty of a visitor appointed under this section to report to the Board on-

(a) the sufficiency of the instructions given to persons attending approved courses of

training at institutions visited by him;

(b) the sufficiency of the instructions given to persons attended by him; and

(c) any other matter relating to the institutions or examinations on which the Board

may, either generally or in particular case, request him to report,

but, no visitor shall interfere with the giving of any instruction or the holding of any

examination.

(3) On receiving a report made in pursuance of this section, the Board shall, as soon as may be,

send a copy of the report to the person appearing to the Board to be in charge of the institution or

responsible for the examinations to which the report relates, requesting that person to make

observations on the report to the Board within such period as may be specified in the request, not

being less than one month beginning with the date of the request.

11. Certificate by the Registrar to be evidence of registration, etc.

A certificate under the hand of the Registrar to the effect that any person was or was not

registered as an estate surveyor and valuer under this Act at any time or during any period

specified in the certificate, or as to any entry in the register of estate surveyors and valuers or as

Page 25: unitedemsa.files.wordpress.com...Pg 2 NIESV STUDY PACK - PPE KNOWLEDGE AND STRUCTURE OF NIGERIA INSTITUTION OF ESTATE SURVEYORS AND VALUERS Learning Objectives At the end of this lecture,

Pg 25

NIESV STUDY PACK - PPE

to any act or proceeding of the Board, shall, until the contrary is proved, be sufficient evidence of

the matters therein specified.

12. Registrar to notify Institution of entries in register

The Registrar shall, as soon as practicable, after the entry in the register of any person's name, or

after the removal of such name from the register, give notice in writing to the Institution of the

entry or removal, as the case may be, together with all the particulars relevant thereto.

Professional discipline

13. Disciplinary powers of the Board

(1) There shall be a Tribunal to be known as the Estate Surveyors and Valuers

Disciplinary Tribunal (in this Act referred to as "the Tribunal") which shall be charged with the

duty of considering and determining any case referred to it by the panel established by the

following provisions of this section and any other case of which the Tribunal has cognizance

under the following provisions of this Act.

(2) The Tribunal shall consist of the chairman of the Board and eleven other members

thereof appointed by the Board of whom not less than four shall be members of the Board

holding office by virtue of paragraph (d) of subsection (1) of section 3 of this Act or where the

number of those members is for the time being less than four, all those members.

(3) There shall be a body to be known as the Estate Surveyors and Valuers

Investigating Panel (in this Act referred to as "the Panel") which shall be charged with the duty

of-

(a) conducting a preliminary investigation into any case where it is alleged that a

person registered has misbehaved in his capacity as an estate surveyor and valuer, or

should for any other reason be the subject of proceeding before the Tribunal; and

(b) deciding whether the case should be referred to the Tribunal.

(4) The panel shall be appointed by the Board and shall consist of seven members

thereof.

(5) The provisions of the Second Schedule to this Act shall, so far as applicable to the

Tribunal and the Panel respectively, have effect with respect to those bodies.

[Second Schedule.]

Page 26: unitedemsa.files.wordpress.com...Pg 2 NIESV STUDY PACK - PPE KNOWLEDGE AND STRUCTURE OF NIGERIA INSTITUTION OF ESTATE SURVEYORS AND VALUERS Learning Objectives At the end of this lecture,

Pg 26

NIESV STUDY PACK - PPE

14. Penalties for unprofessional conduct, etc.

(1) Where-

(a) a person registered or provisionally registered under this Act is judged by the

Tribunal to be guilty of infamous conduct in any professional respect; or

(b) such a person is convicted of an offence in Nigeria or elsewhere by any court

having power to impose imprisonment (whether or not such an offence is

punishable with imprisonment) which in the opinion of the Tribunal is incompatible with the

status of an estate surveyor and valuer; or

(c) the Tribunal is satisfied that the name of any person has been fraudulently

registered, the Tribunal may, if it thinks fit, give a direction reprimanding that person or

ordering the Registrar to strike his name off the relevant part of the register.

(2) The Tribunal may, if it thinks fit, defer or further defer its decision as to the giving

of a direction under subsection (1) of this section until a subsequent meeting of the Tribunal; but-

(a) no decision shall be deferred under this subsection for periods exceeding two

years in the aggregate; and

(b) no person shall be a member of the Tribunal for the purpose of reaching a decision

which has been deferred or further deferred unless he was present as a member of the Tribunal

when the decision was deferred or further deferred.

(3) For the purposes of subsection (1) of this section, a person shall not be treated as

convicted unless the conviction stands at a time when no appeal or further appeal is pending or

may (without extension of time) be brought in connection with the conviction.

(4) When the Tribunal gives a direction under subsection (1) of this section, it shall

cause notice of the direction to be served on the person to whom it relates.

(5) A person to whom such a direction relates may, at any time within 28 days from the

date of service on him of the notice of the direction, appeal against the direction to the Court of

Appeal; and the Tribunal may appear as respondent to the appeal and, for the purpose of

enabling directions to be given as to the cost of the appeal and of proceedings before the

Tribunal, shall be deemed to be a party thereto whether or not it appears on the hearing of the

appeal.

(6) A direction of the Tribunal under subsection (1) of this section shall take effect-

Page 27: unitedemsa.files.wordpress.com...Pg 2 NIESV STUDY PACK - PPE KNOWLEDGE AND STRUCTURE OF NIGERIA INSTITUTION OF ESTATE SURVEYORS AND VALUERS Learning Objectives At the end of this lecture,

Pg 27

NIESV STUDY PACK - PPE

(a) where no appeal under this section is brought against the direction within the time

limited for the appeal, on the expiration of that time;

(b) where an appeal is brought and is withdrawn or struck out for want of prosecution,

on the withdrawal or striking out of the appeal;

(c) where an appeal is brought and is not withdrawn or struck out as aforesaid, if and

when the appeal is dismissed,

and shall not take effect except in accordance with the foregoing provisions of this subsection.

(7) A person whose name is removed from the register in pursuance of a direction of

the Tribunal under this section, shall not be entitled to be registered again except in pursuance of

a direction in that behalf given by the Tribunal on the application of that person; and a direction

under this section for the removal of a person's name from the register may prohibit an

application under this subsection by that person until the expiration of such period from the date

of the direction (and where he has duly made such an application, from the date of his last

application) as may be specified in the direction.

Miscellaneous and general

15. Temporary registration of persons not citizens of Nigeria

(1) Where a person satisfies the Board-

(a) that he has been selected for employment for a specific period in a capacity in

which a person registered as an estate surveyor and valuer under this Act is qualified to be

employed and that he is or intends to be in Nigeria temporarily for the purpose of serving for

that period in the employment in question; and

(b) that he holds a qualification or has passed examinations necessary for

obtaining some qualification granted outside Nigeria which is for the time being accepted by the

Board as respects the capacity in which, if employed, he is to serve, the Board may, if it thinks

fit, give a direction that he shall be temporarily registered as an estate surveyor and valuer.

(2) The temporary registration of a person shall continue only while he is in such

employment as is mentioned in subsection (1) (a) of this section and shall cease at the end of the

period of the employment specified to the Board under that subsection or on the prior

determination of the employment, whichever first occurs:

Page 28: unitedemsa.files.wordpress.com...Pg 2 NIESV STUDY PACK - PPE KNOWLEDGE AND STRUCTURE OF NIGERIA INSTITUTION OF ESTATE SURVEYORS AND VALUERS Learning Objectives At the end of this lecture,

Pg 28

NIESV STUDY PACK - PPE

Provided that nothing in this subsection shall preclude the Board from giving a further

direction under subsection (1) of this section in respect of a specified period whose

commencement coincides with the termination or prior determination of another such

employment.

(3) A person who is temporarily registered shall, in relation to his employment as

mentioned in subsection (1) (a) of this section, and to things done or omitted to be done in the

course of that employment, be deemed to be fully registered, but in relation to all other matters

he shall be treated as not so registered.

(4) In case of doubt as to whether a person's employment has been terminated, the

decision of the Board shall be conclusive for the purpose of subsection (2) of this section.

(5) The Registrar, as directed from time to time by the Board, shall remove from the

register the name of any person ceasing to be entitled to the benefit of this section.

16. Offences

(1) Any person, not being a registered estate surveyor and valuer, who-

(a) for or in expectation of reward, practises or holds himself out to practise as such;

or

(b) without reasonable excuse takes or uses any name, title, addition or description

implying that he is authorised by law to practise as a registered estate surveyor and

valuer, shall be guilty of an offence under this Act.

(2) If any person, for the purpose of procuring the registration of any name,

qualification or other matter-

(a) makes a statement which he believes to be false in a material particular; or

(b) recklessly makes a statement which is false in a material particular,

he shall be guilty of an offence under this Act.

(3) If the Registrar or any other person employed by the Board wilfully makes any

falsification in any matter relating to the register, he shall be guilty of an offence under this Act.

(4) A person guilty of an offence under this Act shall be liable-

(a) on conviction in a court lower than the High Court, to a fine not

exceeding N100 and, where the offence is a continuing one, to a further fine not exceeding N20

for each and every day that the offence continues;

Page 29: unitedemsa.files.wordpress.com...Pg 2 NIESV STUDY PACK - PPE KNOWLEDGE AND STRUCTURE OF NIGERIA INSTITUTION OF ESTATE SURVEYORS AND VALUERS Learning Objectives At the end of this lecture,

Pg 29

NIESV STUDY PACK - PPE

(b) on conviction in High Court, to a fine not exceeding N 1,000 or

imprisonment for a term not exceeding two years or to both such fine and imprisonment and,

where the offence is a continuing one, to a further fine not exceeding N50 for each and every

day that the offence continues.

(5) Where an offence under this section has been committed by a body corporate is

proved to have been committed with the consent or connivance of, or to be attributable to any

neglect on the part of any director, manager, secretary, or any person purporting to act in any

such capacity he, as well as the body corporate, shall be deemed to be guilty of that offence and

shall be liable to be proceeded against and punished accordingly.

17. Miscellaneous supplementary provisions

(1) Subject to the following provisions of this section, a person not otherwise

exempted shall not hold an appointment requiring status as an estate surveyor and valuer under

this Act in the public service of the Federation or of a State or in the armed forces of the

Federation unless he is an estate surveyor and valuer registered under this Act.

(2) An estate surveyor and valuer shall, but to the extent only of his particular

qualifications, be entitled to practice as a registered estate surveyor and valuer throughout the

Federation.

(3) It shall be the duty of any person in charge of any university or institution in the

Federation offering courses leading to an approved qualification intended for persons who are

seeking to become registered estate surveyors and valuers under this Act, to furnish to the

Registrar, not later than 31st day of March in every year, a list of the names, and of such other

particulars as the Board may specify, of all persons who attended any such courses at the

university or institution in question at any time during the preceding year.

(4) In this section "public service" includes service as a registered estate surveyor

and valuer in or with any educational institution, corporation or State-owned company in the

Federation.

18. Regulations, rules and orders

The Minister shall have power to make all such regulations, as may, in his opinion, be necessary

or expedient for giving full effect to the provisions of this Act and for the administration thereof.

Page 30: unitedemsa.files.wordpress.com...Pg 2 NIESV STUDY PACK - PPE KNOWLEDGE AND STRUCTURE OF NIGERIA INSTITUTION OF ESTATE SURVEYORS AND VALUERS Learning Objectives At the end of this lecture,

Pg 30

NIESV STUDY PACK - PPE

Any power to make regulations, rules and orders under this section shall include power to-

(a) make provision for such incidental or supplemental matters as the person or

authority making the instrument considers expedient for the purpose of the investment; and

(b) make different provision for different circumstances.

19. Interpretation

(1) In this Act, unless the context otherwise requires-

"approved qualification" means such qualification which is approved for the time

being by the Board;

"Board" means the Estate Surveyors and Valuers Registration Board of Nigeria

established under section 1 (1) of this Act;

"estate surveyor and valuer" means any person registered as such under this Act;

"Institute" means the Nigerian Institute of Estate Surveyors and Valuers;

"Minister" means Minister charged with responsibility for matters relating to estate

surveying and valuation;

"profession" means the profession of estate surveying and valuation;

"register" means the register maintained under this Act and "registered" shall be

construed accordingly;

"Registrar" means the Registrar appointed in pursuance of section 6 of this Act.

(2) For the purpose of this Act, a person is registered if his name is for the time being

entered in the register.

(3) Any approval, consent, direction, notice, observation, report, representation or

request authorised or required to be given or made by or under this Act shall be in writing and

may, without prejudice to any other method of service, but subject to the provisions of rules

made under the Second Schedule of this Act, be served by post.

Page 31: unitedemsa.files.wordpress.com...Pg 2 NIESV STUDY PACK - PPE KNOWLEDGE AND STRUCTURE OF NIGERIA INSTITUTION OF ESTATE SURVEYORS AND VALUERS Learning Objectives At the end of this lecture,

Pg 31

NIESV STUDY PACK - PPE

20. Short title

This Act may be cited as the Estate Surveyors and Valuers (Registration, etc.) Act.

___________________

SCHEDULES

FIRST SCHEDULE

Supplementary provisions relating to the Board

Qualifications and tenure of office of members

1. (1) A person shall not be appointed a member of the Board unless he is a citizen of Nigeria

and he is registered as an estate surveyor and valuer under this Act.

(2) Subject to the provisions of this paragraph, a person who is a member of the Board

otherwise than by virtue of paragraphs (b) and (c) of subsection (1) of section 3 of this Act, shall

hold office for a period not exceeding three years beginning with the date of his appointment or

election, as the case maybe:

Provided that a person shall not hold office, whether appointed or elected, for a term of less than

two years unless the Minister, after consultation with the Board, otherwise directs.

(3) Any member of the Board holding office otherwise than as mentioned in sub-paragraph

(2) of this paragraph may, by notice to the Board, resign his office.

(4) Subject to section 3 of this Act, a person who has ceased to be a member of the Board

shall be eligible again to become a member of the Board.

(5) Where a member of the Board ceases to hold office before the date when his term of

office would have expired by effluxion of time, the body or person by whom he was appointed or

elected shall, as soon as may be, appoint or, as the case may be, elect a person to fill the vacancy

for the residue of the term aforesaid, so however that (without prejudice to the powers conferred

by paragraph (d) of subsection (1) of section 3 of this Act) the foregoing provisions of this sub-

paragraph shall not apply where a person holding office as a member of the Board in pursuance

of that paragraph ceases to hold office at a time when the residue of his term does not exceed one

year.

(6) The power of appointing a person as chairman of the Board shall-

(a) during the period of three years beginning with the date when this sub-

paragraph comes into force, be exercisable by the Minister; and

(b) after the expiration of that period, be exercisable by the Board,

Page 32: unitedemsa.files.wordpress.com...Pg 2 NIESV STUDY PACK - PPE KNOWLEDGE AND STRUCTURE OF NIGERIA INSTITUTION OF ESTATE SURVEYORS AND VALUERS Learning Objectives At the end of this lecture,

Pg 32

NIESV STUDY PACK - PPE

and where an existing member of the Board is appointed chairman, his office as an existing

member shall become vacant and his term of office as chairman shall begin on the date of his

appointment as chairman.

Powers of the Board

2. (1) Subject to the following sub-paragraph and to any direction of the Minister under this Act,

the Board shall have power to do anything which in its opinion is calculated to facilitate the

carrying on of its activities.

(2) The Board shall not have power to borrow money or to dispose of any property except

with the prior consent of the Minister and it shall not have power to pay remuneration (including

pensions) allowances, or expenses to any member, officer or servant of the Board or to any other

person, except in accordance with the scales approved by the Minister.

Proceedings of the Board

3. Subject to the provisions of this Act and section 27 of the Interpretation Act (which provides

for decisions of a body to be taken by a majority of the members of the body and for the

chairman to have a second or casting vote) the Board may make standing orders regulating the

proceedings of the Board or of any committee thereof.

[Cap. I23.]

4. The quorum of the Board shall be eleven and the quorum of any committee of the Board shall

be determined by the Board.

5. (1) The Board shall appoint one of its members to be the vice-chairman of the Board for such

periods as the Board may determine, so however that a vice-chairman who ceases to be a

member shall cease to be a vice-chairman.

(2) At any time while the office of the chairman is either vacant or the chairman is, in the opinion

of the Board permanently or temporarily unable to perform the functions of his office, the vice-

chairman shall perform those functions, and references in this Schedule to the chairman shall be

construed accordingly.

6. (1) Subject to the provisions of any standing orders of the Board, the Board shall meet

whenever it is summoned by the chairman; and if the chairman is required so to do by notice

given to him by not less than six other members he shall summon a meeting of the Board to be

held within seven days from the date on which the notice is given.

Page 33: unitedemsa.files.wordpress.com...Pg 2 NIESV STUDY PACK - PPE KNOWLEDGE AND STRUCTURE OF NIGERIA INSTITUTION OF ESTATE SURVEYORS AND VALUERS Learning Objectives At the end of this lecture,

Pg 33

NIESV STUDY PACK - PPE

(2) At any meeting of the Board, the chairman, or in his absence, the vice-chairman shall preside,

but if both are absent, the members present at the meeting shall appoint one of their number to

preside at that meeting.

(3) Where the Board desires to obtain the advice of any person on a particular matter, the Board

may co-opt him as a member for such period as it thinks fit; but a person who is a member by

virtue of this sub-paragraph shall not be entitled to vote at any meeting of the Board and shall not

count towards a quorum.

(4) Notwithstanding anything in the foregoing provisions of this paragraph, the first meeting of

the Board shall be summoned by the Minister who may give such directions as he thinks fit as to

the member who shall preside and as to the procedure which shall be followed at the meeting.

Committees

7. (1) The Board may appoint one or more committees to carry out, on behalf of the Board, such

of its functions as the Board may determine.

(2) A committee appointed under this paragraph shall consist of the number of persons

determined by the Board, and not more than one third of those persons may be persons who are

not members of the Board; and a person other than a member of the Board shall hold office on

the committee in accordance with the terms of the instrument by which he is appointed.

(3) A decision of a committee of the Board shall be of no effect until it is confirmed by the

Board.

Miscellaneous

8. (1) The fixing of the seal of the Board shall be authenticated by the signature of the chairman

or of some other member authorized generally or specially to act for that purpose by the Board.

(2) Any contract or instrument which, if made or executed by a person not being a body

corporate, would not be required to be under seal, may be made or executed on behalf of the

Board by any person generally or specially authorized to act for that purpose by the Board.

(3) Any document purporting to be a document duly executed under the seal of the Board shall

be received in evidence and shall, unless the contrary is proved, be deemed to be so executed.

9. The validity of any proceedings of the Board or of a committee thereof shall not be affected by

any vacancy in the membership of the Board or committee, or by any defect in the appointment

of a member of the Board or of a person to serve on the committee, or by reason that a person not

entitled to do so took part in the proceedings.

Page 34: unitedemsa.files.wordpress.com...Pg 2 NIESV STUDY PACK - PPE KNOWLEDGE AND STRUCTURE OF NIGERIA INSTITUTION OF ESTATE SURVEYORS AND VALUERS Learning Objectives At the end of this lecture,

Pg 34

NIESV STUDY PACK - PPE

10. Any member of the Board, and any person holding office on a committee of the Board, who

has a personal interest in any contract or arrangement entered into or proposed to be considered

by the Board or a committee thereof, shall forthwith disclose his interest to the Board and shall

not vote on any question relating to the contract or arrangement.

11. A person shall not, by reason only of his membership of the Board, be treated as holding an

office in the public service of the Federation or of any State thereof.

________________________

SECOND SCHEDULE

Supplementary provisions relating to the Disciplinary Tribunal

and Investigating Panel

The Tribunal

1. The quorum of the Tribunal shall be four.

2. (1) The Chief Justice of Nigeria shall make rules as to the procedure to be followed and

the rules of evidence to be observed in proceedings before the Tribunal.

(2) The rules shall in particular provide-

(a) for securing that notice of the proceedings shall be given, at such time and in

such manner as may be specified by the rules, to the person who is the subject of the

proceedings;

(b) for determining who, in addition to the person aforesaid, shall be a party to the

proceedings;

(c) for securing that any party to the proceedings shall, if he so requires, be entitled

to be heard by the Tribunal;

(d) for enabling any party to the proceedings to be represented by a legal practitioner;

(e) subject to the provisions of section 14 (5) of this Act, as to the costs of

proceedings before the Tribunal;

(f) for requiring, in a case where it is alleged that a person who is the subject of the

proceeding is guilty of infamous conduct in any professional respect, that where the Tribunal

adjudges that the allegation has not been proved, it shall record a finding that the person is not

guilty of such conduct in respect of the matters to which the allegation relates; and

Page 35: unitedemsa.files.wordpress.com...Pg 2 NIESV STUDY PACK - PPE KNOWLEDGE AND STRUCTURE OF NIGERIA INSTITUTION OF ESTATE SURVEYORS AND VALUERS Learning Objectives At the end of this lecture,

Pg 35

NIESV STUDY PACK - PPE

(g) for publishing in the Federal Gazette notice of any direction of the Tribunal which

has taken effect providing that a person's name shall be struck off a register.

3. For the purposes of any proceedings before the Tribunal, any member of the Tribunal may

administer oaths and any party to the proceedings may issue out of the registry of the High

Court, as the case may require, writs of subpoena ad testificandum and ducestecume; but no

person appearing before the Tribunal shall be compelled to-

(a) make any statement before the Tribunal tending to incriminate himself; or

(b) produce any document under such a writ which he could not be compelled to

produce at the trial of an action.

4. (1) For the purpose of advising the Tribunal on questions of law arising in proceedings before

it, there shall in all such proceedings be an assessor to the Tribunal who shall be appointed by the

Board on the nomination of the Chief Justice of Nigeria and shall be a legal practitioner of not

less than seven years' standing.

(2) The Chief Justice of Nigeria shall make rules as to the functions of assessors appointed

under this paragraph, and in particular such rules shall contain provisions for securing that-

(a) where an assessor advises the Tribunal on any question of law as to evidence,

procedure or any other matter specified by the rules, he shall do so in the

presence of every party or person representing a party to the proceedings who appears

thereat or, if the advice is tendered while the Tribunal is deliberating in private, that every such

party or person as aforesaid shall be informed as to what advice the assessor has tendered; and

(b) every such party or person as aforesaid shall be informed if the Tribunal does not

in any case accept the advice of the assessor on such a question as

aforesaid.

(3) An assessor may be appointed under this paragraph either generally or for any particular

proceedings or class of proceedings, and shall hold and vacate office in accordance with the

terms of the instrument by which he is appointed.

The Panel

5. The quorum of the Panel shall be three.

6. (1) The Panel may, at any meeting attended by not less than six members thereof, make

standing orders with respect to the Panel.

Page 36: unitedemsa.files.wordpress.com...Pg 2 NIESV STUDY PACK - PPE KNOWLEDGE AND STRUCTURE OF NIGERIA INSTITUTION OF ESTATE SURVEYORS AND VALUERS Learning Objectives At the end of this lecture,

Pg 36

NIESV STUDY PACK - PPE

(2) Subject to the provisions of any such standing orders, the Panel may regulate its own

procedure.

Miscellaneous

7. (1) A person ceasing to be appointed a member of the Tribunal or the Panel shall be eligible

for re-appointment as a member of that body.

(2) A person may, if otherwise eligible, be a member of both the Tribunal and the Panel; but no

person who acted as member of the Panel with respect to any case shall act as a member of the

Tribunal with respect to that case.

8. The Tribunal or the Panel may act notwithstanding any vacancy in its membership; and the

proceedings of either body shall not be invalidated by any irregularity in the appointment of a

member of that body, or (subject to sub-paragraph (2) of paragraph 7 of this Schedule) by reason

of the fact that any person who was not entitled to do so took part in the proceedings of that

body.

9. The Tribunal and the Panel may each sit in two or more divisions.

10. Any document authorised or required by virtue of this Act to be served on the Tribunal or the

Panel shall be served on the Registrar.

11. Any expenses of the Tribunal or the Panel shall be defrayed by the Board.

12. A person shall not, by reason only of his appointment as an assessor to the Tribunal or as a

member of the Panel, be treated as holding an office in the public service of the Federation or of

any State thereof.

______________________________

ESTATE SURVEYORS AND VALUERS REGISTRATION ACT

SUBSIDIARY LEGISLATION

_______________________________________

List of Subsidiary Legislation

1. Estate Surveyors and Valuers (Disciplinary Tribunal and Assessors) Rules.

_____________________________________

ESTATE SURVEYORS AND VALUERS (DISCIPLINARY TRIBUNAL

AND ASSESSORS) RULES

[S.I. 15 of 1980.]

under paragraphs 2 and 4 of the Second Schedule

Page 37: unitedemsa.files.wordpress.com...Pg 2 NIESV STUDY PACK - PPE KNOWLEDGE AND STRUCTURE OF NIGERIA INSTITUTION OF ESTATE SURVEYORS AND VALUERS Learning Objectives At the end of this lecture,

Pg 37

NIESV STUDY PACK - PPE

[27th August, 1980]

[Commencement.]

Proceedings before the Tribunal

1. Reference of case to Tribunal

In any case where in pursuance of section 13 (3) of the Act the Panel is of the opinion that a

prima facie case is made against an estate surveyor and valuer, the Panel shall prepare a report of

the case and formulate any appropriate charge or charges and forward them to the secretary

together with all the documents considered by the Panel.

2. Parties and appearance

(1) The parties to proceedings before the Tribunal shall be-

(a) the complainant;

(b) the respondent;

(c) any other person required by the Tribunal to be joined or joined by leave of the

Tribunal.

(2) Any party to the proceedings may appear in person or be represented by a legal

practitioner, so however that the Tribunal may order a party to the proceedings to appear in

person if of opinion that it is necessary in the interest of justice but without prejudice to his right

to counsel.

3. Notice of hearing and service

(1) On the direction of the chairman, the secretary shall fix a day for the hearing of the

case and forthwith thereafter shall, in the form set out in the Schedule to these Rules, serve

notice thereof on each party to the proceedings.

[Schedule.]

(2) The secretary shall serve on each party, other than the complainant, copies of the

report and all the charges prepared by the Panel and all documents considered by the Panel.

(3) It shall be sufficient compliance with this rule if any process required to be served is handed

to the party concerned or affected personally or is sent by registered post to the last known place

of residence or abode of the party.

4. Hearing in absence of parties

(1) Subject to the next succeeding paragraph, the Tribunal may hear and determine a

case in the absence of any party.

Page 38: unitedemsa.files.wordpress.com...Pg 2 NIESV STUDY PACK - PPE KNOWLEDGE AND STRUCTURE OF NIGERIA INSTITUTION OF ESTATE SURVEYORS AND VALUERS Learning Objectives At the end of this lecture,

Pg 38

NIESV STUDY PACK - PPE

(2) Any party to the proceedings before a Tribunal who fails to appear or be

represented, or who has previously appeared before the Tribunal but subsequently fails to appear

or be represented, may apply within thirty days after the date when the pronouncement of the

findings and directions of the Tribunal were given, for a re-hearing on the grounds of want of

notice or other good and sufficient reason; and the Tribunal may, in appropriate cases, grant the

application upon such terms as to costs or otherwise as it thinks fit.

5. Witnesses and evidence

The Tribunal may in the course of its proceedings hear such witnesses and receive such

documentary evidence as in its opinion may assist in arriving at a conclusion as to the truth or

otherwise of the charge or charges referred to it by the Panel; and in the application of this rule,

the provisions of the Evidence Act or Law in force in the State where the Tribunal holds its

sittings shall apply to any such proceedings.

[Cap. E14.]

6. Amendment of charges

If in the course of the proceedings it appears to the Tribunal that the charge or charges referred to

it by the Panel require to be amended in any respect, the Tribunal may permit such amendment

as it thinks fit.

7. Proceedings to be in public

The proceedings of the Tribunal shall be held, and its findings and directions shall be

pronounced, in public.

8. Adjournment of hearing

The Tribunal may, of its own motion, or upon application of any party, adjourn the hearing on

such terms as to costs or otherwise as the Tribunal may think fit.

9. False evidence

If any person wilfully gives false evidence on oath before the Tribunal during the course of any

proceedings, or wilfully makes a false statement in any affidavit sworn for the purpose of any

such proceedings, the Tribunal may refer the matter to the Attorney-General of the Federation

for any action he may deem fit.

10. Findings and costs

Page 39: unitedemsa.files.wordpress.com...Pg 2 NIESV STUDY PACK - PPE KNOWLEDGE AND STRUCTURE OF NIGERIA INSTITUTION OF ESTATE SURVEYORS AND VALUERS Learning Objectives At the end of this lecture,

Pg 39

NIESV STUDY PACK - PPE

If, after the hearing, the Tribunal adjudges that the charge of professional misconduct has not

been proved, the Tribunal-

(a) shall record a finding that the respondent is not guilty of such conduct in respect of

the matters to which the charge relates;

(b) may, nevertheless, order any party (except the complainant) to pay the costs of the

proceedings if, having regard to his conduct and to all the circumstances of the

case, the Tribunal thinks fit so to order.

11. Publication of Tribunal's findings

Subject to section 14 (4) of the Act (which relates to appeals), any direction given by the

Tribunal shall be published in the Federal Gazette as soon as may be after the direction takes

effects.

12. Record of proceedings

(1) Shorthand notes of the proceedings may be taken by a person appointed by the

Tribunal and any party who appeared at the Tribunal and any party who appeared at the

proceedings shall be entitled to inspect the transcript thereof.

(2) The secretary shall supply to any person entitled to be heard upon an appeal against

the direction of the Tribunal, and to the Estate Surveyors and Valuers Registration Board of

Nigeria, but to no other person, a copy of the transcript of such notes on payment of such charges

as may be determined by the secretary.

(3) If no shorthand notes are taken, the chairman shall take a note of the proceedings

and, accordingly, the provisions of these Rules as to inspection and supplying of copies shall

apply to such notes.

13. Dispensing with certain provisions

The Tribunal may dispense with any requirement of these Rules respecting notices, affidavits,

documents, service or time in any case where it appears to the Tribunal to be just to do so; and

the Tribunal may in any appropriate case extend the time to do anything under these Rules.

14. Power to retain exhibits pending appeal

Page 40: unitedemsa.files.wordpress.com...Pg 2 NIESV STUDY PACK - PPE KNOWLEDGE AND STRUCTURE OF NIGERIA INSTITUTION OF ESTATE SURVEYORS AND VALUERS Learning Objectives At the end of this lecture,

Pg 40

NIESV STUDY PACK - PPE

The Tribunal may order that any documents or other exhibits produced or used at the hearing

shall be retained by the secretary until the time within which an appeal may Bd3Ebne entered has

expired, and if notice of appeal is given, until the appeal is heard or otherwise disposed of.

15. Appointment and duties of assessor

(1) An assessor, when nominated in accordance with paragraph 4 (1) of the Second

Schedule to the Act, shall be appointed by the Board by instrument, and the assessor shall hold

and vacate office as provided in the instrument; where the appointment is not a general one, it

shall have effect only in respect of a particular sitting of the Tribunal.

[Second Schedule. Cap. E13.]

(2) Subject to the terms of his appointment, an assessor shall attend any meeting of the

Tribunal as and when requested to do so by notice in writing given to him by the secretary not

later than three clear days before the date appointed for the meeting; and he shall there advise the

Tribunal on questions of law arising in proceedings before it.

(3) Except where the Tribunal is deliberating in private, the advice of the assessor on

questions of law as to evidence, procedure and as to compliance with the Act shall be tendered in

the presence of every party or of his counsel.

(4) If the advice by the assessor to the Tribunal is given otherwise than in the presence

of all parties, or as the case may be of their counsel, the assessor shall, as soon as may be

thereafter, inform all the parties as to the nature of the advice and the reaction thereto of the

Tribunal.

(5) Fees to be paid to the assessor shall be such as may be determined by the Board

from time to time with the consent of the assessor.

16. Interpretation

In these rules, unless the context otherwise requires-

"chairman" means the chairman of the Estate Surveyors and Valuers Disciplinary

Tribunal;

"complainant" means the Estate Surveyors and Valuers Investigating Panel or any

member thereof;

Page 41: unitedemsa.files.wordpress.com...Pg 2 NIESV STUDY PACK - PPE KNOWLEDGE AND STRUCTURE OF NIGERIA INSTITUTION OF ESTATE SURVEYORS AND VALUERS Learning Objectives At the end of this lecture,

Pg 41

NIESV STUDY PACK - PPE

"respondent" means the person required to answer any charge of professional

misconduct;

"secretary" means a person appointed to act as the Registrar under section 6 (1) of the

Act.

17. Short title

These Rules may be cited as the Estate Surveyors and Valuers (Disciplinary Tribunal and

Assessors) Rules.

________________________

SCHEDULE

Form (CCB. 1)

Notice of hearing by the Estate Surveyors and Valuers Disciplinary Tribunal

In the matter of the Estate Surveyors and Valuers (Registration, etc.) Act (Cap. E13)

and

In the matter of A.B., a registered estate surveyor and valuer

TAKE NOTICE that the report and charges prepared by the Estate Surveyors and Valuers

Investigating Panel in the above matter are fixed for hearing by the Estate Surveyors and Valuers

Disciplinary Tribunal

at………………………………………………………………………………….………………..

on the………………………………………………….. day

of…………………………………………………………….. 20…………………… Copies of:

(a) the report;

(b) the charges; and

(c) …………………………………

are annexed hereto.

DATED the………………………………………. day

of………………………………………………………………… 20……………………..

……………………………………………………………

Secretary to the Tribunal.” (www.placng.org).

Page 42: unitedemsa.files.wordpress.com...Pg 2 NIESV STUDY PACK - PPE KNOWLEDGE AND STRUCTURE OF NIGERIA INSTITUTION OF ESTATE SURVEYORS AND VALUERS Learning Objectives At the end of this lecture,

Pg 42

NIESV STUDY PACK - PPE

“HOW TO BECOME A MEMBER

(1) Subject to section 15 and to rules made under section 6 (4) of this Act, a person shall be

entitled to be registered as an estate surveyor and valuer if:

(a) he passes the qualifying examination for membership recognized or conducted by The Board

under this Act and immediately before the commencement of this Act, he was a member of The

Institution; or

(b) he holds a qualification granted outside Nigeria which for the time being is accepted by The

Board and he is by law entitled to practice for all purpose as estate surveyor and valuer in the

country in which the qualification was granted and, if The Board so, requires, he satisfies The

Board that he has had sufficient practical experience as estate surveyor and valuer.

(2) Subject as aforesaid, a person shall be entitled to be registered under this Act if

(a) he holds a certificate recognized by the Board and has had not less than two years post

qualification practical experience in the profession; or

(b) he has passed an examination approved by the Board and has had not less than two years post

qualification practical experience in the profession.

(3) An applicant for registration under this Act shall, in addition to evidence of qualification,

satisfy The Board

(a) that he is of good character

(b) that he has attained the age of 21 years; and

(c) that he has not been convicted in Nigeria or elsewhere of an offence involving fraud or

dishonesty.

(4) The Board may in its absolute discretion provisionally accept a qualification produced in

respect of an application for registration under this section, or direct that the application be

renewed within such period as may be specified in the direction.

(5) Any entry directed to be made in the register under section (4) of this section shall show that

registration is provisional, and no entry the Board specified in writing in that behalf.

Page 43: unitedemsa.files.wordpress.com...Pg 2 NIESV STUDY PACK - PPE KNOWLEDGE AND STRUCTURE OF NIGERIA INSTITUTION OF ESTATE SURVEYORS AND VALUERS Learning Objectives At the end of this lecture,

Pg 43

NIESV STUDY PACK - PPE

(6) The Board shall from time to time publish in the Gazette particulars of the qualifications for

the time being accepted for registration under this Act.”(www.esvabon.gov.ng).

Page 44: unitedemsa.files.wordpress.com...Pg 2 NIESV STUDY PACK - PPE KNOWLEDGE AND STRUCTURE OF NIGERIA INSTITUTION OF ESTATE SURVEYORS AND VALUERS Learning Objectives At the end of this lecture,

Pg 44

NIESV STUDY PACK - PPE

ASSOCIATION OF PROFESSIONAL BODIES OF NIGERIA

Learning Objectives

At the end of this lecture, students are expected to have acquired basic understanding of the

structure as well as operations of Association of Professional Bodies of Nigeria and should be

able to:

a. Demonstrate in clear terms good knowledge of the body.

b. Understand the broad idea of the structure of the Association and their inter relationship.

c. Establish an understanding of the basic requirements for membership of the association.

d. Understand the conditions requisite for professional bodies to be admitted into the

membership of the association.

e. Acquire a basic understanding of the general membership conditions.

INTRODUCTION

APBN is the umbrella of institutes, institutions and societies which are recognized and chartered

to regulate the various professions by the law of Nigeria. It is the unified body of professionals

formed to render advice to government on national policy issues.

APBN is set up to speak with one voice on behalf of its member-bodies while at the same time

giving professional and proper technical advice to Government on matters affecting the

professionals and their practices. It is the apex organization comprising, for now, twenty-seven

(27) professional bodies, whose main objective is to provide Nigeria with an effective forum for

transmitting to Government the aggregate views of the nations Professionals on matters of public

interest which they are competent to express informed and enlightened views.

APBN was given official Federal Government recognition as the third leg of the tripod of the

Organized Private Sector (OPS) in January, 1992 to employ multidisciplinary approach to

problem solving.

Page 45: unitedemsa.files.wordpress.com...Pg 2 NIESV STUDY PACK - PPE KNOWLEDGE AND STRUCTURE OF NIGERIA INSTITUTION OF ESTATE SURVEYORS AND VALUERS Learning Objectives At the end of this lecture,

Pg 45

NIESV STUDY PACK - PPE

MEMBERSHIP CONDITIONS

The following conditions were approved by the Board at the Extra-Ordinary General Assembly

meeting of the Association held on November 5, 1993 regarding membership admission which

has since been in force.

GENERAL MEMBERSHIP ADMISSION CONDITIONS

1. A professional body recognized by an Act of Parliament or Decree and registered under

the companies and Allied Matters Decree No 1 of 1990 is eligible.

2. Application for membership of the Association shall be made to the Secretary General in

writing and in such form containing such information as the Board may from time to time

prescribed.

3. Application for admission to membership of the Association shall be considered by the

Board of the Association and the Board shall have full discretion as regards the admission

of any professional body to the membership of the Association.

4. A professional body found qualified and approved by the Board is a member as from the

date of approval, by the Board.

5. Compliance to requirement in section 8 and any other prescribed conditions.

6. A Member-body of the Association shall cease to be a member if such member-body

ceases to be a professional body or Association in the opinion of the Board, provided that

the decision to invalidate the membership of a Member-body shall have to be ratified by

simple majority of the General Assembly before such invalidation takes effect.

SPECIFIC

7. Fully completed Application Form (see Forms B&C attached)

8. Application must be duly sponsored and endorsed by two (2) Member-Bodies of

APBN for consideration

Page 46: unitedemsa.files.wordpress.com...Pg 2 NIESV STUDY PACK - PPE KNOWLEDGE AND STRUCTURE OF NIGERIA INSTITUTION OF ESTATE SURVEYORS AND VALUERS Learning Objectives At the end of this lecture,

Pg 46

NIESV STUDY PACK - PPE

MEMBERS

1. “Chartered Institute Of Bankers Of Nigeria, 19, Adeola Hopewell Street, Victoria Island,

Lagos.

2. Association of national accountants of Nigeria, 250, Herbert Macaulay way, Yaba, Lagos

3. Association of medical laboratory scientists of nig., 12, Messanya Street, Wuse Zone 6,

FCT, Abuja.

4. Chartered institute of administration, 78, old Ojo road, Amuwo-Odofin, Lagos

5. Chartered insurance institute of Nigeria, 27 Lagos Street, Ebute Metta, Lagos

6. Chartered institute of personnel mgt. Of Nigeria, CIPM avenue, Alausa-Ikeja, Lagos

7. Chartered institute of purchasing & supply mgt. Of Nigeria, 27, Shipeolu street, Onipanu,

Lagos

8. Institute of chartered accountants of Nigeria, Pc16, Idowu Taylor street, Victoria island,

Lagos

9. Institute of chartered secretaries & administrators of Nigeria, Elephant cement way,

alausa-ikeja, Lagos

10. institute of public analysts of Nigeria, 443, Herbert Macaulay way, Yaba, Lagos

11. Chartered Institute Of Taxation Of Nigeria, Plot 16 Otunba Jobi Fele way, central

business district, Alausa-Ikeja, Lagos.

12. Nigerian Institute Of Management (Chartered), 22, Idowu Taylor street, Victoria island,

Lagos,

13. Institute Of Chartered Chemists Of Nigeria, 443, Herbert Macaulay way, Yaba, Lagos

14. Institute of Management Consultant Of Nigeria, 4, Thomas Ajufo street, Ikeja, Lagos

15. Nigerian institute of architects, 215, Katampe extension, Garki, Abuja

Page 47: unitedemsa.files.wordpress.com...Pg 2 NIESV STUDY PACK - PPE KNOWLEDGE AND STRUCTURE OF NIGERIA INSTITUTION OF ESTATE SURVEYORS AND VALUERS Learning Objectives At the end of this lecture,

Pg 47

NIESV STUDY PACK - PPE

16. Nigeria computer society, Plot 10, Otunba Jobi-Fele way, Alausa-Ikeja, Lagos

17. Nigerian society of engineers, National engineering centre, (off National Mosque-Labour

House Road) CBD, Abuja

18. Nigerian institution of surveyors, 251 Herbert Macaulay Way, CBD, Abuja, Nigerian

institute of quantity surveyors, 20th, 4th avenue, Gwarimpa, Abuja.

19. Nigerian Institute of Town Planners, NITP-Bawa Bwari house, Plot 2047, Michael

Okpara way, Wuse Zone 5, Abuja

20. Nigerian institution of Estate Surveyors & Valuers, Plot 759, independence avenue

(Bassan plaza) Cbd, Abuja

21. Nigerian mining and GeoSciences Society, Anguldi Zawan, p.o. Box 6192, Anglo Jos,

plateau state

22. Nigerian Medical Association, 8, Benghazi street, Wuse zone 4, Abuja

23. Nigerian Institute of Building, 7, Misau Crescent, off Kumo street, off Ahmadu Bello-

way, Garki, Abuja

24. Nigerian Bar Association, 24, Oro-ago Street, off Muhammed Buhari Way, Garki, Abuja

25. Nigerian Institute of Public Relations plot 1275, Aderemi Adesoji Street, Zone D

Extension, Gudu District, Apo, Abuja.

26. Pharmaceutical society of Nigeria, 32, Faramobi Ajike Street, Anthony Village, Lagos

27. Chartered Institute Of Stockbrokers, Bookshop house (10th floor) Broad Street, Lagos.”

Page 48: unitedemsa.files.wordpress.com...Pg 2 NIESV STUDY PACK - PPE KNOWLEDGE AND STRUCTURE OF NIGERIA INSTITUTION OF ESTATE SURVEYORS AND VALUERS Learning Objectives At the end of this lecture,

Pg 48

NIESV STUDY PACK - PPE

THE INTERNATIONAL REAL ESTATE FEDERATION (FIABCI)

Learning Objectives

At the end of this lecture, students are expected to have acquired basic understanding of the

structure as well as operations of the International Real Estate Federation and should be able to:

a. Demonstrate in clear terms good knowledge of the Federation.

b. Understand the broad idea of the structure of the Association and their inter relationship.

c. Establish an understanding of the basic requirements for membership of the Federation.

d. Understand the various FIABCI designations and to whom they are applied.

e. Acquire a basic understanding of the general examinations that qualities an individual to be

a member.

f. Demonstrate the knowledge of the exchange programmes of the association and who could

benefit from it.

INTRODUCTION

FIABCI, the International Real Estate Federation is a multi-discipline, networking organization

for all professionals associated with real estate transactions. She provides access and opportunity

for real estate professionals interested in gaining knowledge, sharing information and conducting

international business. With members in more than 50 countries and World, Regional and

National Congresses throughout the year, one can travel and meet like-minded professionals of

the highest caliber anywhere in the world. FIABCI also enjoys Special Consultative Status with

the Economic and Social Council of the United Nations.

FIABCI, the International Real Estate Federation, is a business network of real estate

professionals worldwide.

FIABCI MEMBERS

FIABCI members are located in all regions of the world and represent all sectors of the real

estate industry.

FIABCI DESIGNATION

The FIREC (FIABCI International Real Estate Consultant) designation is awarded to FIABCI

members who have satisfied specific criteria based upon education, business experience and

participation in FIABCI Global activities.

Page 49: unitedemsa.files.wordpress.com...Pg 2 NIESV STUDY PACK - PPE KNOWLEDGE AND STRUCTURE OF NIGERIA INSTITUTION OF ESTATE SURVEYORS AND VALUERS Learning Objectives At the end of this lecture,

Pg 49

NIESV STUDY PACK - PPE

Earning the prestigious FIREC designation means becoming part of an elite class of international

real estate industry leaders who transact business on a global basis.

The FIREC designation enables you to:

o Differentiate yourself from your colleagues

o Gain a competitive advantage

o Advance your expertise and industry knowledge

o Increase your earning potential

o Enhance your skills

o Effectively build your global business

To earn the FIREC designation, candidates must complete the following requirements:

o Complete Coursework

o Demonstrate qualifying experience

o Attend FIABCI International Events

The FIABCI Diploma

The FIABCI Diploma is awarded to students graduating from a Master in Real Estate program

that fulfills certain requirements. Academic members of FIABCI may apply to be authorised to

grant the FIABCI Diploma to their students.

FIABCI Diploma Requirements For a University that is an Academic member of FIABCI to

have the honor of awarding the FIABCI Diploma to students graduating from a Master in Real

Estate Program, the following requirements must be met: 1. Provide a summary/overview of the

University and the Real Estate Program 0under consideration; 2. Provide the Admissions

Guidelines for the University Program; 3. The FIABCI course modules must be covered

adequately in the University program to have a University approved. (Program completion

requirements will vary from University/Country to University/Country, however, in general the

Diploma requires successful completion of a minimum of ten approved courses); These modules

will include, at a minimum, a focus on topics such as the following: Taking advantage of

International Real Estate Opportunities; Working with Overseas Cultures, Ethics, Transparency

Page 50: unitedemsa.files.wordpress.com...Pg 2 NIESV STUDY PACK - PPE KNOWLEDGE AND STRUCTURE OF NIGERIA INSTITUTION OF ESTATE SURVEYORS AND VALUERS Learning Objectives At the end of this lecture,

Pg 50

NIESV STUDY PACK - PPE

and Practices; Fundamentals of International Real Estate Markets; International Real Estate

Brokerage; Building an International Real Estate Network and Marketing your service overseas;

Investing and Developing in Overseas Real Estate Markets; Property and Asset Management in

an Overseas Market; and Appraisal, Financing, Legal and Accounting in Overseas Real Estate

Markets. 4. Provide a summary of any changes in the syllabus for the FIABCI Member

University's Masters in Real Estate program. The Application Materials, including the Program

Curriculum, must be submitted to the FIABCI Education and Academic Member Committee

through the Office of the Secretariat in Paris, France (Patricia Delaney at

[email protected]) for approval to award the FIABCI Diploma. The University agrees

that any approval to award the FIABCI Diploma is subject to specific terms and conditions and it

can be revoked, at any time, at the discretion of FIABCI. The Diploma will be sent electronically

to the University for preparation to award to each individual student. Names and contact

information for the FIABCI Diploma recipients must be submitted to the FIABCI Secretariat.

With members in 65 countries, including 100 Professional Associations, 65 Academic

Institutions and 3000 individual members from all professions of the real estate sector, FIABCI

is the most representative organization of the real estate industry in the world. FIABCI holds

Special Consultative Status with the Economic and Social Council of the United Nations.

FIABCI University Members are encouraged to promote student membership in FIABCI, which

includes access to the Exchange Program, the Scholarship Foundation, United Nations activities

and high-level contacts in real estate throughout the world.

PROFESSIONAL EXCHANGES

FIABCI offers its members opportunities to participate in educational exchanges and

professional internships abroad

Through its Exchanges Committee, FIABCI helps young professionals and students in real estate

to organise internships abroad.

The program allows participants to develop their skills, understand differences in real estate

practices in another country and to start building their international career.

Professional exchanges allow FIABCI Young Members to gain added expertise while becoming

Page 51: unitedemsa.files.wordpress.com...Pg 2 NIESV STUDY PACK - PPE KNOWLEDGE AND STRUCTURE OF NIGERIA INSTITUTION OF ESTATE SURVEYORS AND VALUERS Learning Objectives At the end of this lecture,

Pg 51

NIESV STUDY PACK - PPE

familiar with best practice in the chosen country and activity sector. Educational exchanges

allow students to study their chosen real estate discipline abroad at a recognised academic

institution.

In addition to the professional, education and cultural benefits for participants and hosts, the

Exchanges programme provides excellent opportunities for networking and relationship-

building.

WHO CAN JOIN

Professional associations

Individuals and Companies

Public sector institutions

Academic institutions

Young members

Students

Page 52: unitedemsa.files.wordpress.com...Pg 2 NIESV STUDY PACK - PPE KNOWLEDGE AND STRUCTURE OF NIGERIA INSTITUTION OF ESTATE SURVEYORS AND VALUERS Learning Objectives At the end of this lecture,

Pg 52

NIESV STUDY PACK - PPE

COMMONWEALTH ASSOCIATION OF SURVEYING AND LAND ECONOMY

(CASLE)

Learning Objectives

At the end of this lecture, students are expected to have acquired basic understanding of the

structure as well as operations of the Commonwealth Association of Surveying and Land

Economy and should be able to:

a. Demonstrate in clear mission and aims of association.

b. Understand the broad idea of the structure of the Association and their inter relationship.

c. Establish an understanding of the basic requirements for membership of the association.

d. Understand the various FIABCI designations and to whom they are applied.

e. Acquire a basic understanding of the general examinations that qualities an individual to

be a member.

f. Demonstrate the knowledge of the exchange programmes of the association and who

could benefit from it.

INTRODUCTION

CASLE was formed in 1969 as a federation of independent professional societies representing

surveying and land economy in Commonwealth countries. CASLE is an organisation which is

non profit-making in nature, and any monies raised, or any surplus monies made from its

activities are deployed into activities which are intended to support its aims as set out in Article 2

of the Constitution.

It currently comprises over 40 societies in 32 countries; it has approved Associate Members and

correspondents, some of whom are in 19 other countries. The professional societies represented

cover the disciplines of surveying and mapping, land economy, and quantity surveying and cost-

control.

CASLE welcomes membership from professional institutions and societies and associate

membership from individuals and groups across the Commonwealth.

Page 53: unitedemsa.files.wordpress.com...Pg 2 NIESV STUDY PACK - PPE KNOWLEDGE AND STRUCTURE OF NIGERIA INSTITUTION OF ESTATE SURVEYORS AND VALUERS Learning Objectives At the end of this lecture,

Pg 53

NIESV STUDY PACK - PPE

Surveying and Land Economy

Surveying and land economy is professional grouping that contains three broad fields of activity;

surveying and mapping, land economy, and quantity surveying. All three are increasingly

recognized as having an important part to play in the development of every Commonwealth

country, and are well positioned to make significant contributions to the effective development

and economic management of resources.

“Mission, Aim and Objectives

Mission: CASLE has as its mission statement - "We are committed to the advancement of the

profession of Surveying in the Commonwealth, and to the enhancement of the skills of

Surveyors in the management of the natural and built environments for the common good."

Aims:

the encouragement of provision in Commonwealth countries of adequate facilities for

professional and technician education

the transfer of technology throughout the Commonwealth, and

through programmes of continuing professional development, the constant up-dating of

professional skills and the application of rapidly advancing technologies to a wide variety

of surveyors‟ functions.

Objectives:

Its objectives have accordingly been defined as:

fostering the development of the profession in all Commonwealth countries.

fostering appropriate standards of education for surveying and land economy and the

establishment of appropriate facilities for education and training.

helping to develop professional techniques and practices attuned to national needs.

facilitating the transfer of technology within the Commonwealth and assisting national

programmes of continuing professional development designed to keep surveyors up-to-

date.

Page 54: unitedemsa.files.wordpress.com...Pg 2 NIESV STUDY PACK - PPE KNOWLEDGE AND STRUCTURE OF NIGERIA INSTITUTION OF ESTATE SURVEYORS AND VALUERS Learning Objectives At the end of this lecture,

Pg 54

NIESV STUDY PACK - PPE

encouraging dialogue between its member societies and national governments on all

matters of national policy on which the profession is competent to offer informed

opinions and advice.

CASLE achieves these objectives through conferences and seminars, lecture tours, publications,

manpower studies and direct advice to governments, universities, other educational bodies and

its own member societies. In the period 1969 - 2002 CASLE organized eight General Assemblies

and many regional conferences, seminars, workshops and lecture tours. In some instances these

will be offered in the successful CASLE "Road-show" format which consists of CPD activities,

covering a spectrum of surveying topics. These involve a small group of specialist surveyors

consecutively delivering a lecture and workshop programme to adjacent centres within a region

of the Commonwealth.

CASLE also publishes a regular newsletter, reports of general assemblies and regional

conferences and seminars, information of interest to the profession, including advice on

Education for Surveying and Land Economy.

CASLE and Sustainability

CASLE believes that many of the problems facing the world in respect of the use of scarce

resources may be mitigated by careful planning. Within the scope of the three professional fields,

the education and experience of surveyors may be drawn upon to provide advice on the effective

use of resources in new projects, and the long-term resource and cost consequences of planning,

design, and construction decisions.

There are many definitions of the term "sustainability", which tends to obscure the fact that

things can be improved in practice with currently available skills. CASLE believes that the

modification of the environment to serve human needs can and must be more sensitively

managed. Whilst some Governments and developers are already alive to the issues, CASLE is

keen to encourage the wider use of surveyors‟ and others‟ complementary skills, within their

specialist fields of competence, to solve problems. In the latter case, CASLE joined with the

complementary Commonwealth organisations representing Architects, Engineers, and Planners,

to deliver a one-day programme of seminars and workshops on issues relating to sustainability.

Page 55: unitedemsa.files.wordpress.com...Pg 2 NIESV STUDY PACK - PPE KNOWLEDGE AND STRUCTURE OF NIGERIA INSTITUTION OF ESTATE SURVEYORS AND VALUERS Learning Objectives At the end of this lecture,

Pg 55

NIESV STUDY PACK - PPE

This initiative is part of a continuing commitment by CASLE, to encourage the development of

constructive attitudes towards sustainability, at all levels in public and private sector clients and

their professional advisers, throughout the Commonwealth. CASLE followed up these initiatives

in 2001 with a presence at the „Istanbul +5 meeting in New York, and also contributed to the

„pre-CHOGM‟ events in Australia.” (castle.org.).

Page 56: unitedemsa.files.wordpress.com...Pg 2 NIESV STUDY PACK - PPE KNOWLEDGE AND STRUCTURE OF NIGERIA INSTITUTION OF ESTATE SURVEYORS AND VALUERS Learning Objectives At the end of this lecture,

Pg 56

NIESV STUDY PACK - PPE

INTERNATIONAL FEDERATION OF SURVEYORS (FIG)

Learning Objectives

“At the end of this lecture, students are expected to have acquired basic understanding of the

structure as well as operations of the International Real Estate Federation and should be able to:

a. Demonstrate in clear terms good knowledge of the body.

b. Understand the broad idea of the structure of the federation and their inter relationship.

c. Establish an understanding of the basic requirements for membership of the federation.

d. Understand the various roles of FIG.

e. Acquire a basic understanding of the general examinations that qualities an individual to

be a member.

f. Demonstrate the knowledge of the exchange programmes of the federation and who

could benefit from it.

INTRODUCTION

FIG is the premier international organization representing the interests of surveyors worldwide.

It is a federation of the national member associations and covers the whole range of professional

fields within the global surveying community. It provides an international forum for discussion

and development aiming to promote professional practice and standards.

FIG was founded on July 18 1878 in Paris by delegates from seven national associations -

Belgium, France, Germany, Great Britain, Italy, Spain and Switzerland - and was known as the

Fédération Internationale des Géomètres. This has become anglicized to the International

Federation of Surveyors. It is a UN-recognized non-government organization (NGO),

representing more than 120 countries throughout the world, and its aim is to ensure that the

disciplines of surveying and all who practise them meet the needs of the markets and

communities that they serve.

Page 57: unitedemsa.files.wordpress.com...Pg 2 NIESV STUDY PACK - PPE KNOWLEDGE AND STRUCTURE OF NIGERIA INSTITUTION OF ESTATE SURVEYORS AND VALUERS Learning Objectives At the end of this lecture,

Pg 57

NIESV STUDY PACK - PPE

The FIG vision

A modern and sustainable surveying profession in support of society, environment and economy

by providing innovative, reliable and best practice solutions to our rapidly changing and complex

world, acting with integrity and confidence about the usefulness of surveying, and translating

these words into action.

The role of FIG

FIG‟s activities are governed by a work plan, which is approved by the General Assembly and

reviewed by Council as its tenure progresses. The current work plan with the motto “Ensuring

the Rapid Response to Change Ensuring the Surveyor of Tomorrow” guides Council,

Commissions, Networks and Task Force in their activities.

FIG supports the role of a prosperous and sustainable profession of surveyors to provide solution

functionality, reliably, affordably for a complex and rapidly changing world that cannot wait, and

to translate a sustainable development agenda into action. FIG supports international

collaboration among its members for the progress of surveying in all its fields and applications.

FIG has a close cooperation with United Nations relevant bodies, World Bank, and its sister

associations and has been globally recognized as the leading international non-governmental

organization on geospatial information and the management of “land”, the “sea” and the “built”

environment. It is within the surveyors‟ task to determine the size and shape of the earth, to map

its surface and to manage it in a sustainable way.

Who are the members of FIG?

FIG draws its membership from practitioners working in communities with both the public and

private sectors, from the scientific, research and academic community, as well as from the spatial

technologies and services community. FIG functions with the goodwill, resources and

contribution of its memberships and their corp of volunteers from around the world.

Members of FIG consist of:

Page 58: unitedemsa.files.wordpress.com...Pg 2 NIESV STUDY PACK - PPE KNOWLEDGE AND STRUCTURE OF NIGERIA INSTITUTION OF ESTATE SURVEYORS AND VALUERS Learning Objectives At the end of this lecture,

Pg 58

NIESV STUDY PACK - PPE

member associations – national associations representing one or more of the disciplines of

surveying

affiliates – groups of surveyors or surveying organizations undertaking professional activities

but not fulfilling the criteria for member associations

corporate members – organizations, institutions or agencies which provide commercial

services related to the profession of surveyor

academic members – organizations, institutions or agencies, which promote education or

research in one or more of the disciplines of surveying. An individual may be appointed as a

correspondent in a country where no association or group of surveyors exist that is eligible to

join FIG as a member.

Page 59: unitedemsa.files.wordpress.com...Pg 2 NIESV STUDY PACK - PPE KNOWLEDGE AND STRUCTURE OF NIGERIA INSTITUTION OF ESTATE SURVEYORS AND VALUERS Learning Objectives At the end of this lecture,

Pg 59

NIESV STUDY PACK - PPE

ROYAL INSTITUTION OF CHARTERED SURVEYORS

Learning Objectives

At the end of this lecture, students are expected to have acquired basic understanding of the

structure as well as operations of the International Real Estate Federation and should be able to:

a. Demonstrate in clear terms good knowledge of the body.

b. Understand the broad idea of the structure of the Association and their inter relationship.

c. Establish an understanding of the basic requirements for membership of the association.

d. Understand the various FIABCI designations and to whom they are applied.

e. Acquire a basic understanding of the general examinations that qualities an individual to

be a member.

f. Demonstrate the knowledge of the exchange programmes of the association and who

could benefit from it.

INTRODUCTION

The Royal Institution of Chartered Surveyors (RICS) is a professional body that accredits

professionals within the land, property and construction sectors worldwide.

Members holding RICS qualifications may use the following designations after their name:

MRICS (Member), FRICS (Fellow), AssocRICS (Associate). Those with the designation

MRICS or FRICS are also known as chartered surveyors.

History

“The RICS was founded in London as the "Institution of Surveyors" after a meeting of 49

surveyors at the Westminster Palace Hotel on 15 June 1868. The inaugural president was John

Clutton (who founded Cluttons, a property firm still in business today). RICS has occupied

headquarters on the corner of Great George Street and Parliament Square since then. It received a

Royal charter as "The Surveyors' Institution" on 26 August 1881.

Page 60: unitedemsa.files.wordpress.com...Pg 2 NIESV STUDY PACK - PPE KNOWLEDGE AND STRUCTURE OF NIGERIA INSTITUTION OF ESTATE SURVEYORS AND VALUERS Learning Objectives At the end of this lecture,

Pg 60

NIESV STUDY PACK - PPE

The Surveyors' Institution became the "Chartered Surveyors' Institution" in 1930.[3] In 1946,

George VI granted the title "Royal" and in 1947 the professional body became the “Royal

Institution of Chartered Surveyors”.

International presence

The RICS headquarters is in London with its main support functions in Coventry. There are

regional offices in the United Kingdom, across mainland Europe, in China, Hong Kong,

Singapore, Australia, the Middle East, Sub-Saharan Africa, North America and Brazil.

In 2015, there were RICS-qualified professionals in more than 140 countries. The total number

of those accredited worldwide was 120,000, plus student membership of 81,000. The majority of

accredited individuals or members are still based in the United Kingdom with large numbers also

in mainland Europe, Australia and Hong Kong. There is now a strong growth globally, and

particularly in China, India and the Americas.

The RICS has close links with many national surveying institutions and is a founding member

association of the International Federation of Surveyors (FIG). Within the RICS the primary

areas of practice represented at FIG are geomatics (land and hydrographic survey), environment,

planning, construction and valuation.

The RICS works in close collaboration with other professional bodies, central banks and

international organisations such as The United Nations, World Bank and The European Union.

In 2013 the RICS was a founder member of the coalition to develop International Property

Measurement Standards, which launched its first standard – for measuring office space – in

November 2014. In 2014 the RICS was a founder member of the coalition to develop

International Ethics Standards. In each case, coalition member bodies are committed to

implementing the new standards through training and guidance for professional practitioners.

Page 61: unitedemsa.files.wordpress.com...Pg 2 NIESV STUDY PACK - PPE KNOWLEDGE AND STRUCTURE OF NIGERIA INSTITUTION OF ESTATE SURVEYORS AND VALUERS Learning Objectives At the end of this lecture,

Pg 61

NIESV STUDY PACK - PPE

Membership

Entry to membership of the RICS is via four main routes: academic; graduate; technical; and

senior professional. The RICS has links with universities worldwide, with whom they have

accredited approved courses which satisfy part of the qualification requirements to become

trainee surveyors. The RICS also offers expedited routes to membership for qualified

professional members of some partner associations.

The RICS requires members to update their knowledge and competence during their working life

through Continuing Professional Development.

• Associate members may use "AssocRICS" after their names (previously members at this

level were known as Technical Members and used the designation "TechRICS").

• Professional Members may use "MRICS" after their names (previously members at this

level were known as Professional Associates and used the designation "ARICS").

• Fellows may use "FRICS" after their names.

• Honorary members may use “HonRICS” after their names.

Individuals holding the professional MRICS or FRICS designation are entitled to use the

professional title designation "Chartered Surveyor" and variations such as "Chartered Building

Surveyor" or "Chartered Quantity Surveyor", depending on their chosen specialist qualifications

and field of expertise.

Professional Groups

The RICS specifies areas of specialism, each with its own professional group, clustered into

Land, Property and Construction. Within each professional group there may be further

specialisms.

Page 62: unitedemsa.files.wordpress.com...Pg 2 NIESV STUDY PACK - PPE KNOWLEDGE AND STRUCTURE OF NIGERIA INSTITUTION OF ESTATE SURVEYORS AND VALUERS Learning Objectives At the end of this lecture,

Pg 62

NIESV STUDY PACK - PPE

Property Professional

Groups Land Professional

Built Environment

Professional Groups

Arts & Antiquities Environment Building Control

Commercial Property Geometrics Building Surveying

Dispute Resolution Minerals & Waste Project Management

Facilities Management Planning & Development Quantity Surveying &

Construction

Machinery & Assets Rural Dilapidations forum

Management Consultancy Telecom forum Insurance forum

Residential property

Valuation

Building conservation forum

Specialist accreditations

The RICS aims to cover, among its practicing members, property and construction related

expertise generally. Specialized areas of practice expertise for which accreditation is available

include:

Building Information Modelling (BIM) Manager Certification

Building Conservation

Chartered Environmentalists

Dispute Resolution

ECO Assessor Certification

Fixed Charge Receivership Scheme

Valuer Registration.

Page 63: unitedemsa.files.wordpress.com...Pg 2 NIESV STUDY PACK - PPE KNOWLEDGE AND STRUCTURE OF NIGERIA INSTITUTION OF ESTATE SURVEYORS AND VALUERS Learning Objectives At the end of this lecture,

Pg 63

NIESV STUDY PACK - PPE

Past Presidents

Recent Presidents of the RICS have included Jonathan David Harris OBE, Peter Ralph Faulkner,

Peter William Fall, Nick Brooke, Barry Gilbertson, Steve Williams, Graham Chase, David

Tuffin, Peter Goodacre, Max Crofts and Robert Peto.

RICS Matrics

The junior branch of the RICS, known as Matrices (pronounced "matrix"), provides educational

support, charitable and networking activities for surveying students, trainee surveyors (of any

age) and Chartered Surveyors with ten years or less post-qualification experience. It comprises

some 40 local groups across the United Kingdom. Established in 1889 as the Junior Committee,

it became the "Junior Organization" ("JO") in 1928 and was re-branded as "RICS Matrics" in

2003. It also has links with the Young Chartered Surveyors in the Republic of Ireland.

BCIS

BCIS is the Building Cost Information Service of RICS. Established in 1962 BCIS provides

independent cost and price information to the construction industry and anyone else who needs

comprehensive, accurate and independent data. BCIS pioneered elemental cost planning in the

early 1960s that is now the basis of early cost advice in the construction industry today. The

organization holds the largest independent database of cost plans in elemental form.

Services, available to government bodies and private developers, include measuring price

movement, benchmarking, market research, statistical analysis, forecasting, and impact studies,

relating to construction, maintenance, rebuilding and insurance. BCIS International offers

survey-based cost information for markets outside the UK.

Charitable works

Lion heart is the benevolent fund for past and present RICS members and their families. The

charity was established in 1899 and provides financial support, health and well-being packages,

and work-related counselling and befriending support. RICS also supports the Chartered

Surveyors Training Trust, which helps young people enter the profession through

Page 64: unitedemsa.files.wordpress.com...Pg 2 NIESV STUDY PACK - PPE KNOWLEDGE AND STRUCTURE OF NIGERIA INSTITUTION OF ESTATE SURVEYORS AND VALUERS Learning Objectives At the end of this lecture,

Pg 64

NIESV STUDY PACK - PPE

apprenticeships; Charity Property Help, which provides property advice to charities and

voluntary organisations, and The Chartered Surveyors' Voluntary Service (CSVS), a registered

charity providing free property advice to people who would otherwise struggle to access

professional assistance.”(en.wikipedia.org).

Page 65: unitedemsa.files.wordpress.com...Pg 2 NIESV STUDY PACK - PPE KNOWLEDGE AND STRUCTURE OF NIGERIA INSTITUTION OF ESTATE SURVEYORS AND VALUERS Learning Objectives At the end of this lecture,

Pg 65

NIESV STUDY PACK - PPE

PROFESSIONAL ETHICS

Professional ethics is defined as the personal and corporate rules that govern behavior within the

context of a particular profession. An example of professional ethics is ESVARBON Rules that

set ethical rules that govern an estate surveyor and valuer's moral obligations. These are works

etiquettes and moral standards expected from professional in the discharge of his duties to his

clients.

Where a professional estate surveyor dies contrary to the expected ethics, he is said to have

committed either professional misconduct or negligence.

Professional misconduct is the failure of a licensed professional to meet expected standards of

practice. It is measured by doing something against the rules and regulations for estate surveying

practice.

Professional "negligence" on the other hands means an act or an omission in the carrying out of

the work of a practitioner that constitutes a failure to maintain the standards that a reasonable and

prudent practitioner would maintain in the circumstances.

PRINCIPAL-AGENT RELATIONSHIP

Learning Objectives

At the end of this lecture, students are expected to have acquired basic understanding of the

structure as well as operations of the Principal – agent relationship and should be able to:

a. Demonstrate in clear terms good knowledge the concept of Principal.

b. Understand the broad idea of the structure of the purpose and relationship between

principal and agent.

c. Establish an understanding of the fiduciary responsibilities of an agent.

d. Understand the various FIABCI designations and to whom they are applied.

e. Acquire a basic understanding of the general examinations that qualities an individual to

be a member.

Page 66: unitedemsa.files.wordpress.com...Pg 2 NIESV STUDY PACK - PPE KNOWLEDGE AND STRUCTURE OF NIGERIA INSTITUTION OF ESTATE SURVEYORS AND VALUERS Learning Objectives At the end of this lecture,

Pg 66

NIESV STUDY PACK - PPE

f. Demonstrate the knowledge of the exchange programmes of the association and who

could benefit from it.

INTRODUCTION

The principal-agent relationship is an arrangement in which one entity legally appoints another

to act on its behalf. In a principal-agent relationship, the agent acts on behalf of the principal and

should not have a conflict of interest in carrying out the act.

Who can be a Principal?: Any person who has the legal capacity (meaning that they are not

insane, or in certain circumstances a minor) to perform an act may be a principal and empower

an agent to carry out that act. Persons, corporations, partnerships, not-for-profit organizations,

and government agencies may all be principals and appoint agents.

Who can be an Agent?: Any individual capable of comprehending the act to be undertaken is

qualified to serve as an agent.

Purpose of Agency Relationship

Agency as a contract to be made by an agent on behalf of a principal is considered to be the

contract of the principal and not that of the agent. It allows the principal to authorize somebody

to carry out her duties, either for a specific purpose (i.e., purchasing a house) or generally (i.e., to

conduct many transactions). The agency relationship is usually entered into by informal

agreement, but also can occur by formal agreement (in certain cases, the agency relationship

must be specified in writing). The acts must be legal (i.e., principal can not hire agent to kill the

professor).

Basis of the Agency relationship

Inherent in the Principal-Agent (P-A) relationship is the understanding that the agent will act for

and on behalf of the principal. The agent assumes an obligation of loyalty to the principal that

she will follow the principals instructions and will neither intentionally nor negligently act

improperly in the performance of the act. An agent cannot take personal advantage of the

business opportunities the agency position uncovers. A principal, in turn, reposes trust and

Page 67: unitedemsa.files.wordpress.com...Pg 2 NIESV STUDY PACK - PPE KNOWLEDGE AND STRUCTURE OF NIGERIA INSTITUTION OF ESTATE SURVEYORS AND VALUERS Learning Objectives At the end of this lecture,

Pg 67

NIESV STUDY PACK - PPE

confidence in the agent. These obligations bring forth a fiduciary relationship of trust and

confidence between P and A.

Obligations of the Agent to the Principal

1. An agent must obey reasonable instructions given by the P.

2. The Agent must not do acts that have not been expressly or impliedly authorized by the

Principal.

3. The Agent must use reasonable care and skill in performing the duties.

4. Most importantly, the Agent must be loyal to the Principal.

5. The Agent must refrain from putting herself in a position that would ordinarily encourage a

conflict between the agents own interests and those of the principal.

6. The Agent must keep the Principal informed as to all facts that materially affect the agency

relationship.

Responsibilities of the Agent

“The elements of an agent‟s responsibility to the principal are summed up in one word fiduciary.

In other words, the agent owes a fiduciary obligation to the principal or client which warrants

him to perform the following responsibilities:

Accounting: Real estate agent handles money; sometimes, large sums of it. Though these monies

are the agents clients money, and will eventually be remitted to the client, they may be held by

the agent for a considerable amount of time. The duty of accounting for all the funds that are

given to an agent, for safe keeping is part of the agents fiduciary responsibilities. The law

requires that funds that belong to clients and customers be kept in a bank account that‟s separate

from the agent‟s business account to avoid commingling or combining of client and customer

funds with the broker‟s business or personal funds.

Care: Care is best described as agents using their best efforts and skills on behalf of their clients‟

respective interests. Activities such as helping a seller client determine a fair asking price for a

Page 68: unitedemsa.files.wordpress.com...Pg 2 NIESV STUDY PACK - PPE KNOWLEDGE AND STRUCTURE OF NIGERIA INSTITUTION OF ESTATE SURVEYORS AND VALUERS Learning Objectives At the end of this lecture,

Pg 68

NIESV STUDY PACK - PPE

property and then making every reasonable effort to market the property are expectations of the

client under the care provision of fiduciary duties.

Confidentiality: The agent is expected to keep all information that can harm the client‟s interest

in the strictest confidence in addition to any personal information the client wants to be kept

confidential, even if the information won‟t harm the client were it known. For example, seller

client‟s desperate need to sell a property because of a financial situation needs to be kept

confidential. In some places, the duty of confidentiality is considered to be part of the fiduciary

duty of loyalty.

Disclosure: The fiduciary duty of disclosure requires you, as an agent, to reveal any facts you‟re

aware of that benefit your client. Disclosure applies to information that may benefit the client

even if the client hasn‟t asked about it.

As the agent, perhaps you know that the town is undergoing a tax reassessment that drastically

may change the taxes on all the houses in town and your buyer client wants to buy a home for the

first time and doesn‟t have a clue what a reassessment is. As a buyer‟s agent, you‟re required to

tell your client about the proposed reassessment, even though they didn‟t ask about it.

Disclosure also applies to information that can hurt your customer‟s interest and that your

customer has asked you to keep confidential. Perhaps your buyer customer reveals to you some

financial difficulty that may make getting a mortgage difficult. You need to tell your seller client

about the possibility that the buyer may have difficulty getting a mortgage, even if they asked

you to keep it secret.

Keep in mind that the disclosure that is part of an agent‟s fiduciary responsibility isn‟t the same

as a seller‟s obligation to disclose latent and material defects.

Too often, buyer customers begin to get so comfortable with the real estate agent that they reveal

information that can hurt them in a negotiation and which the agent must reveal to his seller

client. The customer either forgets that the agent represents the seller or never fully understands

exactly what that means in terms of fiduciary duties.

Page 69: unitedemsa.files.wordpress.com...Pg 2 NIESV STUDY PACK - PPE KNOWLEDGE AND STRUCTURE OF NIGERIA INSTITUTION OF ESTATE SURVEYORS AND VALUERS Learning Objectives At the end of this lecture,

Pg 69

NIESV STUDY PACK - PPE

Loyalty

Loyalty means always putting your client‟s interests above everyone else‟s, including your own.

You must never profit by doing something against your client‟s interest.

Say, for example, that you represent a buyer who wants to offer N200,000 for a house. You

know the seller will take N 180,000, and you‟re being paid a percentage of the total commission

paid to the seller‟s broker based on the final sale price of the house. That means the higher the

price of the house, the more money you make as the agent, right?

Even though disclosing the seller‟s acceptable price is against your interest and you‟ll make less

money, you nevertheless have to advise your buyer client to make the lower offer. If you don‟t,

you‟re violating the fiduciary duty of loyalty and profiting at the expense of your client.

Obedience

Obedience, as a fiduciary duty, requires you as an agent to follow the instructions of your

principal. The only limitation on adhering to the duty of obedience is if the client‟s instructions

are illegal or unethical. If your seller client gives you instructions that violate some provision of

fair housing laws regarding marketing the property, your fiduciary duty of obedience doesn‟t

require you to break the law.

The obedience requirement also doesn‟t extend to keeping things confidential regarding

problems with the property itself. If your seller client instructs you not to tell potential buyers

about a leaky roof, you wouldn‟t obey the client.” (www.dummies.com).

Page 70: unitedemsa.files.wordpress.com...Pg 2 NIESV STUDY PACK - PPE KNOWLEDGE AND STRUCTURE OF NIGERIA INSTITUTION OF ESTATE SURVEYORS AND VALUERS Learning Objectives At the end of this lecture,

Pg 70

NIESV STUDY PACK - PPE

REFERENCES

John A. Yoegel, What you should know about the agent and principal

relationship for the real estate license exam, from real estate license exams for

dummies, 2nd edition,

Page 71: unitedemsa.files.wordpress.com...Pg 2 NIESV STUDY PACK - PPE KNOWLEDGE AND STRUCTURE OF NIGERIA INSTITUTION OF ESTATE SURVEYORS AND VALUERS Learning Objectives At the end of this lecture,

Pg 71

NIESV STUDY PACK - PPE

Project Coordination and Control

Learning Objectives

At the end of this lecture, students are expected to have acquired basic understanding of project

coordination and control and should be able to:

g. Demonstrate in clear terms the concept of a Project

h. Understand the broad idea of project management and key stakeholders involved

i. Establish an understanding of the project life cycle including project stages, project

constraints and classification by project management life cycle

j. Understand the concept of project initiation document (PID) and purpose of the PID

k. Acquire a basic understanding of the qualities and key roles of a Project Coordinator

l. Illustrate the function of project board controls and how issues, problems and changes are

handled.

CONCEPT OF A PROJECT

A project is a sequence of unique, complex, and connected activities having one goal or purpose

and that must be completed by a specific time, within budget, and according to specification. A

project is also defined as an assignment/task/job that has to be undertaken and completed within

a set time, budget, resources and performance specifications designed to meet the needs of

stakeholder and beneficiaries

For example

The Canadian International Donor Agency (CIDA) has donated R7.5 million to provide RDP

homes to 50 families living in the Joe Slovo informal settlement. On 6 February 2004, the

agency signed a contract with the Department of Housing to implement the project. The

following requirements, amongst others were set in the contract:

1. The RDP houses must meet specifications in line with government policy.

2. In order to ensure sustainability and affordability for the 50 families, the head of each of the

50 families must be given skills development training in small business development and small

Page 72: unitedemsa.files.wordpress.com...Pg 2 NIESV STUDY PACK - PPE KNOWLEDGE AND STRUCTURE OF NIGERIA INSTITUTION OF ESTATE SURVEYORS AND VALUERS Learning Objectives At the end of this lecture,

Pg 72

NIESV STUDY PACK - PPE

business start-up. This is to ensure that the families will be able to afford rentals, maintenance of

the homes and to expand their homes to accommodate the growth of the families in the future.

3. The project must be completed within three years and the handover of the homes to the 50

families must be a high profile public event.

From the example we see:

• a clear task - build RDP homes for 50 families;

• a set time – within 3 years;

• a budget – R7.5 million;

• performance specifications to meet the stakeholder needs – houses that meet the specifications

in line with government policy, training for the head of each family;

• Beneficiaries – 50 families;

• Stakeholders – donor agency, Department of Housing

What is project management?

Project Management is the use of knowledge, skills, tools, and techniques to plan and implement

activities to meet or exceed stakeholder needs and expectations from a project.

Project stakeholders

Project stakeholders are individuals and organisations who are actively involved in the project, or

whose interests may be positively or negatively affected by the project.

Key stakeholders in the example above include:

• Project Manager - the individual responsible for managing the project;

• Project beneficiaries – 50 families who are going to receive the houses;

• Performing organization – the Department of Housing whose employees are most directly

involved in doing the work of the project;

Page 73: unitedemsa.files.wordpress.com...Pg 2 NIESV STUDY PACK - PPE KNOWLEDGE AND STRUCTURE OF NIGERIA INSTITUTION OF ESTATE SURVEYORS AND VALUERS Learning Objectives At the end of this lecture,

Pg 73

NIESV STUDY PACK - PPE

• Sponsor – Canadian International Development Agency.

In addition to these there are many different typical stakeholders:

• Suppliers and contractors – e.g. Construction companies, Skills development and education

and training organisations, legal firms, events management company;

• Project team members and their families;

• Government agencies – e.g. Local municipality.

• Community representatives and organisations

What is project management?

Project Management is the use of knowledge, skills, tools, and techniques to plan and implement

activities to meet or exceed stakeholder needs and expectations from a project.

Project stakeholders

Project stakeholders are individuals and organisations who are actively involved in the project, or

whose interests may be positively or negatively affected by the project.

Key stakeholders in the example above include:

• Project Manager is the individual responsible for managing the project

• Project beneficiaries are the 50 families who are going to receive the houses

• Performing organization the Department of Housing whose employees are most directly

involved in doing the work of the project;

• Sponsor Canadian International Development Agency.

In addition to these there are many different typical stakeholders:

• Suppliers and contractors – e.g. Construction companies, Skills development and education

and training organisations, legal firms, events management company;

Page 74: unitedemsa.files.wordpress.com...Pg 2 NIESV STUDY PACK - PPE KNOWLEDGE AND STRUCTURE OF NIGERIA INSTITUTION OF ESTATE SURVEYORS AND VALUERS Learning Objectives At the end of this lecture,

Pg 74

NIESV STUDY PACK - PPE

• Project team members and their families;

• Government agencies – e.g. Local municipality.

• Community representatives and organisations

The project life cycle

Projects are usually divided into stages which are, definition, planning, execution and delivery

stages for a better management and control. Collectively these project stages are known as the

project life cycle.

The starting point begins the moment the project is given the go- ahead (when a contract

agreement is signed). Project effort starts slowly, builds to a peak, and then declines to delivery

to the customer.

• Definition stage – This stage specifies the project, project objectives are established, the teams

are formed and major responsibilities are assigned.

• Planning stage – In this stage, the plans are developed to determine the project steps,

beneficiaries, timeframes, quality standards and budget.

• Execution stage – This is the stage where the major portion of the project work takes place

both physically and mentally. Time, cost and specification measures are used for control. The

project managers have to ensure that the project is on schedule within the budget and meeting

specifications. They have to also check if any changes are required.

• Delivery stage – This involve delivering the project to the customer and this may involve

customer training and transfer of documents.

Project Constraints

• Projects are limited by their product quality and process quality requirements – Cost – mostly

labor cost, but also hardware, software, training, etc. – Calendar time (schedule) –

Requirements/objectives and/or quality – Resources – people (skills), facilities, equipment, etc. •

Page 75: unitedemsa.files.wordpress.com...Pg 2 NIESV STUDY PACK - PPE KNOWLEDGE AND STRUCTURE OF NIGERIA INSTITUTION OF ESTATE SURVEYORS AND VALUERS Learning Objectives At the end of this lecture,

Pg 75

NIESV STUDY PACK - PPE

In reality we can only control and manage – at most - two of these constraints – Client decision:

Which constraints can you tolerate flexibility in?

Concept of Project Planning • Project Planning involves understanding the fundamentals of a

project: – What business situation is being addressed? – What do you need to do? – What will

you do? – How will you do it? – How will you know you did it? – How well did you do?

Classification by Project Management Life Cycle

Traditional PM – Follows the linear, waterfall model • Incremental planning PM – Delivers the

project in incremental stages – Reduces risk of delivering everything at once • Iterative planning

PM – Allows project to evolve as understanding increases – Allows management and

stakeholder expectations / operational challenges to be clarified • Adaptive planning PM –

Allows project purpose & goals to evolve as project proceeds – Project planned in cycles: goals

& requirements reviewed each cycle • Extreme planning PM – Involves users and/or client in

constant input / review of requirements – Needs small group of project team working closely

“Initiating the Project- Project Initiation is where you create a sound baseline for management of

a project by taking current understanding of the „what‟ and „why‟, as documented in the Project

Brief, and extending it to include a detailed definition of „how‟, „when‟, and by „whom‟ in a

Project Initiation Document.

Project Initiation Document (PID)-The purpose of the PID is to provide the information

required by senior management and stakeholders to enable them to commit to the resources and

timelines proposed. It is a sort of „contract‟ between the Project Manager and Project Board that

defines how the project will be run.

The PID provides a detailed proposition against which success can be measured. To do this the

PID builds on the approved Project Brief by defining in detail how the project will be developed

and when it will be delivered. It provides a more detailed understanding of the costs and benefits

of the project and, in particular, the resources, risks and timelines required for successful

delivery. How the Project Initiation Document (PID) is used. The PID is presented to the Project

Board so that the views of key stakeholders can be considered. This is an essential stage in the

process to ensure engagement and buy-in from all interested parties to the proposed outcomes.

Page 76: unitedemsa.files.wordpress.com...Pg 2 NIESV STUDY PACK - PPE KNOWLEDGE AND STRUCTURE OF NIGERIA INSTITUTION OF ESTATE SURVEYORS AND VALUERS Learning Objectives At the end of this lecture,

Pg 76

NIESV STUDY PACK - PPE

Acceptance of the PID is the start of the next stage of the project where teams are pulled together

to execute the project over the agreed timeline under the accountability of the project board. It is

possible that the detailed analysis undertaken for the PID will uncover increased costs or risks

such that the project is cancelled. This is the strength of the staged project process as it avoids

significant resources being expended on the wrong project.

Developing the Project Initiation Document. The Project Initiation Document is all about

explaining how the project will be delivered and managed. It will update the Project Brief on all

aspects of the project, but specifically it should provide the following:

• Accountabilities, roles and responsibilities of each of the project team, including part time

members (who will do what) • An activity plan (e.g.: a Gantt Chart) on when each deliverable

should be completed (who will do what, and when they will do it by). This will include

dependencies and milestones • An updated assessment of Risks, including their probability and

impact, as well as some mitigation plans and contingency arrangements • Updated Cost/Benefit

analysis, in particular a detailed resource and timing plan (resources and timing often have a

direct impact on each other) • Governance plan that details how the project will be monitored

and controlled in terms of decision points, reports and reporting cycles, including whether

updates will be on an exception or ongoing basis. • Communications Plan that will start to

determine how the project will be communicated to the different audiences, including the press

The PID must document the project management organization and the stakeholders with an

interest in the project's outcome. The following guidance will help you do this.

The Project Manager must identify and gain commitment from appropriate individuals to

participate in project management roles. If any individual appointed to a project role lacks

experience of working in a project environment the project manager should brief them and

manage their expectations to make sure they fully understand the nature of the commitments they

are making in terms of:

• the time and effort they must devote to the project; • the sort of decisions they must take; • the

type of resources they must commit to the project; •the people and organisations they will need

Page 77: unitedemsa.files.wordpress.com...Pg 2 NIESV STUDY PACK - PPE KNOWLEDGE AND STRUCTURE OF NIGERIA INSTITUTION OF ESTATE SURVEYORS AND VALUERS Learning Objectives At the end of this lecture,

Pg 77

NIESV STUDY PACK - PPE

to communicate with; • the sort of risks they will need to consider; • their role in delivering

benefits after the project has completed its work.

Section 3: Running the project Control - the key to a successful project. To appreciate how

project control works you must first understand that, despite all the effort devoted to developing

and gaining commitment to a plan, there is little chance that the resulting project will run

precisely according to that plan. This doesn't mean that you will fail to achieve the objectives of

the plan - on the contrary, you must have a very high level of confidence that you can achieve

those objectives and deliver the full scope, fit for purpose, on time and to budget.

The plan describes what you would like to do but it models just one of the infinite number of

routes from where you are now to where you want to be. In practice your project will follow a

different route to the one shown in your plan, you don't know which one, but you will need

control to make sure it is a route that takes you to where you need to be, when you need to be

there, and at a cost you can afford.

The power of the plan is that it gives you a baseline against which you can compare actual

achievement, cost and time and determine the amount of deviation from plan and hence take

corrective action if required.

The essential requirement for control is to have a plan against which progress can be monitored

to provide the basis for stimulating management action if the plan is not being followed. Control

then becomes a regular, frequent iteration of:

Creating the right environment for control

The basic requirements for control are:

• a plan that is: - realistic - credible - detailed enough to be executed - acceptable to those who

must execute it (Project Manager and Project Teams) - approved by those who are accountable

for its achievement (the Stakeholders and resource „owners‟ (SRO)/ Project Board);

• a process for monitoring and managing progress and resource usage; • a project management

organization of appropriately skilled people with sufficient authority and time to plan, monitor,

report, take decisions and deal with exceptions; • a process to make minor corrections and

Page 78: unitedemsa.files.wordpress.com...Pg 2 NIESV STUDY PACK - PPE KNOWLEDGE AND STRUCTURE OF NIGERIA INSTITUTION OF ESTATE SURVEYORS AND VALUERS Learning Objectives At the end of this lecture,

Pg 78

NIESV STUDY PACK - PPE

adjustments to deal with minor deviations and omissions from the plan; the commitment of those

who will provide the resources indicated in the plan (SRO, • Project Board, Stakeholders and

resource „owners‟ in the parent organization and its related agencies); • explicit authority to

proceed granted by those who are accountable for the • project (i.e. the SRO/ Project Board).

If you do not have all these things there is little point proceeding with the project.

Breaking the project down into manageable stages In all but the smallest or shortest projects you

should think about how to break your project into manageable „chunks‟ called stages. Every

project will have a minimum of two stages - the first being Project Initiation. A large project may

have a number of stages, each of which has its own stage plan. When designing your project's

stage structure look for points where the Project Board should:

• review achievements to date and assess project viability • take key decisions outside the level

of authority of the Project Manager • approve a more detailed plan for the next phase of work •

commit resources in accordance with the project or stage plan • assess the impact of some

significant external event that will influence the project (e.g.: legislation, decision point in other

project, review of business operation).

The Project Manager will also be able to identify stage boundaries by thinking about how far

ahead is it sensible to plan in the fine detail needed for day to day control. In practice, the

detailed plan for a stage will be produced towards the end of the preceding stage, when the

information needed for planning is available.” (www.bis.gov.uk).

Project Coordinator

Who is a Project Coordinator?

Due to the extensive amount of work a Project Manager has on his or her plate it becomes

imperative for the Project Manager to solicit help from a Project Coordinator. The coordinator is

the bond that holds the team and its tasks together to meet deadlines and deliverables for the

project, end to end. The Project Coordinator help Project Managers to deliver projects within the

constraints of Project Scope, Time, and Cost within the Project Governance.

Page 79: unitedemsa.files.wordpress.com...Pg 2 NIESV STUDY PACK - PPE KNOWLEDGE AND STRUCTURE OF NIGERIA INSTITUTION OF ESTATE SURVEYORS AND VALUERS Learning Objectives At the end of this lecture,

Pg 79

NIESV STUDY PACK - PPE

Coordinator Background

Earlier exposure to the Project Management Book of Knowledge (PMBOK) is always a plus.

PMBOK presents a set of standard terminology and guidelines for project management that is

widely used in the industry. A Project Coordinator‟s goal, most often, is to actively work

towards becoming a Project Manager.

A Project Coordinator should possess include skill like excellent oral and written

communication, Excellent prioritization , Customer Relations, Flexibility, Professionalism, Time

Management, Keyboard skills and Proficiency in Microsoft Office

The Role:

The Project Coordinator is an integral member of the project team responsible for delivering

building development projects of varying size and complexity. The Project Coordinator is

responsible for directing, organizing and controlling project activities, under the direction of a

Project Manager (PM) & Project Director.

Scope of Responsibilities:

1. Attend client meetings and assist with determination of project requirements

2. Assist the PM in the drafting and issuance of project proposals, RFP‟s, tenders, budgets, cash

flows and preliminary schedules

3. Prepare project organization and communication charts

4. Chair site meetings and distribute minutes to all project team members

5. Track the progress and quality of work being performed by design disciplines/trades

6. Use project scheduling and control tools to monitor projects plans, work hours, budgets and

expenditures

7. Effectively and accurately communicate relevant project information to the client and project

team

8. Ensure clients‟ needs are met in a timely and cost effective manner

9. Review field inspection reports from Consultants throughout the lifecycle of the project

Page 80: unitedemsa.files.wordpress.com...Pg 2 NIESV STUDY PACK - PPE KNOWLEDGE AND STRUCTURE OF NIGERIA INSTITUTION OF ESTATE SURVEYORS AND VALUERS Learning Objectives At the end of this lecture,

Pg 80

NIESV STUDY PACK - PPE

10. Issue Contracts, Letters of Intent, Purchase Orders, etc.

11. Maintain Contract Execution Tracking Log

12. Assist the PM in the review of Contractor quotations to ensure that only fair and reasonable

pricing is recommended for approval

13. Track & manage contemplated change notices and change orders in the database

14. Prepare substantial completion certificates and ensure all required project close out

documents are obtained

15. Communicate ideas for improving company processes with a positive and constructive

attitude, and for developing this attitude in others

16. Keep the Project Manager (PM) and others informed about project status and issues that may

impact client relationship.

Project Board controls

The controls applied by the Project Board are linked to the Stages in the project and rely on

information about the project current status and future plans being provided by the Project

Manager.

Project Board decisions during delivery

At key milestones during delivery the Project Board might wish to take stock of the project and

satisfy itself that it is sensible and viable to continue. To do this it must be sure that:

• The quality of the deliverables produced to date is acceptable • the required benefits are still

achievable • the actual costs incurred plus revised estimates for future costs are acceptable • the

resources required for planned future work can be made available • the need for the project has

not changed • risks are acceptable • the project is still viable

The Project Manager must provide the Project Board with the information it will need to make

its decision. Often the SRO/ Project Board will need to see updated versions of the Risk Log,

Page 81: unitedemsa.files.wordpress.com...Pg 2 NIESV STUDY PACK - PPE KNOWLEDGE AND STRUCTURE OF NIGERIA INSTITUTION OF ESTATE SURVEYORS AND VALUERS Learning Objectives At the end of this lecture,

Pg 81

NIESV STUDY PACK - PPE

Project Plan and Business Case/Benefits. Highlight Reports to keep the Project Board informed

of stage status, and significant changes and issues that occur between End Stage Assessments,

the Project Manager should present regular Highlight Reports covering:

• Project, date and period covered • Progress achieved as measured against the current plan – e.g.

deliverables completed • Use of Resources (actual versus planned) • Budget status (actual versus

planned) • Actual or potential problems or exceptions • Impact of Issues and Changes (e.g.

requests for changes to requirements) • Products due to be completed during next period •

Revised forecasts for cost and schedule

The frequency of Highlight Reports (usually fortnightly or monthly) should be set by the Project

Board at Project Initiation and should be reviewed/revised when approving a new stage. Other

points to be borne in mind:

• Members of the Project Board should analyze each report carefully and, if necessary, follow it

up informally for clarification or further information.

• The report should be read and acted upon immediately as it may contain early signs of failure.

• Highlight Reporting is an important part of the role of the Project Manager - delays should not

be tolerated.

• If they feel it is worthwhile the members of the Project Board or the SRO may decide to meet

to consider Highlight Reports.

•Project Manager's Controls- As soon as the Project Board gives authority to commence work,

the Project Manager must take control of day-to-day actions and manage the project so that it

runs as close as possible to the approved plan. This means:

• allocating work to the project team(s) in accordance with the plan; • monitoring progress during

development of the deliverables products by the team(s);• ensuring that deliverables meet

specified levels of quality; • ensuring the delivery of completed deliverables to the required

destination(s); • monitoring costs and use of resources; • reporting progress and exceptions to

the SRO/ Project Board via Highlight Reports.

Page 82: unitedemsa.files.wordpress.com...Pg 2 NIESV STUDY PACK - PPE KNOWLEDGE AND STRUCTURE OF NIGERIA INSTITUTION OF ESTATE SURVEYORS AND VALUERS Learning Objectives At the end of this lecture,

Pg 82

NIESV STUDY PACK - PPE

Checkpoint reports are produced by team managers/work stream leaders for the Project Manager

who needs to have early warning of deviations from plan and other problems affecting the

project team. Checkpoints provide regular, frequent comparison of actual progress, resource

usage and forecasts against plans. They provide information for the

Project Manager to apply control, e.g. by correcting small deviations from the plan. The basic

purpose of a checkpoint is to answer the questions:

• „What is going according to plan?‟

• „What is not going to plan?‟

• „What is likely not to go to plan?‟

Checkpoints are essential controls - missed checkpoints are usually an early sign of a failing

project. The information gathered at checkpoints should be documented in Checkpoint Reports

and used in the preparation of Highlight Reports.

Handling significant deviations from plan Project Board members are usually senior managers

with limited time to devote management of the project. In order to achieve „management by

exception‟ the Project Manager should be given authority to deal with the inevitable small

deviations from plan. For larger deviations, such as those resulting from requests for change,

poor estimation, delays in deliveries by external agencies the SRO/ Project Board and Project

Manager will require an agreed exception handling process. This will involve:

• Setting delegated limits (e.g. cost and time „Tolerances‟): The SRO/Project Board should set

limits to the allowable deviations from planned cost and schedule so that the Project Manager

knows how much delegated authority is available to manage deviations from plan; • Exception

reporting: The Project Manager may use an exceptional Highlight Report to notify the SRO/

Project Board of any forecast (or actual) deviations from plan beyond delegated limits. Positive

sorts of exception should also be reported in this way e.g.: finishing work early or using less

resource than planned. • Exception planning and decision making: The SRO/ Project Board may

wish the Project Manager to create a new plan to replace the current one if it is no longer viable.

This plan would be submitted to the SRO Project Board for a decision to proceed.

Page 83: unitedemsa.files.wordpress.com...Pg 2 NIESV STUDY PACK - PPE KNOWLEDGE AND STRUCTURE OF NIGERIA INSTITUTION OF ESTATE SURVEYORS AND VALUERS Learning Objectives At the end of this lecture,

Pg 83

NIESV STUDY PACK - PPE

Handling Issues, Problems and Changes

In all projects issues will arise that may deflect the project from its intended path. In all projects

a straightforward communication mechanism is needed so that anyone associated with the

project can communicate to the Project Manager an issue they think might require management

attention e.g.: • Changes to requirements (e.g. Requests to change to the scope, objectives, target

dates or detailed deliverables of the project). • Faults/errors (e.g. notification that one or more

delivered products that have been signed-off after quality control are subsequently found not to

meet specification). • Problems (e.g. a key stakeholder/agency failing to meet commitments). •

Risks that has become reality. (e.g. supplier failure, industrial action) • Loss of key skills

(resignation, promotion, transfer, sickness). • Concerns about the project and/or its deliverables

(e.g. concerns about the ability to achieve the required benefits). • Questions (e.g. about how the

delivered products will be implemented).

In many cases the Project Manager will have authority to deal with issues as part of the day to

day management. Potential changes that are outside or beyond the Project Manager's authority

will have to be referred to the SRO/ Project Board and covered by an agreed change control

process. On small projects, changes may be handled less formally while for medium to large

projects it is recommended that a more rigorous and formal approach be

adopted.”(www.bis.gov.uk).

References

Guidelines for Managing Projects How to organize, plan and control projects NOVEMBER 2010

Introduction to Project Management: Principles, Techniques and Tools 2013

Project Management Guide

Page 84: unitedemsa.files.wordpress.com...Pg 2 NIESV STUDY PACK - PPE KNOWLEDGE AND STRUCTURE OF NIGERIA INSTITUTION OF ESTATE SURVEYORS AND VALUERS Learning Objectives At the end of this lecture,

Pg 84

NIESV STUDY PACK - PPE

EMPLOYABILITY SKILLS AND COMMERCIAL AWARENESS

INTRODUCTION

In order to ascertain the challenges faced by estate surveying and valuation firms on low

employability skills of Estate Management graduates in Nigeria, it is quite important discuss

insights on employability skills of estate management graduates in Nigeria higher institutions for

productive employment.

Indeed, there are many reasons adduced for studying at a university/polytechnic, including

naturally, a love of the subject to be studied, and the opportunity to experience a different way of

life. It is worthy of note to establish that university/polytechnic education is much more than a

production line for work ready graduates. The two greatest concerns of employers today are

finding good workers and training them. The difference between the skills needed on the job and

those possessed by applicants, sometimes called the skills-gap, is of real concern to human

resource managers and business owners looking to hire competent employees. While employers

would prefer to hire people who are trained and ready to go to work, they are usually willing to

provide the specialized, job-specific training necessary for those lacking such skills. Still and

nevertheless, there is no denying that people see this education as a stepping stone to a good job.

ESTATE MANAGEMENT GRADUATES EMPLOYABILITY SKILLS IN NIGERIA

On employability, a scholar once described it as sets of achievements, skills, understandings and

personal attributes that make graduates more likely to gain employment and be successful in

their chosen occupations, which benefit themselves, the workforce, the community and the

economy. Another described it as a set of attributes, skills and knowledge that all labour market

participants should possess to ensure they have the capability of being effective in the workplace

to the benefit of themselves, their employer and the wider economy.

Employability skills are those basic skills necessary for getting, keeping, and doing well on a job.

These are the skills, attitudes and actions that enable workers to get along with their fellow

workers and supervisors and to make sound, critical decisions. The difference between the skills

needed on the job and those possessed by applicants, sometimes called the „skills-gap‟, is of real

concern to employers looking to hire competent employees. The study is based on the concept of

Page 85: unitedemsa.files.wordpress.com...Pg 2 NIESV STUDY PACK - PPE KNOWLEDGE AND STRUCTURE OF NIGERIA INSTITUTION OF ESTATE SURVEYORS AND VALUERS Learning Objectives At the end of this lecture,

Pg 85

NIESV STUDY PACK - PPE

human capital development. In actual fact, human capital theory rests on the assumption that

education is an investment. It is believed that the goal of human capital is to

portray the role of investment in education in order to boost economic and social achievements.

Human capital posits that an educated population is a productive population.

Thus education improves citizens and helps to upgrade the general standard of living in a society.

Employability can be said to relate to interpersonal skills development, including a person's

knowledge and personal attributes that extend to personality, motivation and other factors unique

to the individual. In addition, the necessary factors that contribute to successful employment

frequently involve the possession of specific knowledge, a set of complex and diverse skills and

personal attributes.

As a matter of fact, there has been no lack of studies indicating the need to re-evaluate the course

curriculum, acknowledge the stakeholders‟ concerns and incorporate improvements in order to

equip the graduates to become more able to address the needs of the industry. Also, with high

expectations of the valuation and property management employers of graduates, it is quite

worrisome that graduates lack the practical skills and knowledge and, as a result, are not able to

relate theory to practice. This is due to the fact that graduates do not have sufficient knowledge

of the Estate Management and property development parlance. On the other hand and most of the

time, employers praise the graduates‟ analytical, computer and communication skills. The burden

however is that there is a need to re-evaluate the curriculum, and that the programmes in

majority of the universities where estate management is domiciled should be improved and more

friendly with stakeholder concerns and needs and incorporate improvements to equip the

graduates to be ready for working in the industry. The suggested changes to the curriculum

include increasing the development of their practical skills, through the use of sandwich

programmes, work experience, case studies and on-site training. Boyd (2005) further suggested

the use of online learning to improve the quality of interaction between students and academics

and enhance the linkage between learning and industry.

Commercial awareness on its own is an integral employability skill. The most important

definition of commercial awareness is a “person‟s ability to understand the economics of

business”. In the real estate parlance, “financial” component is the most important component of

Page 86: unitedemsa.files.wordpress.com...Pg 2 NIESV STUDY PACK - PPE KNOWLEDGE AND STRUCTURE OF NIGERIA INSTITUTION OF ESTATE SURVEYORS AND VALUERS Learning Objectives At the end of this lecture,

Pg 86

NIESV STUDY PACK - PPE

commercial awareness and it forms the largest portion of estate management students‟courses.

The most important skill and attribute for commercial awareness development are “critical

thinking” and “ability and willingness to update professional knowledge”, respectively. Over

time, students and academics have slightly different views on commercial awareness; they are

not very different. Commercial awareness is an important employability skill, thus, making it

necessary for real estate academics to re-visit the curriculum and to ensure learning outcomes

related to commercial awareness have been clearly explained and communicated to students.

Furthermore, it is vital for students to obtain practical experience in order to fully develop their

commercial awareness.

Commercial awareness is the ability to:

Stay informed of what‟s happening at individual property firms (noteworthy deals,

transactions and clients) and within the property sector as a whole (the introduction of

new planning regulations or new initiatives from NIESV).

Form an opinion on current affairs (developments in the economy) from a property

industry perspective.

Decide appropriate advice to give to clients and a business strategy for the property firm

(or as a graduate applicant, to express an opinion on what appropriate advice and

strategies would be and give reasons for them).

True commercial awareness requires an estate management graduate to dig deeper into

property‟s business environment on a regular basis to stay knowledgeable of current trends in

the industries surrounding real estate. A large estate firm‟s clients span a range of industries,

so keeping up with the news across sectors is essential to understanding where clients are coming

from and providing the best services for them. It is important to also note that graduates of estate

management need to form their own opinion and demonstrate a deep understanding of a topical

issue for example on how government policies might affect the residential housing market;

recent regulatory body reviews on negotiations, professional scale of charges, etc

Keeping up with current and 21st century trends in the real estate field and business also includes

the following:

Page 87: unitedemsa.files.wordpress.com...Pg 2 NIESV STUDY PACK - PPE KNOWLEDGE AND STRUCTURE OF NIGERIA INSTITUTION OF ESTATE SURVEYORS AND VALUERS Learning Objectives At the end of this lecture,

Pg 87

NIESV STUDY PACK - PPE

Regular upkeep with the property press.

Regular browsing and surfing the news sections on the estate firms‟ websites.

Research via online media to find examples of the estate firms‟ past and present clients.

Frequent reflections on the impact news will have on the property industry and the advice

that property professionals might give to clients as a result.

Based on the aforementioned, the aspects of employability skills apart from commercial

awareness are discussed below and the essence is to elicit the perception of the employers on the

aspects of employability skill which young graduates of estate management are supposed to

possess before working in their firms.

1. Basic skill

Employers are greatly concerned about reading and listening skills. This means that

employees should listen more to instruction from their superior than carrying out

assignments on their own. Obedience to supervisors is natural in any

work ethics. However, mathematical skill is fundamental in the estate surveying and

valuation profession. Unfortunately the result of this study shows that the employers of

graduates of estate management rated mathematical aptitude to work as poor.Basic skill

also highlights the necessary and primarily fundamental skills estate management

graduates are expected to be armed with including: Reading skill, Mathematical skill,

Listening skill, and Speaking skill

2. Thinking skill

Most of the time, the presumption is that graduates still emphasize

the analysis and understanding of claims learnt in class rooms or workplaces rather than

developing creative thinking which is needed in workplaces. Workplace stresses on what

is rather than what should be. Thinking skill highlights the ability of estate management

graduates to think creatively and also provide an atmosphere for that whether as a team

leader or player, make critical and strategic decisions both as team leader or player, solve

official and administrative problems independently and without much monitoring, and

how to reason critically and productively.

Page 88: unitedemsa.files.wordpress.com...Pg 2 NIESV STUDY PACK - PPE KNOWLEDGE AND STRUCTURE OF NIGERIA INSTITUTION OF ESTATE SURVEYORS AND VALUERS Learning Objectives At the end of this lecture,

Pg 88

NIESV STUDY PACK - PPE

3. Resources Management skill

Resources management skills emphasize the use of a company‟s resources in the most

efficient way possible. Many times, skill in terms of resources management is at a lower

level. This means that there is inefficient management of company‟s resources by

employees, which may contribute to high cost of running companies, thereby reducing

profit margin of companies. Hence, estate management graduates are expected to be

armed with Time management, financial management, Facility management, Risk

management, Human resources management, Work place management.

4. Information skill

Employers most of the time rate and ascertain that computer appreciation and application

as now being taught in schools. This has in a way influenced negatively the writing skills

of both students and graduates. The major cause of this poor writing skill is attributed to

the video generation tendency that eschew the reading of well-written prose and use of

abbreviations in email messages and multi-media stimuli. Information skills are

concerned with the ability of employees to locate information sources and to evaluate,

navigate, organize and communicate the information that they find. This will help

employees to improve information literacy and contribute to create a new knowledge.

Towards advancing this frontier, the necessary ingredients include ability to acquire and

analyze data, organize data and maintain information, interpret information and

communicate, process data using the computer.

5. Interpersonal Skill

Employers are concerned about their employee‟s leadership ability and as such treat this

skill acquisition with utmost importance too. The essence of interpersonal skills is to

enable employees to interact successfully with colleagues in workplaces. It fosters

positive communication at the workplace. It helps to solve relationship problems at work,

motivate and influence the team/group working relationship, reduce the costs of rehiring

and training and leads to positive corporate climate for achieving business performance.

In ensuring that a graduate is well equipped, there is need for apt ability to teach others,

Page 89: unitedemsa.files.wordpress.com...Pg 2 NIESV STUDY PACK - PPE KNOWLEDGE AND STRUCTURE OF NIGERIA INSTITUTION OF ESTATE SURVEYORS AND VALUERS Learning Objectives At the end of this lecture,

Pg 89

NIESV STUDY PACK - PPE

source clients, use and innovate leadership skills, and to negotiate businesses and

transactions well.

6. Personal quality

Employers are also greatly concerned with graduates‟ personal skill. Practitioners are

always of the opinion that personal quality skills of their employees have been at

moderate levels. Employers place great premium on self-management, followed by

punctuality and efficiency at work. However, it has become worrisome for employers

because graduates again do not put these skills into practice. This further reveals that

graduates of estate management must have positive attributes to work. What they need is

training and retraining within the workplace environment to fan the flame of positive

attitude to work. In order to succeed, graduates must be equipped with high sense of

responsibility,self-confidence,sociability,self-management,honesty and punctuality,

efficiency at work ,trainability and flexibility, Self-control and self-direction and above

all, good work attitude.

References

Egbenta, I. R. (2015). Employablity Skills among Graduates of Estate Management in Nigeria.

Journal of Education and Practice, 6(31), 41-49.

Xiaobing, Z., & Xin, Z. (2013). University Students‟ Employability Skills Model Based on

Chinese Employer Perspective. Journal of Human Resource and Sustainability Studies,

2013.

Lowden, K., Hall, S., Elliot, D., & Lewin, J. (2011). Employers‟ perceptions of the employability

skills of new graduates. Edge Foundation, University of Glasgow, SCRE Center.

Robinson, J. P. (2000). What are employability skills. The workplace, 1(3), 1-3.

Questions

1. The two greatest concerns of employers today are finding good workers and training

them. Over the years the perception of employers has grown dim in terms of under-skill

Page 90: unitedemsa.files.wordpress.com...Pg 2 NIESV STUDY PACK - PPE KNOWLEDGE AND STRUCTURE OF NIGERIA INSTITUTION OF ESTATE SURVEYORS AND VALUERS Learning Objectives At the end of this lecture,

Pg 90

NIESV STUDY PACK - PPE

and technical-know-how of Estate Management graduates in Nigeria. You are therefore

required to discuss comprehensively the concept of employability.

2. Enumerate and discuss any five (5) basic employability skills expected of estate

management graduates in Nigeria.

3. The term „commercial awareness‟ is also an important employability skill. Explain this

concept in terms of expectations from employers of labour.

Page 91: unitedemsa.files.wordpress.com...Pg 2 NIESV STUDY PACK - PPE KNOWLEDGE AND STRUCTURE OF NIGERIA INSTITUTION OF ESTATE SURVEYORS AND VALUERS Learning Objectives At the end of this lecture,

Pg 91

NIESV STUDY PACK - PPE

ARBITRATION AND DISPUTE RESOLUTION

INTRODUCTION

Dispute can be defined as a conflict of claims or right; an assertion of a right, claims, or demand

on one side met by contrary claims or allegations on the other. This simply connotes that dispute

can arise where two or more parties with varied rights/claims are into a relationship or contract.

This can be found in any form of relationship including family/marital, friendship, contractual,

business relationship, among others. Dispute or differences in opinion can be between

individuals, individual and corporate entity, or between corporate entities. Professionals are

involved in varied types of relationship on daily basis by their nature of calling, therefore the

need for good knowledge of dispute resolution and management.

In the same vein, dispute is an inevitable aspect of human relations. Due to varied perceptions

and incompatibility in ideas, goals, opinions, belief, motivation and emotion, parties are bound to

have conflict. In fact, formation of relationship is not complete without a laid down means of

settling or resolving disputes when they arise. This is because when dispute arises, majorly from

the conflict of varied interest, the options opened to parties are either to resolve the dispute and

continue the relationship/business or end the relationship/ business abruptly which in itself must

have an agreed procedure to carry out. Hence, every relationship, be it interpersonal or business,

requires that parties set in motion appropriate programme for settling dispute which may ensue in

the course of such relationship.

Every stage of relationship can be susceptible to dispute especially in a contractual relationship.

Having a detailed plan of action for the types of disputes that may occur and how to prevent it

from happening or resolving it if occurred will determine the outcome of such relationship.

Importance of this in any contractual relationship therefore calls for setting up of appropriate

dispute resolution strategy. Because of the role they play, professionals come across varied

aspect of dispute which they are expected to manage professionally. Estate Surveyors and

Valuers should therefore not be caught unprepared when dispute occurs.

There are various ways to settle dispute, each with its own merits and demerits. Therefore,

understanding the pros and cons of each method will give professional the insight into the choice

Page 92: unitedemsa.files.wordpress.com...Pg 2 NIESV STUDY PACK - PPE KNOWLEDGE AND STRUCTURE OF NIGERIA INSTITUTION OF ESTATE SURVEYORS AND VALUERS Learning Objectives At the end of this lecture,

Pg 92

NIESV STUDY PACK - PPE

appropriate approach to apply in addressing any dispute that may ensue from the business

relationship of any sort. However, arbitration has been found to be an effective and business

friendly mean of resolving dispute because of some benefits and advantages it has over other

methods. In this section, therefore, arbitration as a method of dispute resolution shall be

discussed in detail for proper understanding and application.

UNDERSTANDING THE CONCEPT OF ALTERNATIVE DISPUTE RESOLUTION

(ADR)

Conventionally, litigation has been the traditional method of resolving disputes. However,

concerns over cost and delays in litigation procedures together with increasing globalization have

led to more flexible means of resolving disputes which provide alternatives to court-based

litigation governed by the law and procedure of a particular state or country. Legal proceedings

is known to be time consuming, expensive, cumbersome and complex in nature. Furthermore,

the issue involved may be of small amount of money which may not worth pursuing with the

cost of litigation. Asides these, some issues, especially business related ones, are highly sensitive

in nature thereby require some level of confidentiality which cannot be guaranteed in public

court.

In view of the foregoing, alternative means of settling dispute have been advocated from varied

quarters which are referred to as Alternative Dispute Resolution (ADR) methods. Okechukwu et

al. (2012) defined Alternative Dispute Resolution (ADR) as other “appropriate” pragmatic and

adaptive models of resolving disputes/conflicts apart from litigation. It is also referred to as any

method of resolving disputes that uses a consensus or interest-based model. It often includes any

process outside court systems, where disputing parties come to mutual agreement on a solution

with or without the assistance of a third party. The most common alternatives include arbitration,

mediation, negotiation, conciliation, bargaining, and hybrid processes, among others. The choice

of Alternative Dispute Resolution (ADR) model(s) to be employed depends on the peculiarities

of the disputes, subject-matter of the dispute, urgency, finance, consequences, methodology,

parties and relationship between the parties.

At this stage, it is important that brief discussion of some of this is done before going into the

focus of this section – arbitration.

Page 93: unitedemsa.files.wordpress.com...Pg 2 NIESV STUDY PACK - PPE KNOWLEDGE AND STRUCTURE OF NIGERIA INSTITUTION OF ESTATE SURVEYORS AND VALUERS Learning Objectives At the end of this lecture,

Pg 93

NIESV STUDY PACK - PPE

Arbitration: This is a dispute settlement model or technique in which the parties in dispute

appoint dispute settlement expert(s) referred to as arbitrator to hear the parties evidence and

decide the dispute for them, based on evidence weight(s) and concessions of the parties

(Okechukwu et al., 2012). It is normally used in commercial, labour or management dispute.

Mediation: Mediation is an ADR method where a neutral and impartial third party, the mediator,

is invited or allowed to facilitate dialogue in a structured multi-stage process to help parties reach

a conclusive and mutually satisfactory agreement. A mediator assists the parties in identifying

and articulating their own interests, priorities, needs and wishes to each other. It is different from

arbitration in the sense that it is a voluntary and non-binding process. It is a process of

negotiation with the assistance of a neutral third party, and the parties do not reach a resolution

unless all sides agree

Conciliation: This is simple a method of settling dispute in a friendly manner. Conciliation is a

process whereby the parties to a dispute use a conciliator, who meets with the parties both

separately and together in an attempt to resolve their differences. They do this by lowering

tensions, improving communications, interpreting issues, encouraging parties to explore potential

solutions and assisting parties in finding a mutually acceptable outcome. It is also non biding and

flexible in nature.

Bargaining: This refers to a process whereby two or more parties reach an accommodation

acceptable to those parties. It usually involves one or more of the parties undertaking to do or not

do certain things.

Page 94: unitedemsa.files.wordpress.com...Pg 2 NIESV STUDY PACK - PPE KNOWLEDGE AND STRUCTURE OF NIGERIA INSTITUTION OF ESTATE SURVEYORS AND VALUERS Learning Objectives At the end of this lecture,

Pg 94

NIESV STUDY PACK - PPE

ARBITRATION AS A DISPUTE RESOLUTION METHOD

DEFINITION

Arbitration is a technique for the resolution of disputes outside the court. The parties to a dispute

refer it to arbitration by one or more persons of their choice, and agree to be bound by the

arbitration decision (the "award"). A third party reviews the evidence in the case and imposes a

decision that is legally binding on both sides and enforceable in the courts. It is sometimes

referred to as „businessman‟s method of resolving disputes‟. Unlike litigation, arbitration takes

place out of court, the two sides select an impartial third party, known as an arbitrator (an

arbiter); agree in advance to comply with the arbitrator's award; and then participate in a hearing

at which both sides can present evidence and testimony. The arbitrator's decision is usually final,

and courts rarely re-examine it. Impartial person to whom the parties agreed to refer the dispute

is called the Arbiter or Arbitration Panel (in case it is a panel or persons).

Some of the attributes that distinguish arbitration from mediation and conciliation include the

fact that it is usually borne out of willingness of/and agreement between the parties involved

rather than coercion or imposition, and the decision of arbitrations are binding on parties.

Arbitration process is usually governed by the law of the state/country. For example, prominent

among arbitration laws in Britain were Arbitration Acts of 1889 and that of 1934 which were

latter harmonized into Arbitration Acts of 1950. In Nigeria, arbitration law is traceable to many

roots. Some of these include Law of Northern Nigeria Cap 16 of 1914, Cap 23 of 1943 and Cap

13 of 1948; Laws of Nigeria Cap 131 of 1954, Cap 47 of 1955, N.N.L.N Cap 186 of 1963, and

lately, Arbitration and Conciliation Acts Chapter 19, Laws of the Federal Republic of Nigeria

1990.

General principles of arbitration are as follows: The object of arbitration is to obtain a fair

resolution of disputes by an impartial third party without unnecessary expense or delay; Parties

should be free to agree how their disputes are resolved, subject only to such safeguards as are

necessary in the public interest; and Courts should not interfere.

Page 95: unitedemsa.files.wordpress.com...Pg 2 NIESV STUDY PACK - PPE KNOWLEDGE AND STRUCTURE OF NIGERIA INSTITUTION OF ESTATE SURVEYORS AND VALUERS Learning Objectives At the end of this lecture,

Pg 95

NIESV STUDY PACK - PPE

Canons/tenets for Arbitrators

The following are the guiding principles for an arbitrator (Udechukwu, 2008):

i. An arbitrator should uphold the integrity and fairness of the dispute resolution process

ii. An arbitrator should disclose any interest or relationship that affects impartiality or

creates an unfavourable appearance of partiality or bias

iii. In communicating with the parties, an arbitrator should avoid impropriety or the

appearance of impropriety

iv. An arbitrator should be honest and trustworthy and maintain confidentiality

v. An arbitrator should make decision in a just, independent and deliberate manner.

AWARD

Award is the nomenclature given to the decision of the arbitrator or arbitration tribunal which is

binding on the parties. It is similar to a judgment in a court of law. The term award does not

necessarily connote a monetary compensation. It is a „judgment‟ given by appointed arbitrator.

So, it also called an award even when none of the claimant‟s claims is granted and no money

needs to be paid by any party, or the penalty is of a non-monetary in nature.

ADVANTAGES AND DISADVANTAGES OF ARBITRATION

(A) Advantages

Some advantages of arbitration are as follows:

1. Arbitration is Faster

It is a common knowledge that court cases always take a very long time. This may be as a result

of long bureaucracy, the need to fulfil some provisions of the law, delay tactics by the lawyer of

seemingly losing side, and some other procedural activities of court proceedings. Arbitration has

advantages of these over the regular court cases as the time involved is, in most cases, relatively

shorter than court cases because it is devoid of most of the delay factors associated with public

court.

Page 96: unitedemsa.files.wordpress.com...Pg 2 NIESV STUDY PACK - PPE KNOWLEDGE AND STRUCTURE OF NIGERIA INSTITUTION OF ESTATE SURVEYORS AND VALUERS Learning Objectives At the end of this lecture,

Pg 96

NIESV STUDY PACK - PPE

2. It Allows for Specialized Arbiter

The expertise of judges in the courts of law is generally on substantive and procedural law. It

goes without saying that, at times, their knowledge of the law will be of little help when they are

called upon to rule on disputes that raise highly technical or scientific issues, or questions that

require a specialization other than the law.

By resorting to arbitration, parties are able to submit the settlement of their disputes to arbitrators

knowledgeable in the area of specialization which truly forms the basis of the dispute. For

example, a construction dispute could be submitted to engineers or architects; a dispute regarding

commercial leases could be resolved by a panel comprised of chartered appraisers; and a dispute

between shareholders could be settled by accountants and tax experts. It can therefore be said

that Arbitration allows for appointment of specialist as arbitrator thereby fosters a more

professional and unbiased judgement.

3. It Fosters Mutual Respect

In arbitration, parties‟ personality and opinions are respected. Unlike traditional legal

proceeding, parties are involved in decisions regarding every stage of arbitration process. Right

from the choice of arbitrator, the time, the location of hearing and so on. Arbitrators also know

that they are servants to the parties involved and therefore would not treat them with disdain.

This cannot be said of court of law.

4. It Is Less Technical and Convenience

It is obvious that one of the major factors responsible for prolonged cases in court of law is the

observance of technicalities of law and bureaucracy of litigation. In arbitration, formalities are

very low and limited to those that are germane to the case at hand. For example, in arbitration,

the location and time of hearing need not follow any law but by the decision of the parties which

definitely would be to their convenience. This reduces cost and time involved in conflict

resolution when arbitration option is considered.

5. It Is Flexibility

Page 97: unitedemsa.files.wordpress.com...Pg 2 NIESV STUDY PACK - PPE KNOWLEDGE AND STRUCTURE OF NIGERIA INSTITUTION OF ESTATE SURVEYORS AND VALUERS Learning Objectives At the end of this lecture,

Pg 97

NIESV STUDY PACK - PPE

Rigidity of the court system to the procedures prescribed by law is one of the disadvantages of

legal proceedings. There are ample rooms for flexibility in arbitration option. For example, the

often convoluted rules of evidence and procedure do not apply in arbitration proceedings --

making them less stilted and more easily adapted to the needs of those involved. Importantly,

arbitration dispenses with the procedure called discovery that involves taking and answering

interrogatories, depositions, and requests to produce documents -- often derided as a delaying

and game-playing tactic of litigation. In arbitrations, most matters, such as who will be called as

a witness and what documents must be produced, can be handled with a simple phone call.

6. It Respects Parties Confidentiality

Arbitration processes are private and confidential to the parties. For example, in judicial

proceedings before the common courts, barring certain exceptions, any member of general public

may be present at court hearings. Similarly, in principle, every document filed in the court record

is accessible to all, except in some restricted cases. The same is true of testimony and transcripts

thereof, whether these take place in court or out of court. This does not happen in arbitration as

those that will be present are limited to the parties involved and others that they agreed to be part

of it.

(B) Disadvantages

1. It Is Sometimes More Expensive

While it is established that, in most cases, arbitration is less expensive than litigation, it is

important to know that in some few occasions, arbitration can more expensive. For example, in

arbitration, the parties themselves assume the remuneration of the arbitrator(s) called upon to

rule on their dispute. The parties also have to assume other administrative cost associated to the

process, as well as the costs associated with the logistics of the arbitration (renting a room for the

hearing, videoconferences, etc.). Also, each party may want to hire one or more experts to help

in presenting and explaining the technical facets of its case in a way that is favourable to that

party. Depending on the nature case involved, all these may sometimes be more than the cost of

litigation. It is obvious that judicial sector is a public good running on public funding, therefore

Page 98: unitedemsa.files.wordpress.com...Pg 2 NIESV STUDY PACK - PPE KNOWLEDGE AND STRUCTURE OF NIGERIA INSTITUTION OF ESTATE SURVEYORS AND VALUERS Learning Objectives At the end of this lecture,

Pg 98

NIESV STUDY PACK - PPE

the financial contribution of litigants to a dispute heard before a court of law represents but a

minimal portion of the actual costs of that dispute.

2. It Sometimes Takes Longer Period

In certain occasions, especially when multiple arbitrators are involved, arbitration process can

take a longer time than court processes which are sometime accelerated in some peculiar cases.

3. Inability to Effect Restraining Measures Against Parties

In arbitration, parties find it easy to avoid enforcement of award against them because arbitrator

does not have power to order interlocutory measure unlike in court of law where judge can

enforce some other order/measures to restraining parties from circumventing the enforcement of

the judgement given.

4. Possibility of Influence on Arbitrators

Because the arbitrators are like employees to the disputants, if one of the parties has more

economic or political influence with tendency to give more brief to the arbitrator in future, there

is possibility that such party can have influence on the arbiter. The influence may be in form of

threat to lose future business/brief.

5. Parties Have Limited Recourse

A final decision is hard to change. If the arbitrator's award is unfair or illogical, a consumer may

well be stuck with it and barred forever from airing the underlying claim in court.

CLASSIFICATIONS OF ARBITRATION.

1. Commercial Arbitration is the most common of disputes. Just as it sounds, it is a dispute

between two commercial enterprises. This covers all disputes arising from commercial

transactions like construction contract, property management contract, sales contract etc.

Page 99: unitedemsa.files.wordpress.com...Pg 2 NIESV STUDY PACK - PPE KNOWLEDGE AND STRUCTURE OF NIGERIA INSTITUTION OF ESTATE SURVEYORS AND VALUERS Learning Objectives At the end of this lecture,

Pg 99

NIESV STUDY PACK - PPE

2. Consumer/Civil Arbitration: surrounds disputes between a consumer and a supplier of goods

or services. It also includes differences arising from social or civil relations such as trespass etc.

3. Labour/Industrial Arbitration: this involves the settlement of employment related disputes.

For example, dispute between organized labour and employers.

ARBITRATION PROCESS

As earlier mentioned, arbitration is normally governed by state and federal law. Most

states/countries have provisions in their civil practice rules for arbitration which provide basic

template for the arbitration as well as procedures for appointment and termination of arbitrator,

confirmation of an arbitrator‟s award (the document that gives and explains the decision of an

arbitrator), grounds for challenging an arbitrator, a procedure that gives an award the force and

effect, contents of arbitration agreement, among others. In Nigeria, the current law guiding

arbitration process is Arbitration and Conciliation Acts Chapter 19, Laws of the Federal Republic

of Nigeria 1990.

Appointment of Arbitrator or arbitration Tribunal

Arbitrators, or Tribunal members, are commonly appointed by one of three means:

1. Directly by the disputing parties (by mutual agreement, or by each party appointing one

arbitrator)

2. By existing tribunal members (For example, each side appoints one arbitrator and then the

arbitrators appoint a third)

3. By an external party (For example, the court or an individual or institution nominated by the

parties)

Arbitration agreement

This is the document which contains the details about what has been agreed by the parties to

serve as guiding rule the entire arbitration process. It represent contract document for arbitration.

It can form part of original document or form a separate agreement.

Page 100: unitedemsa.files.wordpress.com...Pg 2 NIESV STUDY PACK - PPE KNOWLEDGE AND STRUCTURE OF NIGERIA INSTITUTION OF ESTATE SURVEYORS AND VALUERS Learning Objectives At the end of this lecture,

Pg 100

NIESV STUDY PACK - PPE

Unless a contrary intention is expressed in the arbitration agreement, it shall be irrevocable

except:

i. By agreement of the parties;

ii. By leave of the court or a Judge;

iii. Even, the occurrence of death of either of the parties does not automatically revoke

the arbitration agreement. Rather, it shall be enforceable by or against personal

representatives of the deceased.

Typical Steps in Arbitration

The process of arbitration differs among cases. The following is a list of the main steps in

arbitration; however it should not be viewed as an exhaustive list.

Initiating the Arbitration – A request by one party for a dispute to be referred to arbitration.

Appointment of Arbitrator – Arbitrators may be appointed by one of three ways: (1) Directly

by the disputing parties, (2) By existing tribunal members (For example, each, each side appoints

one arbitrator and then the arbitrators appoint a third), (3) By an external party (For example, the

court or an individual or institution nominated by the parties).

Preliminary Meeting – It is a good idea to have a meeting between the arbitrator and the parties,

along with their legal counsel (if any), to look over the dispute in question and discuss an

appropriate process and timetable.

Statement of Claim and Response – The claimant sets out a summary of the matters in dispute

and the remedy sought in a statement of claim. This is needed to inform the respondent of what

needs to be answered. It summarizes the alleged facts, but does not include the evidence through

which facts are to be proved. The statement of response from the respondent is to admit or deny

the claims. There may also be a counterclaim by the respondent, which in turn requires a reply

from the claimant. These statements are called the „pleadings‟. Their purpose is to identify the

issues and avoid surprises.

Page 101: unitedemsa.files.wordpress.com...Pg 2 NIESV STUDY PACK - PPE KNOWLEDGE AND STRUCTURE OF NIGERIA INSTITUTION OF ESTATE SURVEYORS AND VALUERS Learning Objectives At the end of this lecture,

Pg 101

NIESV STUDY PACK - PPE

Discovery and Inspection – These are legal procedures through which the parties investigate

background information. Each party is required to list all relevant documents, which are in their

control. This is called „discovery‟. Parties then „inspect‟ the discovered documents and an agreed

upon selection of documents are prepared for the arbitrator.

Interchange of Evidence – The written evidence is exchanged and given to the arbitrator for

review prior to the hearing.

Hearing – The hearing is a meeting in which the arbitrator listens to any oral statements,

questioning of witnesses and can ask for clarification of any information. Both parties are

entitled to put forward their case and be present while the other side states theirs. A hearing may

be avoided however, if the issues can be dealt with entirely from the documents.

Legal Submissions – The lawyers of both parties provide the arbitrator with a summary of their

evidence and applicable laws. These submissions are made either orally at the hearing, or put in

writing as soon as the hearing ends.

Award – The arbitrator considers all the information and makes a decision. An award is written

to summarize the proceedings and give the decisions. The award usually includes the arbitrator‟s

reasons for the decision

Page 102: unitedemsa.files.wordpress.com...Pg 2 NIESV STUDY PACK - PPE KNOWLEDGE AND STRUCTURE OF NIGERIA INSTITUTION OF ESTATE SURVEYORS AND VALUERS Learning Objectives At the end of this lecture,

Pg 102

NIESV STUDY PACK - PPE

RELEVANT PRACTICE LAWS AFFECTING REAL ESTATE PRACTICE IN

NIGERIA

Introduction

This section provides details about the various laws and legislations that regulate the operations

of land uses in Nigeria. The laws were enacted to promote health, harmony, safety and peaceful

coexistence of users as well as ensuring and enforcing standards of buildings.

At the end of this chapter, candidates are supposed to understand the meaning of the various

laws, their implications and likewise the challenges they have posed in the day to day

applications.

The legislations are as follows:

1. Land Use act

2. National Housing Policy

3. Rent Control Laws

4. Capital gain Tax

5. Eminent Domain

6. Police Power

7. Zoning Ordinance

8. Value Added Tax

1.0 LAND USE ACT NO 6 OF 1978

The promulgation of Land Use Act (formerly known as Decree) came into existence on the 29th

of March 1978 with the principal aim of regulating the use of land within the country.

Provisions of The Act

Essential Provisions of the Land Use Act are as follows:

Page 103: unitedemsa.files.wordpress.com...Pg 2 NIESV STUDY PACK - PPE KNOWLEDGE AND STRUCTURE OF NIGERIA INSTITUTION OF ESTATE SURVEYORS AND VALUERS Learning Objectives At the end of this lecture,

Pg 103

NIESV STUDY PACK - PPE

1. Vesting all Land in the State Military Governor

According to section one of the Act, “subject to the provisions of this act, all land comprised in

the territory of each state in the Federation are hereby vested in the Military Governor of that

state and such land shall be held in trust and administered for the use and common benefit of all

Nigerians in accordance with the provisions of this Act”.

2. Control and Management of Land

Section 2 (1) states that “all land in urban areas shall be under the control and management of the

Military Governor of each state” while land in the rural areas shall be under the control and

management of the local government.

3. Powers of the Military Governor in Relation to Land

According to Section 5(1) “it shall be lawful for the Military Governor in respect of land whether

or not in an urban area.

(a) To grant statutory rights of occupancy to any person for all purposes.

(b) To grant easements appurtment to statutory rights of occupancy.

(c) To demand rental for any such land granted to any.

(d) To revise the said rental at such intervals as may be specified in the certificate of

occupancy”.

4. Powers of Local Government in Relation to Rural Land

Section 6 (1) of the Act states as follows:

It shall be lawful for a Local Government in respect of land not in an urban area:

(a) To grant customary rights of occupancy to any person or organisation for the use of land

in the local government area for agricultural, residential and other purposes.

(b) To grant customary rights of occupancy to any person or organisation for the use of land

for grazing purposes and such other purposes ancillary to agricultural purposes as may be

customary in the Local Government area concerned.

Page 104: unitedemsa.files.wordpress.com...Pg 2 NIESV STUDY PACK - PPE KNOWLEDGE AND STRUCTURE OF NIGERIA INSTITUTION OF ESTATE SURVEYORS AND VALUERS Learning Objectives At the end of this lecture,

Pg 104

NIESV STUDY PACK - PPE

Section 6 (2) states inter alia: No single customary right of occupancy shall be granted in respect

of an area of land in excess of 500 hectares if granted for agricultural purposes, or 5,000 hectares

if granted for grazing purposes, except with the consent of the Military Governor.

5. Certificate of Occupancy

Section 9 (1) states that: It shall be lawful for the Military Governor:

(a) When granting a statutory right of occupancy to any person or

(b) When any person is in occupation of land under a customary right of occupancy and

applied in the prescribed manner, or

(c) When a person is entitled to a statutory right of occupancy to issue a certificate under his

hand in evidence of such right of occupancy.

Section 9 (2) states that: such certificate shall be termed a certificate of occupancy and there

shall be paid therefore as may be prescribed.

6. Prohibition of Alienation

Section 21 provides that: it shall not be lawful for any customary right of occupancy or

any part thereof to be eliminated by assignment, mortgage, transfer of possession,

sublease or otherwise however.

(a) Without the consent of the Military Governor in cases where the property is to be sold

by or under the order of any court under the provisions of the applicable Sheriffs and

civil process law.

(b) In other cases without the approval of the appropriate local government.

7. Governor‟s Consent

Section 22 provides that: It shall not be lawful for the holder of a statutory right of occupancy

granted by the Military Governor to alienate his right of occupancy or any part thereof by

assignment, mortgage, transfer of possession, sublease of otherwise whosoever without the

consent of the Military Governor first had and obtained.

8. Revocation of Right of Occupancy

Page 105: unitedemsa.files.wordpress.com...Pg 2 NIESV STUDY PACK - PPE KNOWLEDGE AND STRUCTURE OF NIGERIA INSTITUTION OF ESTATE SURVEYORS AND VALUERS Learning Objectives At the end of this lecture,

Pg 105

NIESV STUDY PACK - PPE

By virtue of the provision of section 28 of the Act, the Military governor has power to revoke

a right of occupancy for overriding public interest. Overriding public interest includes:

(a) Requirement of the land by Government for public purposes.

(b) Alienation of the land by the occupier contrary to the provisions of the Act.

(c) Requirement of the land for mining purposes or pipelines or for any purpose connected

therewith.

9. Payment of Compensation

Section 29 stipulates that if a right of occupancy is revoked due to any of the reasons stated

in section 28, the holder and the occupier shall be entitled to compensation for the above

revocation shall be, as respects-

(a) The land, for an amount equal to the rent, if any, paid by the occupier during the year in

which the right of occupancy was revoked.

(b) Building installation or improvements thereon for the amount of the replacement cost of

the building, installation or improvement, that is to say, such cost as may be assessed on

the basis of the prescribed method of assessment as determined by the appropriate officer

less any depreciation, together with interest of the bank rate for delayed payment of

compensation and in respect of any improvement in the nature of reclamation works,

being such cool thereof as may be substantiated by documentary evidence and proof to

the satisfaction of the appropriate officer.

(c) Crops on land apart from any building, installation or improvement thereon, for an

amount equal to the value as prescribed and determined by the appropriate officer.

(d) Interest charged for the period of delay in payment but charged at current bank rate

The Act is however silent on disturbance, severance and injurious affection.

Objectives of the Land Use Act

Compulsory Acquisition by Government

Page 106: unitedemsa.files.wordpress.com...Pg 2 NIESV STUDY PACK - PPE KNOWLEDGE AND STRUCTURE OF NIGERIA INSTITUTION OF ESTATE SURVEYORS AND VALUERS Learning Objectives At the end of this lecture,

Pg 106

NIESV STUDY PACK - PPE

The difficulties which Government experienced in the acquiring land before the

enactment of the Act were thought capable of solution through the enactment of the Act. For it

ownership and allocation of land became vested in the Government, then simple logic dictated

that Government should find no difficulties whatsoever in making use of whatever piece of land

it deemed fit for public purposes.

To Curb Land Speculation

The Act was expected to curb speculation in land values especially in urban areas. It was

believed that once Government was vested with the ownership and disposition of land,

speculators would find alternative outlets for their capital and entrepreneurial ability and this

would contribute to the stabilization of the cost of Government projects especially in urban areas

of the country.

To Unify the Tenurial Systems

As pointed out earlier, there was dualism of tenures in the south while in the north, the Land

Tenure Law of 1962 was in force. In most parts of the south, there was evidence that this

situation gave rise to endless litigations which, however beneficial to the legal profession,

constituted a drag on economic development. There were cases where plans for the location of

some industries were aborted as a result of endless litigation. Siting of some infrastructural

projects such as hospitals, schools, ports, etc. were also aborted due to endless litigations. These

problems were expected to be drastically reduced if not completely eliminated once a systematic

arrangement regarding ownership and distribution of land could be put into effect.

To Encourage Agricultural Modernization

The different tenure arrangements that existed in Nigeria before the enactment of the Act

imposed impediments on agricultural modernization in many respects: The absence of well

defined titles especially in the rural areas effectively prevented the use of land as collaterals for

bank loans for agricultural investments; because of the communal nature of land ownership, the

erection of permanent structures was not universally welcomed. This also largely discouraged

the cultivation of economic trees with long gestation period. Such oil palm, cocoa, colanuts and

coffee since the right to land was vested rather in the community and not in the individual, it was

Page 107: unitedemsa.files.wordpress.com...Pg 2 NIESV STUDY PACK - PPE KNOWLEDGE AND STRUCTURE OF NIGERIA INSTITUTION OF ESTATE SURVEYORS AND VALUERS Learning Objectives At the end of this lecture,

Pg 107

NIESV STUDY PACK - PPE

believed that the Act will remove once and for all, the shackles to economic development

imposed by the archaic and anachronistic tenure arrangement.

To make Land Available to All

To make land available for all individuals, institutions and even government. Before the act came

into force, land was only available to the rich and indigenes of a particular place at the detriment

of the poor. The Act intended to make land available to both the poor and the rich in the country.

The reason for enacting the Act according to the then head of state, was due to „‟lack of

immediate availability of land for use when required by all concerned‟‟.

To Remove Unhealthy Rivalry and Litigations

As a result of a sharp increase in population, there was a corresponding increase in the demand

to acquire land leading to unhealthy rivalry among citizens and eventually culminating into much

litigation in our courts. It was hoped that the land use act would remove such unhealthy rivalries

and litigations.

The Federal Military Government realized the existence of litigations before the act was

enacted and hence stated in the report of the land use panel that „‟while Government does not

intend to dispossess any Nigerians of his acquires property, which is being lawfully used, the

government cannot remain indifferent to the problem of land litigation which is widely

acknowledged as an obstacle to rapid national development. A situation whereby, public

institutions and individuals encounter problems in acquiring land for development should no

longer be allowed to continue in the overall public interest‟‟.

To Protect People‟s Right in Land

One of the objectives of the Land Use Act of 1978 was to preserve and protect the rights

of all Nigerians to use and enjoy land and the natural fruits growing thereof not only for

themselves but for the sustenance of all members of their families. The Act planned to remove

the general insecurity of title that existed in Nigeria before its enactment.

To Control Development

Page 108: unitedemsa.files.wordpress.com...Pg 2 NIESV STUDY PACK - PPE KNOWLEDGE AND STRUCTURE OF NIGERIA INSTITUTION OF ESTATE SURVEYORS AND VALUERS Learning Objectives At the end of this lecture,

Pg 108

NIESV STUDY PACK - PPE

Although the Act encourages universal development throughout the country, it however,

intended to regulate and control development so as to avoid the springing up of slums; avoid

over-spill in cities through the provision of basic social amenities in the rural areas; and to have

uniform development in both urban and rural areas.

To solve Inflationary Problems

The non-availability of land and its usage have been noted to be the cause of the prevailing

inflation in the country and its attendant problems like shortage of food and other essentials. The

Act intended to have even distribution (by way of allocation and issuance of certificate of

occupancy) of land for development by residential and other users. It also planned to guarantee

permanent farming tenancy, a right that is no longer easily destroyed by claims that have roots in

customary law. With price of accommodation lowered and farm products flooding the market

inflation are bound to be reduced drastically keeping the cost of living at a reasonable level. The

Act planned to guarantee all these.

Problems Of The Land Use Act

1. The Act has pronounced itself on outlaw as seen in Section 46(1) by claiming to be

superior to the constitution of the country. This means that even if any illegality is

committed under the Act, the constitution will have no right via the courts of law to

challenge that illegality.

2. It is essentially an urban legislation which only superficially touches the tenure problems

in the rural sector. This is evident in Section 36(2) of the Act which means that the

communal overlords in whom ownership rights were vested before the commencement of

the Act are still being recognized owners of rural land. To this end, the tenure problems

which plagued rural land before the advent of the Act are still existing in our rural areas

even up till now.

.

(3). It has not eliminated speculation in land, it has only driven it underground or even fuelled it.

Page 109: unitedemsa.files.wordpress.com...Pg 2 NIESV STUDY PACK - PPE KNOWLEDGE AND STRUCTURE OF NIGERIA INSTITUTION OF ESTATE SURVEYORS AND VALUERS Learning Objectives At the end of this lecture,

Pg 109

NIESV STUDY PACK - PPE

(4) The insecurity of title to land that characterized the existing tenure before the Land Use Act

came into force still exists will date despite the enactment of the Act, for example, a certificate of

occupancy can be revoked at anytime by the Governor for public purpose or a contravention of

the Act. It is not also clear in a situation where a bank accepts certificate of occupancy from a

prospective farmer and grants loan in return, if such certificate is revoked who gets the

compensation, the bank or land holder? A Governor can revoke a certificate of occupancy in bad

faith at anytime but such revocation is usually on the pretence of public purpose, which shows

the level of insecurity or uncertainty of our title to land under the Act which it had earlier

intended to check.

(5). The Act has further compounded the problems arising from unhealthy rivalry among citizens

in their bid to acquire land thereby resulting to endless litigations.

(6). The Act has created artificial scarcity of land as previous land owners have not allowed have

not allowed the Government to enter their lands to survey. Plan and allocate land applicants is

stipulated in the Act.

(7). The rich and the influential are still the set of people that can get both State and Local

Government grants in land at the detriment of the

Conclusion

Thirty eight years after its adoption, the land use law of 1979 does not appear to have met its

objectives while it also seems to have had many unintended consequences. Therefore, there have

been many agitations calling for the review of the Act.

This review is needed because presently Nigeria is a Federal system comprising of 36 states and

the federal capital territory, each of which function semi-autonomously. This has made the

implementation of the Land Law to vary significantly across the country

2.0 NATIONAL HOUSING POLICY

Introduction

The National Housing Policy was first launched in February, 1991 by the Federal

Government with the intention of finding a lasting solution to the myriads of problems

Page 110: unitedemsa.files.wordpress.com...Pg 2 NIESV STUDY PACK - PPE KNOWLEDGE AND STRUCTURE OF NIGERIA INSTITUTION OF ESTATE SURVEYORS AND VALUERS Learning Objectives At the end of this lecture,

Pg 110

NIESV STUDY PACK - PPE

created by housing needs in Nigeria. The major issues envisaged to be addressed by this

policy include the following:-

- The Restructuring of the Federal Mortgage Bank of Nigeria (FMBN) to serve as the

apex housing finance institution.

- Mobilising of savings through the establishment of a National Housing Fund (NHF)

- Facilitating continual flow of funds from various sources into the apex institution to

aid a smooth lending practices to primary mortgage institutions for direct lending to

individuals.

- Provision for the inclusion of the participation and partnership of the private sector

was especially the hallmark of the reviewed National Housing Policy of 2006.

3.0 RENT CONTROL LAWS

The purpose of the rent control law is to regulate the cost of accommodation in order to make it

affordable by a tenant. The law endeavors to ensure a reduction in housing rental charges at a

rate virtually below market value. The law specifies the maximum value of rent in specific

locations of the city and likewise indicates the mode of any increment to be proposed by housing

owners.

Advantages of Rent Control Laws

1. Reduced / cheap housing cost especially for the poor – The goal of rent control is to

keep the rents low for tenants. This means that tenants have an assured security against

arbitrary increase in rents as well as evictions. This represents a good protection for the

most economically backward categories of the Society (I.e the elderly and the poor).

2. Housing as a fundamental Human right- Rent control is adjudged as one of the basic

way of protecting fundamental right which is access to a decent accommodation by an

average citizen. Rent control provides stability to school-going children and community

organisations.

Disadvantages Of Rent Control

Page 111: unitedemsa.files.wordpress.com...Pg 2 NIESV STUDY PACK - PPE KNOWLEDGE AND STRUCTURE OF NIGERIA INSTITUTION OF ESTATE SURVEYORS AND VALUERS Learning Objectives At the end of this lecture,

Pg 111

NIESV STUDY PACK - PPE

1. Rent control increases housing scarcity – Wherever rents are fixed as against what the

market dictates, landlords would be discouraged to let apartments. This leads to artificial

scarcity which in the long run will not be beneficial.

2. Reduced quality of Housing – Property investors who build in an atmosphere of rent

control will pursue the course of minimizing the cost by all means so as to maximise

profit. This will result into the use of low quality building materials.

3. Rent control is not a pro-poor legislation, but at the detriment of the tenants in the long

run. This means that rent control does not benefits renters as the remain in low cost

housing without any motivation to pursue the development of their own property. Rent

control may keep them as renters for life.

4. Rent Control discourages investors- A reasonable prospective investor will be inhibited

from throwing their resources into property investment in the free of rent control. They

quickly opt for a more profit friendly investment where resources will be protected and

that will bring more profit.

5. Wasting of space- Cheaper apartments make the poor and even few people to hold up

much larger portions of accommodation than they really need, since they pay top

ridiculously low rent, they will not mind holding on to extra space that are not needed.

This prevent extra property to come into the market

6. Rent control causes slum- Investors are discouraged with the policy of ceiling rents.

Therefore rent control kills the incentive to invest in new rental housing. The rent control

reduces supply in the market and increasing the competition for the available properties,

leading to scarcity of rental homes. The resultant is that more number of people tend to

share limited spaces thereby leading to slums.

7. Curtailing the growth of the Cities (Urbanization- As rent control creates incentives

for renters and disincentives to the owners of the property. Investors are discouraged as

they may not be able to guarantee the safety of their investment.

Furthermore the less-interested investors disallow a smooth growth of the cities, therby

hampering urbanisation.

8. Rent Control leads to over stay by Tenants- The incentive of lower rent used to keep

tenants in apartments for too long a period thereby forcing the owner to dispose the

Page 112: unitedemsa.files.wordpress.com...Pg 2 NIESV STUDY PACK - PPE KNOWLEDGE AND STRUCTURE OF NIGERIA INSTITUTION OF ESTATE SURVEYORS AND VALUERS Learning Objectives At the end of this lecture,

Pg 112

NIESV STUDY PACK - PPE

subject property to the sitting tenant at ridiculously low prices. Later, there tenants could

turn around to sell the same property at market prices to others as owners

9. Reduction in chances of Labour Unemployment- Due to reduction in investment in

real estate, labour employment rate will decline too. Construction activities is one of the

major Labour-intensive industry and can generate lots of work opportunities

4.0 CAPITAL GAINS TAX

One of the forms of property-based taxes is referred to as Capital Gain Tax (C.G.T). While the

capital Gain is the profit or windfall which speculators or other property dealers realize from the

sale of real estates or other landed properties, Capital Gain Tax is the compulsory deduction from

the profit (windfalls) which the speculators or other dealers realize from the sale of real estates or

landed property. It is payable by the person making the disposal. The gain or profit (i.e the

difference between the price paid for acquiring the asset and the price you eventually sold it) is

considered to be a taxable income.

The Capital Gains Tax decree of 1967 is the major legislation on capital gains tax in Nigeria. The

law then first applied to the Federal Capital Territory of Lagos. By the finance (miscellaneous)

Taxation provision decree of 1967, the CGT tax Decree became retrospectively applicable

throughout Nigeria in 1975.

The calculations of the Capital gains tax takes into consideration other incidental expenses

incurred in the realization of sale such as marketing charges, agency commission etc. any

„‟capital loss‟‟ is not susceptible to capital Gains deduction

5.0 EMINENT DOMAIN

Eminent domain is the right of the Government to acquire privately owned real estate for public

use. The proposed use must be for the common benefit of the masses. When such rights is

exercised, just compensation must be provided for the owner. The right of the property owner

must be protected by due process of law.

Public use has been defined very broadly by the court to include not only public facilities but

also property that is no longer fit for use and must be used or destroyed. The power of eminent

Page 113: unitedemsa.files.wordpress.com...Pg 2 NIESV STUDY PACK - PPE KNOWLEDGE AND STRUCTURE OF NIGERIA INSTITUTION OF ESTATE SURVEYORS AND VALUERS Learning Objectives At the end of this lecture,

Pg 113

NIESV STUDY PACK - PPE

domain is commonly delegated by the state to public bodies and agencies e.g state housing

corporation.

6.0 POLICE POWER

This refers to Government powers. Every state has the power to enact laws and regulations to

protect the public health, preserve order and safety and promote the general welfare of the

citizens. This authority is referred to as a state police power.

Police powers can be exercised in the following forms:

a. Environmental protection law

b. Zoning ordinances

c. Building regulations and codes.

7.0 Zoning

Zoning principally connotes the division of land into districts with restricted land uses. Such

restrictions are the use of land and buildings, density of the population, the heights of the

buildings, the sizes of land (meter square) etc.

Benefits of Zoning Laws

1. It affects the values of properties positively.

2. It restricts the types of use to which a site can be put.

3. It helps to preserves the natural characters and features of the site.

4. It promotes the congregation of compatible land uses that fosters harmony of uses as

against congestion of uses

5. It is a good measure to regulate housing/ population density.

6. It helps government in making forward planning and provision of infrastructure required

respective zones

Major types of Zoning Regulations

1. Use – induced Regulation e. g Residential, Industrial, Commercial, religions, recreational

etc.

2. Height-induced Regulation for buildings especially in areas of flight routes e. g Airport

neighborhoods.

Page 114: unitedemsa.files.wordpress.com...Pg 2 NIESV STUDY PACK - PPE KNOWLEDGE AND STRUCTURE OF NIGERIA INSTITUTION OF ESTATE SURVEYORS AND VALUERS Learning Objectives At the end of this lecture,

Pg 114

NIESV STUDY PACK - PPE

3. Size-induced Regulation such as building area and population density requirements.

8.0 VALUE ADDED TAX (VAT)

This is also known as goods and services tax (GST). It is consumption tax added on to purchases

for goods and services. It is collected at every stage along the production chain. The law came

into existence through the VAT Act No 102 of 1993, but its implementation actually began in

January 1994. Since its introduction in Nigeria 15 out of 42 sections of the Act was amended.

The categories of goods VAT must be applied include: Banks, food items, medical and

pharmaceutical products, books, news papers, magazines, house rent, commercial vehicles

Value Added tax conforms to the provisions of the sacrifice theory of taxation. The sacrifice

theory premised that an individual should pay tax on that portion of income that is spent on

luxuries. The sacrifice is in respect of individual‟s means over and above substance.

Value Added Tax is charged by all suppliers of goods and services in accordance with the

provisions of value Added Tax Act. The levy touches every individual as long as he or she

purchases some products or services.

References

Butler, S. (2012) Nigerian Land Markets and the Land Use Law of 1978. A paper in focus on

Land in Africa. Placing Land Rights at the Heart of Development. October.

Udechukwu, C.E. (2008). Professional Practice for Real Estate Practitioners. TREEM Nigeria

Limited, Banja Lagos.

Ogunba O.A. (2013) Principles and Practice of Property Valuation in Nigeria. Atlantis

Books, University of Ibadan Post Office, Ibadan, Nigeria.

Ogbuefi, J. (2004). Comparative Property Rating and Taxation: Institute for Development

Studies, University of Nigeria, Enugu Campus, Nigeria.

Page 115: unitedemsa.files.wordpress.com...Pg 2 NIESV STUDY PACK - PPE KNOWLEDGE AND STRUCTURE OF NIGERIA INSTITUTION OF ESTATE SURVEYORS AND VALUERS Learning Objectives At the end of this lecture,

Pg 115

NIESV STUDY PACK - PPE

VALUATION ANALYSIS PROCESS AND MARKET ANALYSIS

PURPOSE

This section introduces property valuation and its process together with the nature of property

market analysis (starting with its evolution within the property economics field and the current

market change factors). With prevalent inaccuracies in property forecasts, it is necessary to

improve upon the existing analysis processes also.

Basic Property Valuation Concepts

Value

“A main consideration in appraising is to determine a property's value: the present worth of

future benefits arising from the ownership of real property. Unlike many consumer goods that

are quickly used, the benefits of real property are generally realized over a long period of time.

Therefore, an estimate of a property's value must take into consideration economic and social

trends, as well as governmental controls or regulations and environmental conditions that may

influence the four elements of value:

Demand - the desire or need for ownership supported by the financial means to satisfy the

desire;

Utility - the ability to satisfy future owners' desires and needs;

Scarcity - the finite supply of competing properties and

Transferability - the ease with which ownership rights are transferred.

Value versus Cost and Price

Value is not necessarily equal to cost or price. Cost refers to actual expenditures; for example,

materials and labor. Price, on the other hand, is the amount that someone pays for something.

While cost and price can affect value, they do not determine value. The sales price of a house

might be N15, 000,000, but the value could be significantly higher or lower. For instance, if a

new owner finds a serious flaw in the house, such as a faulty foundation, the value of the house

could be lower than the price.

Page 116: unitedemsa.files.wordpress.com...Pg 2 NIESV STUDY PACK - PPE KNOWLEDGE AND STRUCTURE OF NIGERIA INSTITUTION OF ESTATE SURVEYORS AND VALUERS Learning Objectives At the end of this lecture,

Pg 116

NIESV STUDY PACK - PPE

Market Value

An appraisal is an opinion or estimate regarding the value of a particular property as of a specific

date. Appraisal reports are used by businesses, government agencies, individuals, investors and

mortgage lenders when making important decisions regarding real estate transactions. The goal

of an appraisal is to determine a property's market value: the most probable price that the

property will bring in a competitive and open market. Market price, the price at which a property

actually sells, may not always represent the market value. For example, if a seller is under duress

because of the threat of foreclosure, or if the property was sold in a private sale without being

exposed to the open market, the property may sell below its market value.

Property Valuation Methods

An accurate appraisal depends on the methodical collection of data. Specific data, covering

details regarding the particular property, and general data, pertaining to the nation, region, city

and neighborhood wherein the property is located, are collected and analyzed to arrive at a value.

Three basic and primary approaches are used during this process to determine a property's value.

Method 1 - Sales Comparison Approach

The sales comparison approach is commonly used in valuing single-family homes and land.

Sometimes called the market data approach, it is an estimate of value derived by comparing a

property with recently sold properties with similar characteristics. These similar properties are

referred to as comparables, and in order to provide a valid comparison, each must:

Be as similar to the subject property as possible;

Have been sold within the last year in an open and competitive market and

Have been sold under typical market conditions.

Comparables

Comparables should be as similar as possible to the subject property, and at least three or four

should be used in the appraisal process. The most important factors to consider when selecting

comparables are the size and the location of the subject and the comparable properties. The

Page 117: unitedemsa.files.wordpress.com...Pg 2 NIESV STUDY PACK - PPE KNOWLEDGE AND STRUCTURE OF NIGERIA INSTITUTION OF ESTATE SURVEYORS AND VALUERS Learning Objectives At the end of this lecture,

Pg 117

NIESV STUDY PACK - PPE

location is extremely important because it can have a tremendous effect on a property's market

value.

Adjustments

Since no two properties are exactly alike, adjustments to the comparables' sales prices will be

made to account for dissimilar features and other factors that would affect value, including:

Age and condition of buildings;

Date of sale, if economic changes occur between the date of sale of a comparable and the

date of the appraisal;

Location, since similar properties might differ in price from neighborhood to

neighborhood;

Physical features, including lot size, landscaping, type and quality of construction,

number and type of rooms, square feet of living space and whether or not a property has

hardwood floors, a garage, kitchen upgrades, a fireplace, a pool, central air, etc. and

Terms and conditions of sale, such as if a property's seller was under duress or if a

property was sold between relatives (at a discounted price).

The market value estimate of the subject property will fall within the range formed by the

adjusted sales prices of the comparables. Since some of the adjustments made to the sales prices

of the comparables will be more subjective than others, weighted consideration is typically given

to those comparables that had the least amount of adjustment.

Method 2 - Cost Approach

The cost approach can be used to estimate the value of properties that have been improved by

one or more buildings. This method involves separate estimates of value for the building(s) and

the land, taking into consideration depreciation. The estimates are added together to calculate the

value for the entire improved property. The cost approach makes the assumption that a

reasonable buyer would not pay more for an existing improved property than it would cost to buy

a comparable lot and construct a building that is comparable in terms of desirability and

usefulness. This approach is useful when the property being appraised is a type of property that

Page 118: unitedemsa.files.wordpress.com...Pg 2 NIESV STUDY PACK - PPE KNOWLEDGE AND STRUCTURE OF NIGERIA INSTITUTION OF ESTATE SURVEYORS AND VALUERS Learning Objectives At the end of this lecture,

Pg 118

NIESV STUDY PACK - PPE

is not frequently sold and is not an income-producing property. Examples include schools,

churches, hospitals and government buildings.

Building Costs

Building costs can be estimated in several ways, including the square-foot method where the cost

per square foot of a recently built comparable is multiplied by the number of square feet in the

subject building; the unit-in-place method where costs are estimated based on the construction

cost per unit of measure of the individual building components, including labor and materials

and the quantity-survey method which estimates the quantities of raw materials that will be

needed to replace the subject building, along with the current price of the materials and

associated installation costs.

Depreciation

For appraisal purposes, depreciation refers to any condition that negatively affects the value of

an improvement to real property, and takes into consideration:

Physical deterioration, including curable deterioration, such as painting and roof

replacement and incurable deterioration, such as structural problems;

Functional obsolescence, which refers to physical or design features that are no longer

considered desirable by property owners, such as low ceilings, outdated fixtures or homes

with four bedrooms but only one bath and

Economic obsolescence, caused by factors that are external to the property, such as being

located close to a noisy airport or polluting factory.

The cost approach for real estate valuation involves five basic steps:

Estimate the value of the land as if it were vacant and available to be put to its highest

and best use, using the sales comparison approach since land cannot be depreciated.

Estimate the current cost of constructing the building(s) and site improvements.

Estimate the amount of depreciation of the improvements resulting from deterioration,

functional obsolescence or economic obsolescence.

Deduct the depreciation from the estimated construction costs.

Page 119: unitedemsa.files.wordpress.com...Pg 2 NIESV STUDY PACK - PPE KNOWLEDGE AND STRUCTURE OF NIGERIA INSTITUTION OF ESTATE SURVEYORS AND VALUERS Learning Objectives At the end of this lecture,

Pg 119

NIESV STUDY PACK - PPE

Add the estimated value of the land to the depreciated cost of the building(s) and site

improvements to determine the total property value.

Method 3 - Income Capitalization Approach

The income approach is the third method of real estate valuation, and is based on the relationship

between the rate of return an investor requires and the net income that a property produces. It is

used to estimate the value of income-producing properties such as apartment complexes, office

buildings and shopping centers. Property valuation using the income capitalization approach can

be fairly straightforward when the subject property can be expected to have a future income, and

when its expenses are predictable and steady.

Direct Capitalization

Property Valuers will perform the following steps when using the direct capitalization approach:

Estimate the annual potential gross income;

Take into consideration vacancy and rent collection losses to determine the effective

gross income;

Deduct annual operating expenses to calculate the annual net operating income;

Estimate the price that a typical investor would pay for the income produced by the

particular type and class of property. This is accomplished by estimating the rate of

return, or capitalization rate and

Apply the capitalization rate to the property's annual net operating income to form an

estimate of the property's value.

Gross Income Multipliers

The gross income multiplier (GIM) method can be used to appraise other properties that are

typically not purchased as income properties but that could be rented, such as one- and two-

family homes. The GRM method relates the sales price of a property to its expected rental

income. For residential properties, the gross monthly income is typically used; for commercial

Page 120: unitedemsa.files.wordpress.com...Pg 2 NIESV STUDY PACK - PPE KNOWLEDGE AND STRUCTURE OF NIGERIA INSTITUTION OF ESTATE SURVEYORS AND VALUERS Learning Objectives At the end of this lecture,

Pg 120

NIESV STUDY PACK - PPE

and industrial properties, the gross annual income would be used. The gross income multiplier

method can be calculated as follows:

Sales Price / Rental Income = Gross Income Multiplier

Recent sales and rental data from at least three similar properties can be used to establish an

accurate GIM. The GIM can then be applied to the estimated fair market rental of the subject

property to determine its market value, which can be calculated as follows:

Rental Income X GIM = Estimated Market Value” (finance.yahoo.com).

The valuation process is accomplished by following specific steps, the number of which depends

on the nature of the valuation assignment and the data available to complete it. In all cases,

however, the valuation process provides the model to be followed

in performing market research and data analysis, in applying appraisal techniques and in

integrating the results of these analytic activities into an opinion of value.

Identification of the Problem

The first step in the valuation process is to identify the problem. This sets the parameters of the

assignment and eliminates any ambiguity about the nature of the assignment. In this step the

estate surveyor and valuer identifies the client and intended users of the valuation exercise, the

intended use of the appraisal, the purpose of the assignment, the effective date of the opinion, the

relevant property characteristics and the assignment conditions (extraordinary assumptions or

hypothetical conditions).

Scope of Work Determination

The scope of work is the amount and type of information researched and the analyses applied in

an assignment. After the problem to be solved is clearly identified, the appraiser must next

determine the appropriate scope of work to solve the problem. The scope of work must be clearly

disclosed in the valuation report.

Page 121: unitedemsa.files.wordpress.com...Pg 2 NIESV STUDY PACK - PPE KNOWLEDGE AND STRUCTURE OF NIGERIA INSTITUTION OF ESTATE SURVEYORS AND VALUERS Learning Objectives At the end of this lecture,

Pg 121

NIESV STUDY PACK - PPE

Data Collection and Property Description

In this step the valuer gathers general data on the market area and specific data on the subject and

comparable properties. The appraiser collects general data related to property values in an area to

understand the economic climate in which properties compete and the interacting forces that

cause values to increase, decrease or remain stable. Specific data are details about the property

being appraised (the subject property) and comparable properties that

have been sold or leased in the local market. Land and building descriptions are specific data that

help an appraiser to select comparable sales and rentals.

Data Analysis

In the analysis of general data, national, regional and local trends are emphasized. Supply and

demand data are studied to understand the competitive position of the property in its market. In

an analysis of specific data, a set of properties most like the subject property is studied. The

analysis of comparable properties helps an appraiser extract specific sale prices, rental terms,

incomes and expenses, rates of return on investments, construction costs, economic life estimates

and rates of depreciation. These figures are used in the calculations that result in indications of

value for the subject property.

Highest and best use is a critical step in the development of a market value opinion. In highest

and best use analysis, the appraiser considers the use of the land as though it were vacant and the

use of the property as it is improved. To qualify as the highest and best use, a use must satisfy

four criteria: it must be legally permissible, physically possible, financially feasible and

maximally productive. The highest and best use is selected from various alternative uses. Market

analysis provides the basis for an appraiser‟s conclusions about the highest and best use of a

subject property, and the remainder of the valuation process follows from these conclusions.

Site Value Opinion

A land value opinion is formed through the application of a variety of methods that are derived

in varying degrees from the three approaches to value. The most reliable procedure for arriving

at a land value estimate is sales comparison. Sales of similar vacant parcels are analyzed,

compared and related to the land being appraised. If sufficient sales are not available for

comparison or the value opinion indicated by sales comparison needs substantiation, the

Page 122: unitedemsa.files.wordpress.com...Pg 2 NIESV STUDY PACK - PPE KNOWLEDGE AND STRUCTURE OF NIGERIA INSTITUTION OF ESTATE SURVEYORS AND VALUERS Learning Objectives At the end of this lecture,

Pg 122

NIESV STUDY PACK - PPE

appraiser may use another procedure such as extraction, allocation, the land residual technique,

ground rent capitalization, or subdivision development analysis.

Accurate real estate valuation is important to mortgage lenders, investors, insurers and buyers

and sellers of real property. While valuations are generally performed by skilled professionals,

anyone involved in a real transaction can benefit from gaining a basic understanding of the

different methods of real estate valuation.

PROPERTY MARKET ANALYSIS

Property market analysis is the foundation for most property professional opinions. The property

professional is employed primarily because they understand the current market and can make

reasoned predictions on future market trends. The fact that property markets are complex and the

determinants of change are not stable over time makes the role of the property professional

challenging and, financially, worthwhile. It is crucial that property market analysis is undertaken

as competently as possible.

The applicability of property market analysis studies can, to some degree, be measured by their

ability to forecast future market movements. The level of uncertainty in capital growth forecasts

is substantially greater than rental growth forecasts and that forecasters had great difficulty in

forecasting yield shifts.

Estate surveyors and valuers have a long history of analyzing property markets, with many early

twentieth century pioneers stressing the importance of real estate supply and demand. The

importance of market analysis was further developed during the evolution of the property

discipline in the mid 20th Century which suggests that property market analysis has even become

more topical because of the ground-gaining velocity of real estate. Property market analysis

success is a major function of efficient property valuation reports. Contents of valuation reports

are herewith listed below:

1. Identification of the client

The Report must be addressed to the client, or the client‟s representatives. The source of the

instructions and the identity of the client must be stated, if different from the addressee.

2. The purpose of the valuation

Page 123: unitedemsa.files.wordpress.com...Pg 2 NIESV STUDY PACK - PPE KNOWLEDGE AND STRUCTURE OF NIGERIA INSTITUTION OF ESTATE SURVEYORS AND VALUERS Learning Objectives At the end of this lecture,

Pg 123

NIESV STUDY PACK - PPE

The purpose of the valuation must be stated clearly and unambiguously. Where the purpose is

not disclosed the Report must include an appropriate statement.

3. The subject of the valuation

Where a number of properties is valued it may be convenient to list them in a schedule that

identifies each unit of valuation. Where the valuation includes a separate valuation for Plant &

Machinery this may also be included in a schedule, which should identify the items as agreed

with the client.

4. The interest to be valued

The legal interest in each property should be stated. The extent to which vacant possession is,

or may be, available, if required, should also be noted.

5. The type of property and how it is used or classified, by the client

For some purposes the uses, categories, or classes of property will have been agreed with the

client. Where formal agreement is not required it is recommended that the Report contain a

brief description

of these matters.

6. The basis, or bases, of the valuation

The Basis of Valuation must be stated and the definition must be provided in full. A

depreciated replacement cost valuation for inclusion in a Financial Statement must be

expressed as being either subject to the test of adequate profitability or continuing viability or

occupation. Where the Report includes a valuation using depreciated replacement cost and the

value for an alternative use on cessation of the business is materially different, a statement to

this effect must be included in the Report.

7. The date of valuation

The date of valuation must be stated. If there has been a material change in market conditions,

or the circumstances of a property or portfolio, between an earlier date of valuation and the

date of the report, the member must draw attention to this. It may also be prudent for the

member to draw the client‟s attention to the fact that values change over time, and that a

valuation given on a particular date may not be valid on an earlier or later date.

8. Disclosure of any material involvement and, if required, a statement of the status of

the valuer

Page 124: unitedemsa.files.wordpress.com...Pg 2 NIESV STUDY PACK - PPE KNOWLEDGE AND STRUCTURE OF NIGERIA INSTITUTION OF ESTATE SURVEYORS AND VALUERS Learning Objectives At the end of this lecture,

Pg 124

NIESV STUDY PACK - PPE

Any disclosures made in accordance with the RICS Standard must be recorded.

Where it is a requirement for the purpose, the valuer will state if he or she is acting as an

Internal Valuer or an External Valuer. Where acting as an Independent Valuer the specific

criteria must be confirmed, together with a statement that the valuer meets those criteria. In

some States the National Associations Standards may require additional disclosures to be

made with regard to the status of the valuer.

9. Where appropriate, the currency that has been adopted

If some valuations have been translated into a currency other than that of the country in which

the property is located, the exchange rate adopted, and its source, is to be noted.

10. Any Assumptions, Special Assumptions, reservations, any special instructions or

Departures

All assumptions made must be stated, together with any reservations that may

be required. Where the assumptions vary in different States the Report must make this clear.

Special assumptions must be clearly stated. Where the valuation is undertaken on the basis of

restricted information, or is a revaluation without an inspection, the Report must include full

particulars of the restriction. Any Departures from the Standards must be stated and explained.

A statement must be made as to whether or not any allowance has been made for liability for

taxation which may arise on disposal, whether actual or notional, and whether or not the

valuation reflects costs of acquisition or realization. In some countries VAT, or similar taxes,

acquisition and sale costs can be substantial. Where statements rely upon the prospect of

future growth in rental and/or capital values, a statement must be made to the effect that such

growth may not occur, and that values can fall as well as rise.

11. The extent of the member‟s investigations

The Report must record the date and extent of the inspection, including reference to any part

of the property to which access was not possible. The valuer must make it clear if the

valuation has been made without an opportunity to carry out an adequate inspection. In the

case of a revaluation, the Report should also refer to any agreement that further inspections

are unnecessary. Where a substantial number of properties are being valued, a generalized

statement of these aspects is acceptable, provided that it is not misleading.

Page 125: unitedemsa.files.wordpress.com...Pg 2 NIESV STUDY PACK - PPE KNOWLEDGE AND STRUCTURE OF NIGERIA INSTITUTION OF ESTATE SURVEYORS AND VALUERS Learning Objectives At the end of this lecture,

Pg 125

NIESV STUDY PACK - PPE

12. The nature and source of information to be relied on by the member

The valuer must make it clear if the valuation has been carried out without the information

normally available when carrying out a valuation. The valuer must indicate in the Report if

(where practicable) verification is needed of any information or Assumptions on which the

valuation is based, or if information considered material has not been provided. If any such

information or Assumption is material to the amount of the valuation, the valuer must make

clear that the valuation report should not be relied on, pending verification. In the case of a

revaluation, a statement of material changes advised by the client or a stated Assumption that

there have been no material changes, should be included. Valuers should also include any

additional information that has been available to, or established by, them that they believe to

be crucial to the client‟s ability to understand and benefit from the valuation, with regard to

the purpose for which it has been prepared.

13. Confirmation that the valuation accords with these Standards

This statement must be unequivocal, but may include a cross reference to any agreed

Departures referred to under item (10). Where the valuation is for inclusion in a Financial

Statement and is in accordance with International Valuation Application 1 (IVA 1) this must

be specifically stated. Statements concerning valuations by other valuers, which have been

included in the report, should also be referred to.

14. A statement of the valuation approach

Reports must include a statement about the extent to which the values are supported by market

evidence, or are estimated using other valuation techniques (which shall be disclosed) because

of the nature of the property, or limited transactions or any combination of these factors.

15. The opinions of value in figures and words

In the main body of the Report the opinion of value is required in words, as well as in figures.

Where the valuation instruction includes a number of properties falling into different

categories it would normally be inappropriate to produce an aggregate valuation of the whole,

although this will depend upon the purpose for which the valuation is required. If the

identification of individual properties and their values is consigned to a schedule(s) appended

to the Report, a summary of values must be included within the body of the Report. „Negative

values‟ must be stated separately, and not set off against the positive values of assets.

16. Signature and date of the Report

Page 126: unitedemsa.files.wordpress.com...Pg 2 NIESV STUDY PACK - PPE KNOWLEDGE AND STRUCTURE OF NIGERIA INSTITUTION OF ESTATE SURVEYORS AND VALUERS Learning Objectives At the end of this lecture,

Pg 126

NIESV STUDY PACK - PPE

The Report must be signed by the person who accepts responsibility for it.

References

Boyd, T. (2014). PROPERTY MARKET ANALYSIS The key to looking forward. In

Proceedings of the PRRES Conference 2014.Pacific Rim Real Estate Society (PRRES).

Oluwunmi, A. O. (2008). A Study of the Quality of Property Valuation Reports in Lagos

Metropolis (Doctoral dissertation, Covenant University).

Yang, Z. (2000).Five essays in property valuation.

Lawson, J. W. W. (2008). Theory of real estate valuation (Doctoral dissertation, Royal

Melbourne Institute of Technology).

Shapiro, E., Mackmin, D., &Sams, G. (2012).Modern methods of valuation.Taylor & Francis.

Kavanagh, P., Benson, J., & Brown, M. (2007).Understanding performance appraisal

fairness.Asia Pacific Journal of Human Resources, 45(2), 132-150.

Questions

1. In the world of property valuation analysis, the terminologies “value” and “cost” appear

to have become debatable and topical. Distinguish between these two terminologies with

special reference to the Nigeria property market.

2. With reference to the Nigerian property market, discuss the property valuation methods

employed by Estate Surveyors and Valuers in the discharge of their duties.

3. Effective property market analysis is a major bedrock for success in property appraisals.

Discuss any five (5) market analysis contents of valuation reports.

Page 127: unitedemsa.files.wordpress.com...Pg 2 NIESV STUDY PACK - PPE KNOWLEDGE AND STRUCTURE OF NIGERIA INSTITUTION OF ESTATE SURVEYORS AND VALUERS Learning Objectives At the end of this lecture,

Pg 127

NIESV STUDY PACK - PPE

ENTREPRENEURSHIP IN REAL ESTATE

Learning Objectives

After reading this chapter, the following will be understood:

Meaning of entrepreneurship

Importance of entrepreneurship

Trait / qualities of entrepreneurs

Opportunities for entrepreneurship in real estate in Nigeria

How to become a real estate entrepreneur in Nigeria

Skills required for successful entrepreneurship in real estate

Challenges / problem of real estate entrepreneur in Nigeria

1.0 Introduction

In human society, everybody engages in one activities or the other to creates, constructs, or

distributes utilities for the use of gaining access to common wealth (Iyun and Alawiye,

2005).The creation of a nation‟s wealth and dynamism depends upon the competitiveness of its

firms and this, in turn, relies fundamentally on the capabilities of its entrepreneurs and managers

(Cuervo1, Ribeiro and Roig, 2006). Among the factors of production namely land, labour, capital

and entrepreneur, the entrepreneur is the sole creative and purposive dynamic factor while the

other factors are in state of inertia. It takes an entrepreneur to combine the other three factors of

production to produce goods and services.

This chapter on entrepreneurship in real estate is aim at preparing probationer to become real

estate entrepreneur. The chapter is divided into nine sections including the introduction which is

the first section. The second and third section is on meaning of entrepreneurship and importance

of entrepreneurship respectively. Trait / qualities of entrepreneurs and opportunities for

entrepreneurship in real estate in Nigeria is in section four and five while section six and seven

covers how to become a real estate entrepreneur in Nigeria and skills required for successful

entrepreneurship in real estate respectively. Section eight is on challenges / problem of real estate

Page 128: unitedemsa.files.wordpress.com...Pg 2 NIESV STUDY PACK - PPE KNOWLEDGE AND STRUCTURE OF NIGERIA INSTITUTION OF ESTATE SURVEYORS AND VALUERS Learning Objectives At the end of this lecture,

Pg 128

NIESV STUDY PACK - PPE

entrepreneur in Nigeria while section nine is the chapter summary. Practice questions and

suggested solution were also provided at the end of the chapter.

2.0 Meaning of Entrepreneurship

The term “Entrepreneur” is derived from the French word “entreprendre”, meaning “to

undertake.” An entrepreneur is therefore one who undertakes to organize, manage, and assume

the risks of a business. Although there is no single acceptable definition for the term“

entrepreneur” like other economic concept however, the definition provided by Joseph

Schumpeter in 1934 appears more popular than other definitions. According to Ahmad and

Seymour (2008), Schumpeter defined entrepreneurs as innovators who implement

entrepreneurial change within markets, where entrepreneurial change has five manifestations:

1) the introduction of a new (or improved) good;

2) the introduction of a new method of production;

3) the opening of a new market;

4) the exploitation of a new source of supply; and

5) the re-engineering/organization of business management processes.

Schumpeter‟s definition therefore equates entrepreneurship with innovation in the business

sense; that is identifying market opportunities and using innovative approaches to exploit them.

Another popular definition of entrepreneur is „persons who are ingenious and creative in finding

ways that add to their own wealth, power, and prestige‟ (Baumol, 1990)

Entrepreneurship can therefore be referred as the process of starting or running a business for

profit. In economic theory entrepreneurship has been modeled as an occupational choice between

self-employment and wage-employment (Naudé, 2013). Hence someone will become an

entrepreneur if profits and the non-pecuniary benefits from self-employment exceed wage

income plus additional benefits from being in wage employment.

Page 129: unitedemsa.files.wordpress.com...Pg 2 NIESV STUDY PACK - PPE KNOWLEDGE AND STRUCTURE OF NIGERIA INSTITUTION OF ESTATE SURVEYORS AND VALUERS Learning Objectives At the end of this lecture,

Pg 129

NIESV STUDY PACK - PPE

3.0 Importance of Entrepreneurship

Entrepreneurship is an essential element for economic progress as it manifests its fundamental

importance in different ways:

a) By identifying, assessing and exploiting business opportunities;

b) By creating new firms and/or renewing existing ones by making them more dynamic;

c) By driving the economy forward – through innovation, competence, job creation- and by

generally improving the wellbeing of society;

d) Assist in increasing government revenue through payment of various taxes and other charges.

Entrepreneurship allows entrepreneur to satisfy personal need and ambition such as:

a) Need to be their own boss

b) Need to be financial independent

c) Need to use own skills and knowledge

d) Need to satisfy a perceived need

e) Need to be wealthy

f) Need to be able to pay up expenses and live a debt free life

f) Need to get rid of idleness

4.0 Traits / Quality of Entrepreneur

“Are Entrepreneurs born or made” is a popular debate. Many would argue that entrepreneurial

drive is a trait that‟s acquired at birth, while others disagree and believe that anyone can become

an entrepreneur if he or she is so trained. Which side of the argument one lean toward, research

indicates that most successful entrepreneurs share certain personal attributes, including:

creativity, dedication, determination, flexibility, leadership, passion, self-confidence, ability to

take risk among others. The traits / qualities of entrepreneur according to U.S Department of

State (2008) are described below:

Page 130: unitedemsa.files.wordpress.com...Pg 2 NIESV STUDY PACK - PPE KNOWLEDGE AND STRUCTURE OF NIGERIA INSTITUTION OF ESTATE SURVEYORS AND VALUERS Learning Objectives At the end of this lecture,

Pg 130

NIESV STUDY PACK - PPE

1. Creativity

Creativity is the spark that drives the development of new products or services or ways to do

business. It is the push for innovation and improvement. It is continuous learning,

questioning, and thinking outside of prescribed formulas.

2. Dedication

Dedication is what motivates the entrepreneur to work hard, especially in the beginning, to

get the endeavor off the ground. Planning and ideas must be joined by hard work to succeed.

Dedication makes it happen.

3. Determination

Determination is the extremely strong desire to achieve success. It includes persistence and

the ability to bounce back after rough times. For the true entrepreneur, money is not the

motivation. Success is the motivator; money is the reward.

4. Flexibility

Flexibility is the ability to move quickly in response to changing market needs. It is being

true to a dream while also being mindful of market realities.

5. Leadership

Leadership is the ability to create rules and to set goals. It is the capacity to follow through to see

that rules are followed and goals are accomplished.

6. Passion

Passion is what gets entrepreneurs started and keeps them there. It gives entrepreneurs the ability

to convince others to believe in their vision. It can‟t substitute for planning, but it will help them

to stay focused and to get others to look at their plans.

7. Self-confidence

Page 131: unitedemsa.files.wordpress.com...Pg 2 NIESV STUDY PACK - PPE KNOWLEDGE AND STRUCTURE OF NIGERIA INSTITUTION OF ESTATE SURVEYORS AND VALUERS Learning Objectives At the end of this lecture,

Pg 131

NIESV STUDY PACK - PPE

Self-confidence comes from thorough planning, which reduces uncertainty and the level of risk.

It also comes from expertise. Self-confidence gives the entrepreneur the ability to listen without

being easily swayed or intimidated.

8. The willingness to take calculated risks in terms of time, equity, or career.

9 The ability to formulate an effective venture team; the creative skill to marshal needed

resources.

10 The fundamental skills of building a solid business plan.

11 The vision to recognize opportunity where others see chaos, contradiction, and confusion.

5.0 Opportunities for entrepreneurship in real estate in Nigeria

Real estate offers a number of opportunities for entrepreneurial activity notably in the areas of

estate agency, real estate development, valuation, property management, feasibility and viability

appraisal, and consultancy services on issues related to land. The real estate sector in Nigeria

offers wide range of opportunities because of the followings:

1. Shelter is a basic necessity of life

Housing is the basic necessity of life after food. Everybody desire and require one form of

housing or the other. There will always be demand for housing as commodity and services such

as real estate related to housing.

2. Small number of registered real estate professional

After more than three decades of recognition of estate surveying and valuation profession, the

registered members are still below five thousand. It is surprising how a population of less than

five thousand estate surveyors and valuers would be able to rendered real estate service

effectively to the entire Nigeria population of about 150 Million people.

Page 132: unitedemsa.files.wordpress.com...Pg 2 NIESV STUDY PACK - PPE KNOWLEDGE AND STRUCTURE OF NIGERIA INSTITUTION OF ESTATE SURVEYORS AND VALUERS Learning Objectives At the end of this lecture,

Pg 132

NIESV STUDY PACK - PPE

3. Housing deficit

According to World Bank (2012), Nigeria needs an annual housing stock of 720,000 units for the

next 20years in order to overcome the deficiency of about 14-16million housing units. As more

houses are built, the services of real estate professional would be required either at the inception

of construction or completion of a house.

4. Availability of many unregistered title document

Only about 5% of land in Nigeria has registered titles. This has turn the remaining 95% of entire

land in the country to dead capital that might not useful to attract mortgage finance. Meanwhile,

this is a huge opportunity for estate surveyors and valuers to fill this economic gap by acting as

consultant for processing of title document from government.

5. Valuation of asset used as collateral is a compulsory requirement for loan application

Valuation is usually carried out to determine the monetary value of asset pledge as collateral for

credit facility. Valuation service are therefore required by mortgagor and mortgagee to have

access to loan. There would therefore be demand for valuation services as long as people apply

for loan. Valuation servicesis an exclusive professional calling of the estate surveyors and

valuers.

6. Nigeria population

Nigeria has a population of about 150 Million people. It is the most populated country in Africa

and the country with the largest population of black people. A large population connotes a large

market with many business opportunities and large patronage for services. There is therefore

potential for real estate service in this populated market.

7. Real estate as investment medium

More than 50% of asset of most nations is held in real estate (Brown &Matysiak, 2000). Many

institutional investors are now including real estate in their portfolio because of its comparative

Page 133: unitedemsa.files.wordpress.com...Pg 2 NIESV STUDY PACK - PPE KNOWLEDGE AND STRUCTURE OF NIGERIA INSTITUTION OF ESTATE SURVEYORS AND VALUERS Learning Objectives At the end of this lecture,

Pg 133

NIESV STUDY PACK - PPE

advantage over other investment medium and diversification potentials. This has open array of

opportunity for real estate entrepreneur to meet the growing need of real estate investors.

8. All activities take place on land

Land is the platform upon which all human activities takes place. There will always be demand

for land and landed properties as long as humanity exist. Inextricably, there will be demand for

service of a profession of real estate that centres on management and allocation of land

resources.

9. Non availability of databank of concluded transaction

The lack of central databank on concluded transactions has make valuation of property difficult

in Nigeria. This has led to valuation variance (difference in opinion of valuer on the same

property) and inaccuracy (difference in valuation and final sales price) with implication of

liability for professional negligence on part of the valuer. Also, as more institutional investor

enters the Nigeria property market, there would be need for property data in term of prices and

rent of concluded transaction, property price indices, performance indices among others. These

needs has created opportunity for entrepreneurs that will render supportive service to existing

estate firm and real estate investors by supplying them data.

6.0 How to Become a Real Estate Entrepreneur in Nigeria

The section explains the step by step approach of becoming a professional real estate

entrepreneur in Nigeria. The steps which are not necessary in chronological order are stated

below:

6.1 Incubation and validation of business idea

Every entrepreneur must have at inception of starting a business a clear idea of what they intend

to do. Every business enterprise starts as an idea which derived from perceived opportunity

which is then translated into a product or service which a customer needs and prepared to pay

for. They must be able to determine which of Schumpeter‟s entrepreneurial change they intend to

make in the market. The entrepreneur must honestly ask question such as:

Page 134: unitedemsa.files.wordpress.com...Pg 2 NIESV STUDY PACK - PPE KNOWLEDGE AND STRUCTURE OF NIGERIA INSTITUTION OF ESTATE SURVEYORS AND VALUERS Learning Objectives At the end of this lecture,

Pg 134

NIESV STUDY PACK - PPE

can I create a special demand for my business output?

what will be the unique selling point of the service I intend to render?

what advantage do I have over existing firms?

what is the particular need I intent to meet in the market?

will people be able to pay for the services I intend to render?

what technical skill have I acquired / developed?

will my idea fill a particular need?

The idea incubation and validation stage of an enterprise is a very important stage. Any mistake

or insincerity at this stage may lead to colossal loss of resources in future.

6.2 Registration of business entity with Corporate Affairs Commission (CAC)

There are three categories of enterprises that the profession of estate surveying and valuation can

be practiced. These are sole proprietorship, partnership and Limited Liability Company. The

later was not possible until lately. Registration of company is usually carried out the Corporate

Affairs Commission. In sole proprietorship, business is owned by an individual and is popularly

referred as “one-man business”. Partnerships are business entity owned by 2-20 persons with

agreed profit / loss sharing ratio. Registration of sole proprietorship and partnership are carried

out by registration of business name. Individual/Proprietors can register business names without

the services of the legal practitioner, chartered Accountant or Chartered Secretary. The procedure

involves:

Search for availability of name at CAC

Submission of duly completed statutory forms with two passport sized photographs of

each applicant attached to the form

Payment of filing fees at the Corporate Affairs Commission

Application form clearly stating: - The name of the business, nature of business, among

other information

Limited liability companies are distinct legal entities from their owners (called directors).

Limited Liability Company are usually incorporated by CAC and are limited by shares. The

minimum number of shares that can be registered for a limited liability company is one million

Page 135: unitedemsa.files.wordpress.com...Pg 2 NIESV STUDY PACK - PPE KNOWLEDGE AND STRUCTURE OF NIGERIA INSTITUTION OF ESTATE SURVEYORS AND VALUERS Learning Objectives At the end of this lecture,

Pg 135

NIESV STUDY PACK - PPE

shares. The services of the legal practitioner, chartered Accountant or Chartered Secretary are

necessary for registration / incorporation of a limited liability company.

6.3 Registration with professional and regulatory body

There are two categories of registration required for any entrepreneur intending to practice estate

surveying and valuation profession; these are the registration of individual and registration of

firm. The registration of individual involves being elected as an associate of the Nigeria

Institution of Estate Surveyors and Valuers (NIESV). This would be followed by being

registered as Estate Surveyor and Valuer (ESV) by Estate Surveyors and Valuers Registration

Board of Nigeria (ESVARBON). The registration of firm is carried out by ESVARBON which

will issue Certificate of Compliance. The procedure for registration of estate surveying and

valuation firm is as follow:

Formal application to the Registrar requesting for Certificate of Compliance.

Applicant must be financially up to date with ESVARBON. Apart from this the applicant

must present:

- Photocopy of Certificate of registration of business entity with CAC with nature of

Business reading "ESTATE SURVEYING & VALUATION"

- Letter of Resignation & Acceptance of Resignation from former employer.

- 2-Nos Passport photographs.

- Letter of Appointment/Employment (Applicable only to lecturers)

6.4 Preparation of business plan

A business plan is a formal statement of business goals, reasons they are attainable, and plan for

reaching them. It covers objectives, strategies, sales, marketing and financial forecasts. The

business plan is an essential roadmap for business success as it helps to clarify business idea,

spot potential problems, set out goals and measure progress. It is necessary the entrepreneur

prepare a business plan that will capture his business ideas and from inception set the template of

how the business entity would be run. A business plan can be prepared by the entrepreneur using

Page 136: unitedemsa.files.wordpress.com...Pg 2 NIESV STUDY PACK - PPE KNOWLEDGE AND STRUCTURE OF NIGERIA INSTITUTION OF ESTATE SURVEYORS AND VALUERS Learning Objectives At the end of this lecture,

Pg 136

NIESV STUDY PACK - PPE

templates that are available on the internet and textbook. It can also be prepared by outsourcing

it to to external consultant.

6.5. Setting up an office

Setting up an office involves getting an office, equipping the office with necessary office

furniture, fittings and equipment, acquiring stationery, acquiring working tools, and setting off.

6.5.1 Accommodation for Office

The office accommodation provides a working space, location for meeting with client and most

importantly, an address the entrepreneur can be linked to. Many start-ups find setting up an

office at home the most attractive option but this might not provide the right working

environment. Another option open to the entrepreneur is to rent a space. However, when renting

a space, consideration should be given, accessibility, size of the space; and affordability of the

rent. It is not necessary to rent a space in the city centre where rent would be very high. Real

estate entrepreneur can start up from any location and move to s preferred location as income

improves.

6.5.2 Office Stationery

Stationery is an important factor in creating your business image. It comprises the following:

Letterhead paper: is stationery used for letters to customers and suppliers. It Include your

business name, logo, address, phone, email and website addresses.

Business / complementary cards: is a small card that contains the entrepreneur business

name, personal name, logo, address, phone, email and website addresses. It gives your

contacts a record of your name and details.

Receipts booklet: receipts are document issue out as evidence of payment. A receipt

booklet that will have your company name can be printed or a receipt can be generated on

the computer when required.

File

6.5.3 Office furniture and equipment

These are movable asset used in the office. It includes chair, table, computer system, laptop,

printer, scanner, internet modem, binding machine among others. It is advisable for entrepreneur

Page 137: unitedemsa.files.wordpress.com...Pg 2 NIESV STUDY PACK - PPE KNOWLEDGE AND STRUCTURE OF NIGERIA INSTITUTION OF ESTATE SURVEYORS AND VALUERS Learning Objectives At the end of this lecture,

Pg 137

NIESV STUDY PACK - PPE

to start with a basic minimum of furniture and equipment. Also, secretariat work like preparation

of letter, report e.t.c. could be outsourced when they arise before the entrepreneur could acquire

equipment to do so.

6.5.4 Working Tools

These are devices used as working tools by the entrepreneur

Motor Vehicle: This is necessary for necessary mobility from one location to another

which real estate practice required.

Digital Camera: This is an apparatus for taking photographs. It is advisable to get a

digital camera with in-built rechargeable battery.

Measuring Tape: This use for taking measurement. It could be a steel, fiber or digital

tape which comes in different length.

Telephone / Mobile phone: this is a devise for transmitting speech by wire or radio

waves. Smart mobile phones can be used to do a wide range of things such as sending

and receiving mails, calls; taking photographs, scanning document amongst others.

6.6 Recruitment of Employee

This relates to the process of hiring of staff by the entrepreneur. Recruitment of employee is one

of the most important decisions the entrepreneur will take to grow his business. This is because

the human capital is the greatest asset any enterprise can possess. An entrepreneur can start the

business alone but as time goes on he will definitely require more hands and recruitment of

employees could be through direct appointment or open competition. Direct appointments are

made when there is a specific individual in mind to serve in a particular position while open

competition is where applicant go through competitive interview. Irrespective of the method of

employment, the successful employee must possess four important qualities which are integrity,

passion, commitment and hard work.

Page 138: unitedemsa.files.wordpress.com...Pg 2 NIESV STUDY PACK - PPE KNOWLEDGE AND STRUCTURE OF NIGERIA INSTITUTION OF ESTATE SURVEYORS AND VALUERS Learning Objectives At the end of this lecture,

Pg 138

NIESV STUDY PACK - PPE

The first employee could be an estate officer who would combine the services of a receptionist,

secretary, accountant and clerk. As challenges and resources increase more employees would be

required.

6.7 Sourcing for briefs and building clientele base

Sourcing and securing brief as start-up entrepreneur is not an easily task. It takes take time to win

confidence of people and build the credibility of your business entity. The first source of

patronage for the business could be friends, family and associates who know the entrepreneur.

Apart from this the entrepreneur should market his service to everybody in his social group such

as church members, club members, Old Student‟s association, Town Union‟s association,

Residents‟ association amongst others. The entrepreneur should extend his tentacles to as many

people as possible in order create awareness for the business especially at the inception of the

business. The entrepreneur also need to learn how make and keep friends because friends and

acquaintances are good sources of instruction and referral for instruction. Generally, sources of

brief / instruction for real estate firm include the following:

Walk-in instruction – people walking in to directly instruct the company to market their

properties for lease / sale

Instruction through people with direct connection to the firm

Instruction could also flow in the course of seeking/sourcing instructions for letting or

management

Advertise in the National Dailies requesting for particular properties to meet specific

needs

Through the grapevine

Relatives and friends

Retaining clients / customers is as important as securing instruction from them because clients /

customers are the lifeblood of any business. Apart from bringing their subsequent transaction to

you in the future, they often refer other customers / client to the organization. Customer relations

are therefore key to business success and can make or break the business. It is therefore

Page 139: unitedemsa.files.wordpress.com...Pg 2 NIESV STUDY PACK - PPE KNOWLEDGE AND STRUCTURE OF NIGERIA INSTITUTION OF ESTATE SURVEYORS AND VALUERS Learning Objectives At the end of this lecture,

Pg 139

NIESV STUDY PACK - PPE

advisable that direct contact and follow-up be maintained with existing clients. Also, networking

and referral building should be employed to win more clients.

6.8 Financing the Enterprise

Finance for a business can either be equity, debt, or variant of both. Finance is necessary for

business registration, setting up the office and payment of office bills and salary until the

business can generate enough funds to do so. Most small businesses usually start with equities

arising from the entrepreneur personal saving and or gift from friend and relatives. Debt funding

could be introduced later when the business has stabilized. It is advisable that the entrepreneur

start with the business with the finance he has available and grow as the business progress. Do

not borrow fund at inception of a business except it is necessary, as such fund could mar the

business.

6.9 Managing the business

Management is the organization process that includes planning, organizing, staffing, leading or

directing, and controlling an organization to accomplish the goal. It involves coordinating the

efforts of people to accomplish the goal of the organization. It is the duty of the entrepreneur to

manage the business to profitability. A clear roadmap should be set on how to generate enough

revenue to cover expenses and return profit. The entrepreneur as the manger should be able to

exhibit the management function such as planning, organizing, leading and controlling to achieve

the goals of the organization.

7.0 Skills Required for Successful Entrepreneurship in Real Estate

The following skills are required for successful entrepreneurship in real estate:

7.1 Business Ideas Generation:

This involves ability to create new business idea of expand or modify existing ideas. It involves

the following:

- Identification of opportunities

- Creating new products

- Defining new processes

- Cost reduction / quality improvement

Page 140: unitedemsa.files.wordpress.com...Pg 2 NIESV STUDY PACK - PPE KNOWLEDGE AND STRUCTURE OF NIGERIA INSTITUTION OF ESTATE SURVEYORS AND VALUERS Learning Objectives At the end of this lecture,

Pg 140

NIESV STUDY PACK - PPE

7.2 Business Structuring Skill

This is the skill require to restructure the business when need be. This could be in term of new

business formation, forming alliance r joint ventures with existing firm and floating a special

purpose vehicle (SPV) ventures

7.3 Business Planning and Strategies Skill

This skill is required for the continuance of the business. It involves

- Survival strategy

- Growth and expansion

- Downsizing / contraction

7.4 Human Resources Management Skill

This is the skill required in employment and managing people. It involves:

- recruitment and selection of staff,

- determining of wages/salaries and other incentives,

- promotion structure

- creating a career path for staff

- employment relationship

7.5 Communications Skill

Real estate practice involves constant communication which could be oral (face-to-face or

telephone), through email, letter, report, proposal e.t.c. A good communication is therefore

essential for a successful real estate practice.

7.6 Decision making skill

This involves making intelligent choices within a wide problem solving range of choices. It

involves:

defining the problem

Page 141: unitedemsa.files.wordpress.com...Pg 2 NIESV STUDY PACK - PPE KNOWLEDGE AND STRUCTURE OF NIGERIA INSTITUTION OF ESTATE SURVEYORS AND VALUERS Learning Objectives At the end of this lecture,

Pg 141

NIESV STUDY PACK - PPE

setting objectives

identifying alternative solutions

compare alternatives

implementation

evaluation

7.7 Marketing Skill

This is the process involve in identifying, anticipating, and satisfying customer requirement

profitably. In relation to real estate practice, the article of trade is goods (property) and services,

therefore it involves marketing of goods and services. Marketing is one of the most important

skill require to a successful entrepreneur. The entrepreneur should therefore be the chief

marketing officer of his business entity if he wants to be successful.

Marketing of real estate services involves the following activities:

(i) Registration with Financial Institutions

Many financial institutions require Estate Surveying and Valuation firms to be registered with

them before commissioning them for job. It is pertinent that for the entrepreneur to get his firm

registered with all financial institutions in the country.

Registration is usually pursued from the legal or risk department of the head office of the

particular financial institution. This is done by submitting a professional job proposal to the

financial institution and follow up until registration is effect.

(ii) Registration with Government Parastatals and Agencies

Like financial institutions, some government parastatals and agencies require registration with

them before one could be commissioned for a job. This should be pursued to conclusion.

Page 142: unitedemsa.files.wordpress.com...Pg 2 NIESV STUDY PACK - PPE KNOWLEDGE AND STRUCTURE OF NIGERIA INSTITUTION OF ESTATE SURVEYORS AND VALUERS Learning Objectives At the end of this lecture,

Pg 142

NIESV STUDY PACK - PPE

(iii) Pursuance of job from organizations which the company is registered with

Regular visit should be made to both private and public organizations in which the firm is

registered with in order to be considered for job. Such visit should be methodological and

scientific. For financial institutions (bank), the branch managers and the marketers are the main

staff to be market; regular call should follow after visit.

(iv) Marketing to Individuals and Corporate Organisations.

Every organization irrespective of size requires real estate services because of its multi-

dimensional range of professional services such as agency, property management, valuation,

feasibility and viability appraisal, property development and real estate consultancy. It is

therefore the duty of the entrepreneur as the chief marketing officer of his firm to let anyone he

come across or existing client see reasons why they might require real estate services.

In similar vein, marketing of real estate products (property) involves the following:

Using appropriate medium varying from newspaper advertisement, newsletters,

placement of „For Sale‟ / “To Let” board, advertisement on the internet and some specific

web page like Yahoo, Facebook, YouTube amongst other;

Introducing the properties to some identified corporate organization, government

agencies and individuals

Preparation of sale brochure and promotion

Sending of bulk sms and emails to prospective buyers / renter and colleague

7.8 Time Management Skill

This is the skills required to avoiding time wasting activities and be accountable for every single

second spend. This involves listing and prioritizing activities with no procrastination. This skill

helps avoid stress, breakdown, and burn out.

7.9 Finance Management Skill

Page 143: unitedemsa.files.wordpress.com...Pg 2 NIESV STUDY PACK - PPE KNOWLEDGE AND STRUCTURE OF NIGERIA INSTITUTION OF ESTATE SURVEYORS AND VALUERS Learning Objectives At the end of this lecture,

Pg 143

NIESV STUDY PACK - PPE

This is the skill required to be able to identify and assess sources of funds for the business and

also cultivate financial discipline. It includes:

- Book keeping

- Stock keeping

- Resource costing

- Prompt remittance of rent

- Separation of client, company and personal accounts

- Banking transaction management

- Cash and investment management

7.10 Business Evaluation and Innovation Skill

This involves periodical evaluation of the business in terms of technical capacity, financial

capacity, managerial capacity, marketing capacity, competitors‟ comparative achievement and

bringing changes to the capacity as may be required.

7.11 Business Ethics Skill

Ethics are cardinal guiding principles, moral codes, rules of behavior, character conduct,

etiquette, which are recognized as the standard of work ethics expected of member of a

profession, trade or business. Without a code of ethics, and ethical behavior, there is no

profession. The fundamental purpose of professional code of ethics is to maintain high ethical

standard, integrity, and credibility for the professional members, client and public at large. Being

ethical mean being a good business person who is trustworthy, has integrity, honesty and practice

fairness to all. The level of business ethics the society expects from a profession is higher than

that expected from business men. This is the reason while most professional body has a code of

conduct. Decree 24 of 1975; the law regulating estate surveying and valuation practice in Nigeria

set out code of conduct for professional Estate Surveyor and Valuers (ESV). The Code of

Conduct should be adhered to as breach of the Code can attract sanction.

Page 144: unitedemsa.files.wordpress.com...Pg 2 NIESV STUDY PACK - PPE KNOWLEDGE AND STRUCTURE OF NIGERIA INSTITUTION OF ESTATE SURVEYORS AND VALUERS Learning Objectives At the end of this lecture,

Pg 144

NIESV STUDY PACK - PPE

8.0 Challenges / Problem of Real Estate Entrepreneurship in Nigeria

The problems of real estate entrepreneurship in Nigeria are as follows:

8.1 Lack of financial discipline

Many entrepreneurs lack financial discipline and knowledge in keeping accounting records,

especially separating client account from company‟s account. Many entrepreneurs have been

sanctioned by failing to give account or remit client money in their custody.

8.2 Lack of entrepreneurial skills.

Lack of the eleven entrepreneurial skills such as marketing skills, communication skill earlier

discussed in section 7 can affect the success of a real estate entrepreneur (Moore, 2003).

8.3 Lack of legislation governing some aspect of real estate profession

There is lack of legislation governing some aspect of the real estate profession in Nigeria. For

instance, anyone above 18 years of age can practice agency in Nigeria, this has allowed incursion

of quack into this aspect of the profession.

8.4 Poor remuneration

The scale of charges approved by the professional and regulatory body for real estate is not

accepted by the member of the public. Remuneration for real estate professional services in most

cases is negotiated below the scale of the profession charges thereby impoverishing the real

estate entrepreneur.

8.5 Enlistment for professional job

Most financial institutions and government agencies required enlistment with them before

practitioners can secure job from them. Procedure for such enlistment most time are elaborate

and require certain conditions that are difficult for newly established entrepreneur to meet.

Page 145: unitedemsa.files.wordpress.com...Pg 2 NIESV STUDY PACK - PPE KNOWLEDGE AND STRUCTURE OF NIGERIA INSTITUTION OF ESTATE SURVEYORS AND VALUERS Learning Objectives At the end of this lecture,

Pg 145

NIESV STUDY PACK - PPE

8.6 Unhealthy rivalry

There is high level of competition and unhealthy rivalry among practitioners within and between

professional in the built environment. These make many professional firms to quote for job

below the stipulated scale of charges to secure the job ahead of their colleague. There is also

incursion into roles other members of the built environment professional thereby causing rivalry

within and among the built environment professionals.

8.7 Unethical conduct

Breaching the code of conduct through unethical behaviour for pecuniary or other reason can

attract sanctions from the professional and regulatory body. A professional can also be held

liable for negligence if his professional calling. The entrepreneur should be cautious of unethical

conduct as this pre-maturely ends his professional career.

8.8 Lack of technical know how

Many entrepreneurs do not have the necessary technical know-how before setting up their

business enterprises. If you do not know how to do the job, how will you satisfy your clients.

8.9 External environment factor

This is the sum total of factors outside the control of the entrepreneur such as inflation rate,

economic condition of the country, government legislation amongst others.

9.0 Chapter summary

This chapter is on entrepreneurship in real estate. The chapter is divide into nine sections

including the introduction which is the first section. The second and third section is on meaning

of entrepreneurship and importance of entrepreneurship respectively. Trait / qualities of

entrepreneurs and opportunities for entrepreneurship in real estate in Nigeria is in section four

and five while section six and seven covers how to become a real estate entrepreneur in Nigeria

and skills required for successful entrepreneurship in real estate respectively. Section eight is on

Page 146: unitedemsa.files.wordpress.com...Pg 2 NIESV STUDY PACK - PPE KNOWLEDGE AND STRUCTURE OF NIGERIA INSTITUTION OF ESTATE SURVEYORS AND VALUERS Learning Objectives At the end of this lecture,

Pg 146

NIESV STUDY PACK - PPE

challenges / problem of real estate entrepreneur in Nigeria while section nine is the chapter

summary. Question and suggested answer were also provided at the end of the chapter.

References

Ahmad, N., and Seymour, R. G. (2008). Defining Entrepreneurial Activity: Definitions

Supporting Frameworks for Data Collection (No. 2008/1). OECD Publishing.

BHP Information Solutions Ltd (2011) Setting up an office.London:Althorp House.

Baumol, W.J. (1990). „Entrepreneurship: Productive, Unproductive and Destructive‟, The

Journal of Political Economy 98(5): 893-921.

Brown, G. and Matysaik, G. (2000) Real estate investment: A capital market approach, 1st

edition. London: Pearson Education Limited

Cuervo1, A., Ribeiro, D., and Roig, S. (2006)Entrepreneurship: Concepts, Theory

andPerspective. Introduction.

Iyun, R. and Alawiye, J. (2005) Entrepreneurship development and small scale business

management. Akure. Aroman Publisher.

Moore, M., M. (2003) Black entrepreneurship in the real estate sector in Namibia: Constraints

and prospects. Paper delivered at International Conference with the theme

Entrepreneurship in Africa: Towards economic independence held at Windhoek, Namibia

on 17th

-18th

September, 2003

Naudé, W. (2013) Entrepreneurship and Economic Development: Theory, Evidence and Policy.

IZA Discussion Paper Series IZA DP No. 7507

Onashile, A. (2015) Developing entrepreneurial skills for the built environment professionals: A

quantity surveyor‟s viewpoint. Paper presented at the 11th Faculty Distinguished Guest

Lecture at Obafemi Awolowo University, Ile Ife.

Page 147: unitedemsa.files.wordpress.com...Pg 2 NIESV STUDY PACK - PPE KNOWLEDGE AND STRUCTURE OF NIGERIA INSTITUTION OF ESTATE SURVEYORS AND VALUERS Learning Objectives At the end of this lecture,

Pg 147

NIESV STUDY PACK - PPE

U.S. Department of State (2008). “Principles of entrepreneurship.” Available online at

http://ait.org.tw/infousa/zhtw/.../enterp.pdf

Recommended / Suggested Reading

Drucker, P. F. (1993) Innovation and Entrepreneurship.London, HarperCollins

Hemuka, N. (2007) Professional Practice (Theory, Principles and Practice). Benin City. Real

Estate Publication.

McGrath, R. T. (2000) The Entrepreneurial Mindset. London. Harvard Press.

Randel, J. A. (2006)Confessions of a Real Estate Entrepreneur. New York. McGraw-Hill.

Practice Questions

1. What are the likely motivations for real estate entrepreneurship in Nigeria property market.

2. Who is a real estate entrepreneur. What are the qualities of real estate entrepreneur?

3. Advice a friend who intend to set up a profession estate surveying and valuation firm how to

go about it.

4. What are the likely challenges a real estate entrepreneur can encounter in Nigeria. How can

these challenges be surmounted?

5. Who is an entrepreneur? what are the importance of entrepreneurship in real estate.

6. What are the skills required to be a successful real estate entrepreneur?

7. According to Joseph Schumpeter (1934)“entrepreneurs as innovators who implement

entrepreneurial change within markets”. What are the manifestations of entrepreneurial change in

the market? Relate with example the entrepreneurial change to real estate practice in Nigeria.

Page 148: unitedemsa.files.wordpress.com...Pg 2 NIESV STUDY PACK - PPE KNOWLEDGE AND STRUCTURE OF NIGERIA INSTITUTION OF ESTATE SURVEYORS AND VALUERS Learning Objectives At the end of this lecture,

Pg 148

NIESV STUDY PACK - PPE

8. Advice a newly established real estate entrepreneur how to source for briefs, market the briefs

and build clientele base.

9. Justify your occupational choice between self-employment and wage-employment?

Suggested solution

1. The student is expected to discuss reasons why there are opportunities for entrepreneurs

in real estate sector in Nigeria as explained in Section 5

2. Definition of entrepreneur is provided in Section 2 while qualities of real estate

entrepreneur is available in section 4

3. The question is about how to become a real estate entrepreneur. The step by step

approach of becoming a real estate entrepreneur is provided in Section 6.

4. Challenges of real estate entrepreneurship in Nigeria is provide in Section 8. The student

is to provide ingenious solution to each of the identified challenges

5. Definition of entrepreneur is provided in Section 2 while importance of entrepreneurship

is available in section 3

6. The eleven skills required to be a successful real estate entrepreneur is provided in

Section 7

7. Entrepreneurial change has five manifestations: the introduction of a new (or improved)

good / service; the introduction of a new method of production / service; the opening of a

new market; the exploitation of a new source of supply; and the re-

engineering/organization of business management processes. Students are to relate each

of these to real estate sector.

8. Sourcing for brief and building clientele base is available in Section 6.7

Page 149: unitedemsa.files.wordpress.com...Pg 2 NIESV STUDY PACK - PPE KNOWLEDGE AND STRUCTURE OF NIGERIA INSTITUTION OF ESTATE SURVEYORS AND VALUERS Learning Objectives At the end of this lecture,

Pg 149

NIESV STUDY PACK - PPE

9. Self-employment means becoming an entrepreneur. The question expect student to

importance of becoming an entrepreneur over being an employee. The importance of real

estate entrepreneur is provided in Section 3

Page 150: unitedemsa.files.wordpress.com...Pg 2 NIESV STUDY PACK - PPE KNOWLEDGE AND STRUCTURE OF NIGERIA INSTITUTION OF ESTATE SURVEYORS AND VALUERS Learning Objectives At the end of this lecture,

Pg 150

NIESV STUDY PACK - PPE

HUMAN RESOURCES MANAGEMENT, OFFICE ADMINISTRATION AND

PROBLEM SOLVING TECHNIQUE

After reading this chapter, the following will be understood:

Meaning and function of human resources management (HRM)

Meaning and function of office administration

Problem solving techniques

1.0 Human Resources Management

1.1 Meaning of Human Resources Management

Human resources management (HRM) refers to the management of people in organizations. It

comprises the activities, policies, and practices involved in obtaining, developing, utilizing,

evaluating, maintaining, and retaining the appropriate number and skill mix of employees to

accomplish the organization‟s objectives. According to Bratton and Gold (1999) Human

resources management is that part of the management process that specializes in the management

of people in work organizations. HRM emphasizes that employees are critical to achieving

sustainable competitive advantage, that human resources practices need to be integrated with the

corporate strategy, and that human resource specialists help organizational controllers to meet

both efficiency and equity objectives.

The goal of HRM is to maximize employees‟ contributions in order to achieve optimal

productivity and effectiveness, while simultaneously attaining individual objectives (such as

having a challenging job and obtaining recognition), and societal objectives (such as legal

compliance and demonstrating social responsibility)

1.2 Function of Human Resources Management

The main functions of HRM are as follow:

1.2.1 Staffing

These deals with obtaining people with appropriate skills, abilities, knowledge and experience to

fill jobs in the work organization. Pertinent practices are human resource planning, job analysis,

recruitment and selection.

Page 151: unitedemsa.files.wordpress.com...Pg 2 NIESV STUDY PACK - PPE KNOWLEDGE AND STRUCTURE OF NIGERIA INSTITUTION OF ESTATE SURVEYORS AND VALUERS Learning Objectives At the end of this lecture,

Pg 151

NIESV STUDY PACK - PPE

1.2.2 Rewards

Involves the design and administration of reward systems. Practices include job evaluation,

performance appraisal, and benefits.

1.2.3 Employee Development

Is analyzing training requirement to ensure that employees possess the knowledge and skills to

perform satisfactorily in their jobs or to advance in the organization. Performance appraisal can

identify employee key skills and „competence‟.

1.2.4 Employee Maintenance

Is the administration and monitoring of workplace safety, health and welfare policies to retain a

competent workforce and comply with statutory standards and regulations.

1.2.5 Employee Relations

This involves a range of employee involvement/participation schemes in union or non-union

workplaces. In a union environment, it also includes negotiations between management and

union representatives over decisions affecting the employment contract.

1.3 Importance of Human Resources Management to an Organization

The importance of HRM to an organization includes:

•To assist the organization in employing the right employees to fulfill its strategic and

operational goals

•To help create a climate in which employees are encouraged to develop and utilize their skills to

the fullest.

• To help maintain performance standards and increase productivity through effective job design;

providing adequate orientation, training and development; providing performance-related

feedback; and ensuring effective two-way communication

• To help establish and maintain an harmonious employer/employee relationship

• To help create and maintain a safe and healthy work environment

• To develop programs that will meet the economic, psychological, and social needs of the

employees by helping the organization to retain productive employees

Page 152: unitedemsa.files.wordpress.com...Pg 2 NIESV STUDY PACK - PPE KNOWLEDGE AND STRUCTURE OF NIGERIA INSTITUTION OF ESTATE SURVEYORS AND VALUERS Learning Objectives At the end of this lecture,

Pg 152

NIESV STUDY PACK - PPE

•To ensure that the organization is in compliance with provincial/territorial and federal laws

affecting the workplace (such as human rights, employment equity, occupational health and

safety, employment standards, and labour relations legislation).

2.0 Office Administration

Administration department is the backbone of an organization and the administrator is the link

between an organization‟s various departments and he ensures the smooth transfer of information

from one place to the other. Thus without an effective administration, an organization would not

run professionally and smoothly. Office administration is one of the key elements associated

with a high level of workplace productivity and efficiency.

2.1 Function of Office Administration

The functions of office administration and specific roles of an office administrator includes:

2.1.1 Prepare organizational structure chart / organogram

An organizational structure chart is a diagram that represents an organization, identifying its

employees and departments and their relationships to each other. This will allow the staff team

and visitors to the office can visualize how the organization is structured and what its reporting

lines are.

2.1.2 Design appointment Appointments System Visitors will have more guaranteed access

to the staff member visited, staff will be less disrupted by visits and a record of visitors will be

kept.

2.1.3 Design and Manage Reception System

This will allow messages to be properly recorded and visitors to be well attended to.

2.1.4 Management of Retrieval System

A revival system is provided in order to raise function that must be completed periodically at the

required time. A system for bringing up (BU) activities must be design and implemented by the

administrator so task that required follow-up would be properly handled.

Page 153: unitedemsa.files.wordpress.com...Pg 2 NIESV STUDY PACK - PPE KNOWLEDGE AND STRUCTURE OF NIGERIA INSTITUTION OF ESTATE SURVEYORS AND VALUERS Learning Objectives At the end of this lecture,

Pg 153

NIESV STUDY PACK - PPE

2.1.5 Ensure effective Office communications and arrange period staff / management

meeting

The administrator must ensure that there is smooth communication between the top management

and staff through internal memos and notices. The administrator must also arrange periodic

management / staff meetings to facilitate communications within the team.

2.1.6 Filing and record keeping

A filling system that will ensure that different types of information are stored appropriately and

all documents are securely filed and easy to access should be designed by the administrator.

2.1.7 Management of Resource Centre

The office resource centre is a place where reference materials such as books, magazines,

training materials, and as other information that an organization wants to make available are

stored in an orderly way. The administrator should ensure that books and other important

documents are well organized, classified, indexed and appropriately shelved so that staff and

others can find what they need quickly.

2.1.8 Periodic Renewal of Practicing and Municipal Licenses and Insurance Policies

The administrator should ensure that the professional practicing licenses are renewed at the right

time. Also, municipal licenses such as vehicle license and insurance policies should also be

renewed at the right time.

2.1.9 Preparation of Expression of Interest (EIO) and Request for Proposal (RFP)

It is the duty of the administrator to search for EIO and RFP when they are advertised and

prepare them as advertised.

2.2 Importance of Office Administration

Enhance the office staff‟s ability to manage and organize office effectively and

professionally

Ensure filing of documents are done in the proper way and filing standard

Page 154: unitedemsa.files.wordpress.com...Pg 2 NIESV STUDY PACK - PPE KNOWLEDGE AND STRUCTURE OF NIGERIA INSTITUTION OF ESTATE SURVEYORS AND VALUERS Learning Objectives At the end of this lecture,

Pg 154

NIESV STUDY PACK - PPE

Develop an appropriate office management strategy

Develop an appropriate assets management strategy

Develop administrative procedures

Plan and control administrative budget

Ensure the organization run smoothly

Remove communication gap between the management and staff

Improve productivity and efficiency in the office

3.0 Problems Solving Techniques

3.1 Introduction

A problem exists when there is a gap between what you expect to happen and what actually

happens. These problems may be technical or issue-based. Problems must be resolved for

organizations to function properly. Problem solving involves defining a problem and creating

solutions for it.

3.2 Meaning of Problem Solving Technique

Problem solving technique is a structured, systematic approach to solving problems and making

improvements. According to Restructuring Associates Inc. (2008), a systematic approach to

problem solving allows for:

• decision making based on data, rather than hunches

• determining root causes of problems, rather than reacting to superficial symptoms

• devising permanent solutions, rather than relying on quick fixes

3.3 Six Steps Problem Solving Technique

The problem solving technique is a six step sequential and continuous model for solving

organization problem. The steps include definition of problem, determination of cause of

problem, development of alternative solutions, selection of solution, implementation of solution

and evaluation of outcome. Each of the steps is therefore described below:

Page 155: unitedemsa.files.wordpress.com...Pg 2 NIESV STUDY PACK - PPE KNOWLEDGE AND STRUCTURE OF NIGERIA INSTITUTION OF ESTATE SURVEYORS AND VALUERS Learning Objectives At the end of this lecture,

Pg 155

NIESV STUDY PACK - PPE

3.3.1 Step One: Definition of the Problem

The first step, identifying the problem, is a broad review of the current. In this first stage, the

symptoms and scope of the problem is identified. That is, it determines what “hurts,” the degree

to which the symptoms are shared, and the urgency of relieving the symptoms. Tools such as

brainstorming, interviewing, and completing questionnaires are employed to gather information.

3.3.2 Step Two: Determination of the Cause(s) of the Problem

Once the symptoms of the problem are recognized and tentatively defined the problem,

information can be collected about the nature of the problem to zero in on underlying causes of

the problem. In this way, you determine the root causes of the problem. This stage involves

diagnosing the situation so that the focus is on the real problem, not just on its symptoms. After

this stage, efforts over the next two steps can thus be directed to finding solutions that address

the roots of a documented problem, not merely its random symptoms

3.3.3 Step Three: Develop Alternative Solutions

Once a problem has been defined, the next step is to create alternative solutions. However,

creative problem solving requires you to explore a full range of viable solutions before reaching

a conclusion. To assemble a variety of solutions from which to choose a final solution, you must:

• generate as many potential solutions as possible

• relate each solution to the causes of the problem

• merge similar or related solutions.

The essence of this stage is simply want to reduce redundancy, and eliminate any possibilities

that don‟t address the causes you identified earlier.

3.3.4 Step Four: Select a Solution

As a fourth step, evaluate each potential solution for its strengths and weaknesses. Selecting a

solution entails searching for the most effective solution by applying two general criteria. An

effective solution that is technically feasible and acceptable to those who will have to implement

it. Selecting a solution requires a choice that will be effective – one that has sufficient technical

quality to resolve the problem, and is acceptable to those who will implement it.

Page 156: unitedemsa.files.wordpress.com...Pg 2 NIESV STUDY PACK - PPE KNOWLEDGE AND STRUCTURE OF NIGERIA INSTITUTION OF ESTATE SURVEYORS AND VALUERS Learning Objectives At the end of this lecture,

Pg 156

NIESV STUDY PACK - PPE

3.3.5 Step Five: Implementation of the Solution

Choosing a solution does not immediately solve a problem. Putting a solution into action may

prove as difficult as deciding on one. The implementation stage requires action planning:

What must be done?

Who will do it?

When will it be started?

When will key milestones be completed?

How will the necessary actions be carried out?

Why are these actions a solution?

3.3.6 Step Six: Evaluate the Outcome

In simplest terms, evaluation is the monitoring that any project needs to ensure that milestones

are met, costs are contained, and work is completed. The use of feedback mechanisms can be

employed to detect the need for midcourse corrections and to ensure that the problem is solved

without creating new problems.

4.0 Chapter Summary

This chapter is on human resources management, office administration and problem solving

techniques. The chapter is divided into four sections. The first section which is on human

resources management (HRM), show the meaning of HRM, function of HRM and importance of

HRM to an organization. Office administration is the second section. The section reveals the

function and importance of office administration to an organization. Section three is an

exposition on problem solving while section four is the chapter summary. Question and

suggested answer were also provided at the end of the chapter.

Reference

Bratton, J. and Gold, J. (1999) Human Resource Management -Theory and Practice. - London,

Macmillan Press Ltd.

National Open University of Nigeria (2012) Lecture note on human resources management in

education.

Page 157: unitedemsa.files.wordpress.com...Pg 2 NIESV STUDY PACK - PPE KNOWLEDGE AND STRUCTURE OF NIGERIA INSTITUTION OF ESTATE SURVEYORS AND VALUERS Learning Objectives At the end of this lecture,

Pg 157

NIESV STUDY PACK - PPE

Practice Questions

1. Justify the need for the service of human resources manager in an estate surveying and

valuation firm

2. “…without an effective administration, an organization would not run professionally and

smoothly…” Discuss this statement with emphasis on function of an administrator in an

estate surveying and valuation firm

3. What is the problem solving technique? Discuss the six steps problem solving technique.

Suggested Solutions

1. The student is expected to explain the function and importance of HRM to an

organization as explained in Section 1.2 and 1.3

2. Explanation of the function of administrator in relation to estate surveying and valuation

office is expected from students. This is available in Section 2.1

3. Answer is available in Section 3.2 and 3.3

Page 158: unitedemsa.files.wordpress.com...Pg 2 NIESV STUDY PACK - PPE KNOWLEDGE AND STRUCTURE OF NIGERIA INSTITUTION OF ESTATE SURVEYORS AND VALUERS Learning Objectives At the end of this lecture,

Pg 158

NIESV STUDY PACK - PPE

Meaning of Office Procedures

In order to run things smoothly in a professional environment, irrespective of the nature of

office, it is important to have a written procedure visible for all employees to see. This is a need

for a written set of procedures developed by the superior and office manager visible in the front

office.

Office procedures set the standard for how a staff works together in the office. It is often a set of

rules or policies guiding the operations of an office or small business. As such, every office has a

set of rules, guidelines and procedures that have to be done every day.

Office procedures are necessary to ensure efficiency in the workplace. Examples of activities that

need procedure include: handling calls in and out, paperwork, filing, taking messages, customer

interaction and even furniture arrangement can all fall under office procedure.

Office procedures can keep things running smoothly and professionally. Office procedures, in

real estate office, are procedures that the estate surveyor and valuer adopt in performing the

various professional and administrative activities in the office.

Even in a one-person office, organizing office procedures is essential to the efficient operation of

a business. Creating a set of organized...

Benefits

Office procedures create a uniform way of doing things that create consistency, efficiency and

professionalism within the office environment.

Effects

Office procedures often fall in line with company policies. For instance, a company policy may

be that no liquid is allowed near the computers. With that policy may come a procedure for staff

members to safely have water or soft drinks a safe distance from the computers such as in a

break room. Following that office procedure would be mandatory. Disobeying it could cost staff

members their jobs.

Page 159: unitedemsa.files.wordpress.com...Pg 2 NIESV STUDY PACK - PPE KNOWLEDGE AND STRUCTURE OF NIGERIA INSTITUTION OF ESTATE SURVEYORS AND VALUERS Learning Objectives At the end of this lecture,

Pg 159

NIESV STUDY PACK - PPE

Examples

Office procedures mandate an employee's actions while at work (i.e., no personal phone calls,

checking voicemail in the mornings, no unnecessary conversations with clients). They can also

include the handling of office equipment such as reloading empty copy machines and refilling

staplers. These procedures are in place to make the office run efficiently.

Significance

Often office procedures are written out for new employees. They hardly ever come as a surprise.

By knowing office procedures and allowing them to become part of a job description, employees

can focus on daily tasks as a priority.

References

CHIRO: Introduction to Office Procedure

General Office Practices & Procedures

How to Learn Modern Office Practices & Procedures

By Charlie Brooks

Policy Manual Definition

A policy manual is a formalized human resources document that presents a broad overview of

standard operating policies and procedures for an office. It is produced for every department or

section of the organization, and compiled together to form what is known as the Policy Manual

of the organization.

Manuals are generally of two types: Operation and Staff.

Staff manual: is a written document of the rights, privileges and obligations of an employee. It

specifies what an organization expects from an employee and what he, as an employee are

Page 160: unitedemsa.files.wordpress.com...Pg 2 NIESV STUDY PACK - PPE KNOWLEDGE AND STRUCTURE OF NIGERIA INSTITUTION OF ESTATE SURVEYORS AND VALUERS Learning Objectives At the end of this lecture,

Pg 160

NIESV STUDY PACK - PPE

entitled to, as a staff of the company. It contains the rights of the employee to annual leave,

confirmation, promotion and pension.

Operation Manual: This is a well a prepared written procedure of how the various activities of

the organization should be done. It serves as a guide to how the organization expects that every

professional activity be carried out. It is a result of well tested mode of operations after several

years of operation. It therefore gives a breakdown and detailed procedure of stage by stage

involvement in the activity to be carried out by employees of the organization.

In preparation of the policy manual for administrative department, office hours should be the

first thing mentioned, and all employees should be at work on time and prepared for the day. If a

customer comes to the office and no one is there are likely to have a negative view of the

business and may never return. Also, the office should be neat, clean and well supplied. Make

sure a check-in roster and pens are available for customers to sign in, and if possible, make

coffee and water available. First impressions are everything in a successful business. In addition,

every customer should be treated with respect and professionalism, whether they are in the office

or on the telephone. If a call comes in for a supervisor, the employee should get all the

information prior to connecting them to the caller, including name, nature of the call and phone

number in case the connection is lost. If the call is for an appointment, they can find out the

nature of the visit and let them know what‟s available, ensuring to get a phone number in case of

cancellation.

EXPERT WITNESS AND PROOF OF EVIDENCE OF VALUATION

An expert witness is a person who is an expert in the field or area in which he is called upon to

give a witness e.g. a medical doctor could be called to tender a medical report concerning an

injured victim in such a situation he is called an expert witness. Likewise an Estate Surveyor and

valuer who has received the requisite training and expertise to ascertain the value of interest in

real property for various purposes could be called upon to give an expert witness or evidence in

respect of the valuation he carried out where the value of the subject property is now a subject

of controversy in a Law Court.

According to Evidence Act 2011, an expert is a person who is specially skilled in the field case in

which he is giving evidence. S. 57(1)(2) see shell petroleum company Ltd V. Otogo (1996)

Page 161: unitedemsa.files.wordpress.com...Pg 2 NIESV STUDY PACK - PPE KNOWLEDGE AND STRUCTURE OF NIGERIA INSTITUTION OF ESTATE SURVEYORS AND VALUERS Learning Objectives At the end of this lecture,

Pg 161

NIESV STUDY PACK - PPE

6NWLR Pt. 159. It also stated that opinions of experts are relevant and admissible where the

court is to form an opinion on a point of science or art. Such expert must be specially skilled in

the area of science or art he is called upon to give evidence in his area of speciality, otherwise

his evidence will not be relevant. (See Ogunze V. State 1998, 5NWLR pt.581 at 521).

PROOF OF EVIDENCE OF VALUATION

Evidence has been defined in the Evidence Act 2011, pg. 129 as the means by which facts in

issues are established by judicial tribunal, such evidence could be oral, hear say, documents,

things of facts which a court will accept as the facts on issue in a given case.When the value of a

property is in contention in a law court, the proof of the value of such a property becomes

necessary. Usually an Estate Surveyor and Valuer is called upon to carry out valuation ad render

his report in writing i.e. his valuation report. Such valuation report is now tendered in a law

court to enable the court rely on the report to form an opinion as to the value of the property.

His valuation report is now a proof of evidence of valuation. In most cases the valuation report

could be tendered through the Estate Valuer.

This procedure is usually necessary to avoid speculative opinion as to the value of the property

which is insufficient in proof of evidence.

RULES OF EVIDENCE

a). Upon whom rests, the burden of proving facts?

The answer is not a clear cut one, e.g.if ‘A’ is accused of ‘B’ of stealing his shoe. The burden of

proof rests on ‘B’ and not on ‘A’. However ‘A’ is accused of no defence if he pleads insanity as a

defence. In this case the burden of proof rests on ‘A’ since he is in the best position to proof his

insanity.

b). What fact ought to be proved?

A party must prove every material fact upon which he intends to rely by adducing sufficient

evidence. e.g. Mr. A is accused of stealing Mr. B’s shoe. The prosecution must establish by

adducing sufficient evidence that A fraudulently took B’s shoe or that he fraudulently converted

it for his own use or to the use of another person.

c). What facts ought to be excluded from court’s cognizance?

Page 162: unitedemsa.files.wordpress.com...Pg 2 NIESV STUDY PACK - PPE KNOWLEDGE AND STRUCTURE OF NIGERIA INSTITUTION OF ESTATE SURVEYORS AND VALUERS Learning Objectives At the end of this lecture,

Pg 162

NIESV STUDY PACK - PPE

Not all facts relating to a case or a matter may be presented before the courts, proofs of facts in

issue, and of facts relevant to fact in issue are admissible e.g. the fact that A was seen wearing

B’s shoe is relevant to the fact in issue which is the theft of the shoe. Not all facts relating to a

case may be presented to the Law Courts.

d). What Mode is proof acceptable?

Some particular kinds of proof are recognized by the Nigerian Legal System e.g. proof by oral

evidence, by real evidence, by documentary evidence and by electronic gadgets.

TYPES OF EVIDENCE

The following types of evidence are identified:

(i) Real Evidence which is also known as objective or demonstrative evidence and involves

any material object presented or produced for inspection of the court. Areas in which real

evidence may be required include; where objects are moveable and can be presented before

the court and where they are not moveable or that they can inspected out of court.

(ii). Direct Evidence involves evidence of a fact that could be seen, heard or perceived by

any person. It is the fact in issue during trial e.g. the evidence of a witness who saw the accused

in the process of committing the crime directly.

(iii). Indirect or circumstantial evidence which does not involve the fact in issue but from

other facts from which the fact in issue can be informed. It becomes useful where there is no

direct evidence.

iv). Oral Evidence is the statement of a witness in court which is offered as evidence of truth

of that which has been stated or said in the Law Court.

v). Documentary Evidence is usually a statement in a document procedure as a means of

establishing or proving a fact. Document here means books, plans, photographs, reports or

anything expressed or descended or any substance by means of letters, figures, marks to be

used for the purpose of recording.

vi) Hear Say is the evidence of a statement made by a person not called as a witness for the

purpose of proving what is contained in it.

Of all these types of evidence, the only one applicable to Estate Surveyors and Valuers in the

preparation of valuation report as a proof of evidence is the documentary evidence.

Page 163: unitedemsa.files.wordpress.com...Pg 2 NIESV STUDY PACK - PPE KNOWLEDGE AND STRUCTURE OF NIGERIA INSTITUTION OF ESTATE SURVEYORS AND VALUERS Learning Objectives At the end of this lecture,

Pg 163

NIESV STUDY PACK - PPE

THE VALUATION PROCESS/MARKET ANALYSIS AND THE USES OF THE VALUATION GUIDANCE

NOTE

The valuation process is the sequential arrangement of all the activities that a valuer engages to

arrive at an estimated value. The valuation process can be depicted in the diagram below.

SOURCE: Ogunba (2013)

STAGE 1: INSTRUCTIONS AND TERMS OF REFERENCE

STAGE 2: ADOPTION OF A VALUATION BASIS AND METHODOLOGY

STAGE 3: VALUATION AND MARKET INSPECTIONS

STAGE 4: DATA ANALYSIS

STAGE 5: APPLICATION OF ONE OR MORE OF THE

VALUATION MODELS

STAGE 6: RECONCILIATION OF VALUE INDICATIONS

STAGE 7: VALUATION REPORT

Page 164: unitedemsa.files.wordpress.com...Pg 2 NIESV STUDY PACK - PPE KNOWLEDGE AND STRUCTURE OF NIGERIA INSTITUTION OF ESTATE SURVEYORS AND VALUERS Learning Objectives At the end of this lecture,

Pg 164

NIESV STUDY PACK - PPE

a). The first stage is where the Valuer receives a brief from his client to value a particular

property for a particular purpose. The Valuer at this stage requests for certain information to

guide him in carrying out his assignment such information according to Ogunba (2013) include:

- Appraising a copy of deed, survey plan, purchase agreement or other paper pertaining

to the property.

- Ascertaining if there is a mortgage. If there is, he should find out who has it, when it was

placed, for how much, the type of mortgage and the interest rate.

- If the property is an income-producing one, he should obtain a breakdown of income

and expenses for the last year or two and a copy of the leases.

b). Choice of basis of Valuation and Valuation Methodology: Based on the use in which the

valuation is to be put to, the Valuer adopts the appropriate basis of the valuation and

consequently the methodology.

c). Valuation and Market Analysis:Market analysis for valuation purposes involve an

economic assessment of the neighbourhood where the property is located; market survey is

done so as to ascertain and obtain data on sales of comparables etc. General inspection of the

property is also carried out for effective valuation of the property.

d). Data Analysis:Data gathered from property inspection and market analysis will be

analyzed so as to be used for the appropriate valuation methodology e.g analysis of relevant

construction cost indices, analysis of comparable sales, etc.

e). Application of one or more of the approaches to value:

The data obtained in stage (d) is used in arriving at the estimated value in three different

approaches namely. The sales comparison approach, the cost approach and the income

approach.

f). Reconciliation of value indications: Reconciliation of value indications is usually done

when more than one approach to value is used. The value of the most applicable approach is

used e.g. cost method for specialized properties, sales comparison for most residential

properties.

g). Report Presentation: A valuation report is finally prepared by the Valuer stating the final

estimate of value which is the Valuer’s opinion of value.

Page 165: unitedemsa.files.wordpress.com...Pg 2 NIESV STUDY PACK - PPE KNOWLEDGE AND STRUCTURE OF NIGERIA INSTITUTION OF ESTATE SURVEYORS AND VALUERS Learning Objectives At the end of this lecture,

Pg 165

NIESV STUDY PACK - PPE

USES OF VALUATION GUIDANCE NOTES

The valuation standards and guidance notes of NIESV mandated Valuers to comply strictly with

the ethics and standards stipulated by the International Valuation Standards Committee (IVSC)

in their valuation construction and reporting. The standards set a framework for best practice in

the execution and delivery of valuation for different purposes.

- The guidance note also aims at ensuring uniformity in the presentation of valuation

report.

- It seeks at making members internationally competitive so as to provide a

benchmark for common understanding of terms and our clients also.

- The guidance note is aimed at addressing the haphazard style of carrying out

valuations amongst Nigerian Valuers.

- It focuses on the approaches, principles and bases.

- NIESV adopted the guidance note so as to improve consistency and quality of

valuation among Estate Surveyors and Valuers in Nigeria.

ESTATE AGENCY

Estate agency is one aspect of the multi-faceted profession of Estate Management. Estate

agency in its simplest form means acting on behalf of a principal in the disposal, purchase, lease

or development of a landed property. Landed property here could be bare land or land and

buildings. (Ubosi, 2001). In United Kingdom, it is called house agency. It is a profession or

commercial operation which has affected so many people and lives. Estate Agency Institute

Chamber (UK) defines as agent as someone who buys, sells or let property and the interest

therein. Research Institute for Consumer Affairs “RICA” (UK) defines it as the selling of houses.

Agency relationship is the relationship that exist between one person called the principal and

another person called agent. Estate Agency involves;

1. The disposal by way of sale of both freehold and leasehold interest.

2. Letting of residential, commercial and agricultural properties.

It needs a detailed knowledge of valuation and law as it applies to the estate management field.

The agent needs to be well informed or have a basic knowledge of building construction

Page 166: unitedemsa.files.wordpress.com...Pg 2 NIESV STUDY PACK - PPE KNOWLEDGE AND STRUCTURE OF NIGERIA INSTITUTION OF ESTATE SURVEYORS AND VALUERS Learning Objectives At the end of this lecture,

Pg 166

NIESV STUDY PACK - PPE

planning and the remedy opened to anybody that is affected, the basic compensation rules in

town planning both local or regional and national settings, the rent act, development land tax,

rating taxation and site value tax. Agency is a relationship between an agent and a

vendor/principal.

Types of Agency

1. Sole agency: - this is when an agent is taking sole authority to the property. This is when one

firm is given a property to sell or let.

2. Joint agency: - when two firms are given a property to let or sell. They must agree on a

common board.

3. Multiple agency: - it is a situation where the vendor gives to more than an agent to sell or let

his property. This situation occurs when a property is difficult to let especially high rise building.

Advantages of Sole Agency

1. It eliminates abortive work

2. It creates relationship between agent and vendor/principal

3. It reduces cost

4. In a buoyant market, there is tendency for the seller to ‘ Gasump’

Disadvantages of Sole Agency

1. Elimination of speed

2. One agent judgement as to price

3. Less exposure to market

Sole selling right

If an agent is in charge of a property, and if it is sold by another agent, the agent in charge will

still be given his share on commission for he has the selling right.

Authority given to Agent by Vendor

An agent is instructed to find a purchaser, tenant, or lease.

Termination of Agency

1. Death or bankruptcy

2. By performances

Page 167: unitedemsa.files.wordpress.com...Pg 2 NIESV STUDY PACK - PPE KNOWLEDGE AND STRUCTURE OF NIGERIA INSTITUTION OF ESTATE SURVEYORS AND VALUERS Learning Objectives At the end of this lecture,

Pg 167

NIESV STUDY PACK - PPE

Duties of the Agent

1. Must not take bribe, secret discount or commission

2. Must act honestly

3. Exercise reasonable care

4. Duty to carry out his client’s lawful instruction

5. Exercise reasonable care and skill.

6. Must not buy vendors property or act otherwise

7. Confidentiality

8. Must not delegate performance.

Entitlement of the Agent to Commission

This depends on the terms of the contract between the agent and his principal according to

‘Mordoch’ in order for an agent to claim commission, it is necessary for the agent to show that

the event has occurred.

1. Not only that the event has occurred on which payment is made

2. And also that it is an effective cause.

Ways of securing instruction (letting/sale)

Instruction could come in form of the following ways

-Walk in instruction: people come directly to instruct the company to market their properties.

Instruction could flow in from people with direct connection with the firm.

-Proposal: write to corporate bodies that you have tenant

-Through advertisement in the national dailies requesting for particular properties to meet

specific needs.

-Sourcing: moving around if there is any vacant property

-By appointment

-Through grapevine.

Agency documents

Letting: letting register, letting enquiries/request

Sale: sale or client register, sale enquiry/request

Page 168: unitedemsa.files.wordpress.com...Pg 2 NIESV STUDY PACK - PPE KNOWLEDGE AND STRUCTURE OF NIGERIA INSTITUTION OF ESTATE SURVEYORS AND VALUERS Learning Objectives At the end of this lecture,

Pg 168

NIESV STUDY PACK - PPE

Biodata form/Acquaintance sheet: Tenants bio-data form, landlord bio-data form include

information like C of O

Lease agreement: registrable document/instrument

Tenancy agreement: prepare for two years, not registrable

Page 169: unitedemsa.files.wordpress.com...Pg 2 NIESV STUDY PACK - PPE KNOWLEDGE AND STRUCTURE OF NIGERIA INSTITUTION OF ESTATE SURVEYORS AND VALUERS Learning Objectives At the end of this lecture,

Pg 169

NIESV STUDY PACK - PPE

REFERENCES

Aigbokhaevbo, V. (2011). Lecture note on land law II (LAW 422). National Open University of

Nigeria. Retrieved on 28th

July, 2016, from

http://www.nou.edu.ng/uploads/NOUN_OCL/pdf/Laws/Law%20422%20Land%20Law

%20II.pdf

Barret, P. (1995). Facilities Management: Towards better Practice. Blackwell Science Inc.,

Oxford, U.K.

Chalkley, R. (1994). Professional Conduct: A Handbook for Chartered Surveyors. Surveyors

Holding Ltd., London.

Emiedafe, W. (2015). Housing in Nigeria (Part 2): 9 practical solutions to the 17 million housing

deficits in Nigeria. Retrieved on 28th

July, 2016, from

http://sapientvendors.com.ng/solutions-to-housing-deficits-in-nigeria

Federal Republic of Nigeria (1978). Land Use Act. Federal Government Printer, Lagos.

Federal Republic of Nigeria (1990). Capital Gains Tax Act, Cap 354 LFN. Federal Government

Printer, Lagos.

Federal Republic of Nigeria (1991). National Housing Policy. Federal Government Printer,

Lagos.

Federal Republic of Nigeria (1993). Value Added Tax Decree No. 102. Federal Government

Printer, Lagos.

Federal Republic of Nigeria (2006). National Housing Policy. Federal Government Printer,

Lagos.

Federal Republic of Nigeria (2007). Value Added Tax (Amendment) Act. Federal Government

Printer, Lagos.

Fowler, T. (2016). Synopsis of value added tax administration in Nigeria. Guardian 23rd

June.

Ifekwuna, N. (2014). The basics of capital gains tax. Vanguard 31st March

Ivoh H. Seeley (1984). Quantity Surveying Practice. Macmillan Publishers Ltd.

IFMA (2003). IFMA: Scope and Definitions of Facilities Management. HUT/ NOCs Course in

Facilities Management, Atkin, Brain.

Olawale, S.B., Lawal, A.A. & Alabi, J.O. (2015). Nigeria housing policy: any hope for the poor?

Page 170: unitedemsa.files.wordpress.com...Pg 2 NIESV STUDY PACK - PPE KNOWLEDGE AND STRUCTURE OF NIGERIA INSTITUTION OF ESTATE SURVEYORS AND VALUERS Learning Objectives At the end of this lecture,

Pg 170

NIESV STUDY PACK - PPE

American Research Journal of Humanities and Social Sciences, 1(4).

Ritz, G. J. (1993). Total Construction Projection Management. Civil Engineering Series,

McGraw Hill International Eds.

Then, D. S. (1992). A Framework for defining Facilities Management Research Directions. Ed

Peter Barret, RICS Business Services Ltd., London.

Thorncroft, M. (1995). Principles of Estate Management. Estate Gazzettes Ltd, London.

Ward, P. (1979). Organization and Procedures in the Construction Industry. Macdonald.

Worzala, E. (1996). ARES and the formation of the International Real Estate Society. The

Journal of Real Research.

Fowler, T. (2016). Synopsis of value added tax administration in Nigeria. Guardian 23rd

June,

2016.

Olawale, S.B., Lawal, A.A. & Alabi, J.O. (2015). Nigeria housing policy: any hope for the poor?

American Research Journal of Humanities and Social Sciences, 1(4).

Fowler, T. (2016). Synopsis of value added tax administration in Nigeria. Guardian 23rd

June,

2016.

Page 171: unitedemsa.files.wordpress.com...Pg 2 NIESV STUDY PACK - PPE KNOWLEDGE AND STRUCTURE OF NIGERIA INSTITUTION OF ESTATE SURVEYORS AND VALUERS Learning Objectives At the end of this lecture,

Pg 171

NIESV STUDY PACK - PPE

COURSE TITLE: APPLIED VALUATION

(LAND AND BUILDINGS)

PREMIUM

A premium is a sum of money that is paid by a lessee at the commencement or during the

period of a lease in order to get consideration for a reduction in rent.

At a low rent signifies a rent below full rental value (rack rent), and the other benefits

will be, as a rule, financial, having the same effect as a reduction in rent.

Examples relating to the existence of premium are the tenant paying for repairs that

would normally be the landlord‟s responsibility, and the tenant financing the extension of

the property without being charged an increase in rent. A premium is often paid on the

grant or renewal of a lease, but there may be more than one premium, payable at anytime

during the lease term. It entails a cash gain coupled with a loss of rent for the landlord

because the usual result of charging a premium will be a letting at less than FRV. The

landlord is therefore selling part of his income. The tenant will be paying a lump sum in

return for a lease at a rent below the FRV, effectively buying a profit rent. The payment

of a premium has many advantages to the tenant. It could be concluded that premiums

will usually be paid where there is a seller‟s market, that is, where there is or there exists

a competition among prospective tenants to secure an agreement with the prospective

landlord.

Although the amount of the premium will reflect the discounting of future income, into

an immediate lump sum receivable instead of a future flow of income, this is often more

attractive due to the „time value of money‟. The landlord may prefer a lump sum in order

to meet an immediate expense or to make any kind of cash investment. Receipt of a lump

sum immediately may reduce the diminishing effect that inflation has on the value of

future income in real terms, especially if rent review periods are longer than is favourable

to the landlord. A premium should increase the landlord‟s security of income. Once the

tenant has paid a premium, he has invested money in his occupation of the premises in

expectation of making an actual or notional profit rent. As a result, he is more likely to

remain in occupation of the premises and should be a more reliable tenant. Some of the

risk attached to the investment from the landlord‟s point of view may be reduced. A

premium may be used as a loss or deduction to be made from profits when being assessed

for income tax or capital gains taxpaying a premium may be advantageous to a tenant

when his financial circumstances are such that he prefers to part with capital in order to

reduce his future recurring expenses. However, the landlord will usually enjoy the greater

benefits, and premiums will only usually be paid when the property in question attracts

many prospective tenants.

Page 172: unitedemsa.files.wordpress.com...Pg 2 NIESV STUDY PACK - PPE KNOWLEDGE AND STRUCTURE OF NIGERIA INSTITUTION OF ESTATE SURVEYORS AND VALUERS Learning Objectives At the end of this lecture,

Pg 172

NIESV STUDY PACK - PPE

In discussing premium it should be stated here that this is an aspect of Landlord and

Tenant Valuation. The income approach to property valuation principally centres

particularly on the relationship between landlord and tenant. We will examine several of

the valuation problems raised by this relationship.

VALUATION TECHNIQUE

From the lessor‟s point of view, the receipt of a premium has the advantages of securing

an immediate capital sum, which may have certain tax advantages and increasing the

security of his income from the property in so far as a tenant is perhaps more likely to

default when purchasing the full rental value.

From the lessee‟s point of view, it is also possible for a premium to produce certain tax

advantages. Where a premium is paid, the lessor regards the transaction as part of his

income for the period of the lease; and from the lessee‟s view point, he is investing

money in the property, by purchasing a profit rent.

The capital invested by the lessee depreciates as the term runs out; therefore, provisions

must be made for a sinking fund. Hence, the valuation problem may take one of two

forms:

1. What premium should be paid in consideration of a specified reduction in rent?

2. What reduction should be made in the rent in consideration of a specified

premium?

A lessee may request a specified deduction in rent per annum upon the granting of a

lease, the premium to be paid should be equivalent to the capital value of the annual

profit rent.

Question: Mr. Babaginda has been granted a 14-year lease of a shop having a net

rack rental value of N3, 000 per annum. He has agreed to pay N2, 000 per annum

subject to the immediate payment of a premium. Calculate the premium. Assume a

yield of 7% and tax liability of 40K in the Naira.

Solution:

Net rack rental value =N3, 000 pa

Less Rent to be paid per annum =N2, 000

Profit rent per annum =N1, 000

YP for 14yrs @7% and 2.5% (tax 40k in N)

(Remember the Dual rate formular affected by a tax factor

Page 173: unitedemsa.files.wordpress.com...Pg 2 NIESV STUDY PACK - PPE KNOWLEDGE AND STRUCTURE OF NIGERIA INSTITUTION OF ESTATE SURVEYORS AND VALUERS Learning Objectives At the end of this lecture,

Pg 173

NIESV STUDY PACK - PPE

= YP = 1

i + s (1/1-x)

= YP = 1

i + i/(1+i)n (1/1-x)

=YP = 1

0.07 + 0.025/(1.025)14

(1/1-0.4)

YP = 1

0.07 + 0.025/(1.025)14

(100/60)

YP = 5.852

Premium = N5, 852

Alternatively the premium may be agreed and the reduction in rent per annum will need

to be calculated. The lessee will forgo a capital sum and he will expect in return a

reduction in rent totaling:

1. The interest foregone on the premium at the leasehold rate of interest i and

2. A sum sufficient to recover the initial capital outlay by means of a sinking fund

(adjusted for tax liabilities) at the time the lease expires s(1/1-x), so as to result to

a total of i +s (1/1-x).

This is the annual equivalent of the premium or the annuity N1 will purchase. The

reduction may be calculated as Premium x Annuity N1 will purchase or more

conveniently: Premium/YP for the term of the lease.

Question: A lock up store is to be let on a 14-year full repairing and insuring lease

having a net rack rental value of N2, 400 per annum. The tenant has agreed to pay a

premium of N4, 000 at the commencement of the lease. Calculate the rent per

annum to be paid assume a yield of 7% and a tax liability of 40K in the Naira is

envisaged

Solution:

Net rack rental value =N2400 per annum

Page 174: unitedemsa.files.wordpress.com...Pg 2 NIESV STUDY PACK - PPE KNOWLEDGE AND STRUCTURE OF NIGERIA INSTITUTION OF ESTATE SURVEYORS AND VALUERS Learning Objectives At the end of this lecture,

Pg 174

NIESV STUDY PACK - PPE

Less reduction in rent

Annual equivalent of the premium

= 4, 000

YP 14yrs @7% and 2.5% net (tax 40k in N) = 5.852 =N684

Rent to be paid per annum = =N1, 716

There may be circumstances where the tenant agrees to pay a premium at a future date. In

order to calculate the reduction in rent per annum, the present cost of the future premium

may be determined and the annual equivalent then found.

Question: Chief Okoro has been granted a 14-year full repairing and insuring lease

of premises having a net rack rental value per annum of N4,000. The tenant has

agreed to pay a premium of N6, 000 in 5 years time. Calculate the rent per annum

he should pay assuming a yield of 8% and a tax liability of 40K in the Naira.

Solution:

Net rack rental value =N4, 000 p.a.

Less reduction in rent

Present cost of premium=

N6, 000 x PV of N1 in 5 years @ 2.5%

=N6, 000 X 0.884 = N5, 304

Annual equivalent = N5, 304

YP 14yrs @ 8% and 2.5% (tax 40K in Naira)

= N5, 304

5.528 = N960

Rent to be paud per annum N3, 040

Page 175: unitedemsa.files.wordpress.com...Pg 2 NIESV STUDY PACK - PPE KNOWLEDGE AND STRUCTURE OF NIGERIA INSTITUTION OF ESTATE SURVEYORS AND VALUERS Learning Objectives At the end of this lecture,

Pg 175

NIESV STUDY PACK - PPE

Where a tenant has a liability to pay a premium at a known future date, he may provide

for this by investing a capital sum or by an annual sinking fund. In either of the cases, the

rate of interest for investment to provide the capital sum should be a low-risk-free rate,

that is, an accumulative rate of interest sum as low as 2.5% or 3% net.

For instance considering the previous example where a tenant agreed to pay a premium of

N6, 000 in 5 years time, this may be provided by investment of an initial capital sum or

an annual sinking fund.

Hence, (i) initial capital sum = N6, 000 x PV of N1 in 5 yrs @2.5%

= N6, 000 x 0.884 = N5, 304

(ii) Annual sinking fund = N6, 000 x Annual Sinking fund to provide N1 in 5yrs @ 2.5%

= N6, 000 x 0.190 =N1140

However the value of the future premium to the landlord at the commencement of the

lease is N6, 000 deferred by a remunerative rate of interest that is N6, 000 x PV of N1 in

5 yrs @ 8%

=N6, 000 x 0.681 = N4086

It is usual however to calculate the premium or rent from the tenants view point.

VIRTUAL RENT AND COSTS IN USE

Virtual or sitting rent is the term applied to the true annual cost of premises to a lessee. It

is the rent paid together with the annual equivalent of any capital sums he may have

expended on the premises from time to time. It should be noted that when a person

expends capital on property in which he has only a terminable interest, such capital

invested would have been invested elsewhere and would have borne interest at a fair rate

percent but also that an annual sinking fund ought to be provided to replace this capital

by the time the party‟s interest in the property expires. For instance if a lessee whose

term has thirty years still to run spends N5, 000 on improvements to the property, not

only will the expenditure cost him interest on the N5, 000 throughout the term but he

ought also to provide for the replacement of the N5, 000 by the time his lease runs out in

30 years time. The annual amount of interest and sinking fund on the sum in question is

known as its annual equivalent and can be found by either multiplying the capital sum by

the annuity which N1 will purchase or by dividing it by the appropriate figure of years

Page 176: unitedemsa.files.wordpress.com...Pg 2 NIESV STUDY PACK - PPE KNOWLEDGE AND STRUCTURE OF NIGERIA INSTITUTION OF ESTATE SURVEYORS AND VALUERS Learning Objectives At the end of this lecture,

Pg 176

NIESV STUDY PACK - PPE

purchase for the period which the lease still has to run. The effect of either method is to

spread the expenditure over the term and show its true annual cost to the lessee. The rate

percent at which interest on capital is allowed will be that which might reasonably be

expected from a security of similar type to the property in question.

Question: What is the annual equivalent of a capital sum of N5, 000 expended on

property by a Alhaji Deji whose term has still thirty years to run assuming interest

on capital at 7 percent

Solution:

Capital Expenditure = N5, 000

YP 30 yrs @ 7% & 2.5% (tax 40k in N) =8.47

N5, 000

8.47

Equivalent annual cost = N590. 32

Capital expenditure by a lessee may be of two principal types

(i) A premium paid on taking up of a lease

(ii) Alterations, improvements and other capital works to the property itself

In either case the annual equivalent of the capital sum or sums expended must be added

to the rent actually paid in order to find the lessee‟s virtual or sitting rent. However, in

regards to the second scenario, the capital sum expended must be spread over the number

of years still remaining for the lease at the time when the expenditure was made, since

this is the period over which the lessee should be providing a sinking fund for the

replacement of the sum in question.

Additions or improvements to a property such as the building of a garage or the putting in

a new shop in front are clearly capital expenditure but a large sum spent on internal and

external decorations and repairs after a lesee has been in possession for a number of years

may properly be regarded as merely accrued annual repairs. On the other hand if a lessee

spends a considerable sum on repairs immediately after taken up a lease, it may be

assumed that the rent reserved was lower on the condition of the premises and that the

expenditure made is in form of a premium which should be spread over the term of the

lease in calculating the sitting rent.

Page 177: unitedemsa.files.wordpress.com...Pg 2 NIESV STUDY PACK - PPE KNOWLEDGE AND STRUCTURE OF NIGERIA INSTITUTION OF ESTATE SURVEYORS AND VALUERS Learning Objectives At the end of this lecture,

Pg 177

NIESV STUDY PACK - PPE

When a lessee carries out alterations to a property he may have to enter into a covenant

with his landlord that he will restore the premises to their original form at the end of the

lease. The estimated cost of this work must be considered in calculating virtual rent. The

lessee will not only lose interest on this sum during the term but he should be setting

aside a sinking fund to provide for it and this will form part of the annual equivalent cost

of the premises to him. Where premises are used for a trade, business or profession a

tenant who makes an improvement to the property may be entitled to a compensation for

it at the end of his tenancy under the terms of the Landlord/Tenant agreement or he may

be entitled to a new lease on the expiration of the present term under the Landlord/Tenant

agreement at a rent which excludes the value of the improvement. Where either is the

case it is not necessary to provide for setting aside of a sinking fund for whole or part of

the expenditure incurred.

Repairs, rates and other outgoings which the lessee pays are not usually taken into

consideration in finding the virtual rent or annual equivalent cost of the premises to him.

Hence, virtual rent is usually on the same basis as net rent. The two main justifications

for calculating virtual rent are:

(i) To enable the lessee have knowledge of the annual cost of his occupation

(ii) To enable a prospective occupier decide between alternatives

A prominent method which allows comparison to be made between/amongst alternatives

is that of “Cost in Use”.

Cost in Use which seems similar to the Virtual rent needs to be differentiated in various

areas. For instance virtual rent represents the annual cost of occupying a property while

Cost in Use seems to indicate the same thing it is applicable to any item of capital

expenditure. Virtual rent is computed on a net basis and as such ignores items such as

repairing cost whereas cost in use takes into consideration costs of repairs and other

similar costs of outgoings. Cost in Use considers various costs in such a way that all

annual costs are put in consideration ranging from cost of lighting and heating, additional

labour cots incurred due to prefence to a poorly laid out factory, additional handling cost

incurred for preference to particular designs and installations. Cost in use is principally

used for comparison amongst alternatives. This comparison takes note of the initial

purchase costs and the running cost. A method adopted in this comparison would be to

capitalize the anticipated running cost during the estimated life of the facility together

with any anticipated capital expenses in the future and to add to this figure the original

cost of procurement. The total cost of each alternative is compared before decision is

Page 178: unitedemsa.files.wordpress.com...Pg 2 NIESV STUDY PACK - PPE KNOWLEDGE AND STRUCTURE OF NIGERIA INSTITUTION OF ESTATE SURVEYORS AND VALUERS Learning Objectives At the end of this lecture,

Pg 178

NIESV STUDY PACK - PPE

made. Another method adopted is to calculate the annual equivalent of all the capital

costs and add same to the total running cost.

SURRENDER AND RENEWAL OF LEASES

When the tenancy/lease of a lessee is about to expire the occupier or tenant of the

premises will often be anxious to still remain in occupation. This is usually to continue

with the goodwill they have established in the premises over certain period of time

particularly as a business outfit. Other reasons that alluded could be considerable

expenses involved in moving business, fear of loss of trade and consequently loss of

profit. Due to the attachment in such premises resulting from occupation in the past years

the lessee may decide to surrender the balance of his present term in exchange for the

grant of a new lease based on either the current term or condition or for some other

agreed term and condition. For any of the scenario the questions that might be answered

are: What is the appropriate premium payable to the lessor by the lessee to enjoy the

proposed extension or renewal and what new rent supposing the lessee does not wish to

pay a premium in lieu of the old rent.

The premise behind these enquiries are that for the lessee to be interested in surrendering

and renewing his existing lease there is a latent benefit accruing to him. If the true rental

value of the premises exceeds the rent reserved under the present lease, the lessee has to

compensate the lessor for the proposed extension. How the compensation is been handled

is a matter of negotiation between the lessee and lessor. The agreement could be in the

form of the fresh lease being granted at the same rent and on the same term as at present

this might entail the lessee paying a premium to the lessor or the payment of a capital

sum in consideration of the lessee paying an increased rent throughout the proposed new

term. There may also be an agreement of the payment of a certain sum as premium and

also an increased rent throughout the term with the addition obligation on the lessee to

make some capital improvement to the premises when the new lease is granted. The

landlord on his own part will likely accede to the offer of renewal of lease as landlords

are also anxious to retain good/cool tenants and tend to avoid void. Landlords do

commence negotiation for a new lease a few years to the end of the existing lease.

Where tenant decides to give up an existing lease to enable negotiation of a new lease, it

is called surrender. In surrender the current lease is yet to expire whereas a new

negotiation for a longer term is entered into. This is referred to as Surrender and Renewal

Page 179: unitedemsa.files.wordpress.com...Pg 2 NIESV STUDY PACK - PPE KNOWLEDGE AND STRUCTURE OF NIGERIA INSTITUTION OF ESTATE SURVEYORS AND VALUERS Learning Objectives At the end of this lecture,

Pg 179

NIESV STUDY PACK - PPE

(S&R). S&R arises from the landlord as well as from the tenant‟s point of view,

comparing their present situation with that under the proposed lease

The valuer can be called upon to act on behalf of any of the party either the lessee or the

lessor hence; it is good for valuers to be conversant with the calculation from the point of

view of both parties. The lessee should however be credited with the improved value of

the property for the unexpired term of the existing lease. For instance if the balance of the

lessee‟s term is ten years and he is occupying the premises at a profit rent of N8, 000 a

year, it is obvious he has a valuable interest in the property which he will be giving up in

exchange for the new lease. He is therefore entitled to have the value of the surrendered

portion of his term set off against any benefits which he may derive from the proposed

extension. Hence, whether the calculation is made from the lessor‟s or lessee‟s point of

view the principle involved in the calculation is that:

(i) Estimating the value of the party‟s interest in the property, assuming no

alteration in the present term was made; and

(ii) Estimating the value of the party‟s interest assuming that the proposed renewal

or extension was granted.

The difference between these two figures should indicate the extent to which the lessee

will gain or the lessor loses by the proposed extension. The figures derived are seldom a

matter of precise mathematical calculations. Estimates made from the lessor‟s standpoint

and that made from the lesse‟s standpoint are not usually alike, hence a need for

negotiation between both parties. The valuer not withstanding his client is advisable to

look at his calculations on both standpoints as this will put him in a vantage point of

advising his client appropriately.

even though most times the balance of the lease which the lessee is willing to surrender is

usually a short one the incidence of income tax on sinking fund should not be discarded

in valuing his interest under the existing lease and ultimately considered in valuing the

proposed new lease.

Page 180: unitedemsa.files.wordpress.com...Pg 2 NIESV STUDY PACK - PPE KNOWLEDGE AND STRUCTURE OF NIGERIA INSTITUTION OF ESTATE SURVEYORS AND VALUERS Learning Objectives At the end of this lecture,

Pg 180

NIESV STUDY PACK - PPE

TENANTS‟ POINT OF VIEW

Under the existing lease, a profit rent is enjoyed by the tenant. So that the surrender of the

lease by the tenant would imply that a surrender of a valuable leasehold interest in the

property has been made by him. The tenant will also be enjoying a profit rent in the new

arrangement. However a premium is likely to be demanded by the landlord. The tenant

would reasonably be expected to be compensated by a reduction in rent under the

proposed lease. The amount of such compensation would be determined by the value of

the leasehold interest surrendered. There is therefore the need to determine the value of

the tenant‟s existing lease as well as that of the value of the proposed lease.

LANDLORDS‟ POINT OF VIEW

Considering the fact that the landlord anticipates reversion to full rental value at the

expiration of the existing lease, the landlord would not easily or readily agree to any

indiscriminate extension of the tenant‟s existing profit rent. He would for instance like to

compare his position under the existing tenancy with his position under the proposed

tenancy to ensure that he is not worse off by the proposed extension. There is therefore

the need to value the present interest of the landlord as well as the proposed interest for

the purpose of comparison.

VALUERS‟ ROLE

Just as earlier stated a valuer acting for either or both parties would therefore consider the

position of both parties under the present and proposed tenancy so as to be well informed

and destroy any bias or prejudice in carrying out his statutory role. This will involve at

least four separate valuations for every problem on surrender and renewal valuation.

PRINCIPLES INVOLVED IN THE CALCULATION OF SURRENDER AND

RENEWAL

1. Calculations should be made from the point of view of both parties, i.e., the lessor

and lessee‟s points of view, with the lessee being credited with the improved value

of the property for the unexpired term of the existing lease.

2. The valuer should estimate the value of the parties‟ interest in the property,

assuming no surrender was proposed during the present term

3. Estimate the value of the parties‟ interest, assuming that the proposed renewal or

extensions were granted.

Page 181: unitedemsa.files.wordpress.com...Pg 2 NIESV STUDY PACK - PPE KNOWLEDGE AND STRUCTURE OF NIGERIA INSTITUTION OF ESTATE SURVEYORS AND VALUERS Learning Objectives At the end of this lecture,

Pg 181

NIESV STUDY PACK - PPE

4. The difference between these two figures (i.e 2 & 3 above) should indicate the

extent to which the lessee will gain or the lessor will lose by the proposed

extension.

Question: Alhaji Umaru holding a shop on a repairing lease for 40 years, of which

six years are unexpired, desires to surrender his lease and to obtain a fresh lease for

40 years at the same rent. The rent reserved under the present lease is N10, 000 per

annum while the rack rent is N25, 000 per annum. Calculate a reasonable premium

that can be negotiated between both parties. (Assume 8% & 2.5% with a tax factor

of 40%)

Solution:

(A) Lessee‟s Point of View:

Proposed Interests

Profit rent N15, 000 p.a.

YP 40 years @ 8% & 2.5% (40% tax) 9.55

CV N143, 250

Present Interest

Profit rent N15, 000

YP 6 years @ 8% & 2.5% (40% tax) 2.93

CV N43, 950

Hence, the gain to the Lessee N99, 300

(B) Lessor‟s Point of View

Present Interest

Next 6 years rent reserved N10, 000

YP 6 years @ 6% (due to more security) 4.92 N49, 200

Reversion to full rental value N25, 000

YP per deferred 6 years @ 7% 9.52 N238, 000

CV N287, 200

Proposed Interests

Next 40 years proposed rent N10, 000

YP 40 years @ 6% 15. 046 N150, 460

Reversion to full rental value N25, 000

YP per deferred 40 years @ 7% 0.954 N23, 850

CV N174, 310

Page 182: unitedemsa.files.wordpress.com...Pg 2 NIESV STUDY PACK - PPE KNOWLEDGE AND STRUCTURE OF NIGERIA INSTITUTION OF ESTATE SURVEYORS AND VALUERS Learning Objectives At the end of this lecture,

Pg 182

NIESV STUDY PACK - PPE

Hence, the loss to the lessor N112, 890

For a conclusive premium agreeable by both parties to pay the average amount from both

the gain accruing to the lessee and the loss accruing to the lessor can inform the decision.

Thus, the premium payable =

N99, 300 + N112, 890

2

Premium Payable = N106, 095

Question: Assume in the preceding question that instead of paying a premium, the

lessee should pay an increased rent in the new term of lease. What should be the

appropriate rent payable

Solution:

(A) Lessee‟s Point of View:

Full rental value = N25, 000

Deduct annual equivalent of value of present interest =

(This can be seen above: N43, 950)

N43, 950

YP 40 years @ 8% & 2.5% (40% tax) 9.55 = N4, 602

Reasonable rent for new lease = N20, 398

(B) Lessor‟s Point of View:

Value of present interest (as seen above) N287, 200

Deduct value of proposed reversion to N25, 000

(as seen above) N23, 850

Value of proposed term N263, 350

Divided by YP 40 years @ 6.5% 14.22

Reasonable rent for new lease N18, 520

To arrive at a consensus the appropriate rent payable will be an average of both parties

view points

Thus: N20, 398 + N18, 520 = N19, 495

2

There are instances where the lessee has to make an expenditure on the property/premises

in consideration for an extension of an existing lease. The value ascribed to the

property/premises will definitely revert to the lessor at reversion; hence the lessee should

be given credit for the value due to his expenditure which will revert to the lessor at the

end of the term. Such sums expended must be taken into consideration when considering

the cost of the new lease to the lessee.

Question:

Page 183: unitedemsa.files.wordpress.com...Pg 2 NIESV STUDY PACK - PPE KNOWLEDGE AND STRUCTURE OF NIGERIA INSTITUTION OF ESTATE SURVEYORS AND VALUERS Learning Objectives At the end of this lecture,

Pg 183

NIESV STUDY PACK - PPE

A property used for banking hall in a city centre is held on a lease having 8 years

unexpired terms at N27, 000 per annum. The present rental value is N40, 000 per

annum. The lessee is willing to spend N100, 000 for improvements and alterations

affecting only the interior of the building which will increase the rental value by

N16, 000 per annum on the condition that the lessor will accept a surrender of the

present lease and grant a new lease for a term of 30 years. The lessee is left with the

option of paying either a fair rent under the new lease or a premium. The lessor will

appreciate a rent of N25, 000 per annum in the new covenant which must enforce

the carrying out of the improvements and the payment of a reasonable premium.

Advice the lessee of the appropriate premium to pay if the new rent is N25, 000 p.a?

(Assume freehold yield @8%)

Solution:

(A) Lessee‟s point of view

Proposed Interest

Rental Value N40, 000

Add value due to outlay N16, 000

N56, 000

Head rent N25, 000

Profit rent N31, 000

YP 30yrs @ 10% & 2.5% (40% tax) 7.25

N224, 750

Deduct expenditure on improvement N100, 000

N124, 750

Page 184: unitedemsa.files.wordpress.com...Pg 2 NIESV STUDY PACK - PPE KNOWLEDGE AND STRUCTURE OF NIGERIA INSTITUTION OF ESTATE SURVEYORS AND VALUERS Learning Objectives At the end of this lecture,

Pg 184

NIESV STUDY PACK - PPE

Present Interest

Rental Value N40, 000

Rent paid N27, 000

Profit rent N13, 000

YP 8yrs @ 10% & 2.5% (40% tax) 3.44

N44, 720

Gain to lessee N80, 030

(B) Lessor‟s point of view

Present Interest

First 8 years N27, 000

YP 8yrs @ 8% 5.75

N155, 250

Reversion

Reversion to rental value after improvements N56, 000

YP in perp @ 9% 11.11

N622, 160

Deduct cost of improvements N100, 000

N522, 160

PV N1 in 8 years @ 9% 0.502

N262, 124

Value of present interest N417, 374

Proposed Interest

First 30 years N25, 000

YP 30yrs @ 8% 11.26

N281, 500

Reversion to full rental value N56, 000

YP Perp deffered 30yrs @9% 0.84

N47, 040

N328, 540

Loss to lessor N88, 834

Page 185: unitedemsa.files.wordpress.com...Pg 2 NIESV STUDY PACK - PPE KNOWLEDGE AND STRUCTURE OF NIGERIA INSTITUTION OF ESTATE SURVEYORS AND VALUERS Learning Objectives At the end of this lecture,

Pg 185

NIESV STUDY PACK - PPE

In a situation where a fair rent is to be determined and not the payment of premium, the

analysis will be as follows:

(A) Lessee‟s point of view

Rental value after embarking on the improvement N56, 000

Deduct annual equivalent of improvement cost N100, 000

Present Interest

Profit rent N13, 000

YP 8 years @10% & 2.5% (tax 40%) 3.44 N44, 720

N144, 720

Divided by YP 30yrs @10% &2.5%

(Tax 40%) 7.25 N19, 961

Appropriate rent for new lease N36, 039 p.a.

Page 186: unitedemsa.files.wordpress.com...Pg 2 NIESV STUDY PACK - PPE KNOWLEDGE AND STRUCTURE OF NIGERIA INSTITUTION OF ESTATE SURVEYORS AND VALUERS Learning Objectives At the end of this lecture,

Pg 186

NIESV STUDY PACK - PPE

(B) Lessor‟s point of view

Value of present interest (as shown above) N417, 374

Deduct value of proposed reversion to

Full rental value after embarking on improvement

(As shown above) N47, 040

Value of proposed term N370, 334

Divided by YP 30yrs @ 8% 11.26

Appropriate rent for new lease N32, 889

Assumption:

Let‟s assume that both parties agreed on non-payment of premium but rather rents are

being made to be just as much as the

It is observed that the envisaged improvement will result to an increase in rent that

depicts about 16% returns which is obviously greater than the rate of interest expected

from the property when let at its rack rent. It is therefore advisable for the lessor to

embark on the improvement himself if he has the means.

Page 187: unitedemsa.files.wordpress.com...Pg 2 NIESV STUDY PACK - PPE KNOWLEDGE AND STRUCTURE OF NIGERIA INSTITUTION OF ESTATE SURVEYORS AND VALUERS Learning Objectives At the end of this lecture,

Pg 187

NIESV STUDY PACK - PPE

VALUATION OF VARRYING INCOME

It is obvious that rents collected on properties don‟t continue forever. After the expiration

of certain number of years there is need for a re-negotiation of rent to reflect the realities

in the property market. This clause either inserted in the lease agreement particularly for

the long leases or a need for renogiation after the expiration of an ongoing lease. There

are instances where concessions are made between landlord and tenant. For instance a

new tenant coming to start up a business in a property could be considered for certain

number of years to get grounds and clientele after which the landlord will enforce the

collection of the rack rent when such tenant has been established. The inflationary trend

in the economy is another vital factor that causes the variation in rental values so as to

meet current economic realities. This inflationary issue in the economy dissuades

landlords from going into long lease agreement with tenants who prefer short leases

where rent could be reviewed to accommodate the economic realities. However, the

tenants don‟t feel secured in short lease arrangement as that might threaten their security

of occupation. In a bid to satisfy the interest of both parties, long lease covenants are

made with a provisory of rental increase for specific future dates. There are also

provisions for rent review clauses fixed in advance in a predetermined manner even if a

predetermined rental increment is not stated from the onset.

An example on ways of valuing varying income will suffice.

Question:

Value a freehold property to be let for 15 years at N20, 000 in the first five years and

subsequently an addition of N10, 000 increment in rent is made in the review after

every five years

Solution:

First five years

Rent reserved N20, 000

YP 5yrs @ 8% 3. 99

N79, 800

Second five years

Rent reserved N30, 000

YP 5yrs @ 9% 3. 89

PV N1 5yrs @ 9% 0.65

YP 5yrs @9% deffered 5yrs 2.53 N75, 900

Page 188: unitedemsa.files.wordpress.com...Pg 2 NIESV STUDY PACK - PPE KNOWLEDGE AND STRUCTURE OF NIGERIA INSTITUTION OF ESTATE SURVEYORS AND VALUERS Learning Objectives At the end of this lecture,

Pg 188

NIESV STUDY PACK - PPE

Third five years

Rent reserved N40, 000

YP 5yrs @ 10% 3. 79

PV N1 10yrs @ 10% 0.3855

YP 5yrs @10% deferred 10yrs 1.46 N58, 400

Reversion to perpetuity

Rent reserved N50, 000

YP perp @ 11% 9. 09

PV N1 15yrs @ 11% 0.2090

YP perp @11% deffered 15yrs 1.8998 N94, 990

CAPITAL VALUE N309, 090

Varying and Deferred Income during Lease Period

In a leasehold interest there are also times when varying income does exist. This is

primarily experienced in two ways: when the rent payable to the lessor is regularly

reviewed during the period of the lease and when rent receivable from the sub-lessee is

reviewed during the sublease period.

Question:

A shop facility has been let by Chief Okon on a full repairing and insuring lease

having 14 years unexpired term. The rent reserved for the next 7 years is N20, 000

per annum after which a review of N25, 000 p.a. will be made for the remaining part

of the term. The premises has immediately been sublet which expires 2 days before

the expiration of the head lease. The rent reserved for the first 7 years is N23, 000

p.a. after which a rent of N30, 000 p.a. will be charged the sub-lessee. The current

rack rent is N35, 000 p.a. Value the leasehold and sub leasehold interest (Assume

freehold yield to be 6%)

Solution:

Leasehold Interest

First 7 years

Rent received N23, 000

Less rent paid N20, 000

Profit rent p.a. N3, 000

YP 7 years @7% & 2.5% net (tax 40%) 3.44

N10, 320

Page 189: unitedemsa.files.wordpress.com...Pg 2 NIESV STUDY PACK - PPE KNOWLEDGE AND STRUCTURE OF NIGERIA INSTITUTION OF ESTATE SURVEYORS AND VALUERS Learning Objectives At the end of this lecture,

Pg 189

NIESV STUDY PACK - PPE

Second 7 years

Rent received N30, 000

Less rent paid N25, 000

Profit rent p.a. N5, 000

YP 7 years @8% & 2.5% net (tax 40%) 3.32

PV N1 7yrs @ 8% 0.5835

YP 7 years @8% & 2.5% net (tax 40%)

Deferred 7 yrs 1.9372

N9, 686

Capital Value N20, 006

Sub- leasehold Interest

First 7 years

Rack rent N35, 000

Less rent paid N20, 000

Profit rent p.a. N15, 000

YP 7 years @8% & 2.5% net (tax 40%) 3.32

N49, 800

Second 7 years

Rack rent N35, 000

Less rent paid N25, 000

Profit rent p.a. N10, 000

YP 7 years @9% & 2.5% net (tax 40%) 3.22

PV N1 7yrs @ 9% 0.5470

YP 7 years @9% & 2.5% net (tax 40%)

Deferred 7 yrs 1.7613

N17, 613

Capital Value N67, 413

Page 190: unitedemsa.files.wordpress.com...Pg 2 NIESV STUDY PACK - PPE KNOWLEDGE AND STRUCTURE OF NIGERIA INSTITUTION OF ESTATE SURVEYORS AND VALUERS Learning Objectives At the end of this lecture,

Pg 190

NIESV STUDY PACK - PPE

VALUATION FOR MORTGAGE PURPOSES

A mortgage is a security created by a contract for the payment of a debt already due or to

be due or of a present or future advance affected by the means of an actual or executing

conveyance of real or personal property.The mortgaged property is charged with the

payment of the money secured, redeemable at law only according to strict legal

conditions of the conveyance, but redeemable in equity independent of such conditions

and enforceable in default of payment by foreclosure.

Essential Elements in a Mortgage

1. A transfer of property or an interest in property by the mortgagor to the mortgagee;

2. The transfer is made as a security for payment of money or grant of money‟s worth;

3. On repayment of the grant, the mortgage property shall be transferred (released) to the

mortgagor having paid all the charges (capital and interest), this therefore means that the

interest in that property granted to the mortgagee shall cease

4. In default of compliance by the mortgagor, will the expressed or implied conditions

and terms in the mortgage deed (contract) enforced.

5. The mortgage deed (document) contains the promise under seal that the loan will be

repaid on an agreed date or by agreed installments. The covenant also contains the

promise to pay interest at a stipulated rate.

Page 191: unitedemsa.files.wordpress.com...Pg 2 NIESV STUDY PACK - PPE KNOWLEDGE AND STRUCTURE OF NIGERIA INSTITUTION OF ESTATE SURVEYORS AND VALUERS Learning Objectives At the end of this lecture,

Pg 191

NIESV STUDY PACK - PPE

Mortgage Repayment could be arranged in one of two (2) ways

• Interests only method:

This method is normally used for relatively short term loans. The capital is repaid in a

lump sum at maturity, ie, @ the end of the mortgage period. Alternatively, repayment

may be arranged in tranches, spaced over the mortgage period. For instance a 30 year

mortgage may be divided into 3-capital repayment period of after 10years, after 20years

and at maturity with interest calculated only on the capital at risk ie the outstanding

capital.

• Annuity method of repayment:

By this method, an annual amount is repaid periodically, usually monthly or yearly over

the period of the loan to cover both interest and capital repayment. This arrangement

ensures that some capital is returned or repaid at the end of each year so that the amount

on capital outstanding reduces year by year. As such, interest on capital decreases and

more of the equal installments is available to repay the capital outstanding. The return on

capital for each year is calculated on capital outstanding only (ie capital at risk). This

method assumes no change in the level of interest rate and it is the method usually

adopted by primary mortgage institutions.

In time past, fixed interest mortgages were predominant. Nowadays, because of the effect

of inflation, variable interest mortgage is becoming more popular. Mortgage could be

upward of 30years on a variety of terms to suit the needs of individual project and

borrower. It could cover both the development period (ie bridging/short term) and long

term.

(Details of mortgage repayment such as the Constant Amortizing loan and (CAM) and

the Fully Amortizing Constant Payment Mortgage Loan (CPM) etc are topics best apt

for lectures in property finance)

Analysis involved in a Mortgage Transaction

APPROACH A: The amount of mortgage to be advanced in a particular transaction can

traditionally be limited by 2-criteria:

• 1. The sum advanced must be a fraction of the OMV of the property

mortgaged usually 2/3 or 75%.

Page 192: unitedemsa.files.wordpress.com...Pg 2 NIESV STUDY PACK - PPE KNOWLEDGE AND STRUCTURE OF NIGERIA INSTITUTION OF ESTATE SURVEYORS AND VALUERS Learning Objectives At the end of this lecture,

Pg 192

NIESV STUDY PACK - PPE

• 2. The net income from the property must exceed the interest (under interest

only repayment) or

(APPROACH B): for interest and capital repayment (in annuity method repayment).

Question:

Mr. Bolaji wishes to take a mortgage with his property a 5bedroom detached house

in Victoria Island Lagos. Comparable property currently let for N3.5million PA net

with a yield of 5% on freehold rack rental basis. Advice the mortgagee on the

maximum amount to advance. The mortgage grants 25year term @a rate of 12.5%

PA

Solution:

(Under Approach A)

(A) Capital Value Analysis

Net Income N3, 500, 000 p.a.

YP in perp @ 5% 20

ECV N70, 000, 000

Advance 2/3 of N70, 000, 000

Recommended Loan (Say) N46, 500, 000

(B) Rental Income Analysis

Mortgage value N46, 500, 000

Annual interest

(Assuming interest only repayment) @ 12.5% 0.125

Annual interest @ 12.5% N5, 812, 500

Based on the policy of the mortgagee, the net income generated by the property cannot

service the recommended mortgage loan of N46, 500, 000. The loan to be advanced to

Mr. Bolaji should therefore be reduced. To obtain the maximum loan to be given, we

work backward from the rent receivable which must be at least 12.5% of the loan that

Page 193: unitedemsa.files.wordpress.com...Pg 2 NIESV STUDY PACK - PPE KNOWLEDGE AND STRUCTURE OF NIGERIA INSTITUTION OF ESTATE SURVEYORS AND VALUERS Learning Objectives At the end of this lecture,

Pg 193

NIESV STUDY PACK - PPE

should be advanced. i.e. if N3, 500, 000 represents 12.5% of the maximum loan to

disburse, what should the amount be?

0.125X = N3, 500, 000

X= N3, 500, 000/0.125 = N28, 000, 000

Hence, N28, 000, 000 represents the maximum amount that N3, 500, 000 annual rent can

support or service for a 25year mortgage term @ 12.5% interest under the interest only

repayment system even though N28, 000, 000 represents just 40% of the estimated

capital value of the property.

In Approach B under the annuity system or method which allows for periodic repayment

of both interest and capital, a much lower figure can be advanced as loan.

APPROACH B: Mortgage usually stipulates that repayment on borrowed funds be

made at regular intervals and at an equal amount/sum.

• Let M=amount on mortgage, P=annual repayment.

• Assuming the mortgagee advances M to the mortgagor for a period of „n‟ years

(n=n integer).

• The mortgagor, not just parts with M but also the amount of interest (i) which M

could have accumulated to over the period.

• (ie, the total amount advanced or forgone is M+accrued interest in n years.

From the mortgagors point of view, the annual repayment (P) by the mortgagor at the end

of the period would accumulate to an amount at i compound interest. What is that

amount/ we therefore apply the following formula to arrive at that amount

Annual Repayment P = M ((1+i)n x i)

(1 + i)n – 1

Mortgage M =P (1+i)n–1

Page 194: unitedemsa.files.wordpress.com...Pg 2 NIESV STUDY PACK - PPE KNOWLEDGE AND STRUCTURE OF NIGERIA INSTITUTION OF ESTATE SURVEYORS AND VALUERS Learning Objectives At the end of this lecture,

Pg 194

NIESV STUDY PACK - PPE

(1+ i)n x i

M= Mortgage loan sum

P= Annual Repayment

P= M A i

A-1

M= P(A -1)

Ai

Question:

A prospective mortgagor can afford N250,000 annual repayment. If the mortgage is

over 25years and at the rate of 10%PA. What is the maximum amount he can

borrow?

Solution:

P= N250, 000

i= 10%

M=?

n= 25 years

M = P (1+i)n–1

(1+ i)n x i

M = 250000 (1.1)25

-1

(1.1)25

x 0.1

250000 ((10.8347) -1)

1.0835

= 250000 ((10.8347) -1)

1.0835

Page 195: unitedemsa.files.wordpress.com...Pg 2 NIESV STUDY PACK - PPE KNOWLEDGE AND STRUCTURE OF NIGERIA INSTITUTION OF ESTATE SURVEYORS AND VALUERS Learning Objectives At the end of this lecture,

Pg 195

NIESV STUDY PACK - PPE

250000 (9.8347)

1.0835

Maximum amount that can be borrowed = N2, 269, 198

Question:

A mortgage institution is prepared to advance a N10million mortgage to their client

who is prepared to repay over a period of 20years and at an interest rate of 15%.

What will be his annual repayment (P)?

Solution:

M = N10, 000, 000

i= 15%

n= 20years

P= M ((1+i)n x i)

(1 + i)n – 1

P = 10, 000, 000 (1.15)20

x 0.15

(1.15)20

-1

P = 10, 000, 000 (16.3665) x 0.15

(16.3665) -1

P = 10, 000, 000 (16.3665) x 0.15

(16.3665) -1

p = 10, 000, 000 (2.4550)

(15.3665)

10, 000, 000 x 0.1597

Annual repayment = N1, 597, 631

APPROACH C: Approach C is to use the mortgage installment table which gives the

monthly installment to redeem every N100 of borrowed capital over a period of n years at

Page 196: unitedemsa.files.wordpress.com...Pg 2 NIESV STUDY PACK - PPE KNOWLEDGE AND STRUCTURE OF NIGERIA INSTITUTION OF ESTATE SURVEYORS AND VALUERS Learning Objectives At the end of this lecture,

Pg 196

NIESV STUDY PACK - PPE

a given rate of compound interest. It is computed on a fixed annual basis without

allowing for interest to compound on each monthly installment.

The formula for the table is adopted from the formula „annuity N1 will purchase single

rate basis‟.

The mortgage installment formula is:

(i + s) 100

12

Where

i = interest received on the capital outlay ie, the remunerative rate on each N1

(return on capital)

s = replacement/return of capital over a given known period ie, the annual

sinking fund to replace the capital invested (return of capital or amortization)

Question:

Mr. Okafor borrowed N100, 000 from a mortgage bank to purchase a house for his

occupation. Interest is to be 10% and the loan repayment is over 25 years. Without the

use of valuation table, calculate what Mr. Okafor will pay monthly to the bank

Page 197: unitedemsa.files.wordpress.com...Pg 2 NIESV STUDY PACK - PPE KNOWLEDGE AND STRUCTURE OF NIGERIA INSTITUTION OF ESTATE SURVEYORS AND VALUERS Learning Objectives At the end of this lecture,

Pg 197

NIESV STUDY PACK - PPE

Solution:

Using the mortgage installment formula

(i + s) 100

12

s= sinking fund of the single rate

0.1 +( 0.1

(1.1)25

– 1) x 100

12

0.1 + 0.010168

12

0.110168x 100

12

0.918

X1000 (remaining multiple of N100, 000 or better still discard the 100 in the formula and

multiply the amount out rightly)

0.918 x 1000

Monthly repayment for 25 yrs @ 10% = N918

Note that:

1. In the calculation above, compounding is on an annual basis that is why we divide by

12. The annual installment, dividing by 12 does allow for compound interest each month.

2. The calculation is on single rate basis ie interest only system without provision for

capital recovery.

3. Using this method, it is possible to calculate the amount of capital outstanding at a

particular time by multiplying the annual repayment by the YP for the unexpired term of

the mortgage period.

Page 198: unitedemsa.files.wordpress.com...Pg 2 NIESV STUDY PACK - PPE KNOWLEDGE AND STRUCTURE OF NIGERIA INSTITUTION OF ESTATE SURVEYORS AND VALUERS Learning Objectives At the end of this lecture,

Pg 198

NIESV STUDY PACK - PPE

Question:

Madam Jegede borrowed N60, 000, 000 over 25 years at 10.5% compound interest.

The annual repayment was N6, 864, 400. What capital will be outstanding after 10

years?

Solution:

Capital outstanding = Annual repayment x YP for the unexpired term

= N6, 864, 400 x 1/i + s

N6, 864, 400 x 1/0.105 +0.105

(1.105)15

-1

Capital outstanding = N6, 864, 400 x 7.40 = N50, 796, 560

CONTEMPORARY VALUATION MODELS

Critics have looked at the traditional valuation methods and raised eyebrow on its

absence of implicit non-growth potentials. Perhaps it is believed not to give a good

representation of the property market and also isolates property investment from other

types of investments. The crux of the matter is how to properly account for growth which

is inevitable and assess the correct yield with reference to the methodology in the other

forms of investment. Amongst the contemporary models are:

• Equated Yield Analysis

• Rational Valuation Model

• Real Value Approaches

• Real Value/Equated Yield Model

Certain variables are germane in the study of these growth explicit models. Hence the

inputs of these variables are very essential. They are:

The equated yield (e)

The growth rate in rentals (g)

Page 199: unitedemsa.files.wordpress.com...Pg 2 NIESV STUDY PACK - PPE KNOWLEDGE AND STRUCTURE OF NIGERIA INSTITUTION OF ESTATE SURVEYORS AND VALUERS Learning Objectives At the end of this lecture,

Pg 199

NIESV STUDY PACK - PPE

The review period in years (t)

The inflation risk free yield (i)

All risk yield (k)

The Equated Yield: This is the inflation prone target yield. This yield as a general note is

subject to the vagaries of inflationary measures. It tends to capture the growth in future

income and is described as the conventional internal rate of return benchmarked at a little

higher yield that the most secured government gilts of a little above 2% or 3%.

Growth Rate in Rentals: This is the envisage growth pattern in rentals usually on an

annual basis to capture the practical reality in the property market over time. Hence

interval changes in incremental income on property. This results to an annual rate of

incremental income on property investment.

Review Period in Years: A pattern of periodic changes in rental evidence to capture the

present realities in the property market. This could be predetermined biennially, once in

three years, five years as the case may be.

The inflation risk free yield: This is also known as the inflation proof yield. This yield is

said to act on a totally inflation proof income and as such regarded as the real return on

such investment. It has the assumption that the rent represents the true value of the

investment and cannot be eroded by the powers of inflation as the rent on its own has the

propensity to review to a new one thereby cancelling the effect of inflation.

All Risk Yield: This can be regarded as the market capitalization yield. This yield

invariably is submerged in the conventional valuation methods as they capture all events

in the market. It is said to make provisions also for the growth pattern experienced in

rental value however in an implicit manner.

These variables have certain connectivity as one can be derived from the other using

certain mathematical models. For instance given this formular:

e-K = (ASF@e)((1+g)t -1)

Other variables can be derived

e-K = (ASF@e)((1+g)t -1)

(e-K)/(ASF@e) +1 = (1+g)t

Page 200: unitedemsa.files.wordpress.com...Pg 2 NIESV STUDY PACK - PPE KNOWLEDGE AND STRUCTURE OF NIGERIA INSTITUTION OF ESTATE SURVEYORS AND VALUERS Learning Objectives At the end of this lecture,

Pg 200

NIESV STUDY PACK - PPE

(e-k) (((1+e)t -1)/e) +1 =(1+g)

t

t

√ ((e-k) (((1+e)t-1)/e) +1) = 1+g

g= t

√ ((e-k) (((1+e)t-1)/e) +1) -1

Question:

A freehold shop property let on a 25 year lease with provision for 3 yearly rent

reviews at N150, 000 has just been sold for N3, 000, 000. Redemption yield on gilts is

10%. What is the implied growth rate?

Solution:

k= 150, 000/ 3, 000, 000 = 0.05 = 5%

e = 10% + 2= 12%

t= 3 yrs

g = t

√ ((e-k) (((1+e)t-1)/e) +1) -1

g = 3

√ ((0.12-0.05) (((1+0.12)3-1)/0.12) +1) -1

√ ((0.07) (((1.12)3-1)/0.12) +1) -1

3

√ ((0.07) ((0.404928)/0.12) +1) -1

3

√ ((0.07) (3.3744) +1) -1

3

√ (0.236208 +1) -1

3

√ (1.236208) -1

(1.0732) -1

g= 0.0732 = 7.32%

Also

Page 201: unitedemsa.files.wordpress.com...Pg 2 NIESV STUDY PACK - PPE KNOWLEDGE AND STRUCTURE OF NIGERIA INSTITUTION OF ESTATE SURVEYORS AND VALUERS Learning Objectives At the end of this lecture,

Pg 201

NIESV STUDY PACK - PPE

((1+g) ((1/(1+e))) = ((1/(1+i))

(1+g) (1+i) = (1+e)

i = (((1+e)/(1+g)) -1)

g = (((1+e)/(1+i)) -1)

e = (((1+g) (1+i)) -1)

i = (((1+e)/(1+g)) -1)

i = ((1+0.12)/(1+0.0732))-1

1.04361-1

i = 0.04361 or 4.361%

There is also the 3 –YP formula which is expressed as follows in deducing unknown

variables:

(1+g)t = YP perp @ K – YP t yrs @ e

YP perp @ K x PV t yrs @ e

Page 202: unitedemsa.files.wordpress.com...Pg 2 NIESV STUDY PACK - PPE KNOWLEDGE AND STRUCTURE OF NIGERIA INSTITUTION OF ESTATE SURVEYORS AND VALUERS Learning Objectives At the end of this lecture,

Pg 202

NIESV STUDY PACK - PPE

Equated Yield Analysis

This is a Discounted Cash Flow Model built into the calculation. It involves the use of an

equated yield which is the discounted rate which when applied to the projected income

allowing for growth produces a net present value of zero.

Question:

A freehold investment has been purchased for N780, 000 having an annual rental value

of N45, 000. If the property is let on a lease for growth at 7.88% per annum, determine

the equated yield. (Assume a trial rate of 13%)

Solution:

Period Growth

factor

Cashflow PV Factor Deferred YP PV of Slice

0 (780, 000) (780, 000)

1-5 1.0000 45, 000 1.0000 3.5172 158, 274

6-10 1.4612 65, 754 0.5428 1.9091 125, 531

11-15 2.1351 96, 080 0.2946 1.0362 99, 558

16 - perp 3.1197 140, 387 0.1599 2.7716 389, 097

NPV -7, 540

Reduce trial rate to say 12%

Period Growth factor

Cash flow PV Factor Deferred YP PV of Slice

0 (780, 000) (780, 000)

1-5 1.0000 45, 000 1.0000 3.6048 162, 216

6-10 1.4612 65, 754 0.5674 2.0454 134, 493

11-15 2.1351 96, 080 0.3220 1.1607 111, 520

16 - perp 3.1197 140, 387 0.1827 3.1668 444, 578

NPV +72, 807

Page 203: unitedemsa.files.wordpress.com...Pg 2 NIESV STUDY PACK - PPE KNOWLEDGE AND STRUCTURE OF NIGERIA INSTITUTION OF ESTATE SURVEYORS AND VALUERS Learning Objectives At the end of this lecture,

Pg 203

NIESV STUDY PACK - PPE

Therefore equated yield = 0.12 + (72807/80347) × 0.01

=0.1291 = 12.91%

Rational Valuation Model

• BASIC FEATURES:

• It values the current (fixed) income stream separately from the reversionary

income flow

• It takes account of the number of years to the next review(s) irrespective of

whether the property is currently rack rented or not

• It uses only a rack rented investment yield capitalization rate at which the property

would be valued in the market if it were leased on regular rent reviews

• It allows growth of the current rental value from the date of the valuation in line

with the long term growth rate implied by the low investment yield rate

The figures below illustrates the cash flow projection assumed by the model for a

reversionary investment with the current rental income, r, current rental value, R and n

years to the next rent review. It is assumed that rental income remains fixed until the first

review at which time the income is increased not to R but R(1+g)n

where g is the implied

rental value growth rate.

Rent Implied rental value

Growth rate (g)

Reversionary Income

R(1+g)n

R

Current fixed income R(1+g)n

r y(1+d)n

r/d – r/d(1+d)n

n Years

Page 204: unitedemsa.files.wordpress.com...Pg 2 NIESV STUDY PACK - PPE KNOWLEDGE AND STRUCTURE OF NIGERIA INSTITUTION OF ESTATE SURVEYORS AND VALUERS Learning Objectives At the end of this lecture,

Pg 204

NIESV STUDY PACK - PPE

CV= (r/d – r/d(1+d)n) + R(1+g)n/y(1+d)n

• CV = Capital Value

• r= Current Rental Income

• R= Estimated Current Rental Value

• y= Rack Rented Capitalization Rate

• n= number of years to next review

• d= risk adjusted opportunity cost of capital

• g= implied growth rate (to be calculated as

• g= (((d-y)/d) ((1+d)t -1) +1)

1/t -1

REAL VALUE APPROACH

The model is founded on the principle that an income can be reviewed at each rent

payment date to a new rental which matches inflation over the intervening period. By so

doing an investor has an inflation proof investment with rental growth in monetary terms

both a static real value profile into perpetuity.

The rate of return required on such an income is the interest rate required for giving up

the capital taking into account all risks attached to the investment but excluding any extra

return for the effects of future inflation. The founder of this model referred to this real

return as the “inflation risk free yield” (i)

If we assume rents are paid annually at the end of each year, then the valuation of an

inflation proofed income would be as follows:

The rent at the end of year × PV N1 @ i

=N1 × 1/(1+i) = 1/(1+i)

In the case of an inflation prone income receivable at the end of the year, the valuation

would be at a higher rate of return i.e. real return adjusted for the inflation rate. In other

words, the valuation of an inflation prone income would be:

Rent at the end of year × PV N1 @ (1+i) (1+d)

=1/((1+i) (1+d))

The determination of yields for the inflation proofed and prone incomes are then

incorporated into the valuation of a periodic rental flow. If it is assumed as the rent

review term and n the whole term, the valuation can be built up in blocks of the review

pattern terms. The present value of N1 per annum formula (1-PV/i) can be substituted for

the PV N1 formula when the rent is to remain static in money terms but declining in real

value. When the rent is assumed static and declining in real terms, the yield is adjusted as

for the PV of N1. Thus,

Page 205: unitedemsa.files.wordpress.com...Pg 2 NIESV STUDY PACK - PPE KNOWLEDGE AND STRUCTURE OF NIGERIA INSTITUTION OF ESTATE SURVEYORS AND VALUERS Learning Objectives At the end of this lecture,

Pg 205

NIESV STUDY PACK - PPE

PV N1 p.a. @i adjusted for d

=(1-(1/(1+i)(1+d)))/((1+i)(1+d)-1)

The rent is assumed to remain static at review in real terms and the capital value of each

block is discounted at the inflation risk free yield. The valuation of a periodically

reviewed rent is undertaken by constructing a series and summating to obtain the real

value formula for the YP of a rising income:

• Ist t years rent NR p.a.

• YP t years (prone) (1-(1/(1+i)(1+d)t ))/((1+i)(1+d)-1) say A

• 2nd

t years rent NR p.a.

• YP t years (prone) as above

• PV t years (proofed) 1/(1+i)t

• 3rd t years rent NR p.a.

• YP t years (prone) as above

• PV 2t years (proofed) 1/(1+i)2t

• Up to and including last t years rent NR p.a.

• YP t years (prone) as above

• PV (n-t) years (proofed) 1/(1+i)n-t

• Summating, we have

• Value= R × A (1 +1/(1+i)t + …..+ 1/(1+i)

n-t )

• Value/R = A (1 +1/(1+i)t + …..+ 1/(1+i)

n-t )

• Multiplying both sides by 1/(1+i)t and since value/R = YP

• YP(1/(1+i)t ) = A (1/(1+i)

t + 1/(1+i)

2t + ……+1/(1+i)

n )

• Subtracting this equation from the former

• YP(1-1/(1+i)t )= A(1-1/(1+i)

n )

• YP = A((1-1/(1+i)n )/(1-1/(1+i)

t )))

• Where A is as defined above

• The valuation of a rising income can be undertaken using the above formula. The

valuer however needs to determine i, d, t and n.

• The greatest criticism of this formula is the complexities involved as it cannot be

said to be easy to comprehend.

• Hence, the formulation of the “Real Value/Equated Yield Approach”

THE REAL VALUE/EQUATED YIELD APPROACH

Page 206: unitedemsa.files.wordpress.com...Pg 2 NIESV STUDY PACK - PPE KNOWLEDGE AND STRUCTURE OF NIGERIA INSTITUTION OF ESTATE SURVEYORS AND VALUERS Learning Objectives At the end of this lecture,

Pg 206

NIESV STUDY PACK - PPE

The main difference between the real value and equated yield model is that while the

former looks at an income profile in terms of its purchasing power (and therefore

discounts fixed income by a yield comprising real return, needs free yield and inflation),

the latter views growth in rents in monetary terms by increasing the rent at the end of the

first year by g, and then discounting at the equated yield, e

Crosby reconciled both approaches by assuming that growth and inflation are the same

thing. He compared the end of year value of an inflation proofed income with one where

the equated yield is used and a growth factor applied to the rent. He argued that there was

no difference between the two methods and that they should produce the same result

METHOD 1: Equated Yield

Value= N1 × PV N1, 1 year @ e= ((1+g) (1/(1+e)))

METHOD2: Real Value

Value = N1 × PV N1, I year @ i adjusted for g

=1/(1+i)

• ((1+g) ((1/(1+e))) = ((1/(1+i))

• (1+g) (1+i) = (1+e)

• i = (((1+e)/(1+g)) -1)

• g = (((1+e)/(1+i)) -1)

• e = (((1+g) (1+i)) -1)

• i = inflation risk free yield

• g = rent growth rate

• e = the equated yield

The valuation of both techniques of a proofed income for one year illustrates the

reconciliation. The equated yield technique increases the rent by g before discounting at

e. The real value approach assumes a reinstatement at the end of the year and discounts

the real value of the rent at a real return. In other words:

• Method 1: Equated Yield

• N1 × (1+g) × PV of N1 in I Year @e = (1+g)/(1+e)

• Method 2: Real Value

• N1 × PV N1 in 1 Year @ i =1/(1+i)

The assumption is that both valuations must equate

=(1+g)/ (1=e) =(1/(1+i))

The above equation is the same as previously found

If n is assumed to be perpetual. The valuation is the addition of a perpetual number of

deferred capital values. The capital value of each block is constant, that is, the rent and

the YP for the term review remain the same. The only change is the number of years in

the present value

Page 207: unitedemsa.files.wordpress.com...Pg 2 NIESV STUDY PACK - PPE KNOWLEDGE AND STRUCTURE OF NIGERIA INSTITUTION OF ESTATE SURVEYORS AND VALUERS Learning Objectives At the end of this lecture,

Pg 207

NIESV STUDY PACK - PPE

Assuming rent is N1 and a rent review is on a 5 yearly pattern. The valuation is of the

following form:

Years 0-5 PV =N1

Years 5-10 PV = 1/(1+i)t

Years 10-15 PV = 1/(1+i)2t

Years 15-20 PV = 1/(1+i) 3t

Last Block PV = 1/(1+i) n-t

The above forms a geometric progression which can be summated as follows:

Sum of YP‟s S = YP t @ e (1+1/(1+i)t + 1/(1+i)

2t + ……. 1/(1+i)

n-t)

Multiply both sides by 1/(1+i)t

S(1/(1+i)t ) = YP t @ e (1/(1+i)

t + 1/(1+i)

2t + ……. 1/(1+i)

3t +……..+ 1/(1+i)

n)

• Deducting the latter equation from the former

S(1-(1/(1+i)t )) = YP t @ e (1-(1/(1+i)

n ))

S = YP t @ e (1-(1/(1+i)n ))

(1-(1/(1+i)t ))

This can be expressed as

YP t @ e ((1-(1/(1+i)n ))/i)

((1-(1/(1+i)t ))/i)

YP t @ e ×(YP n @ i)

( YP t @ i)

VALUATION OF SPECIALIZED PROPERTIES

This talks about valuation of properties that rarely change hands. Gone are the days when

valuation was stereotyped. Thus, valuation has gone beyond the day to residential

apartments, office complex where valuers can give values at the snap of their fingers

particularly as it regards structures having near sale comparable features. In contemporary

times valuation has transcended “the normal every day and ever available structures” to

“bizarre structures” such as shrines, tomb site, mines, deserts, oasis, quarry, worship

grounds, amongst others. This has led to areas like agricultural valuation, environmental

valuation, mineral valuation, wet lands amongst others. In order to buttress these types of

valuations certain questions and answers are provided as a guide for further studies.

Question:

Discuss the various classifications of Agricultural Properties

Solution:

(Discuss on) Crop Farms; Dairy Farms; Perennial Farms; Forestry/Timberland Farms;

Irrigated Land; Livestock Ranches/Stations; Special Purpose Properties

Page 208: unitedemsa.files.wordpress.com...Pg 2 NIESV STUDY PACK - PPE KNOWLEDGE AND STRUCTURE OF NIGERIA INSTITUTION OF ESTATE SURVEYORS AND VALUERS Learning Objectives At the end of this lecture,

Pg 208

NIESV STUDY PACK - PPE

Question:

Discuss the purposes of mineral valuation.

Solution:

(Discuss on) Company balance sheet; Outright sale/purchase; Mergers; Acquisitions; Pricing

of initial public offering of stock, Litigation; Expropriation; Compensation; Income tax

matters; Insurance claim; Exploration lease/concession bids; Initial public offerings; Stock

market transactions; Financial collateral; Insurance claims; Assessment of bank security;

Securities reporting; Accounting purposes, book value adjustments

Question:

Determine the Capital Value of a farm by capitalizing rent as a proportion of farm

profits. From records the farm output includes: Crops- 6 ha Maize × 5 tonnes/ha @

N200, 000/tonnes; Dairy – 70 dairy cows each producing 5000 litres per annum @N120

per litre; Livestock fattening- 45 fatted cows @ N80, 000/cow. Certain expenditures

incurred include feeding stuff 2500kg @N1080/kg; Maize seeds 60 ha @N30,000/ha;

Labour- 12 men(N750,000/Man); Fuel, oil, repairs of machinery and other

miscellaneous @15% of the total cost of expenditure as stated above. (Assume farm

yield rate @7% and the rent proportion of profit @25%)

Solution:

Farming Output N Farm Expenditure

Estimate

N

Crops

(6×5×200000)

6, 000, 000 Feeding Stuff

(2500×1080)

2, 700, 000

Dairy

(70×5000×120)

42, 000, 000

Maize seeds

(60×30000)

1, 800, 000

Livestock

Fattening

(45×80000)

3, 600, 000

Labour

(12×750000)

9, 000, 000

Page 209: unitedemsa.files.wordpress.com...Pg 2 NIESV STUDY PACK - PPE KNOWLEDGE AND STRUCTURE OF NIGERIA INSTITUTION OF ESTATE SURVEYORS AND VALUERS Learning Objectives At the end of this lecture,

Pg 209

NIESV STUDY PACK - PPE

Miscellaneous 2, 025, 000

Total 51, 600, 000 Total 15, 525, 000

Profit margin = output – expenditure

51600000-15525000 = N36, 075, 000

Rent porportion of profit @25% = N9, 018, 750

Rent = N9, 018, 750

YP Perp @7%= 14.29

CV= N128, 877, 938

Question:

Differentiate between Severance and Injurious Affection in compensation valuation

Solution:

Severance - Part of a property being acquired (severed) the remaining part retained has its

value reduced

Injurious affection – Depreciation caused to the remaining land of a claimant as a result

of the new use of the land severed

Question:

Discuss the Development Valuation Process

Solution:

1. Valuer receives instruction and peruses all relevant documents

2. Valuer should consult client and determine valuation basis to suit client

requirement. The basis could be development value for the highest and best use

Page 210: unitedemsa.files.wordpress.com...Pg 2 NIESV STUDY PACK - PPE KNOWLEDGE AND STRUCTURE OF NIGERIA INSTITUTION OF ESTATE SURVEYORS AND VALUERS Learning Objectives At the end of this lecture,

Pg 210

NIESV STUDY PACK - PPE

while the valuation method could be comparison (where practicable) or residual

valuation.

3. Valuer will undertake valuation and market survey/inspection to seek the

following: market area data; inspect property to determine strong visual reference

to matter which affect value and cost; examine comparable property data;

4. Valuer analyses gathered data

5. Valuer prepares and submits his development valuation report

Question:

Using the zonal travel cost approach, value the economic benefit of a National Park

used for recreation. From the records of staff of the park of the visitors in the

various concentric zones to the park the following are gathered: Distance time

cost=Cost/km is N0.40; Travel time cost =Cost/minute is N0.25. The first to fourth

concentric zones have these respective details of round trip travel distance and

round trip travel time as 0Km/0Minutes; 22Km/35Minutes; 45Km/80Minutes

and90Km/135Minutes. Entrance fee to the park is N20 while the number of visits

from each zone per 1,000 of the population with the approximate population of each

zone is given as 315/1000; 196/2000; 105/4000 and 25/8000.

Solution:

Zone Round Trip

Travel

Distance(Km)

Round Trip

Travel

Time(Min)

Distance

times

cost/Km

(N0.40)

Travel Time

time

Cost/Minute

(N0.25)

Total Travel

Cost/Trip

0 0 0 0 0 0

1 22 35 N8.8 N 8.75 N 17.55

2 45 80 N 18 N 20 N 38

3 90 135 N 36 N 33.75 N 69.75

Entrance fee to the park is given as N 20. 00

Page 211: unitedemsa.files.wordpress.com...Pg 2 NIESV STUDY PACK - PPE KNOWLEDGE AND STRUCTURE OF NIGERIA INSTITUTION OF ESTATE SURVEYORS AND VALUERS Learning Objectives At the end of this lecture,

Pg 211

NIESV STUDY PACK - PPE

Zone Travel Cost

plus N20.

00

Visits/1000 Population Total Visits Capital

Value

(Column 2 x

Column 5)

0 N 20 315 1000 315 N 6, 300

1 N 37.55 196 2000 392 N 14, 719.6

2 N 58 105 4000 420 N 24, 360

3 N 89.75 25 8000 200 N 17.950

1327 N 63, 329.6

Question:

Discuss the basis of environmental valuation with its components part

Solution:

Basis of environmental valuation is Total Economic value.

This is made up of Use and Non-use value.

Use Value (Direct and Indirect Use Value)

Non- Use Value (Exitence value; Bequest Value; Option Value

Question:

Discuss the factors that affect the value of mineral assets

Solution:

Legal rights and interests; Quantity of mineral; Quality of mineral; Access to markets;

Government controls; Capital and operating costs; Likely output to be justified by the

potential market; Taxation and royalties; Required capital; Timing and Completion of

Capital projects; Services and infrastructure provided

Page 212: unitedemsa.files.wordpress.com...Pg 2 NIESV STUDY PACK - PPE KNOWLEDGE AND STRUCTURE OF NIGERIA INSTITUTION OF ESTATE SURVEYORS AND VALUERS Learning Objectives At the end of this lecture,

Pg 212

NIESV STUDY PACK - PPE

Question:

Determine the freehold value of limestone stocks in quarry along the Lagos Abeokuta

Expressway using the Hotelling method. It is observed that the annual quantity of the

resources extracted is 2,500,000 tonnes. The stock of the remaining recoverable or

established reserves is 87,550,000. The annual fees, taxes and royalties paid by the

mineral licensees to the Government amounts to N106, 850,000. Assume the Net

Current Replacement Cost (NCRC) of the Plant and Machinery is N90, 000,000 and the

estimated value of market share of the quarry is N1, 000,000

Solutions:

Value (Reserves)

= (RR/Q)/S

106850000X87550000

2500000

= N3, 741, 887, 000

Add replacement cost of plant vand machinery and estimated value of market shares

Capital Value = N90, 000, 000 + N1, 000, 000

=N3, 832, 887,000

Question:

In a sequential order discuss the past and present laws governing compensation in

Nigeria

Solution:

Discuss on the various laws

1. Public Land Acquisition Act Cap 167 of 1917 Modified by the Public Lands

Acquisition (Miscellaneous Provisions) Act of 1958

2. The Northern Nigeria Land Tenure Law of 1962

Page 213: unitedemsa.files.wordpress.com...Pg 2 NIESV STUDY PACK - PPE KNOWLEDGE AND STRUCTURE OF NIGERIA INSTITUTION OF ESTATE SURVEYORS AND VALUERS Learning Objectives At the end of this lecture,

Pg 213

NIESV STUDY PACK - PPE

3. State Lands Act (Cap 45) and the State Lands (Compensation) Decree 38 of 1968

4. Oil Pipeline Act 1956 (as amended by the 1962, 1965 and 1990 Acts) and the

Petroleum Act Cap. 350 of 1990

5. The Public Land Acquisition (Miscellaneous Provisions) Decree No. 33 of 1976

6. Land Use Decree (now Act) No. 6 of 1978

Question:

Discuss the uses of Environmental valuation

Solution:

Discuss these

1. Cost-benefit analysis

2. Full pricing for compensation and other issues

3. Implementation of the polluter pays principle (PPP)

4. Greening National Balance Sheet

5. Valuation of Sacred Properties and properties of cultural heritage

6. Non-market valuations

7. Property pricing

Question:

Discuss the Agricultural Valuation Process

Solution:

Discuss these

1. Receives Instructions

2. Determine valuation basis

3. Valuation and market inspection/survey

4. Analysis of data gathered

5. Prepares and submit farm valuation

Question:

Giving any workable example of your choice discuss the development valuation

method in valuing real estate

Page 214: unitedemsa.files.wordpress.com...Pg 2 NIESV STUDY PACK - PPE KNOWLEDGE AND STRUCTURE OF NIGERIA INSTITUTION OF ESTATE SURVEYORS AND VALUERS Learning Objectives At the end of this lecture,

Pg 214

NIESV STUDY PACK - PPE

Solution:

This is based on the principle of GDV – TDC =Residual value. Use that as a basis for a

practical example of your choice.

Page 215: unitedemsa.files.wordpress.com...Pg 2 NIESV STUDY PACK - PPE KNOWLEDGE AND STRUCTURE OF NIGERIA INSTITUTION OF ESTATE SURVEYORS AND VALUERS Learning Objectives At the end of this lecture,

Pg 215

NIESV STUDY PACK - PPE

COURSE TITLE: APPLIED VALUATION 11 (PLANT AND MACHINERY)

INTRODUCTION

Machines

„Machine‟ is a term derived from a Latin Word „Machina. As a scientific definition a

“machine" is any device that transmits or modifies energy. In common usage, the

meaning is that of devices having parts that perform or assist in performing any type of

work or a particular type of work. Machines normally require some energy source

("input") and always accomplish some sort of work ("output"). Machines of all types

make work easier by changing the size or direction of an applied force. A basic

distinction between a machine and a tool is that the latter is a device that has no rigid

moving parts. People have used mechanisms to transform one form of motion or energy

into another. Modern power tools, automated machine tools, and human-operated power

machinery are tools that are also machines.

It behooves here to reiterate that machine is a device having a unique purpose that

augments or replaces human or animal efforts for the accomplishment of physical tasks.

All machines have an input and output device. The outputs of some machines constitute

the input of others such as generators. The outputs of generators (electrical, hydraulic and

pneumatic energy) are used to operate many other machines like material processing,

packaging, conveying and washing machines.

TYPES OF MACHINES

There are two major types of machines: The simple and the complex machine. Simple

machines are types of machines that works with one movement or if that do perform their

action in one movement. The machines may be used to support industrial applications,

although devices lacking rigid movable parts are not commonly considered machines.

Hydraulics are mostly used in heavy equipment industries, automobile industries,

aeronautical industries, construction equipment industries, industries. However, there are

6 simple machines; the inclined plane, the wedge, the screw, the lever, the pulley, and the

wheel and axle. Of these, the lever, the pulley, and the inclined plane are the primary; the

wheel and axle, the wedge and the screw are referred to as secondary machines. A

combination of wheel and axle is a rotary lever, while the screw may be considered an

inclined plane cut around a core. The wedge is a double inclined plane. On the other

hand, Complex machines comprise elementary machines called simple machines, such as

the wedge and the pulley. A wheelbarrow can be described as an example of a complex

machine that uses a lever and a wheel and axle. Complex machines are selected based

Page 216: unitedemsa.files.wordpress.com...Pg 2 NIESV STUDY PACK - PPE KNOWLEDGE AND STRUCTURE OF NIGERIA INSTITUTION OF ESTATE SURVEYORS AND VALUERS Learning Objectives At the end of this lecture,

Pg 216

NIESV STUDY PACK - PPE

on the operations they perform; the complicated devices used for sawing, planning, and

turning, for example, are known as sawing machines, planning machines, and turning

machines respectively and as machine tools collectively.

Machines transform other forms of energy (as heat) into mechanical energy are known as

engines. The electric motor transforms electrical energy into mechanical energy. Its

operation is the reverse of that of the electric generator, which transforms the energy of

falling water or steam into electrical energy. The various types of simple machines may

be discussed as below:

The inclined plane: This is a plane surface set at an angle, other than a right angle,

against a horizontal surface. The inclined plane allows one to overcome large

resistance by applying a relatively small force through a longer distance than

required to raise the load.

The figure below gives an illustration of an inclined plane.

Figure1: An Inclined Plane:

Examples of Inclined plane include: Ramps, Sloping roads, e.t.c.

Wedges: The wedge is an active twin of the inclined plane. Unlike an inclined

plane that is stationary, the wedge functions by moving. This machine consists of a

set of inclined planes set together that can sustain relative sliding or rolling

motion. By moving one plane relative to the other, a wedge is capable of building

relative force in a direction perpendicular to that of the moving wedge. Force

increase varies inversely with the size of the wedge angle; a sharp wedge (small

inclined plane) yields a large force. With adequate friction at the interfaces, the

Force

Resistance

Inclined plane

Page 217: unitedemsa.files.wordpress.com...Pg 2 NIESV STUDY PACK - PPE KNOWLEDGE AND STRUCTURE OF NIGERIA INSTITUTION OF ESTATE SURVEYORS AND VALUERS Learning Objectives At the end of this lecture,

Pg 217

NIESV STUDY PACK - PPE

wedge becomes a separating, holding and stopping device with countless

mechanical applications. In the broadest sense wedges include all devices for

cutting and piercing. Examples include kitchen knife, chisel, hatchet, carpenter

planes, air hammer, e.t.c.

Figur2: A Wedge:

Screw: A simple screw is a helical (a three-dimensional twisted shape) inclined

plane. A screw can convert a rotational force (torque or moment) to a linear force

and vice versa. The ratio of threading determines the mechanical advantage of the

machine. More threading increases the mechanical advantage. A rough

comparison of mechanical advantage can be done by taking the circumference of

the shaft of the screw and divide by the distance between the threads.

A screw is a shaft with a helical groove or thread formed on its surface and

provision at one end to turn the screw. Its main uses are as a threaded fastener

used to hold objects together, and as a simple machine used to translate torque into

linear force. It can also be defined as an inclined plane wrapped around a shaft.

Lever: A lever is a simple machine that makes work easier for use; it involves

moving a load around a pivot using a force. Levers are simple machines used to

lift weights. From a layman‟s point of view it is pertinent to define some terms

germane to the study of this simple machine. These are: the load, the effort, and

the pivot (fulcrum). A load is the thing you're lifting. A fulcrum is the thing that

makes the load lighter. An effort is the person pushing to make the objects move.

There are three different types of levers. They are the first class, second class and

third class levers.

First class lever- A first-class lever has its fulcrum located between the input

effort and the output load. In operation, a force is applied (by pulling or pushing)

LOAD (LARGE)

EFFORT (SMALL)

Wedge

Page 218: unitedemsa.files.wordpress.com...Pg 2 NIESV STUDY PACK - PPE KNOWLEDGE AND STRUCTURE OF NIGERIA INSTITUTION OF ESTATE SURVEYORS AND VALUERS Learning Objectives At the end of this lecture,

Pg 218

NIESV STUDY PACK - PPE

to a section of the bar, which causes the lever to swing about the fulcrum,

overcoming the resistance force on the opposite side. The fulcrum may be at the

center point of the lever as in a seesaw or at any point between the input and

output. This supports the effort arm and the load. Apart from the seesaw other

types of the first class lever include: bicycle hand brake, can opener, hammer

(when pulling the nail with the hammers claw), pliers (double lever), scissors

(double lever), oars, shoehorn e.t.c.

Second class lever- In a second class lever the input effort is located at one end of

the bar and the fulcrum is located at the other end of the bar, opposite to the

input, with the output load at a point between these two forces. Examples include:

dental elevator, door, nutcracker, springboard, wheelbarrow, bottle opener, e.t.c

Third class lever- For this class of levers, the input effort is higher than the output

load, which is different from second-class levers and some first-class levers.

However, the distance moved by the resistance (load) is greater than the distance

moved by the effort. Since this motion occurs in the same length of time, the

resistance necessarily moves faster than the effort. Thus, a third-class lever still

has its uses in making certain tasks easier to do. In third class levers, effort is

applied between the output load on one end and the fulcrum on the opposite end.

Examples include: baseball bat, boat paddle, broom, fishing rods, hockey sticks,

hoe, nail clippers, tweezers, stapler, e.t.c.

fulcrum

Effort Load

Effort Load

fulcrum

m

Page 219: unitedemsa.files.wordpress.com...Pg 2 NIESV STUDY PACK - PPE KNOWLEDGE AND STRUCTURE OF NIGERIA INSTITUTION OF ESTATE SURVEYORS AND VALUERS Learning Objectives At the end of this lecture,

Pg 219

NIESV STUDY PACK - PPE

Pulley: A pulley (also called a sheave or block) is a wheel with a groove between

two flanges around its circumference. A rope, cable or belt usually runs inside the

groove. Pulleys are used to change the direction of an applied force, transmit

rotational motion, or realize a mechanical advantage in either a linear or rotational

system of motion. Pulley systems are used in the real world to lift large masses

onto tall heights. The pulley system consists of one or more pulleys and a rope or a

cable. The number of pulleys used may increase or decrease the mechanical

advantage of the system. Generally, the higher the mechanical advantage is, the

easier it is to lift the object that is being lifted. Overall, no matter how easy it is to

use the pulley system, the system itself is not very efficient due to the force of

friction. For example, one has to pull two meters of rope of cable through the

pulleys in order to lift an object one meter. Examples of pulley include crane and

flag pole.

Wheel and axle

The wheel and axle is a simple machine. The form consists of a wheel that turns an

axle and in turn a rope converts the rotational motion to linear motion for the purpose

of lifting. By considering the machine as a torque multiplier, i.e. the output is a

torque, items such as gears and screwdrivers can fall within this category.

A wheel and axle is a lever that rotates in a circle around a center point or fulcrum.

The larger wheel (or outside) rotates around the smaller wheel (axle). Bicycle wheels,

ferris wheels and gears are all examples of a wheel and axle. Wheels can also have a

solid shaft with the center core as the axle such as a screwdriver or drill bit or the log

in a log rolling contest.

A wheel is considered a lever that can turn 360 degrees and can have an effort or

resistance applied anywhere on that surface. The effort or resistance force can be

applied either to the outer wheel or inner wheel (axle). Steering wheels and door knob

are good examples.

Load Effort

Fulcrum

Page 220: unitedemsa.files.wordpress.com...Pg 2 NIESV STUDY PACK - PPE KNOWLEDGE AND STRUCTURE OF NIGERIA INSTITUTION OF ESTATE SURVEYORS AND VALUERS Learning Objectives At the end of this lecture,

Pg 220

NIESV STUDY PACK - PPE

The wheel and axle is similar in appearance to a pulley, with one major difference: the

wheel is fixed to the axle, as is the steering wheel of a car. A user applies effort to the

large outer wheel of the steering wheel to move the load at the axle. The Mechanical

Advantage of a wheel and axle is equal to the radius of the wheel divided by the

radius of the axle. The radius of the wheel, and therefore its circumference, is usually

much larger than the radius of the axle. Therefore, the distance over which the effort

is applied is much greater than the distance the load, which is placed at the axle,

moves. The difference in the sizes of the wheel and axle can result in a large

mechanical advantage. Some common examples of a wheel and axle are a doorknob

and a round water faucet handle.

MECHANICAL ADVANTAGE AND EFFICIENCY OF MACHINES

Mechanical advantage (MA) of a simple machine is the ratio between the force it

exerts on the load and the input force applied. In a layman‟s view MA can be

described as the amount of effort saved when using machines. This does not entirely

describe the machine's performance, as force is required to overcome friction as well.

The mechanical efficiency of a machine is the ratio of the actual mechanical

advantage (AMA) to the ideal mechanical advantage (IMA). Functioning physical

machines are always less than 100% efficient.

Machinery

Machinery on the other hand can be defined as an assemblage of machines or

mechanical apparatus. The word machinery is believed to be clouded with varied

explication. Notwithstanding, these diverse definitions, that proposed by the

Companies Act which defines the term “Plant and Machinery” as the fixed assets of a

company other than land and buildings tend to suffice on global detection. Motor

vehicles, mobile plants, ships, locomotives, airplanes and similar assets (which

patently are not physically fixed) will normally be considered to be plant and

machinery.

Plant

The term “Plant” refers to all total of machines used in a company for production

process. A straight jacketed definition of plant could sway professional judgment even

though that forms the basis for the recognition of any plant. However, due to the

susceptibility of diverse meanings attributed to “plant” depending upon its settings

and context of assessment, it could be impossible to define it exhaustively. After the

recognition of a fixed asset being the combination of series of machine for anything to

Page 221: unitedemsa.files.wordpress.com...Pg 2 NIESV STUDY PACK - PPE KNOWLEDGE AND STRUCTURE OF NIGERIA INSTITUTION OF ESTATE SURVEYORS AND VALUERS Learning Objectives At the end of this lecture,

Pg 221

NIESV STUDY PACK - PPE

be considered as “plant” it must have the following attributes: it must have some

degree of durability. So anything which is quickly consumed or worn out in the

course of a few operations or within a short time cannot be called “plant”. It must

fulfill the function of a plant in the activity to which it is employed. i.e. it must be a

tool of business- a means of carrying on a business. It would not cover the stock in

trade i.e. goods ought or made for sale by a businessman. It would not include an

article which is merely a part of the premises in which the business is carried on.

Hence, a plant may include any article or object fixed or moveable, live or dead used

by a businessman for carrying on his business. It is pertinent to state here that most

times plant and machinery can be thought of as asset of a company other than land

and buildings. However, motor vehicles, mobile plant, ships, locomotives and similar

assets not physically fixed are not necessarily considered as plant and machinery

except when they are utilized or form part of the production cycle.

Any difference amongst Plant, machinery and other related appellations

The term plant has been described as the combination of these machines geared

towards achieving the same purpose. Hence machinery and plant have been

attributed to mean the same thing. This calls for why agitation is being made to

change the appellation of this course from “plant and machinery valuation” to

“machines and plant valuation”. As the latter term will cover all small devices

geared towards making work easy and faster to accomplish. More so the plant and

machinery valuation faculty of the Royal Institution of Chartered Surveyors has

changed its name to machinery and business assets faculty. The American Society

of Appraisers prefers to call its faculty machinery and technical specialties. In a nut

shell plant and machinery, machines and plant, plant and equipment, machinery and

business assets or whatever name is chosen refers to the installations and support

facilities especially designed and installed to perform predetermined function in the

course of manufacturing or production process whether singly or in the combination

of other items. Therefore plant and machinery or whatever appellation is adopted

include: industrial building housing the plant and production process; building

service installations normally included in the valuation of land and buildings;

process plant; plant or equiptment which might have been installed wholly in

connection with the operator‟s industrial or commercial process together with

furniture, furnishings, fixtures and fittings, vehicle, tools, etc.

This first lecture is aimed at giving students a base on getting acquainted with certain

basic terms and recognition of these terms as the study goes along. For instance terms

Page 222: unitedemsa.files.wordpress.com...Pg 2 NIESV STUDY PACK - PPE KNOWLEDGE AND STRUCTURE OF NIGERIA INSTITUTION OF ESTATE SURVEYORS AND VALUERS Learning Objectives At the end of this lecture,

Pg 222

NIESV STUDY PACK - PPE

like simple machines, complex machines synonymous with machinery and plants

should no longer be strange as these form part of assets of companies. Students of

Estate Management should note that asset valuation is part of their exclusive reserves

as valuers after graduation. However, plant and machinery valuation is only confined

to valuation of machinery and plants. In the case of asset valuation other equipments

and apparatus such as furniture, fittings and fixtures become inclusive for valuation

except where these devices can be categorically classified as machines. The simple

machines are however very pertinent in the study of plant and machinery as they form

the basis of operation of any form of device designed to make the work and activities

of man easier and accomplishable at record time.

CONSTITUENT OF MACHINES AND PLANT

Generally, any equipment which provides part of the services to the property and which

would normally be sold with the property on the open market will be considered as part

of the building. On the other hand, all equipments primary concerned with the process of

production should be thought of as plant and machinery.

Plant and machinery forms an integral part of the capital assets of companies and

establishments, especially in production and manufacturing concerns where their

activities involves intensive use of plant and machinery. There could be confusion even

among valuers as to what constitute plants and machinery particularly in a non-

production concern; hence caution needs to be taken in identification.

A company in existence is made up of various assets. These assets are broadly classified

into fixed assets and current assets. Current assets are those items which are expected to

be consumed or sold in the course of trading connected with the business or production

cycle. Thus, theses cannot be regarded as plant and machinery. The fixed assets of a

company on the other hand could be defined as those assets which are intended for use on

a continuing basis for the purpose of the company‟s activities in the production or supply

of goods and services. Fixed asset are sub-divided into the following three categories:

Tangible assets: these include

Land and buildings

Plant and machinery

Motor vehicles

Other fixtures and fittings, tools and equipment; and,

Payments on account of tangible assets in course of construction

Intangible assets:

Cost of research and development

Page 223: unitedemsa.files.wordpress.com...Pg 2 NIESV STUDY PACK - PPE KNOWLEDGE AND STRUCTURE OF NIGERIA INSTITUTION OF ESTATE SURVEYORS AND VALUERS Learning Objectives At the end of this lecture,

Pg 223

NIESV STUDY PACK - PPE

Patents, licenses, trademarks if required for valuable consideration or created as an

asset

Goodwill to the extent that it may be acquired for valuable consideration; and

Payments on account of intangible assets

Financial assets:

Own shares

Investments

Loans to third parties; and

Shares in affiliates

Taking a look at the various forms of assets of a company, it is obvious that plant

and machinery come under the fixed asset. A general classification of items that

constitute plant and machinery is as follows:

Plant and Machinery used for:

The generation, storage, primary transformation or main transformation of power

i.e. generators, transformers, switchgears etc

The heating, cooling, ventilating, lighting, drainage or supply of water i.e. boilers,

compressors, water pumps, water reservoirs, water and effluent treatment plant etc

Lifts and elevators used for passengers:

Railway and train way lines, tracks, coaches:

Pipelines, gas and electricity pipes:

Petrol station equipment i.e. pumps, fuel reservoirs, oil tankers etc

Firefighting equipment

Motor vehicles, including trailers, trucks, buses etc

Cranes and mechanical equipment e.g. tippers, pay loaders, forklift, drilling

equipment

Ship and barges

Aeronautical equipment, including planes, helicopters, space ships etc

Communication equipment such as telephones and telecommunication

installations, computers, etc

Farm implements such as tractors, harrows, incubators, hatchers etc

Plant and Machinery in the nature of a building or structure including blast

furnaces, coke ovens, tar distilling plants cupolas or water towers with tanks

Page 224: unitedemsa.files.wordpress.com...Pg 2 NIESV STUDY PACK - PPE KNOWLEDGE AND STRUCTURE OF NIGERIA INSTITUTION OF ESTATE SURVEYORS AND VALUERS Learning Objectives At the end of this lecture,

Pg 224

NIESV STUDY PACK - PPE

As a reminder it should be noted that in some companies motor vehicles could also

be classified as plant and machinery depending on its mode of operation. From the

aforementioned students should begin to have a clear picture of what constitutes

plant and machinery in an organization. This identification of plant and machinery

in any organization is the bane to most valuers. A successful

recognition/completion of what constitutes plant and machinery in every

organization has perhaps solved more than half of what is required in the valuation

exercise.

PURPOSES OF MACHINE AND PLANT VALUATION

Having described and indentified the types and nature of plant and machinery,

there is the need to discover why plant and machinery needs to be valued. We all

know that the business world is highly industrialized. Many jobs previously

accomplished manually is being done by machines. Hence, due to the large sums

of money spent on acquisition and installation of plants and equipments in

factories, industries, offices, and other organizations all over the world, there is

need to keep abreast of developments in the plant and machinery business climate.

Also there have been situations that had caused people uncomfortable and many

businesses ruined for no cause of their own through fire outbreak, flood and

earthquakes etc. for these unforeseen circumstances, the issue of insuring these

machines and equipments become mandatory to mitigate loss. Also, for updating

the accounts of a company or for the determination of the premium payable to the

insurance company, in an attempt at guiding against under-insurance, plant and

machinery valuers are also called upon to determine the worth of these equipments

periodically. There are instances whereby the equipments in question will have to

change hands. Thus, it can be observed that plant and machinery valuation is

required for varied reasons which can be summarized into three broad categories:

Financial valuation:

This involves the establishment of the economic worth of the plant and machinery

for an ongoing concern. That is, the plant and machinery remain in the same

position, the same hands and contributing to the profitability of the organization.

This exercise may be carried out for the following reasons:

Merger and takeover

Insolvency

Settlements of disputes

Balance sheet

Page 225: unitedemsa.files.wordpress.com...Pg 2 NIESV STUDY PACK - PPE KNOWLEDGE AND STRUCTURE OF NIGERIA INSTITUTION OF ESTATE SURVEYORS AND VALUERS Learning Objectives At the end of this lecture,

Pg 225

NIESV STUDY PACK - PPE

Internal performance analysis

Published financial statements

Investigation

Nationalization

Funding

Takeover defence

Floatation

Taxation

Privatization

Open market valuation:

Open Market Valuation is required when a company actually intends to enter into

a transaction or it wishes to borrow money using its plant and machinery as

security for the loan. Hence it is required by:

Sellers wanting to know how much they might expect to achieve

Accountants investigating companies with financial problems

Buyers wishing to know how much to offer

Bankers and lenders for loan security purpose

Accountants handling the affairs of failed companies

These categories of people seek to establish the worth of the plant and machinery

assets if sold outside the business of the company, provided that the plant and

machinery would be sold by private treaty assuming:

- An able and willing buyer

- A willing seller

- A reasonable period within which to negotiate the sale taking into

consideration the nature of the plant and machinery and the state of the market

- The parties in the transaction are knowledgeable, prudent and without

compulsion

- Values would remain stable throughout the reasonable period of negotiation at

prevailing market condition

- The plant and machinery would be freely exposed to the market

- No account would be taken of an additional bid by a special purchaser

- No account would be taken of expenses of realization which may arise in the

event of disposal

Insurance valuation:

Page 226: unitedemsa.files.wordpress.com...Pg 2 NIESV STUDY PACK - PPE KNOWLEDGE AND STRUCTURE OF NIGERIA INSTITUTION OF ESTATE SURVEYORS AND VALUERS Learning Objectives At the end of this lecture,

Pg 226

NIESV STUDY PACK - PPE

Insurance valuation determines the value of plant and machinery which represents

the actual amount to indemnify the insured in the event of occurrence of loss or

risks against which insurance has been taken. The two basis of insurance valuation

are as follows:

- Reinstatement basis of valuation: This simply implies giving the insured a new

one to replace the old one in case of total loss if there is no under insurance

clause.

There is need to distinguish between Insurable Replacement Cost (IRC) and

Gross Replacement Cost (GRC) approach.

Gross replacement cost valuation makes allowance for finance costs and

charges, which when considered for a capital intensive plant may be enormous.

In Insurable replacement cost, finance costs and charges are not considered

rather each department in the factory is grouped according to its possibility of

catching fire. Thus, each department is rated into the level of risk.

- Indemnity insurance basis of valuation: In this case items insured are placed

exactly the position or condition it was immediately before the occurrence of

the loss. The claim settlement is either by repair, replacement with equivalent

one where obtainable or by payment of cash equivalent.

Page 227: unitedemsa.files.wordpress.com...Pg 2 NIESV STUDY PACK - PPE KNOWLEDGE AND STRUCTURE OF NIGERIA INSTITUTION OF ESTATE SURVEYORS AND VALUERS Learning Objectives At the end of this lecture,

Pg 227

NIESV STUDY PACK - PPE

Note that:

1. The cost of reinstating the plant and machinery damaged if it is not a

total loss or if destroyed, the plant and machinery will be replaced as in

the condition it was immediately before the incidence plus

2. Provision for loss of rent during the reconstruction/repair period plus

3. Cost of alternative accommodation during the repairs or reconstruction

period

4. The inflation rate will be assumed constant during the construction

period

BASES OF MACHINES AND PLANT VALUATION

Just as you have been thought in your conventional valuation practices that involve

the valuation of land and buildings, it is also applicable in the valuation of plant and

machinery that purpose defines the basis while the basis determines the method of

valuation to adopt. Bases can be described as the ground on which the valuation

should take place. The purpose of valuation could answer the question why should

valuation be carried out in the first place. Bases should explain what the valuation

exercise intends to achieve while the method of valuation expounds on how the

valuation exercise should be carried out. The method of valuation used and the final

value estimates are largely determined by the Basis of valuation employed which

has to reflect the requirements of the instruction, the use to which the property is, or

will be put and the type of property. The requirements of the instruction are varied

and depend upon the particular need of the client. Property types, on the other hand,

include „specialized‟ and „non-specialized‟ held either as owner-occupied,

investment or surplus to the requirement of the business.

For the sake of convenience, the bases for plant and machinery valuation are

classified under two broad groups. Thus: „the going concern basis‟ and „the gone

concern basis‟

In carrying out the valuation of plant and machinery, it is important to identify

which of the two concepts is relevant to the intended purpose.

Going-concern basis: the concept as the name suggests assumes that the

assets being valued will continue in the processes for which they are

purchased and installed. Going-concern basis means the same as „existing

use‟ or „value-in-use basis. Other names ascribed to the going concern

basis of valuation include „accounting purposes valuation‟, and the „in-

Page 228: unitedemsa.files.wordpress.com...Pg 2 NIESV STUDY PACK - PPE KNOWLEDGE AND STRUCTURE OF NIGERIA INSTITUTION OF ESTATE SURVEYORS AND VALUERS Learning Objectives At the end of this lecture,

Pg 228

NIESV STUDY PACK - PPE

house accounting valuation‟. The going-concern basis of valuation is

described in the RICS guidance note and the International Valuation

Standards Committee (IVSC) notes on the valuation of plant and

machinery as „Value to the Business‟. The IVSC guidance notes define

„Value to the Business‟ as the value, assuming that plant and machinery

will continue in its present existing use in the business of the company.

Both the RICS and the IVSC guidance notes recommend this approach

for both private and public companies alike, as long as their intention is

to continue using the assets in their business for a reasonable period in

the future. Going-concern basis reflects the potential use limitations of

the business with all its strengths and weaknesses. It does not account of

the alternative uses to which the assets can be put. From the forgoing, it

could easily be seen that for balance sheet purposes, for instance, the

going-concern basis is to be adopted, otherwise it would run counter to

the accounting principle of value in the business making use of the assets

being subjected to valuation. Other purposes for which the basis could be

used include mortgage, stock financing, taxation, insurance, acquisition

and mergers, company accounts, annual financial report, business

performance analysis e.t.c.

Page 229: unitedemsa.files.wordpress.com...Pg 2 NIESV STUDY PACK - PPE KNOWLEDGE AND STRUCTURE OF NIGERIA INSTITUTION OF ESTATE SURVEYORS AND VALUERS Learning Objectives At the end of this lecture,

Pg 229

NIESV STUDY PACK - PPE

Content of the valuation for going-concern:

The instruction to determine „going concern value‟ is will in normal cases refer to

the entire plant and machinery asset of a company. Hardly will a company require

the „value in use‟ of a single item or one single production line.

Plant and machinery in this context will be part of the fixed assets of the company

and will not include items that are to be consumed by the company in the normal

course of its business. It is the fixed assets of a company other than its land and

building.

Motor vehicles, mobile plant, ships, locomotives, aeroplanes and similar assets,

which are not physically fixed, are part of plant and machinery unless they form

part of the stock in trade of the company.

If motor vehicles and other similar items not physically fixed are to be included in

the valuation, the basic information required will be found in their registration

books. Physical inspection of these items must be made noting mileage and

general condition. Where it is not possible to inspect physically all the items of

motor vehicles and the like, a note should be made in the report to this effect. In

the case of company, which deals in plant and machinery, its items of plant held

for trading purposes will not qualify to be classed as part of fixed assets (plant and

machinery).

All plant and equipment, which provides a service to the building in which a

company carries on its business and which will normally be sold with the

buildings, will be classified as part of the buildings. All other plant and equipment

connected with the business of the company will be valued as plant and machinery

unless otherwise requested in the instructions. Any such instructions should be

clearly stated in the final report.

Items on hire purchase should be included in the valuation without any

accounting taken of the outstanding amount on the hire purchase agreement,

unless the company specifically requests that items on hire purchase agreement be

included only at a figure, which represents their equity in the asset at the date of

the valuation.

Items on operating leases in which the ownership remains with the lessor are to be

excluded from the value in use of the lessee‟s assets. Such items will not appear on

the lessee‟ balance sheet in any case and should be ignored in the valuation.

Items on finance leases in which the risk and benefit of ownership is with the

lessee will be included in the valuation, because the lessee merely funds

ownership of the items by a sort of borrowing.

Page 230: unitedemsa.files.wordpress.com...Pg 2 NIESV STUDY PACK - PPE KNOWLEDGE AND STRUCTURE OF NIGERIA INSTITUTION OF ESTATE SURVEYORS AND VALUERS Learning Objectives At the end of this lecture,

Pg 230

NIESV STUDY PACK - PPE

Product-dedicated items are items used in connection with the manufacture of an

individual product. They have no general use besides the manufacture of an

individual product and are not considered as party of the production plant. They

include items such as injection moulds, drilling and inspection jigs, etc. they can

only be used in conjunction with the production plant to produce just the item for

which they were designed and made. The economic value is dependent on the

demand for the product for which they are used, and as soon as there is no further

demand for that product, the economic life of these items end, and they become

more or less of no value to the company. In most cases these should be excluded

from the valuation, or at best reflected in the value of the plant to which they are

attached should the client demand to see them in the valuation. The description of

the item to which they are attached should state what items of this type are used

with it.

Redundant products are items that either due to any form of obsolescence,

climatic pressures or as a result of efflux of time has become outmoded thereby

resulting to disuse. These products should not be included in the valuation. One

guiding principle for valuers of plant and machinery on the bases of going concern

should always be products relevant to operation/production of such companies.

Gone concern basis:

„Gone-concern value‟ and „alternative use value‟ mean the same thing. Gone –

concern valuation assumes that the business will be discontinued or altered. The

values obtained using this basis will reflect the potentials for alternative uses, if

any, which can be realized by the company if the assets were used for other

purposes other than that for which they were installed (Aluko and Ajayi, 1992).

For instance, a sweet making factory wanting to incorporate the manufacture of

biscuits and macaroni will be valued on a gone concern basis. Other purposes for

which gone concern valuations can be applied include sale and purchase of assets,

privatization, division under the terms of a will, investment decisions, etc.

Items that should form part of the valuation for this basis should be at the

discretion of the valuer, which is a function of the type of valuation and the

specification of duty.

From observation it can be deduced that the financial purpose of valuation is

synonymous with the Going concern basis while the open market purpose is

synonymous with the Gone concern basis (see previous lecture (2) for

reference). The insurance purpose is not necessary confined to any of the above

Page 231: unitedemsa.files.wordpress.com...Pg 2 NIESV STUDY PACK - PPE KNOWLEDGE AND STRUCTURE OF NIGERIA INSTITUTION OF ESTATE SURVEYORS AND VALUERS Learning Objectives At the end of this lecture,

Pg 231

NIESV STUDY PACK - PPE

mentioned basis; more so insurance can be carried out on all company’s’ assets

of which plant and machinery forms part. Details of insurance valuation will be

focus of discourse later.

FACTORS AFFECTING THE VALUE OF MACHINES AND PLANT

Before delving into the determination of assets values it is pertinent that students

identify the various factors that affect values of plant and machinery. These factors

include:

Physical factors: plant and machinery value is affected by depreciation

due to age, wear and tear, physical and functional obsolescence. Intensity

of use and number of shifts operated affect wear and tear of plant and

machinery, which invariably affect the value. The nature of materials

handled may also affect the life span in much the same way as adverse

climatic conditions. Apart from these, the availability of spare parts is an

indispensable tool in good repairs and preventive maintenance etc, and

determines to a large extent the quality of services available.

Statutory regulations: Government legislation on import, such as the

banning of the importation of certain items of plant have the effect of

pushing up local demand for the relevant manufacturing plant or, in the

case of plant and machinery, of enhancing greatly the second hand values

of some of the items of plant and machinery. Again, the level of import

duties payable on plant and machinery is also an important factor. For the

higher the import duty, the higher the value of the existing plant. In

Nigeria, the custom duty is surcharged on imported plant and machinery

and this is an additional cost to the plant.

Level of expertise: various plants and machineries require specialize

approach in their installation and operation. In countries were such

machines are not being manufactured, cost of operation and installations

of such machines could be substantial notwithstanding a comparison

paltry cost of purchase.

Economic factors: the general economic climate and the specific

situation of the particular industry must be taken into account. While

temporary fluctuations in the levels of business can usually be ignored,

the underlying trends cannot. Anything that is said to have value has a

monetary worth attached to it, Plant and machinery inclusive. Monetary

worth is likewise affected by monetary and fiscal policies of government.

Page 232: unitedemsa.files.wordpress.com...Pg 2 NIESV STUDY PACK - PPE KNOWLEDGE AND STRUCTURE OF NIGERIA INSTITUTION OF ESTATE SURVEYORS AND VALUERS Learning Objectives At the end of this lecture,

Pg 232

NIESV STUDY PACK - PPE

Such effects are usually pronounced in situations where such assets will

have to change hands. Factors like changes in consumer demand or

government legislation can make some plant obsolete before their

physical life is over, and where they cannot be used for other purposes,

their market value will be adversely affected. Each proposed

sale/purchase of plant and machinery will be unique in terms of its

contents, location and timing, and it will be normal for identical items

offered in different sales to attract different market values, due to the

following reasons:

Type of sale

If the sale is by unction, it is well known that the higher the attendance, the

greater the chances of realizing high price.

The attendance level at an auction is a function of the quality and quantity of

the items on offer for sale. It is therefore important when assessing the value

of an item to take into consideration the volume and quality of the total

package of items to be sold.

Location

Buyers are usually less prepared to look for items to purchase in remote and

hidden locations because of the extra transportation and handling costs in

getting the items to their premises. Such costs should be considered in

assessing value in the cases of most items except where the sale is to local

purchasers, who may even be willing to pay a premium.

Time of sale

In some cases, such as insolvency, the vendor may not be in a position to

choose an optimum time for selling his items. On the other hand, demands

for some items vary with the time of the year. It is well known for instance,

that the market value of certain items goes up when festive seasons are near.

Removal costs

It is not impossible to come across plant and equipment, which can only be

removed from its present location after removing some part of the building

structure such as the roof or walls. The value of such an item will naturally

be affected by the amount of demolition and remedial work necessary to

enable the buyer take home his purchase.

Special dismantling costs

Page 233: unitedemsa.files.wordpress.com...Pg 2 NIESV STUDY PACK - PPE KNOWLEDGE AND STRUCTURE OF NIGERIA INSTITUTION OF ESTATE SURVEYORS AND VALUERS Learning Objectives At the end of this lecture,

Pg 233

NIESV STUDY PACK - PPE

This will include such costs as will be incurred in the purging of a chemical

plant. This type of cost in most cases can be so enormous as to result in a

negative value.

Category of purchaser

The value attainable can vary significantly depending on whether the buyer

is a dealer, agent or an end user. The surveyor should keep in mind

wholesale and retail value factor in his assessment. The definition of market

value states that the surveyor must disregard in his valuation, the prospects

of abnormally high offers by a special purchaser.

The special purchaser will include any buyer with an urgent and specific

requirement for the particular item on offer and who is prepared to pay over

and above the normal market value in order to secure the purchase.

Selling costs

These will include agents‟ commission, costs of advertising and all other

costs incurred in putting up items for sale. They must all be deducted from

all market valuations to arrive at a reasonable estimated Net Realizable

Value. It is however also pertinent to report gross figures and add a note that

the selling costs have not been taken into account rather than provide net

figures, which may not be accurate due to uncertain costs.

METHODS OF MACHINES AND PLANT VALUATION

As we are all conversant with conventional methods of valuation such as the cost

method, income approach, residual method, profit method, comparative method, it

is worthwhile mentioning here that these methods are suitable usually when

valuing land and buildings. However, for plant and machinery valuation, the

methods usually adopted are: the market data approach; the profits method; and

the cost or summation method. For the particular requirements of the plant and

machineries to be valued, however, the profit, the market data or the summation

methods could be employed, depending on the nature of the market and the

Assignment:

Apart from physical factors, statutory regulations, level of expertise

and economic factors, students are advised to go and learn more

factors that affect values of plant and machinery

Page 234: unitedemsa.files.wordpress.com...Pg 2 NIESV STUDY PACK - PPE KNOWLEDGE AND STRUCTURE OF NIGERIA INSTITUTION OF ESTATE SURVEYORS AND VALUERS Learning Objectives At the end of this lecture,

Pg 234

NIESV STUDY PACK - PPE

availability of reliable data. Although, in practice the method usually adopted is

the cost or summation method. The following is, therefore, a cursory look at the

suitability of each of the above methods adopted in plant and machinery

valuations.

The market data approach: this approach is appropriately based on the

assumption that an informed purchaser, without undue influence, will pay no

more for a property than the cost of acquiring an existing one with the same utility

and located in the same or similar neighborhood. This method is, therefore,

mainly predicated on the comparison or recent sales prices of properties having

similar attributes with the subject being appraised. Some of the attributes include

location, physical state, tenure/title, time within which the transaction was

concluded, construction, layout, equipment and size. The effective comparison of

these attributes and the making of proper adjustments where necessary are the

bane of the method. To achieve reasonable results, experience and a fair good

knowledge of that segment of the property market are essential ingredients. It is

however true that the number of subjective considerations associated with the use

of the method is reducible to a manageable level with the availability of sufficient

and reliable data and evidence of comparable transactions making the method a

more dependable means of estimating value.

The profits method: this method converts anticipated future benefits (profits) of

property ownership into an estimate of present value. The steps involved require

adequate market data and reliable books of account, which were not always

sufficiently available. It is now being canvassed that under circumstances of

dearth of supportable data, the placement of value on plant and machinery is

almost impossible under this method. This is true because it is often difficult to

determine from the records of most establishments, the percentage contribution of

each asset of the company. In the light of the above considerations, therefore, the

profit method was, at best useful only as a check for the overall value of the assets

as determined using the cost approach.

The cost or summation method: the cost or summation approach is based on the

premise that an informed purchaser, under normal circumstances, would pay no

more for a property than the cost of producing or acquiring a substitute property

of similar utility within the same or like neighborhood. The method essential

involves the estimation of the total cost of construction of such plant and machine

Page 235: unitedemsa.files.wordpress.com...Pg 2 NIESV STUDY PACK - PPE KNOWLEDGE AND STRUCTURE OF NIGERIA INSTITUTION OF ESTATE SURVEYORS AND VALUERS Learning Objectives At the end of this lecture,

Pg 235

NIESV STUDY PACK - PPE

depreciated to reflect physical, functional and economic obsolescence. The major

problem in the use of this method of valuation is the determination of

obsolescence due to aging (physical deterioration), unfavourable national and/or

international economic factors and functional problems. It is, therefore, suggested

that its application is more acceptable if the plant is in its highest and best use

(new state, say not more than three years) or made as good as that through regular

and sufficient maintenance and refurbishment. These are necessary because,

estimating even minor forms of obsolescence involves, in most cases,

unsupportable judgmental considerations that weaken the method‟s credibility.

However, with experience and sufficient knowledge of that segment of the

market, acceptable value estimate could be obtained from its use.

The method is also considered the most alternative in the placement of value,

under the circumstances of plant and machinery that rarely change hands (rarely

sold in the open market). As highlighted earlier, this method is mostly used in

valuation of plant and machinery because, apart from the reasons given for not

using the other approaches dealt with above, it is most suitable for the type of

specialized nature of plant for which companies are noted for; specialized in terms

of design, layout, construction, quality, type of materials and equipment.

From the aforementioned it has been revealed that the cost or summation method

is the widely used method in the valuation of plant and machinery. However, for

an appropriate grasp of this method, valuers should be conversant with the concept

of depreciation and its application.

DEPRECIATION OF MACHINES AND PLANT

Depreciation is the term used to describe the reduction in the value of an asset

over a period of time. Depreciation may be observed in forms of physical

deterioration, locational and economic obsolescence and functional obsolescence.

It is the measure of the wearing out, consumption or other loss of value of assets

whether from use, effluxion of time or obsolescence through technological and

market changes. There are so many factors likely to accelerate depreciation in

values of plant and machinery. Some of these include rate of use or general

carelessness as regards use. Good management on the opposite side of reduces the

rate of depreciation on an asset. Experience has shown that two similar properties

could lie in the same location, have similar characteristics but upon valuation by

depreciated replacement cost method, the value of one is found to be less than that

of the other. It is therefore wrong for a valuer to neglect this very important factor

which greatly affect the values of the properties and which could lead to a false

Page 236: unitedemsa.files.wordpress.com...Pg 2 NIESV STUDY PACK - PPE KNOWLEDGE AND STRUCTURE OF NIGERIA INSTITUTION OF ESTATE SURVEYORS AND VALUERS Learning Objectives At the end of this lecture,

Pg 236

NIESV STUDY PACK - PPE

presentation of value when neglected. It may sometimes take more than one

professional to decide what rate of depreciation should be used in depreciating a

plant or machinery. Having ascertained the gross value of a plant and machinery

by the appropriate method of valuation, the valuer can direct his attention to

depreciating this figure in order to arrive at the existing-use-value of the plants and

machinery. Depreciation is indeed an issue in valuation of plant and machinery.

Fundamentally, the existing use value of plant and machinery calculated by the

depreciated replacement cost method is that part of the gross value that is

attributed to the unexpired portion of the total economic working life. The factors

that affect depreciation or determine the assessment and allocation of depreciation

are:

General condition

Standard of maintenance

Workload to which plant and machinery is subjected

Replacement of component parts since new

Other relevant factors

Depreciation and what it means

Before we delve into the various methods of depreciation it is crucial at this

moment for us to take a look at the four technical meanings ascribed to

depreciation. They include:

Loss of value: when the value of an asset is computed at two different

dates, and the value at the later date is subtracted from the value at the

earlier date, the difference is a measure of the amount of depreciation that

the asset has experienced. This is the meaning generally applied to the word

depreciation when used in everyday speech. Of course, when talking about

value, one must make the distinction between market value and value to the

owner. Depreciation may apply to either of these two definitions of value

whether they are determined by actual market price, by an appraisal, or by

any other method.

Cost amortization: from an accounting point of view, the cost of an asset is

an operating expense which has been prepaid and which must be

apportioned according to some systematic procedure among the years of the

asset‟s life. In standard accounting terminology it is cost and not value that

is apportioned. A report of the committee on terminology of the American

Institute of Certified Public Accountants define depreciation as follows:

“Depreciation accounting is a system of accounting which aims to

Page 237: unitedemsa.files.wordpress.com...Pg 2 NIESV STUDY PACK - PPE KNOWLEDGE AND STRUCTURE OF NIGERIA INSTITUTION OF ESTATE SURVEYORS AND VALUERS Learning Objectives At the end of this lecture,

Pg 237

NIESV STUDY PACK - PPE

distribute the cost or other basic value of tangible capital assets, less

salvage value (if any), over the estimated useful life of the unit (which may

be a group of assets) in a systematic rational manner. It is a process of

allocation, not of valuation. Depreciation for the year is the portion of the

total charge under such a system that is allocated to the year. Although the

allocation may properly take into account occurrences during the year, it is

not intended to be a measurement of the effect of all such occurrences”.

The term book value is used to describe the difference between the

depreciable cost of an asset and the total amount of depreciation charges

that have been made to date against the asset.

The difference in value between an old asset and the reproduction of the

identical but new asset, which is used as a standard to compare values:

the appraisal concept of depreciation is based upon this premise. One way

to appraise old assets is based upon replacement cost, which is defined as

the difference between the cost of reproduction of the asset and the

depreciation that it has experienced through time to date of appraisal.

Wear and tear, which result in decreased serviceableness: as structures

and equipment become older, the wear and tear often results in impaired

functional efficiency. This is an engineering concept of depreciation. It is

not a value concept. The fact that an asset may be perfect and as good as

new, does not insure than its value is the same as when it was new, for the

asset may have undergone a certain amount of technological obsolescence.

That is, more recently produces assets may perform the service more

efficiently or may have a broader range of capabilities.

It is extremely important to recognize at the outset that depreciation is a book of

account entry (a paper loss). Hence, depreciation could not just be the only factor

that affects before tax cash inflow as other factors ranging from managerial

inefficiency, marketing procedures and other adverse financial climate could affect

such. However, depreciation is usually considered for issues like tax matters; an

escrow account for replacements is not set up and consequently, monies from the

cash flow stream are not diverted for this purpose.

COMPOSITE METHODS OF DEPRECIATION

Having studied the four technical meanings ascribed to depreciation, students by

now should have a great grasp of the concept of depreciation. However, it is one

thing to know about depreciation, it is another to be able to apply it. Every

professional exposed to the utilization of the concept, depreciation, encounter

Page 238: unitedemsa.files.wordpress.com...Pg 2 NIESV STUDY PACK - PPE KNOWLEDGE AND STRUCTURE OF NIGERIA INSTITUTION OF ESTATE SURVEYORS AND VALUERS Learning Objectives At the end of this lecture,

Pg 238

NIESV STUDY PACK - PPE

problems, as there are divergent opinions on its application. Professionals are faced

with myriad of options in determining depreciation. However, regardless of the

depreciation method to be used, it is important that two elements be considered

before depreciation charge is calculated. The first is the depreciation base, which is

the aggregate of all costs incurred in acquiring the asset, installing it and making it

ready for use LESS or minus any residual value expected at the time of retirement,

plus the anticipated costs of removing the asset when retired. The second element is

the useful life of the asset, which is a function of the physical wear and tear to

which the asset is subjected and of technological change and innovation. Having

said this, it behooves us to take a look at the various methods of determining the

depreciation of plant and machinery. They are:

Straight-line method: the plant and machinery is seen as providing an

equal amount of service in each year with maintenance cost either constant

or insignificant. The benefits derived from using the asset are also assumed

to spread evenly over the useful life of the asset. The annual depreciation

charge is obtained by dividing the depreciation base by the estimated

number of years for which the asset is useful and the charge is the same

every year. The rate of depreciation may be obtained by converting the

reciprocal of the useful life into a percentage. The rate when applied by the

depreciation base gives the depreciation charge. Annual depreciation under

this method is computed by dividing the depreciable book value of the

asset over its full service life in years. Students should note that the

depreciable book value is the same as the depreciation base, which is

defined mathematically as (acquisition cost indexed to present value, less

items that can be written off immediately such as fees related to taxation if

any, less salvage value).

Illustration:

Assuming a machine that has a depreciable book value of N1, 050, 000 and

with a full service life of 25 years. You are required to compute its annual

depreciation.

Solution:

Depreciable book value/base = N1, 050, 000

Useful life = 25 years

Annual depreciation = N1, 050, 000

25

= N42, 000

Page 239: unitedemsa.files.wordpress.com...Pg 2 NIESV STUDY PACK - PPE KNOWLEDGE AND STRUCTURE OF NIGERIA INSTITUTION OF ESTATE SURVEYORS AND VALUERS Learning Objectives At the end of this lecture,

Pg 239

NIESV STUDY PACK - PPE

Thus, the annual depreciation is 1 or 4% of its depreciable book value.

25

Sum-of-the-years’-digits method: this method is a way of achieving

decreasing charges by applying diminishing rates to the depreciation base.

The rates are obtained by setting the sum of the digits fro one through the

useful life of the asset in years as the denominator, while the numerators

are the individual year numbers taken in descending order per year. Under

the method of declining charge for depreciation of a fixed asset is made as

the asset ages and usually there are heavy early-year charges to

depreciation counterbalanced by lower charges in the later years. Under

this method, the digits that correspond to the number of years of service

life of an asset are added. Mathematically this can be computed as n (n+1)

where n = number of years in the life.

2

Illustration:

Assuming a machine that has a depreciable book value of N1, 050, 000 and

with a full service life of 25 years. You are required to compute its annual

depreciation.

Solution:

Depreciable book value/base = N1, 050, 000

Useful life = 25 years

Thus, for n = 25, n (n+1) = 25(25+1) = 25(26) = 325

2 2 2

Thus, the depreciation for the first year of life is then computed as

25 * 1050000 = N80, 769. 23

325

The second year, the depreciation is 24 * 1050000 = N77, 538. 46

325

The third year, the depreciation is 23 * 1050000 = N74, 307. 69

325

And so on until the end of the 25th

year. Students who are vigilant will discover

that the charges decrease by N3, 230. 77 yearly.

Declining balance depreciation: there are two facets of this type of depreciation

method they are the 100% declining balance and the double declining balance. In any

declining balance depreciation a given rate is applied each year to the remaining book

Page 240: unitedemsa.files.wordpress.com...Pg 2 NIESV STUDY PACK - PPE KNOWLEDGE AND STRUCTURE OF NIGERIA INSTITUTION OF ESTATE SURVEYORS AND VALUERS Learning Objectives At the end of this lecture,

Pg 240

NIESV STUDY PACK - PPE

value of the asset. The remaining book value is that portion of the depreciable book of an

asset or assets that has not been written off in the previous years. This method assumes

that as the fixed assets gets older, the benefits derived from them decreases, while

maintenance and repair costs incurred on them increase. The method entails the

application of a constant depreciation rate to the declining basic value of an asset at the

beginning of successive years. The result is that a declining depreciation charge is

obtained as the asset ages. Any prospective salvage value for the asset is disregarded in

computing the permissible declining balance rate because the declining balance method

automatically generates a salvage value at the end of the asset‟s life.

The depreciation rate under the 100% declining balance approach is

computed as: 100%

Estimated life of asset in years

While the depreciation rate under the double declining balance approach is

computed as:

200%

Estimated life of asset in years

Mathematically, let P represent the initial depreciable book of an asset and

r represent the declining balance rate expressed as a decimal. It can be

shown that the book value of the asset at the end of n years is given by:

S.V. = P (1-r)n .

Illustration:

Assuming a machine that has a depreciable book value of N1, 050, 000 and

with a full service life of 25 years. You are required to compute its annual

depreciation.

Solution: (100% or single declining balance approach)

Depreciable book value/base = N1, 050, 000

Useful life = 25 years

100% = 4%

25years

Because the initial depreciable book value is N1, 050, 000, the depreciation

charge

during the first year of life would be 0.04(N1, 050, 000) = N42, 000. The

remaining

book value would be N1,050,000 – N42, 000 = N1, 008, 000

Thus, the depreciation during the second year is 0.04(N1, 008,000) =N40, 320.

The

remaining book value would be N1,008,000 – N40, 320 = N967, 680

Page 241: unitedemsa.files.wordpress.com...Pg 2 NIESV STUDY PACK - PPE KNOWLEDGE AND STRUCTURE OF NIGERIA INSTITUTION OF ESTATE SURVEYORS AND VALUERS Learning Objectives At the end of this lecture,

Pg 241

NIESV STUDY PACK - PPE

Thus, the depreciation during the third year is 0.04(N967, 680) =N38, 707.2 and

so on.

The salvage value at the end of the 25 years is thus:

S.V. = P (1-r)n = S.V. = N1, 050, 000 (1-0.04)

25 = N378, 416.55

Illustration:

Assuming a machine that has a depreciable book value of N1, 050, 000 and

with a full service life of 25 years. You are required to compute its annual

depreciation.

Solution: (200% or double declining balance approach)

Depreciable book value/base = N1, 050, 000

Useful life = 25 years

200% = 8%

25years

Because the initial depreciable book value is N1, 050, 000, the depreciation charge during

the first year of life would be 0.08(N1, 050, 000) = N84, 000. The remaining book value

would be N1,050,000 – N84, 000 = N966, 000

Thus, the depreciation during the second year is 0.08(N966, 000) =N77, 280. The

remaining book value would be N966,000 – N77, 280 = N888, 720

Thus, the depreciation during the third year is 0.08(N888, 720) =N71, 097.6 and so on.

The salvage value at the end of the 25 years is thus:

S.V. = P (1-r)n = S.V. = N1, 050, 000 (1-0.08)

25 = N130, 583

Let‟s take a quick look at the comparison of depreciation methods. The table

below gives annual depreciation charges and end-of-year book values for straight

line depreciation, double declining balance, 100% declining balance and sum of

the years digits for our hypothetical machine previously considered. In each case

the depreciable book value is N1, 050, 000 and the service life, is 25 years.

Table showing hypothetical machine’s annual depreciation charge (N)

Years Straight line Double

declining

balance

100%

declining

balance

Years digit

0

1 42, 000 84, 000 42, 000 80, 769

2 42, 000 77, 280 40, 320 77, 538

3 42, 000 71, 098 38, 707 74, 308

Page 242: unitedemsa.files.wordpress.com...Pg 2 NIESV STUDY PACK - PPE KNOWLEDGE AND STRUCTURE OF NIGERIA INSTITUTION OF ESTATE SURVEYORS AND VALUERS Learning Objectives At the end of this lecture,

Pg 242

NIESV STUDY PACK - PPE

4 42, 000 65, 410 37, 159 71, 077

5 42, 000 60, 177 35, 673 67, 846

6 42, 000 55, 363 34, 246 64, 615

7 42, 000 50, 934 32, 879 61, 385

8 42, 000 46, 859 31, 561 58, 154

9 42, 000 43, 110 30, 298 54, 923

10 42, 000 39, 662 29, 086 51, 692

11 42, 000 36, 489 27, 923 48, 462

12 42, 000 33, 570 26, 806 45, 231

13 42, 000 30, 884 25, 734 42, 000

14 42, 000 28, 413 24, 704 38, 769

15 42, 000 26, 140 23, 716 35, 538

16 42, 000 24, 049 22, 768 32, 308

17 42, 000 22, 125 21, 857 29, 077

18 42, 000 20, 355 20, 983 25, 846

19 42, 000 18, 727 20, 143 22, 615

20 42, 000 17, 229 19, 338 19, 385

21 42, 000 15, 850 18, 564 16, 154

22 42, 000 14, 582 17, 82 12, 923

23 42, 000 13, 416 17, 109 9, 692

24 42, 000 12, 342 16, 424 6, 462

25 42, 000 11, 355 15, 767 3, 231

Let‟s also take a look at the salvage value of our hypothetical machine at the end

of various useful lives.

Table showing hypothetical machine’s annual depreciation charge (N)

Years Straight line Double

declining

balance

100%

declining

balance

Years digit

0 1, 050, 000 1, 050, 000 1, 050, 000 1, 050, 000

1 1, 008, 000 966, 000 1, 008, 000 969, 231

2 966, 000 666, 720 967, 680 891, 693

Page 243: unitedemsa.files.wordpress.com...Pg 2 NIESV STUDY PACK - PPE KNOWLEDGE AND STRUCTURE OF NIGERIA INSTITUTION OF ESTATE SURVEYORS AND VALUERS Learning Objectives At the end of this lecture,

Pg 243

NIESV STUDY PACK - PPE

3 924, 000 817, 62 928, 973 817, 385

4 882, 000 752, 213 891, 814 746, 308

5 840, 000 692, 036 856, 141 678, 462

6 798, 000 636, 673 832, 896 613, 847

7 756, 000 585, 739 789, 020 552, 462

8 714, 000 538, 880 757, 459 494, 308

9 672, 000 495, 769 727, 161 439, 385

10 630, 000 456, 108 698, 074 387, 693

11 588, 000 419, 619 670, 151 339, 231

12 546, 000 386, 050 643, 345 294, 000

13 504, 000 355, 166 617, 611 252, 00

14 462, 000 326, 752 592, 907 213, 231

15 420, 000 300, 612 569, 191 177, 693

16 378, 000 276, 563 546, 423 145, 385

17 336, 000 254, 438 524, 566 116, 308

18 294, 000 234, 083 503, 584 90, 462

19 252, 000 215, 357 483, 440 67, 847

20 210, 000 198, 128 464, 102 48, 462

21 168, 000 182, 278 445, 538 32, 308

22 126, 000 167, 696 427, 717 19, 385

23 84, 000 154, 280 410, 608 9, 693

24 42, 000 141, 938 394, 184 3, 231

25 0 130, 583 378, 417 0

Note: students are encourage to study the above tables and begin to make

crucial observations.

Other methods of depreciation of plant and machinery, obtainable, are the

revaluation and productivity or usage method.

Revaluation method: this method involves the valuation of the assets

according to their class, at the beginning of the year and also at the end of

the year. By making adjustments in respect of additional assets purchased

within the year, a measure of the value of the asset used up during the year

is arrived at. This value is then charged to the income statement as

depreciation in respect of such assets. The method can be applied to loose

Page 244: unitedemsa.files.wordpress.com...Pg 2 NIESV STUDY PACK - PPE KNOWLEDGE AND STRUCTURE OF NIGERIA INSTITUTION OF ESTATE SURVEYORS AND VALUERS Learning Objectives At the end of this lecture,

Pg 244

NIESV STUDY PACK - PPE

tools, or where the useful life of the asset may turn out to be short due to

loss, theft, breakage, damage etc. or where it may be cumbersome and

unnecessary to keep individual records for each of the items because of the

fact that their individual values are immaterial in the context of the whole

organization.

Illustration:

The following were obtained from the books of Goalmex Engineering

Corporation. In the first year, Processing machines were bought valued at

N400, 000. The following values were given by the expert that valued the

assets as follows:

Year ended year 1 = N250, 000

Year ended year 2 = N100, 000

Year ended year 3 = N70, 000

Year ended year 4 = N52, 000

Year ended year 5 = N35, 000

Year ended year 6 = N10, 000

Determine the depreciation charged for the six years.

Solution:

Year 1 = N400, 00 – N250, 000 = N150, 000

Year 2 = N250, 00 – N100, 000 = N150, 000

Year 3 = N100, 00 – N70, 000 = N30, 000

Year 4 = N70, 00 – N52, 000 = N18, 000

Year 5 = N52, 00 – N35, 000 = N17, 000

Year 6 = N35, 00 – N10, 000 = N25, 000

The Table below gives a lucid picture of how the asset stands at the end of the six

years period.

Table showing comprehensive details of machine (N)

Year Book value at

the beginning

of the year

Depreciation

charge in the

year

Accumulated

depreciation at

the end of the

year

Book value

at the end of

the year

1 400, 00 150, 000 150, 000 250, 000

2 250, 000 150, 000 300, 000 100, 00

3 100, 00 30, 000 330, 000 70, 000

4 70, 000 18, 000 348, 000 52, 000

5 52, 000 17, 00 365, 000 35, 000

Page 245: unitedemsa.files.wordpress.com...Pg 2 NIESV STUDY PACK - PPE KNOWLEDGE AND STRUCTURE OF NIGERIA INSTITUTION OF ESTATE SURVEYORS AND VALUERS Learning Objectives At the end of this lecture,

Pg 245

NIESV STUDY PACK - PPE

6 35, 000 25, 00 390, 000 10, 000

Productivity or usage method: this method takes the productivity capacity

of the asset into consideration and, therefore, bases the depreciation

charges on the machine hours used or production units. The useful life of

the asset is hereby disregarded and the total machine hours or production

units are estimated and used to calculate the annual depreciation charge

which will then vary with the level of production or the number of machine

hours achieved. This often results in fluctuating charges. The method can

be used for assets, which wear out in proportion to their usage, and in the

estimation of computation of cost of production.

Illustration:

A machine cost N200, 00 with a budgeted capacity of 600, 000 units. The

following units were thereafter produced:

Year 1 = 150, 000 units

Years 2 = 80, 000 units

Year 3 = 100, 000 units

Years 4 = 70, 000 units

Year 5 = 65, 000 units

Years 6 = 50, 000 units

Calculate the depreciation charge.

Solution:

Cost of acquisition = N200, 000 = 0.33/units

Maximum capacity 600, 000

Year 1 = 0.33 * 150, 000 units = N49, 500

Year 2 = 0.33* 80, 000 units = N26, 400

Year 3 = 0.33 * 100, 000 units = N33, 000

Year 4 = 0.33* 70, 000 units = N23, 100

Year 5 = 0.33 * 65, 000 units = N21 450,

Year 6 = 0.33 * 50, 000 units = N16, 500

The Table below gives a lucid picture of how the asset stands at the end of the six

years period:

Table showing comprehensive details of machine (N)

Page 246: unitedemsa.files.wordpress.com...Pg 2 NIESV STUDY PACK - PPE KNOWLEDGE AND STRUCTURE OF NIGERIA INSTITUTION OF ESTATE SURVEYORS AND VALUERS Learning Objectives At the end of this lecture,

Pg 246

NIESV STUDY PACK - PPE

Year Book value at

the beginning

of the year

Depreciation

charge in the

year

Accumulated

depreciation at

the end of the

year

Book value

at the end of

the year

1 200, 00 49, 500 49, 500 150, 500

2 150, 500 26, 400 75, 900 124, 100

3 124, 100 33, 000 108, 900 91, 100

4 91, 100 23, 100 132, 000 68, 000

5 68, 000 21,450 153, 450 46, 550

6 46, 550 16, 500 169, 950 30, 050

ASSIGNMENT: HAVING STUDIED THE VARIOUS DEPRECIATION

METHODS, YOU ARE REQUIRED TO WRITE AN ESSAY AND

CRITICALLY SUGGEST THE BEST METHOD OF DEPRECIATION THAT

SHOULD BE ADOPTED IN THE VALUATION OF MACHINES AND

PLANTS.

PROCESSES OF PLANT AND MACHINERY VALUATION

Just like in all activities that is worth the while plant and machinery valuation has to be

carried out in a systematic way. Any valuer that is conscious of the approach preceding

the actual process of actual valuation of plant and machinery will circumvent any glitch

likely to be envisioned during the process. In the valuation exercise, the major steps

involved after securing the brief include the establishment of the scope and basis of the

valuation; identification; classification and compilation of inventory of items of plant and

machinery; and finally value the asset on the appropriate basis.

Securing the brief:

In any form of valuation there must be a commitment by the client for the need for such

job. This commits the client and obliges the valuer in carrying out the professional task.

The instruction could be written but the intention of contract is what seems paramount

here.

Defining the scope of the valuation:

Invariably when the scope is defined, it automatically means that the purpose of the

valuation has earlier been ascertained. The purpose of the valuation will not result

without a brief for the exercise in the first place. Students should recall the third lecture

Page 247: unitedemsa.files.wordpress.com...Pg 2 NIESV STUDY PACK - PPE KNOWLEDGE AND STRUCTURE OF NIGERIA INSTITUTION OF ESTATE SURVEYORS AND VALUERS Learning Objectives At the end of this lecture,

Pg 247

NIESV STUDY PACK - PPE

on basis of plant and machinery valuation for further details. When the scope of valuation

is successfully defined, the valuer is said to have established a route of operation.

Identification of plant and machinery:

As it has severally been highlighted earlier on, the major bane of this aspect of valuation

is the identification of what actually constitutes a plant/machinery in any organization.

This is the first step in the valuation process after the identification of the scope of

valuation and has been a Herculean task to even the most experienced valuers. The scope

of valuation determines what items are to be regarded as plant and machinery. Also the

requirements of the clients‟ will also be a guiding factor. However, one prominent guide

in identifying plant and machineries is to determine the use to which the item is put. If a

boiler for instance is used to heat the building mainly, then it should be valued with the

building, but if it is purposely used to provide steam energy for the manufacturing

process then it should be classified and valued as an item of plant and machinery. With

respect to valuation norms Students should note that all moveable assets, which concerns

any organizational will fall under one of the following group: plant and machinery,

service system, loose tolls and equipments, fixtures and fittings, office and other

furniture, office equipment (computer hardware, software, typewriters, printing, Xerox

and others), vehicles as well as raw materials and finished stocks. As has been stated

earlier, in conventional plant and machinery valuation, vehicles used as official cars also

form part of items as plant and machinery.

Classification of plant and machinery:

In the classification of plant and machinery, the principal factor here is the nature of

organizational outfit. Is it mining, manufacturing or processing concern? For instance

plants can be classified as follows:

Automobile plants: these include trucks, buses, trolleybuses, road tractors, trailers, cranes

etc.

Chemical plants: chemical plants are industrial process plant that manufactures

chemicals, usually on a large scale. Machinery required for chemical plants include:

chemical dryer, chemical mixer, chemical process pumps, compressed air dryer,

distillation columns, fertilizer plants, industrial mixer, laboratory centrifuge, rotary

vacuum dryers, etc.

Gas plants: plants that come under this classification include Oxygen plant, Hydrogen

plant, Nitrogen plant, Ammonia plant, Acetylene plant, Carbon-dioxide (CO2) generator,

Dry ice plant, methane plant, Biogas plant, LNG plant, CNG plant, Natural gas plant,

Page 248: unitedemsa.files.wordpress.com...Pg 2 NIESV STUDY PACK - PPE KNOWLEDGE AND STRUCTURE OF NIGERIA INSTITUTION OF ESTATE SURVEYORS AND VALUERS Learning Objectives At the end of this lecture,

Pg 248

NIESV STUDY PACK - PPE

Liquid oxygen pumps, Propane generator, Nitrous Oxide plant, Liquid Nitrogen Pump,

Oxygen generator, Cryogenic liquid plants, etc.

Pharmaceutical plant: plants that are classified under this category include: drum

blender, dry heat sterilizer, dust collector, empty capsule filling machine, feeze drying

machine, fermentation house equipment, fertilizer process equipment, furnace ducts,

herbal extraction plant, homogenizer, ointment mixer, oscillating granulator among

others.

Diary plants: plants that come under this category include concentration separation

equipment, conveyors, cream separators, filling machines, flow meters, heat exchangers,

homogenizers, insects control system, pasteurizes, pipes, pneumatic blowers and

boosters, pressure vessels, process pumps, programmable controllers, solenoid pneumatic

valves, temperature controllers, thermic fluid heaters, centrifugal pumps, built milk

coolers, butter chumers, milk pasteurization plant, etc.

Power plant: certain plants in this category include steam power plant, geothermal power

plant, gas power plant, nuclear power plant, wind power plant, hydroelectric dams, coal

plants, fossil fuel power plants, etc.

The list is endless. The prevalent situation determines the classification to adopt.

Some valuers usually omit this stage especially in very small industrial out fits that focus

on a particular line of operation. However, there should be a point of caution here. When

machines are classified under subset as regards what they are focused on in various era of

operation it enables the valuer to capture every machine in the various era of operation

and as such the omission of certain machines will to a great extent be reduced if not

eliminated.

Compilation of inventory of plant and machinery:

Having identified and classified the various plants and machinery valuation, the next

stage is to compile the inventory of existing plants and machinery. Compilation of plant

and machinery follows a definite pattern. The amount of details necessary for the asset

schedule to form the basis of the valuation should be enough information to enable the

client to identify the assets being valued at the same time, to give the valuer enough

information to calculate the worth of the machinery. In describing the machine, it must be

in such a way as to leave no doubt in mind of the manufacturer or supplier asking for the

price. The general format is:

Maker‟s name

Model or type designation

Capacity

Description

Page 249: unitedemsa.files.wordpress.com...Pg 2 NIESV STUDY PACK - PPE KNOWLEDGE AND STRUCTURE OF NIGERIA INSTITUTION OF ESTATE SURVEYORS AND VALUERS Learning Objectives At the end of this lecture,

Pg 249

NIESV STUDY PACK - PPE

Serial number

Note on age and condition

Other relevant comment

However, it should be noted that when preparing the schedule of company‟s assets within

its premises that certain plant and machineries might be with the subcontractors; away on

repair; working on an off-site project, or at the house of one of the managers or employee

or anywhere, such as the helicopters or mobile plants. As a result of this, the valuer must

liaise with the client and the building surveyor in order to make sure that that the

borderline assets are not totally omitted nor included in both the plant and property

valuation. Hence, it is necessary to ask about equipment not on site.

The actual valuation:

A scrupulous adherence to the aforementioned steps will curb unnecessary errors. At this

stage the valuer determines the method of valuation to adopt, although in the Nigerian

practice the cost or summation approach is usually adopted. From the information

gathered at the inventory stage, the appropriate type and adoption of depreciation is

considered. Economic factors prevalent and those that have been experienced since the

manufacture/purchase of such assets are also important factors to consider.

Placing values on the said assets becomes the prerogative of the valuer at this stage.

Reporting

At the end of any valuation exercise, valuers are required to report their professional task.

This is function of the basis of the valuation job being handled. However, certain features

are always germane. This includes: The value arrived at, the date, purpose of the

valuation and the beneficiary of such job.

VALUATION OF OBSOLETE MACHINES

If change is the law of life, there is no reason to suppose that the rate of change will be

such as to leave nothing obsolete. With the rapid pace of technological change today, it is

common for valuation surveyors to set levels of value for items no longer in current

production. In the fields of machine tools, textile plants, computer equipment and process

control, the pace of change has been particularly rapid. The assessment of value of

obsolete machinery for presentation in company‟s accounts is becoming increasingly

important as the speed of technological change advances even more rapidly.

It has earlier been established that the proper basis of value of machinery for a going

concern is the “Existing use value”, which is usually determined by:

Page 250: unitedemsa.files.wordpress.com...Pg 2 NIESV STUDY PACK - PPE KNOWLEDGE AND STRUCTURE OF NIGERIA INSTITUTION OF ESTATE SURVEYORS AND VALUERS Learning Objectives At the end of this lecture,

Pg 250

NIESV STUDY PACK - PPE

“Depreciating the gross current replacement cost in order to reflect the value

attributable to the remaining useful economic working life of the machine, taking due

account of wear, tear, age, obsolescence, scarcity value and other relevant factors

including residual value at the end of the asset‟s useful economic working life”

However, it should also be recalled that two other methods of valuation of plant and

machinery do exist, thus, the profit method and that of market data. The profit method is

seldom used due to the intricacies involved and certain parameters that might appear

arcane when considered. In the case of market data approach, “where suitable market

evidence exists this value should be compared with the cost of acquiring a similar asset

in the open market which has the same remaining working life as the existing asset”

From the aforementioned, it becomes evident that it is not necessary to establish a gross

value, that is a new cost, and then depreciate it in every case when valuing certain plants

and machinery. However, the snag with the market data approach is to get a proper

market that showcases the machine in question having the same features and

peculiarities. This market is colloquially called the second hand market (Tokunbo) and

appellation common in this part of the world. Certain types of machine tools, textile

machines or some of the simpler machines in pharmaceutical plants are perhaps good

examples of machinery which is commonly bought and sold in the second hand market.

The question then is what about the most sophisticated plants and machinery?

It should be noted that the key world in the fairly used market as often referred to in

relation to valuation of plant and machinery is “suitable market evidence”. The word

suitable is obviously open to various interpretations, but in this context implies that a

ready and active market must exist in a certain type of plant and contain a sufficient

number of transactions for Data to be analysed and trends monitored. These transactions

must be free and fair and conducted in industries where purchase of the plant at the

market rate can be expected to offer a prudent purchaser an economic return on his

invested capital assuming the machines he has purchased are employed for the use they

were intended, rather than for onward sale. The market value we are seeking to obtain is

the best price likely to be paid by a machine user for an asset in the same condition as the

item been valued rather than a dealer. This is because as defined much earlier machines

are principally used for ongoing business that anticipates future streams of profit, not the

breakup of the plant for sale.

Before a valuer begins to carry out valuation of obsolete machines, the first condition to

be established is what type of obsolescence is being considered, even though this term

might seem vague in explanation. Thereafter just as was learnt in the valuation of other

modern machines, the scope of the valuation is of preeminence.

Page 251: unitedemsa.files.wordpress.com...Pg 2 NIESV STUDY PACK - PPE KNOWLEDGE AND STRUCTURE OF NIGERIA INSTITUTION OF ESTATE SURVEYORS AND VALUERS Learning Objectives At the end of this lecture,

Pg 251

NIESV STUDY PACK - PPE

For a machine whose useful life has ended the value of such machine is its “salvage

value”. However, when such machine is still in operation, the task of determining its

value becomes herculean. Notwithstanding the challenge facing the valuer, certain points

need to be made. Firstly, a machine that has exceeded its useful life no matter the good

state it commands and its level of productivity perhaps due to effective maintenance

policy has little more time to pack up. A manufacturer is the best personnel to describe a

machine. Based on the specifications, peculiarities and other recondite features of any

machine, useful life are placed of which a machine is deemed to function, thus, any more

extra years of operation will be short-lived, hence such machines will not be much better

than their salvage value while their value of operation will not exceed that of the final

specified year. Although, the discretion of the valuer is paramount in placing a final

figure, having taking note of specific features prevalent. Obsolete machines under this

category could be described as scrap.

To some economic school of thought, a machine is regarded as being obsolete when a

new technological advancement makes it uneconomical to utilize such equipment even

when such equipment is still physically capable of contributing to production. There are

machines which do to pace in technology have become obsolete but do have some useful

life running, the approach will be to depreciate for new as used during the

cost/summation method studied earlier but better still make recourse to market data

evidence as discussed above, particularly when the manufacturers of such machines

cannot be contacted or approached and invoice of purchase cannot be reached either. In

some industries however, particularly process industries, chemical and food plants are

good examples here, plants are rarely sold second hand. The same may be said of highly

specialist machine tools such as special purpose drilling machines used in the automotive

industry. When existing use values of these obsolete plants are to be calculated, fairly

ground rule has been laid according to the RICS guidelines on the basis of their modern

equivalent asset. Referring to building valuations the relevant guidance notes state:

“When valuing for gross current replacement cost, where a modern building of the same

gross internal floor area might cost substantially less than an identical replacement of

the one on site, then the lower figure for the modern building should be used as a basis

for estimating net current replacement cost”

The above statement is equally relevant to the valuation of plant and machinery. In

practical fact, what represents a modern equivalent asset may be a matter of debate, but

the principle is plain.

What is a modern equivalent asset? How do we determine what is considered an adequate

replacement? To decide this it is perhaps useful to consider what the owner of the

machinery would do in the event of its replacement. He has two choices. He may replace

Page 252: unitedemsa.files.wordpress.com...Pg 2 NIESV STUDY PACK - PPE KNOWLEDGE AND STRUCTURE OF NIGERIA INSTITUTION OF ESTATE SURVEYORS AND VALUERS Learning Objectives At the end of this lecture,

Pg 252

NIESV STUDY PACK - PPE

his plant with equivalent plant from another source, or with modern computer controlled

plant of the latest technology.

Let‟s take the first case that is replacing his plant with equivalent plant from another

source. In many cases the particular obsolete machines being considered may not be

available from the original supplier, but may still be available as a new machine from

countries where the state of technological development is not as high. For example in the

United Kingdom today there are large number of machine tools, plastics machinery and

amounts of general factory equipment being imported from Eastern Europe and South

East Asia to do work previously done by British, German or American machines. It has

been argued that these machines are not of the same quality as those produced by the

manufacturers they are replacing. However, the standard of quality of manufacture and

sophistication of machines has increased progressively over time. The demand for more

reliable products has led to the need to manufacture to closer tolerances and to produce

goods of better quality than was the case in the past, and necessary improvements in

machinery have been to make this possible. A second rank of machine of today‟s

manufacture may easily produce work of equal quality to that of an obsolete machine

when it was new some twenty or thirty years ago.

It‟s simply like comparing a modern Eastern European car with a Morris Oxford, or an

Old Mercedes. If a factory owner would consider replacing his machinery in this way

then this can be considered as an appropriate estimation of gross that is new value. To

produce a net value appropriate for presentation in accounts, it is then only necessary to

depreciate this figure to reflect the wear, tear and remaining life of his current machinery.

On the other hand gross or new value may be decided by comparing existing assets to

modern computer controlled machinery. Consultation with the client‟s engineering staff

can usually provide a likely scheme for replacement. It is not necessary to consider each

individual machine in this situation, but the client‟s intention towards groups of

machines. He may replace three older spinning machines with one modern item for

example, or a bank of five capstan lathes with a machining centre. In this situation each

of the obsolete machines may be valued at one-third or one-fifth of the cost of the modern

asset. To produce a figure for existing use value, yet again this figure would be

depreciated to reflect the wear, tear, age condition and remaining life of the plant being

valued.

In summary, the kind of obsolescence determines the approach of valuation to be adopted

for such plant and machinery. As an addendum, it behooves us to have a working

knowledge of the various kinds of obsolescence prevalent.

Page 253: unitedemsa.files.wordpress.com...Pg 2 NIESV STUDY PACK - PPE KNOWLEDGE AND STRUCTURE OF NIGERIA INSTITUTION OF ESTATE SURVEYORS AND VALUERS Learning Objectives At the end of this lecture,

Pg 253

NIESV STUDY PACK - PPE

KINDS OF OBSOLESCENCE

Technological Obsolescence

This relates to the different relationship between the design and material of the subject

machinery item and that used in present day machine. Another example is in the area of

space requirement. If for instance the subject machinery requires a floor space of 95m2

for its operation while the modern type requires a floor space of 50m2.

This is a difference

of 45/95 or 47.37% less the space required of the previous machine.

Management of industries will always tend to maximize space by working increasing

productivity per square meter of plant area and floor space requirement of machine an

important condition for desiring the purchase of a particular machine.

Functional Obsolescence

This relates to difference in production rates or efficiency and other characteristics

between a new machine and the one under consideration. Another consideration could be

direct technical requirements. For instance a seven-year-old machine could be replaced

with a machine that is more expensive but increases the efficiency in production by 15%.

This 15% will incorporate labour and energy difference, down time, ease of repair, level

and spread of accuracy of finished products. Again, a new machine may produce 100

units per time frame compared with 70 units produced in the same period b y the machine

being valued. Machines are usually designed or adapted for a specific use. This could be

defined as the highest and best use of a machine, or the most profitable and likely use to

which a machine can be put. By implication, the ability of a property to be utilized at its

highest and best use can be associated with its value. Therefore, any use to which the

machine is put less than its highest and best use will be a contributing factor to

depreciation as it represents a loss from the upper limit of value. This limitation may be

termed functional obsolescence.

Economic Obsolescence

Economic obsolescence deals with influences external to the machinery itself. It seems

the most misunderstood and the most difficult for appraisers to quantify. It is defined as

the impairment of the useful life of a machine arising from economic forces such as,

change in optimum use, legislative enactments which restrict or impair a machine and

changes in its demand and supply relationship. These factors can be measured and

expressed in percentage of the subject machine productivity or potential use. It is the loss

in asset value created by factors unrelated to the asset itself, unlike physical and

functional obsolescence that are intrinsic to the asset.

Economic obsolescence is caused by factors external to the asset. Consequently it is often

referred to as external obsolescence, since it is beyond the control of the asset owner.

Page 254: unitedemsa.files.wordpress.com...Pg 2 NIESV STUDY PACK - PPE KNOWLEDGE AND STRUCTURE OF NIGERIA INSTITUTION OF ESTATE SURVEYORS AND VALUERS Learning Objectives At the end of this lecture,

Pg 254

NIESV STUDY PACK - PPE

Reduction in asset value attributable to economic obsolescence are usually beyond the

ability of the asset owner to influence and could not be cured by asset modification.

However, an examination of financial impact attractable to the external forces acting on

the asset may be obtained in quantifying the level of impact f economic obsolescence on

an asset. A reduction in selling prices and investor‟s returns are examples of potential

source for the creation of economic obsolescence. Summarily, the following are some of

the causes of economic obsolescence: reduction in the demand for the product, increased

computation, changes in supply of raw materials, increased cost of raw material, cost of

labor or utilities without a corresponding price increase of the product, inflation, high

interest rates, environmental considerations and legislation.

OBSOLESCENCE, DEPRECIATION AND THE USEFUL LIFES OF MACHINES

Depreciation is loss in value due to any cause. There is hardly any factor more relevant in

the valuation of plant and machinery than depreciation and obsolescence. It is necessary

to understand the clear distinction in meaning of depreciation as a different form of

obsolescence that has emerged in the evolution of concepts associated with technical

developments. Depreciation is a universal phenomenon. It is an attribute of all physical

objects that are subject to wear and tear whether in use or not in use. Remember the

catchphrase „every machine begins to depreciate immediately after being manufactured‟.

Thus, depreciation is therefore a premium for existence in the sense that decay is

inevitable. Obsolescence on the other hand is stimulated by exogenous factors, be it

technological, functional or economic. Students should recall that during the discourse of

the various types of obsolescence, another machine is used as a base in determining

whether the machine in question can be classified as obsolete. Hence, obsolescence is

externally motivated. The effect no doubt of obsolescence is depreciation in value but it

is not depreciation itself.

The useful life of Plant and machinery is paramount in determining the level of

obsolescence and depreciation of such items. The longer the useful life of a plant and

machine, perhaps the quicker or slower the obsolescence that is likely to be recorded

whereas for the sake of depreciation it is usually slower when the useful life is longer if

and only if the productivity of the machine is time oriented. The scenario is different

when the useful life is shorter as obsolescence could again be slower or quicker whereas

depreciation will be so fast.

Just as the valuation of land and buildings, the valuation of plant and machinery also

entails the valuer‟s subjective judgment. The sum figure of depreciation in relation to the

gross current replacement value can be summarized as follows:

Page 255: unitedemsa.files.wordpress.com...Pg 2 NIESV STUDY PACK - PPE KNOWLEDGE AND STRUCTURE OF NIGERIA INSTITUTION OF ESTATE SURVEYORS AND VALUERS Learning Objectives At the end of this lecture,

Pg 255

NIESV STUDY PACK - PPE

Condition Depreciation (%)

New (N) 0-5

Excellent (E) 6-10

Very Good (VG) 11-20

Good (G) 21-50

Fair (F) 51-60

Poor (P) 61-90

Scrap (S) 91-100

As the level of depreciation increases, the machine is tending towards obsolescence.

Thus, students should have noticed by now that a thin line is said to exist between

obsolescence and depreciation, but one point of interest is that while obsolescence is a

function of other external factors like the manufacturers, other available machines that

can serve as a substitute and other economic factors amongst others, whereas

depreciation on its part is a function of the existence of the machine.

VALUATION OF MACHINES AND PLANT FOR INSURANCE PURPOSES

Valuation of plant and machinery for insurance purposes is the most important purpose of

plant and machinery valuation. Perhaps, literature has revealed that insurance valuation is

the source of the greatest number of instructions received by valuers. Life itself is a risk

hence situations occur that causes people uncomfortable and many businesses ruined for

no cause of their own through fire outbreak, flood, earthquakes, other personal and

corporate carelessness etc. for these unforeseen circumstances and more, the issue of insuring these machines and equipments becomes mandatory to mitigate loss. Insurance valuation is carried out for the determination of the premium payable to the insurance company, in an attempt at guiding against under-insurance. Insurance valuation is carried out so as to provide the client with an accurate assessment of the value of the plant at risk based on the terms and conditions of the insurance policy. It is to guide against under-insurance also to guide against higher payment of premiums. Students should note that this particular purpose of plant and machinery has been singled out because while the open-market purpose is more akin to the basis of gone concern, the financial purpose of plant and machinery valuation can be regarded as being synonymous with the going concern basis. Insurance valuation for plant and machinery on its own part cannot be necessarily confined to any of these bases of plant and machinery valuation. It however has its bases of either indemnity or re-instatement. Moreover, insurance valuation could be undertaken to capture all acclaimed assets of the company of which plant and machinery forms part.

Page 256: unitedemsa.files.wordpress.com...Pg 2 NIESV STUDY PACK - PPE KNOWLEDGE AND STRUCTURE OF NIGERIA INSTITUTION OF ESTATE SURVEYORS AND VALUERS Learning Objectives At the end of this lecture,

Pg 256

NIESV STUDY PACK - PPE

In a nut shell this purpose of plant and machinery valuation is important and peculiar as a result of the following:

Insurance valuation for machines and plant is the source of the greatest

number of instructions received by valuers as it is willingly requested for

by clients (not necessarily statutorily engendered or coarsed)

Insurance valuation for machines and plant is not an end but a means to

an end. It is required to calculate premium by the actuarial scientist

payable by the insured.

Insurance valuation for machines and plant is not subject to the known

going concern basis of financial valuation and gone concern basis of open

market valuation rather it has its own typical bases of indemnity and re-

instatement.

Insurance valuation of machines and plant can be undertaken to capture

all acclaimed assets of the company of which machines and plant forms

part as against the other purposes.

The consequences of under-insurance: the condition of average “Whenever a sum insured is declared to be subject of average, if the property covered thereby shall, at the breaking out of any fire or at the commencement of any destruction of, or damage to, such property by any peril hereby insured against, be collectively of greater value than the sum insured, then the insured shall be considered to be their own insurers for the difference and shall bear a rate able share of the loss accordingly” . E.g. Value at the risk of the contents of the factory N5,000, 000 Contents of the factory at the time of the fire, were insured for N3, 000, 000 Actual loss sustained (agreed by both parties (insured & insurer)N1, 600, 000 The actual settlement would be calculated according to the below formula: Agreed loss sustained X Sum Insured

Value at risk = Amount payable

1, 600, 000 X 3, 000, 000

5, 000, 000 = N960, 000

Page 257: unitedemsa.files.wordpress.com...Pg 2 NIESV STUDY PACK - PPE KNOWLEDGE AND STRUCTURE OF NIGERIA INSTITUTION OF ESTATE SURVEYORS AND VALUERS Learning Objectives At the end of this lecture,

Pg 257

NIESV STUDY PACK - PPE

Note: a valuation of plant and machinery to be insured ensures that the insured party is

not subject to downward adjustment due to under insurance in case he needs to make a

claim, and also that he does not pay unnecessary premium. A downward adjustment will

occur on a claim if the sum insured is less than the true value at risk at the time of the

loss, in which case only a proportion of the proven loss will be paid, even where the loss

is below the sum insured. This in insurance practice is known as the “Condition of

Average”.

VALUATION FOR INSURANCE: Techniques and principles involved

Once there is an instruction from a client, a current specification of insurance together

with a print of the insurance plan showing the layout of the buildings and reference

numbers must be obtained from the client or his insurance broker.

The specification of insurance will contain the following information:

The present basis of insurance

Division of the total insurance into groupings

The scope of cover

Present Basis of Insurance:

There are two basis of insurance in general use for plant and machinery. These are –

reinstatement with new and indemnity.

Reinstatement with new value:

The reinstatement with new value of an item is simply the cost of replacing the existing

item with a substantially similar new item. It is the sum total of the new ex-works price

(terms of sale signifying that the price involved or quoted by a seller includes charges

only up to the seller‟s factory or premises. All charges from there on are to be borne by

the buyer) of a new replacement item less an allowance for age and obsolescence, plus

the costs of transportation, installation, taxes, duties and fees. The main sources of

information for the ex-works costs would be.

Up-to-date price lists and recent information from the surveyor’s own database:

As noted during the course of study of becoming of an Estate Surveyor, students should

be reminded that surveyors work well with information. Hence, every aspiring plant and

machinery surveyor must collect as much information as possible; on the prices of every

item he comes across either in the trade journals or advertisements. This would be formed

into a cross-reference database, which may come in useful someday.

Enquiries from direct contact with the manufacturer:

Page 258: unitedemsa.files.wordpress.com...Pg 2 NIESV STUDY PACK - PPE KNOWLEDGE AND STRUCTURE OF NIGERIA INSTITUTION OF ESTATE SURVEYORS AND VALUERS Learning Objectives At the end of this lecture,

Pg 258

NIESV STUDY PACK - PPE

This is possible only where the surveyor has full details concerning type, model, capacity

and serial number. Where the enquiry reveals that the existing item has been replaced by

an updated version, suitable adjustments in the valuation would have to be made for any

substantial improvement in the new model, as the insurers will not likely pay for such

improvement. What the adjustment would be and what constitute substantial

improvement could be subjective. It is important to agree this with the client or their

broker.

The original purchase price (the historical cost approach):

Students will remember that this approach has earlier been used during the valuation of

plants and machinery. This entails the adjustment of historical costs by application of

trade index. Surveyors are always tempted to use this method, as it seems convenient.

Getting in contact with manufacturers could seem thorny and the database system of

surveyors is controvertible. Thus, surveyors are left with this option.

However, it should be stated here that this approach also has its glitch. For instance, such

costs could have been in respect of sample or prototype machines whose costs would bear

no relationship to subsequent mass-produced ones. Again, where the original cost is to be

taken from the client‟s ledger, it may be difficult to determine the precise items to which

the ledger entry relates, and the entry may not even show whether the figures are for

complete item delivered and installed or not, except perhaps with the assistance of the

client or his broker.

In the long run, the historical cost approach when used should be corroborated with

adequate and appropriate information.

Having established the current new cost of an item, cost of transportation and the cost

of installation must be added to arrive at the reinstatement with new value.

Transportation costs may not be quoted in the ex-works price but they are easy to

establish by reference to local transporters who can usually be contacted near the

supplier‟s premises.

Installation costs will vary considerably depending on the item. While some items

will require no installation at all, some others may need specially made bases and

foundations to receive them. With some major process plant where a lot of services

together with deep foundations, steel supporting structures, etc. are necessary for their

installations, the cost may be anything up to half of the total cost. Fortunately for the

surveyor the supplier of most complex items of plant and machinery usually quote for

a full installation and commissioning of such items. Foundations may not necessary

Page 259: unitedemsa.files.wordpress.com...Pg 2 NIESV STUDY PACK - PPE KNOWLEDGE AND STRUCTURE OF NIGERIA INSTITUTION OF ESTATE SURVEYORS AND VALUERS Learning Objectives At the end of this lecture,

Pg 259

NIESV STUDY PACK - PPE

be subject to destruction by insured risks such as fire or flood, but their cost must be

included in the installation cost. This is because the new plant may not necessarily fit

the existing foundation, and at any rate the reinstatement may be on another spot.

Where the client confirms the existing foundation will be appropriate for the

installation of a replacement item, the cost of foundation will be excluded from the

valuation and a note made to that effect in the report.

The technical definition of reinstatement found on insurance specification is:

The carrying out of the aforementioned work, namely:

“Where property is destroyed, the rebuilding of the property, if it is a building or

in the case of other property, its replacement by similar property, in a condition

equal to but not better or more extensive than its condition when new”

“Where property is damaged, the repair of the damage and the restoration of the

damaged portion of the property to a condition substantially the same as but not

better or more extensive than its condition when new”

Reinstatement with new: New for old but not better than former = replacing existing

assets with identical or substantially similar equipment the manufacturer‟ current prices +

costs of transportation, installation, commissioning and import duties plus consulting

engineer‟s fees (where necessary)

Indemnity: monetary value of damage sustained- places the item insured at exactly the

position or condition it was immediately before the occurrence of the loss. The claim

settlement is either by repair, replacement with equivalent one where obtainable or by

payment of cash equivalent.

However, in recent literatures a third basis of insurance valuation is said to exist. This is

known as the debris removal cost. This is the basis of insurance in the case of a plant,

which is due for decommissioning in the near future. It generally applies to a complete

factory or production section rather than individual machine or equipment.

Division of the total insurance into groupings:

Lets take a look at an extract from a typical insurance specification in order to aid our

discussion.

Page 260: unitedemsa.files.wordpress.com...Pg 2 NIESV STUDY PACK - PPE KNOWLEDGE AND STRUCTURE OF NIGERIA INSTITUTION OF ESTATE SURVEYORS AND VALUERS Learning Objectives At the end of this lecture,

Pg 260

NIESV STUDY PACK - PPE

SPECIFICATION OF INSURANCE

-In the name of-

BAJULAWSON ENGINEERING LTD.

23, Anibalewa off Alli baba Street

Victoria Island

Lagos

Nigeria

(1) (2) (3)

On the buildings On machinery, plant On stock and

including landlord‟s and all other contents materials in trade

fixtures and fittings therein and thereon

therein and thereon and the property of

on the walls. Gates and the insured or held by

fences around and them in trust for which

belonging thereto they are responsible

(excluding landlord‟s

Fixtures and fittings, stock

Materials-in-trade)

Item

No

Plan No Description Col.1

N „000

Col.2

N „000

Col.3

N „000

Total

N „000

1 1-4 Machine

Shop,

Fabrication

Shop and

Compressor

House

165, 000 550, 000 - 715, 000

2 5 Stacking

Area

3,000 9,000 23, 000 35, 000

3 6-9 Offices and

Canteen

45, 000 16, 000 - 61, 000

4 10(a) Gate House 2, 000 600 - 2, 600

5 10(b) Generator

House

1, 500 12, 000 - 13, 500

6 12,13&18 Heavy

Machine

Shop

265, 000 375, 000 - 640, 000

Page 261: unitedemsa.files.wordpress.com...Pg 2 NIESV STUDY PACK - PPE KNOWLEDGE AND STRUCTURE OF NIGERIA INSTITUTION OF ESTATE SURVEYORS AND VALUERS Learning Objectives At the end of this lecture,

Pg 261

NIESV STUDY PACK - PPE

7 14-17 Foundry 120, 00 66, 000 - 186, 000

TOTAL 601, 500 1, 028,

600

23, 000 1, 653, 000

From an extract of specification as showcased above, it is evident that insurance

policies of companies cover certain assets such as land and buildings, machineries and

stocks amongst other classifications as prescribed. From extract above one can easily

see that in each location certain premium had been paid for the insurance cover for the

various assets. Thus, the insurance premium for plant and machinery so far is in the

tune of N1, 028, 600.

The content of the valuation will be stated in the specification as shown above and the

valuation must be prepared strictly in accordance with the details of the specification.

The extract is typical of the content of an insurance covering plant and machinery,

and as a general rule the valuation should include all items that are at risk from

insured perils and are the property or the responsibility of the insured, apart from

buildings, stock and materials-in-trade. Items generally regarded as landlord‟s fixtures

and fittings will be insured with the buildings rather than the plant and

machinery. Any doubt about what should be designated building or plant and

machinery should be discussed as agreed with the client or his insurance broker.

Where a surveyor is instructed to classify as plant and machinery an item, which

should be part of the building he must make a note of this in his report.

The scope of cover:

Plant and machinery valuations are carried out to provide the insured with an

assessment of the value at risk of the plant and other equipment for which he is

responsible in accordance with the terms of the insurance policy. The insurance cover

generally takes care of:

The plant and machinery (including special purpose machinery) all other contents

therein or thereon the property of the insured the property held in trust by the

insured for which they are responsible excluding landlord‟s fixtures and fittings

excluding stock and material-in-trade. Unless otherwise stated, items in the nature of

landlord‟s fixtures and fittings will normally be insured with buildings rather than

plants and machinery.

Information required includes: manufacturer‟s name, trade name or model name;

capacity of the machine; serial number of the machine; date of manufacture of the

machine; ancillary plant ad equipments; small tools and equipments; items outside the

factory-underground installations.

Page 262: unitedemsa.files.wordpress.com...Pg 2 NIESV STUDY PACK - PPE KNOWLEDGE AND STRUCTURE OF NIGERIA INSTITUTION OF ESTATE SURVEYORS AND VALUERS Learning Objectives At the end of this lecture,

Pg 262

NIESV STUDY PACK - PPE

Sources of cost information are: up-to-date price lists; direct contact with the

manufacturer; original purchase price from the client‟s records. Transportation costs

and installation costs are added to either of these.

Where there are supplementary conditions and memoranda attached to the insurance

specifications such as “all other contents”, this will mean that proper records of the

following should be taken care of:

1. Money and stamps: where this is not covered by a separate “all risks policy”

2. Documents, manuscripts and business books: this is to the value of the materials with

the cost of clerical labour expended in writing up.

3. Computer systems records: this is to the value of the materials, cost of clerical labour

and computer time expended in reproducing such records

4. Plans, design, etc: cost of replacement

5. Personal effects of employees and visitors: reinstatement clause does not cover

personal effects. The correct basis of valuation is indemnity.

For plant and machinery held on trust, lease, or rented, the question of insurance

liability must be considered first.

Insurance on a “first loss basis policy” in the context of plant and machinery, the

following items are usually concerned: debris removal cost, consultants‟ fees, disused

or little used plant, plant foundations and piling, administration and computer records,

specific product items

Page 263: unitedemsa.files.wordpress.com...Pg 2 NIESV STUDY PACK - PPE KNOWLEDGE AND STRUCTURE OF NIGERIA INSTITUTION OF ESTATE SURVEYORS AND VALUERS Learning Objectives At the end of this lecture,

Pg 263

NIESV STUDY PACK - PPE

VALUATION OF PLANT AND MACHINERY FOR INSURANCE

PURPOSES

In this section, the detailed scope of insurance valuation for plant and machinery will be

the focus of discourse. Perhaps this can also be regarded as the contents of insurance

valuation for plants and machineries. Thus, apart from core plant and machineries other

items likely to be considered in this purpose of valuation include:

- Special Purpose Item:

Unless such an item was recently made for the client, the manufacturer may not be able

to help with the price of the new one. It is even possible that such item has been

built in the client‟s own workshop and no prices are available. The surveyor will

therefore have to adopt one of the following options:

Estimate the value by reference to details available in the client‟s workshop, plant book

and payment ledgers. This estimate should exclude the cost of the construction

drawings and records, which will be valued with other documents.

Base the value on a similar item that is currently available by estimating the current cost

of such item and then making allowance for any modifications or improvement to

the existing item.

Updating the historic costs

- Money and stamps:

Where money is to be insured on a separate basis it may not be necessary to include it in

the valuation. In other cases enquiries should be made into the maximum amount

of cash that may be held on the company‟s premises at any given time, such order

is to be included in the valuation.

- Documents:

Insurance cover on documents, manuscript and business records kept on the company‟s

premises are to be included in the valuation especially for the materials only

together with the costs of clerical work involved in writing them. Insurance cover

on these items would not include the commercial value of information contained

in the items. Estimate of the valuation of these items in circumstance can only be

an approximation. It is not usually feasible to calculate the exact cost of

reinstating records in such items. In making the approximation, records, which

should be taken into account include: ledgers, order books, personnel records,

Page 264: unitedemsa.files.wordpress.com...Pg 2 NIESV STUDY PACK - PPE KNOWLEDGE AND STRUCTURE OF NIGERIA INSTITUTION OF ESTATE SURVEYORS AND VALUERS Learning Objectives At the end of this lecture,

Pg 264

NIESV STUDY PACK - PPE

stores and works progress records, market research documentation, plant register,

laboratory records

- Computer data and software:

It is now the general practice to make duplicate copies of computer records and software

regularly and the maximum loss in respect of such items would be the cost of

taking copies and reproducing the work. The surveyor must seek assurance that

that the duplicate copies are kept in other premises besides where the insured risk

is located. If this is established the valuation would include only the cost of

reproduction.

- Product- dedicated items:

These are items used in connection with the manufacturing of an individual product.

They have no general application and are not considered as part of the process

plant. Insurance covers usually stipulates that in the event of loss, the basis of

settlement will be the reinstatement costs for those product dedicated items in

current use and which the insured would wish to replace in the event of damage or

destruction by any peril insured against. The valuation will therefore need to

establish the cost of replacing these items at current new prices. A great deal in

this regard will depend on the accuracy and veracity of the records kept by the

company, because while it may not be possible to establish an exact figure, a

realistic and practical value must be provided. If the company keeps accurate

records, there will likely be noticed a constant demand for such items, and with

this, it should be possible to establish a level for the current new price without

much difficulty. If suitable records are not available, the only way to establish

quantities of these items will be to organize a physical inspection which can be

time wasting. On the other hand it is crucial that a thorough investigation into the

value of this asset be done, because in some industries, such as plastic moulding

and foundries, the insurance value of these items could exceed that of the process

plant.

- Personal effects:

Allowance must be made in the valuation for the personal effects of directors, employees

and visitors that could be kept on the insured premises at a given time and not

otherwise insured. The valuation should make sure the value limit in respect of

these items in the insurance memorandum is realistic, but if not, the client should

Page 265: unitedemsa.files.wordpress.com...Pg 2 NIESV STUDY PACK - PPE KNOWLEDGE AND STRUCTURE OF NIGERIA INSTITUTION OF ESTATE SURVEYORS AND VALUERS Learning Objectives At the end of this lecture,

Pg 265

NIESV STUDY PACK - PPE

be so advised and an opinion of a reasonable value given. The correct basis of this

valuation is indemnity.

- Items off site:

These are properties of the company not available at the time of inspection, either on loan

to a sub-contractor or have been sent for repairs and overhaul. If such items are

covered by the insurance policy, they must be included in the valuation.

- Items held on trust:

There may be items on the insured premises, which are not the property of the insured.

These will include items on loan or lease to the insured. The agreement for each

lease or loan will include liability for insurance and this should be studied

carefully as the relevant as the relevant clause in the agreement may state for how

much it must be insured. Where the agreement is silent on insurance value, the

surveyor must agree a figure with the client. If the loaned item is to be excluded

on the instructions of the client, this should be stated in the report.

In the cases of items held by the client on behalf of customers, the client will have to

decide whether such items should be included in the valuation or not. Where such

items are included in the client‟s insurance policy as an extension they must be

included in the valuation.

- Separate policies:

Items that can easily be stolen or damaged are usually insured under an all-risks policy

(an insurance policy covering personal possessions against many risks but not, of

course, all risks. A policy of this kind does not list the risks covered; instead it lists

only the exclusions. Such wide cover often merits very high premiums and items

covered on the basis often include jewellery, photographic or electronic

equipment, and other valuables), which may include risks covered by the main

insurance. In that case, it is not necessary to include such items in the valuation.

Items of plant and machinery, which are usually covered by a “First Loss” policy (an

insurance policy for goods in which a total loss is extremely unlikely and the

insurer agrees to provide cover for a sum less than the total value of the property),

where the total value at risk is spread over separate areas of risk include: Debris

removal costs, Consultants‟ fees, Plant foundation and piling, Administration and

computer records, Specific product items. These items, which are covered by

specific insurance, should be excluded from the valuation and a note to this effect

should be included in the report.

Page 266: unitedemsa.files.wordpress.com...Pg 2 NIESV STUDY PACK - PPE KNOWLEDGE AND STRUCTURE OF NIGERIA INSTITUTION OF ESTATE SURVEYORS AND VALUERS Learning Objectives At the end of this lecture,

Pg 266

NIESV STUDY PACK - PPE

- Motor vehicles:

These are covered by the main policy only to the extent that they are not covered by their

specific motor policy. In the case of damage due to any peril covered by the main

policy, claims can only be made on the difference between the amount recoverable

under the specific policy and the indemnity value. The reinstatement clause does

not normally apply to motor vehicles.

Motor vehicles and their contents are usually specifically insured and as such excluded

from the policy except in respect of the amount over and above that recoverable

under the more specific insurance. Thus, the difference between the amount

recovered on the specific motor vehicle policy and the indemnity value is equal to

the amount by which the vehicle was under-insured on the motor vehicle policy at

the time of loss.

However, unregistered internal transport fleets will normally be included with the plant

and machinery on the fire insurance policy.

- Professional fees:

Consultants‟ fees for estimates, designs, specifications, supervision and commissioning

that will be incurred in the reinstatement of an item will be taken into account in

the valuation. It is possible that the design, supervision, etc. would be done by the

insured‟s own staff, but allowance must still be made since direct expenditure and

overheads would be involved.

- Uncompleted plant:

During the valuation survey, there could be items, which are still being installed. If they

are small items, which are quickly installed, they could be included. However, the

treatment of major process plant, which could be expensive and may take months

to install, will depend on the installation agreement.

If the contractor is expected to insure against risks already covered by the factory policy,

the correct approach is to exclude the item from the valuation. The report will

however include a recommendation to the client to amend the main policy at the

time of commissioning the item.

If the client is liable for insurance cover, either the total estimated contract price is

incorporated in the valuation or a suitable amount included to cover work

completed up to date. A note on this should be included in the final report when

submitting the valuation.

Page 267: unitedemsa.files.wordpress.com...Pg 2 NIESV STUDY PACK - PPE KNOWLEDGE AND STRUCTURE OF NIGERIA INSTITUTION OF ESTATE SURVEYORS AND VALUERS Learning Objectives At the end of this lecture,

Pg 267

NIESV STUDY PACK - PPE

Where the valuation survey reveals that a major item is to be installed in the near future,

no allowance would be made for the item in the valuation, and the report will

make a note of this.

- Public authorities’ requirements

Where the public authorities‟ requirements clause is attached to the specification of

insurance, the amount recoverable under the policy will extend to include such

additional costs of reinstatement of the destroyed or damaged property that may be

incurred by reason of the necessity to comply with any Act of Parliament or Local

Authority bye law.

Same goes for a plant or factory that represents a major risk or hazard near a residential

area irrespective of whether or not the plant is entirely within the factory building

or not. Where public authority requirements are likely to have a profound effect on

the reconstruction of the building in the event of a major loss that may have a

knockout effect on the type of plant, which could be accommodated, each case

should be assessed on its merit.

- Debris removal:

Most fire insurance policies will cover the cost of removal of debris together with costs

of dismantling. These must be reflected in the valuation as it can form a significant

proportion of the total value at risks, especially for complicated items like

chemical or petroleum plants. The sum total of the value at risk to be insured will

be the assessed value of each item plus the cost of debris removal where

appropriate. Industrial fire insurance policies usually cover the cost of removal of

debris together with costs of dismantling and where necessary, shoring up or

propping up.

- Loss assessors’ fees:

Unless specifically stated the costs of compiling and negotiating a claim in the event of a

loss or damage by an insured peril will not be payable under the terms of most

industrial fire insurance.

-Value added tax

Insurance valuations are generally reported exclusive of value added tax.

- Inflation protection:

Page 268: unitedemsa.files.wordpress.com...Pg 2 NIESV STUDY PACK - PPE KNOWLEDGE AND STRUCTURE OF NIGERIA INSTITUTION OF ESTATE SURVEYORS AND VALUERS Learning Objectives At the end of this lecture,

Pg 268

NIESV STUDY PACK - PPE

Valuation of risks to be insured will be based on prevailing prices at the date of the

valuation while in fact the loss or damage being insured against may not take place

for a long time during the life of the policy. It is necessary to give due

consideration to changes in prices, which may occur during the life of the policy

and in the case of reinstatement with new policies, during the reinstatement period

after the loss.

Inflation will not be of importance in an indemnity valuation where the value represents

the worth at the time of the loss and not at the date of reinstatement. It is however

very important with reinstatement with new valuation, especially with items which

take considerably long period to re-install such as chemical plants and refineries.

In reinstatement with new policies, it is usual to find clauses provided for the cost to rise

automatically in line with inflation.

The report:

As an absolute minimum, the surveyor‟s report must include the following amongst

others to be discussed under report writing.

1. The name and address of the insured

2. The location of items in the valuation

3. Date of the valuation

4. Basis of the valuation

5. Content of the valuation

6. Grouping of the contents

7. Relevant points needed to be highlighted

8. Projections for future inflations

9. Surveyor‟s name and seal

Unless otherwise requested the valuation for insurance should be reported in the local

currency with a note in the report on the exchange rate between the local currency

and the currency of the country of supply of the items at the date of the valuation.

The report should endeavour as much as possible to follow the format of the insurance

specifications when showing the separation of the total amounts into groups. The

value of buildings and plants and machinery should be reported in one document,

but in different columns. (See insurance specification of BAJULAWSON

ENGINEERING LTD. In lecture above)

Page 269: unitedemsa.files.wordpress.com...Pg 2 NIESV STUDY PACK - PPE KNOWLEDGE AND STRUCTURE OF NIGERIA INSTITUTION OF ESTATE SURVEYORS AND VALUERS Learning Objectives At the end of this lecture,

Pg 269

NIESV STUDY PACK - PPE

BACKGROUND AND HISTORY OF PLANT AND MACHINERY

It can be said that the Rating and Valuation Act 1925 was the first legislation that made

specific reference to the rating of plant and machinery. However, it cannot be said

categorically plant and machinery that existed before then had not been rated. This can be

evidenced in “The Domes day Book” which can be regarded as the first comprehensive

record for tax purposes, amongst others, of plant and machinery. The law of rating stems

from the Poor Relief Act 1601 (or even earlier), which required the taxation of every

inhabitant and of every occupier of lands, houses, tithes, etc but which did not specify

what personal property in particular should be included in the assessment. (The Poor Rate

Exemption Act 1840 exempted stock-in-trade from liability to rates).

Just as before embarking on the task of valuation of plant and machinery, it was pertinent

to identify what constitute plant and machinery. Likewise before taking a look at rate

ability as it affects plant and machinery, a quick look at the identification of plant and

machinery is also apt.

SI 2001 No.846 The Valuation for Rating (Plant and Machinery ) ( England ) (

Amendment ) Regulations 2001 and

SI 2001 No.2357 ( W.195 ) The Valuation for Rating ( Plant and Machinery ) Wales

) ( Amendment ) Regulations 2001

which made changes to the pre-amble to Class 1.

(see LINK to PN 3 )

2.10 Micro generation

The 2000 Regulations were further amended in 2008 by inserting a new Regulation 2A

which provides for a temporary exclusion of the value of defined micro generation plant

from the rateable value of a hereditament.

SI 2008 No.2332 The Valuation for Rating ( Plant and Machinery ) ( England ) (

Amendment ) Regulations 2008

The regulations apply to England and have effect from 1st October 2008. Similar

regulations came into force for Wales, effective from 1st April 2010.

SI 2010 No.146 W21 Valuation for Rating (Plant and Machinery) (Wales)

(Amendment) Regulations.

Page 270: unitedemsa.files.wordpress.com...Pg 2 NIESV STUDY PACK - PPE KNOWLEDGE AND STRUCTURE OF NIGERIA INSTITUTION OF ESTATE SURVEYORS AND VALUERS Learning Objectives At the end of this lecture,

Pg 270

NIESV STUDY PACK - PPE

(See LINK to PN 2)

3 Recommended Practice

(The following text is written with reference to England, if you are dealing with a Welsh

hereditament the appropriate Welsh Regulations are noted above.)

3.1 General

If an item or structure in or on the hereditament can be classified as plant or machinery

then it will be rateable if it can be identified with a Class in the Schedule to the

Regulations, provided that is not subject to exemption or exception. Please see PN 1 for

further discussion on the individual Classes.

3.2 Approach to Rate ability

Key Point

Firstly establish that you are dealing with plant (If the item is not plant, it may still

be rateable, e.g. mezzanine floors)

If you are dealing with plant, then refer to the Regulations to establish rateability.

The valuer in considering whether an item should properly be included in the valuation

needs to adopt a two-stage approach.

First it is necessary to conclude whether the item is plant and/or machinery, and the

definition of plant which received House of Lords approval is to be found in Lindley LJ's

judgement in Yarmouth v France (1887), (see para 2 above)

In Jarrold v John Good & Sons Ltd (1963), a distinction was drawn between premises

in which business is carried on and plant with which business is carried on. (see para 2)

For example, in Rogers (VO) v Evans 1985 LT 275 EG 727 the Member agreed with

the VO that a mezzanine floor ( supported first floor ) was not plant but “ more a part of

the setting than part of the apparatus for carrying on the trade ” and that “ Its function is

.... to provide places in which the business is carried on, not to be plant with which it is

carried on ”.

If it is concluded that the item is plant and/or machinery it is then necessary, as a second

stage, to establish whether it can be identified with an item named in the Schedule to SI

2000 number 540, (the Regulations). It will only be rateable if it can be so identified.

It should be noted that the Regulations only apply to plant and machinery. If a chattel

which cannot be defined as either plant or machinery is sufficiently attached to or

Page 271: unitedemsa.files.wordpress.com...Pg 2 NIESV STUDY PACK - PPE KNOWLEDGE AND STRUCTURE OF NIGERIA INSTITUTION OF ESTATE SURVEYORS AND VALUERS Learning Objectives At the end of this lecture,

Pg 271

NIESV STUDY PACK - PPE

enjoyed with the land, it can be rated as enhancing the value and forming part of the

rateable hereditament. ( Storehire ( UK ) Ltd v Wojcik LT 1991 RA 39 )

4 Identification of Rateable Plant and Machinery

Key Point

The Regulations list items of plant under four headings, or Classes. Class 1 deals

with „power‟ items, Class 2 „services‟, Class 3 „transport‟, and Class 4 „structures‟.

The approach to considering rateability within each Class is considered in more

detail in Practice Note 1 : LINK

The Regulations comprise a comprehensive list of items, in most cases it will be clear as

to whether an item is included or not. However whilst some items are described in

generic terms, others have a specific description.

Classes 1, 2, and 4 have specific pre ambles which further identifies when an item

can be rateable. Class 3 items are subject to their own specified limitations. The List

of Accessories is only applicable to Classes 1 and 2.

In cases of doubt, the terms which are used in the Regulations and which are such that

they have a clear meaning in everyday language should be so interpreted, whilst those

which are familiar only to the technician should be given the meaning which would be

accorded to them by a person familiar with the particular plant.

Reference should be made to Post Office v Escott (VO) & Kerrier RDC 1974 LT RA

97 in which J H Emlyn Jones referred to previous authorities on the matter, and to

Elliott's Bricks Ltd v Hartley (VO) 1990 LT RA 161 where the Lands Tribunal upheld

the VO's contentions that two cylinders used for storing liquid petroleum gas (described

by the ratepayers as vessels) were tanks and therefore a named item in Class 4. ( Under

the1989 P & M Regulations “ vessels ” had to be used for a specific purpose and the

storage of liquid petroleum gas was not a specified use. Vessels are unqualified as to

use in the 1994 and 2000 Plant and Machinery Regulations )

See also Chesterfield Tube Co Ltd v Thomas (VO) [1970] RA 471 (

Whitfield (VO) v National Transcommunications Ltd [1994] RA 214 and

Thames Water PLC v Handcock (VO) [2008] RA 413.

5 Referencing

Key Point

Page 272: unitedemsa.files.wordpress.com...Pg 2 NIESV STUDY PACK - PPE KNOWLEDGE AND STRUCTURE OF NIGERIA INSTITUTION OF ESTATE SURVEYORS AND VALUERS Learning Objectives At the end of this lecture,

Pg 272

NIESV STUDY PACK - PPE

Where it is safe to do so, and you have the ratepayer‟s permission, always take

photos of the plant or machinery found on site, together with any identification plate

showing the type, size and output of the item. Note what the item does, (if in doubt

ask). But above all, do not take risks while inspecting, measuring or taking photos.

Obey all site requirements for personal protective equipment and obey any health

and safety instructions given to you.

The caseworker should ensure that details of all rateable plant and machinery are

recorded when inspecting the hereditament. In the majority of hereditaments, there will

be an element of rateable plant and machinery in or on the hereditament which will

comprise of service plant which will normally be reflected in the price or cost that is

applied to the building being valued.

Recording of this service plant is essential, not only to ensure that the extent, quality and

condition of the plant is properly taken into account in the valuation but also so that the

caseworker can respond, if necessary, to requests to supply particulars of the plant and

machinery that has been assumed to form part of the hereditament (P and M Regulation 3

- Supply of written particulars).

In addition appropriate details will be required of all plant and machinery which is to be

separately costed/valued, not only to ensure that the item has been properly recorded, but

also so that the item can be matched and/or compared with similar items which are

detailed in any standard cost guidance from which the caseworker can establish an

appropriate cost for the item under consideration (In many cases relevant details can be

obtained from the manufacturer's name plate on the item or by enquiries of the site

engineer / electrical engineer).

Thus prior to inspection it may be necessary to refer to the relevant standard cost

guidance (and/or such referencing guides as may be issued) to confirm the details that are

required for a particular item so that the information obtained and the unit of measure

recorded on site will be compatible with the basis of the unit prices contained in the

guide.

These laws as its still ongoing evidenced in the UK can be anchored as the basic laws on

which most commonwealth nations hinge upon in dealings of plant and machinery

valuation. There are however no specific laws that deal with such assets directly in

Nigeria. However, impliedly certain laws make recourse to plant and machinery and not

necessarily its valuation. Such Act include: Company Income Tax Act CAP 60 LFN

1960; Act on Factories, Value added Tax etc.

The ACT governing the act of valuation involving plant and machinery if not set into

motion by our regulatory body might fall into the hands of “learned men” who apparently

Page 273: unitedemsa.files.wordpress.com...Pg 2 NIESV STUDY PACK - PPE KNOWLEDGE AND STRUCTURE OF NIGERIA INSTITUTION OF ESTATE SURVEYORS AND VALUERS Learning Objectives At the end of this lecture,

Pg 273

NIESV STUDY PACK - PPE

have no knowledge of valuation thereby eventually directing our course of operation.

This is a wake-up call.